Sunteți pe pagina 1din 166

G.R. No.

81561 January 18, 1991

PEOPLE OF THE PHILIPPINES, plaintiff-appellee


vs.
ANDRE MARTI, accused-appellant.

The Solicitor General for plaintiff-appellee.


Reynaldo B. Tatoy and Abelardo E. Rogacion for accused-appellant.

BIDIN, J.:

This is an appeal from a decision * rendered by the Special Criminal Court of Manila (Regional Trial Court, Branch XLIX)
convicting accused-appellant of violation of Section 21 (b), Article IV in relation to Section 4, Article 11 and Section 2 (e) (i),
Article 1 of Republic Act 6425, as amended, otherwise known as the Dangerous Drugs Act.

The facts as summarized in the brief of the prosecution are as follows:

On August 14, 1987, between 10:00 and 11:00 a.m., the appellant and his common-law wife, Shirley Reyes, went to the booth of
the "Manila Packing and Export Forwarders" in the Pistang Pilipino Complex, Ermita, Manila, carrying with them four (4) gift
wrapped packages. Anita Reyes (the proprietress and no relation to Shirley Reyes) attended to them. The appellant informed Anita
Reyes that he was sending the packages to a friend in Zurich, Switzerland. Appellant filled up the contract necessary for the
transaction, writing therein his name, passport number, the date of shipment and the name and address of the consignee, namely,
"WALTER FIERZ, Mattacketr II, 8052 Zurich, Switzerland" (Decision, p. 6)

Anita Reyes then asked the appellant if she could examine and inspect the packages. Appellant, however, refused, assuring her
that the packages simply contained books, cigars, and gloves and were gifts to his friend in Zurich. In view of appellant's
representation, Anita Reyes no longer insisted on inspecting the packages. The four (4) packages were then placed inside a brown
corrugated box one by two feet in size (1' x 2'). Styro-foam was placed at the bottom and on top of the packages before the box
was sealed with masking tape, thus making the box ready for shipment (Decision, p. 8).

Before delivery of appellant's box to the Bureau of Customs and/or Bureau of Posts, Mr. Job Reyes (proprietor) and husband of
Anita (Reyes), following standard operating procedure, opened the boxes for final inspection. When he opened appellant's box, a
peculiar odor emitted therefrom. His curiousity aroused, he squeezed one of the bundles allegedly containing gloves and felt dried
leaves inside. Opening one of the bundles, he pulled out a cellophane wrapper protruding from the opening of one of the gloves.
He made an opening on one of the cellophane wrappers and took several grams of the contents thereof (tsn, pp. 29-30, October 6,
1987; Emphasis supplied).

Job Reyes forthwith prepared a letter reporting the shipment to the NBI and requesting a laboratory examination of the samples he
extracted from the cellophane wrapper (tsn, pp. 5-6, October 6, 1987).

He brought the letter and a sample of appellant's shipment to the Narcotics Section of the National Bureau of Investigation (NBI),
at about 1:30 o'clock in the afternoon of that date, i.e., August 14, 1987. He was interviewed by the Chief of Narcotics Section.
Job Reyes informed the NBI that the rest of the shipment was still in his office. Therefore, Job Reyes and three (3) NBI agents,
and a photographer, went to the Reyes' office at Ermita, Manila (tsn, p. 30, October 6, 1987).

Job Reyes brought out the box in which appellant's packages were placed and, in the presence of the NBI agents, opened the top
flaps, removed the styro-foam and took out the cellophane wrappers from inside the gloves. Dried marijuana leaves were found to
have been contained inside the cellophane wrappers (tsn, p. 38, October 6, 1987; Emphasis supplied).

The package which allegedly contained books was likewise opened by Job Reyes. He discovered that the package contained
bricks or cake-like dried marijuana leaves. The package which allegedly contained tabacalera cigars was also opened. It turned out
that dried marijuana leaves were neatly stocked underneath the cigars (tsn, p. 39, October 6, 1987).

The NBI agents made an inventory and took charge of the box and of the contents thereof, after signing a "Receipt"
acknowledging custody of the said effects (tsn, pp. 2-3, October 7, 1987).

Thereupon, the NBI agents tried to locate appellant but to no avail. Appellant's stated address in his passport being the Manila
Central Post Office, the agents requested assistance from the latter's Chief Security. On August 27, 1987, appellant, while
claiming his mail at the Central Post Office, was invited by the NBI to shed light on the attempted shipment of the seized dried
leaves. On the same day the Narcotics Section of the NBI submitted the dried leaves to the Forensic Chemistry Section for
laboratory examination. It turned out that the dried leaves were marijuana flowering tops as certified by the forensic chemist.
(Appellee's Brief, pp. 9-11, Rollo, pp. 132-134).

Thereafter, an Information was filed against appellant for violation of RA 6425, otherwise known as the Dangerous Drugs Act.

After trial, the court a quo rendered the assailed decision.

In this appeal, accused/appellant assigns the following errors, to wit:

THE LOWER COURT ERRED IN ADMITTING IN EVIDENCE THE ILLEGALLY SEARCHED AND SEIZED OBJECTS
CONTAINED IN THE FOUR PARCELS.

THE LOWER COURT ERRED IN CONVICTING APPELLANT DESPITE THE UNDISPUTED FACT THAT HIS RIGHTS
UNDER THE CONSTITUTION WHILE UNDER CUSTODIAL PROCEEDINGS WERE NOT OBSERVED.

THE LOWER COURT ERRED IN NOT GIVING CREDENCE TO THE EXPLANATION OF THE APPELLANT ON HOW
THE FOUR PARCELS CAME INTO HIS POSSESSION (Appellant's Brief, p. 1; Rollo, p. 55)

1. Appellant contends that the evidence subject of the imputed offense had been obtained in violation of his constitutional rights
against unreasonable search and seizure and privacy of communication (Sec. 2 and 3, Art. III, Constitution) and therefore argues
that the same should be held inadmissible in evidence (Sec. 3 (2), Art. III).

Sections 2 and 3, Article III of the Constitution provide:

Sec. 2. The right of the people to be secure in their persons, houses, papers and effects against unreasonable searches and seizures
of whatever nature and for any purpose shall be inviolable, and no search warrant or warrant of arrest shall issue except upon
probable cause to be determined personally by the judge after examination under oath or affirmation of the complainant and the
witnesses he may produce, and particularly describing the place to be searched and the persons or things to be seized.

Sec. 3. (1) The privacy of communication and correspondence shall be inviolable except upon lawful order of the court, or when
public safety or order requires otherwise as prescribed by law.

(2) Any evidence obtained in violation of this or the preceding section shall be inadmissible for any purpose in any proceeding.

Our present constitutional provision on the guarantee against unreasonable search and seizure had its origin in the 1935 Charter
which, worded as follows:

The right of the people to be secure in their persons, houses, papers and effects against unreasonable searches and seizures shall
not be violated, and no warrants shall issue but upon probable cause, to be determined by the judge after examination under oath
or affirmation of the complainant and the witnesses he may produce, and particularly describing the place to be searched, and the
persons or things to be seized. (Sec. 1 [3], Article III)

was in turn derived almost verbatim from the Fourth Amendment ** to the United States Constitution. As such, the Court may
turn to the pronouncements of the United States Federal Supreme Court and State Appellate Courts which are considered doctrinal
in this jurisdiction.

Thus, following the exclusionary rule laid down in Mapp v. Ohio by the US Federal Supreme Court (367 US 643, 81 S.Ct. 1684, 6
L.Ed. 1081 [1961]), this Court, in Stonehill v. Diokno (20 SCRA 383 [1967]), declared as inadmissible any evidence obtained by
virtue of a defective search and seizure warrant, abandoning in the process the ruling earlier adopted in Moncado v. People's
Court (80 Phil. 1 [1948]) wherein the admissibility of evidence was not affected by the illegality of its seizure. The 1973 Charter
(Sec. 4 [2], Art. IV) constitutionalized the Stonehill ruling and is carried over up to the present with the advent of the 1987
Constitution.

In a number of cases, the Court strictly adhered to the exclusionary rule and has struck down the admissibility of evidence
obtained in violation of the constitutional safeguard against unreasonable searches and seizures. (Bache & Co., (Phil.), Inc., v.
Ruiz, 37 SCRA 823 [1971]; Lim v. Ponce de Leon, 66 SCRA 299 [1975]; People v. Burgos, 144 SCRA 1 [1986]; Roan v.
Gonzales, 145 SCRA 687 [1987]; See also Salazar v. Hon. Achacoso, et al., GR No. 81510, March 14, 1990).
It must be noted, however, that in all those cases adverted to, the evidence so obtained were invariably procured by the State
acting through the medium of its law enforcers or other authorized government agencies.

On the other hand, the case at bar assumes a peculiar character since the evidence sought to be excluded was primarily discovered
and obtained by a private person, acting in a private capacity and without the intervention and participation of State authorities.
Under the circumstances, can accused/appellant validly claim that his constitutional right against unreasonable searches and
seizure has been violated? Stated otherwise, may an act of a private individual, allegedly in violation of appellant's constitutional
rights, be invoked against the State?

We hold in the negative. In the absence of governmental interference, the liberties guaranteed by the Constitution cannot be
invoked against the State.

As this Court held in Villanueva v. Querubin (48 SCRA 345 [1972]:

1. This constitutional right (against unreasonable search and seizure) refers to the immunity of one's person, whether citizen or
alien, from interference by government, included in which is his residence, his papers, and other possessions. . . .

. . . There the state, however powerful, does not as such have the access except under the circumstances above noted, for in the
traditional formulation, his house, however humble, is his castle. Thus is outlawed any unwarranted intrusion by government,
which is called upon to refrain from any invasion of his dwelling and to respect the privacies of his life. . . . (Cf. Schermerber v.
California, 384 US 757 [1966] and Boyd v. United States, 116 US 616 [1886]; Emphasis supplied).

In Burdeau v. McDowell (256 US 465 (1921), 41 S Ct. 547; 65 L.Ed. 1048), the Court there in construing the right against
unreasonable searches and seizures declared that:

(t)he Fourth Amendment gives protection against unlawful searches and seizures, and as shown in previous cases, its protection
applies to governmental action. Its origin and history clearly show that it was intended as a restraint upon the activities of
sovereign authority, and was not intended to be a limitation upon other than governmental agencies; as against such authority it
was the purpose of the Fourth Amendment to secure the citizen in the right of unmolested occupation of his dwelling and the
possession of his property, subject to the right of seizure by process duly served.

The above ruling was reiterated in State v. Bryan (457 P.2d 661 [1968]) where a parking attendant who searched the automobile to
ascertain the owner thereof found marijuana instead, without the knowledge and participation of police authorities, was declared
admissible in prosecution for illegal possession of narcotics.

And again in the 1969 case of Walker v. State (429 S.W.2d 121), it was held that the search and seizure clauses are restraints upon
the government and its agents, not upon private individuals (citing People v. Potter, 240 Cal. App.2d 621, 49 Cap. Rptr, 892
(1966); State v. Brown, Mo., 391 S.W.2d 903 (1965); State v. Olsen, Or., 317 P.2d 938 (1957).

Likewise appropos is the case of Bernas v. US (373 F.2d 517 (1967). The Court there said:

The search of which appellant complains, however, was made by a private citizen — the owner of a motel in which appellant
stayed overnight and in which he left behind a travel case containing the evidence***complained of. The search was made on the
motel owner's own initiative. Because of it, he became suspicious, called the local police, informed them of the bag's contents, and
made it available to the authorities.

The fourth amendment and the case law applying it do not require exclusion of evidence obtained through a search by a private
citizen. Rather, the amendment only proscribes governmental action."

The contraband in the case at bar having come into possession of the Government without the latter transgressing appellant's
rights against unreasonable search and seizure, the Court sees no cogent reason why the same should not be admitted against him
in the prosecution of the offense charged.

Appellant, however, would like this court to believe that NBI agents made an illegal search and seizure of the evidence later on
used in prosecuting the case which resulted in his conviction.

The postulate advanced by accused/appellant needs to be clarified in two days. In both instances, the argument stands to fall on its
own weight, or the lack of it.
First, the factual considerations of the case at bar readily foreclose the proposition that NBI agents conducted an illegal search and
seizure of the prohibited merchandise. Records of the case clearly indicate that it was Mr. Job Reyes, the proprietor of the
forwarding agency, who made search/inspection of the packages. Said inspection was reasonable and a standard operating
procedure on the part of Mr. Reyes as a precautionary measure before delivery of packages to the Bureau of Customs or the
Bureau of Posts (TSN, October 6 & 7, 1987, pp. 15-18; pp. 7-8; Original Records, pp. 119-122; 167-168).

It will be recalled that after Reyes opened the box containing the illicit cargo, he took samples of the same to the NBI and later
summoned the agents to his place of business. Thereafter, he opened the parcel containing the rest of the shipment and entrusted
the care and custody thereof to the NBI agents. Clearly, the NBI agents made no search and seizure, much less an illegal one,
contrary to the postulate of accused/appellant.

Second, the mere presence of the NBI agents did not convert the reasonable search effected by Reyes into a warrantless search and
seizure proscribed by the Constitution. Merely to observe and look at that which is in plain sight is not a search. Having observed
that which is open, where no trespass has been committed in aid thereof, is not search (Chadwick v. State, 429 SW2d 135). Where
the contraband articles are identified without a trespass on the part of the arresting officer, there is not the search that is prohibited
by the constitution (US v. Lee 274 US 559, 71 L.Ed. 1202 [1927]; Ker v. State of California 374 US 23, 10 L.Ed.2d. 726 [1963];
Moore v. State, 429 SW2d 122 [1968]).

In Gandy v. Watkins (237 F. Supp. 266 [1964]), it was likewise held that where the property was taken into custody of the police
at the specific request of the manager and where the search was initially made by the owner there is no unreasonable search and
seizure within the constitutional meaning of the term.

That the Bill of Rights embodied in the Constitution is not meant to be invoked against acts of private individuals finds support in
the deliberations of the Constitutional Commission. True, the liberties guaranteed by the fundamental law of the land must always
be subject to protection. But protection against whom? Commissioner Bernas in his sponsorship speech in the Bill of Rights
answers the query which he himself posed, as follows:

First, the general reflections. The protection of fundamental liberties in the essence of constitutional democracy. Protection against
whom? Protection against the state. The Bill of Rights governs the relationship between the individual and the state. Its concern is
not the relation between individuals, between a private individual and other individuals. What the Bill of Rights does is to declare
some forbidden zones in the private sphere inaccessible to any power holder. (Sponsorship Speech of Commissioner Bernas ,
Record of the Constitutional Commission, Vol. 1, p. 674; July 17, 1986; Emphasis supplied)

The constitutional proscription against unlawful searches and seizures therefore applies as a restraint directed only against the
government and its agencies tasked with the enforcement of the law. Thus, it could only be invoked against the State to whom the
restraint against arbitrary and unreasonable exercise of power is imposed.

If the search is made upon the request of law enforcers, a warrant must generally be first secured if it is to pass the test of
constitutionality. However, if the search is made at the behest or initiative of the proprietor of a private establishment for its own
and private purposes, as in the case at bar, and without the intervention of police authorities, the right against unreasonable search
and seizure cannot be invoked for only the act of private individual, not the law enforcers, is involved. In sum, the protection
against unreasonable searches and seizures cannot be extended to acts committed by private individuals so as to bring it within the
ambit of alleged unlawful intrusion by the government.

Appellant argues, however, that since the provisions of the 1935 Constitution has been modified by the present phraseology found
in the 1987 Charter, expressly declaring as inadmissible any evidence obtained in violation of the constitutional prohibition
against illegal search and seizure, it matters not whether the evidence was procured by police authorities or private individuals
(Appellant's Brief, p. 8, Rollo, p. 62).

The argument is untenable. For one thing, the constitution, in laying down the principles of the government and fundamental
liberties of the people, does not govern relationships between individuals. Moreover, it must be emphasized that the modifications
introduced in the 1987 Constitution (re: Sec. 2, Art. III) relate to the issuance of either a search warrant or warrant of
arrest vis-a-vis the responsibility of the judge in the issuance thereof (SeeSoliven v. Makasiar, 167 SCRA 393 [1988]; Circular No.
13 [October 1, 1985] and Circular No. 12 [June 30, 1987]. The modifications introduced deviate in no manner as to whom the
restriction or inhibition against unreasonable search and seizure is directed against. The restraint stayed with the State and did not
shift to anyone else.

Corolarilly, alleged violations against unreasonable search and seizure may only be invoked against the State by an individual
unjustly traduced by the exercise of sovereign authority. To agree with appellant that an act of a private individual in violation of
the Bill of Rights should also be construed as an act of the State would result in serious legal complications and an absurd
interpretation of the constitution.

Similarly, the admissibility of the evidence procured by an individual effected through private seizure equally applies, in pari
passu, to the alleged violation, non-governmental as it is, of appellant's constitutional rights to privacy and communication.

2. In his second assignment of error, appellant contends that the lower court erred in convicting him despite the undisputed fact
that his rights under the constitution while under custodial investigation were not observed.

Again, the contention is without merit, We have carefully examined the records of the case and found nothing to indicate, as an
"undisputed fact", that appellant was not informed of his constitutional rights or that he gave statements without the assistance of
counsel. The law enforcers testified that accused/appellant was informed of his constitutional rights. It is presumed that they have
regularly performed their duties (See. 5(m), Rule 131) and their testimonies should be given full faith and credence, there being no
evidence to the contrary. What is clear from the records, on the other hand, is that appellant refused to give any written statement
while under investigation as testified by Atty. Lastimoso of the NBI, Thus:

Fiscal Formoso:

You said that you investigated Mr. and Mrs. Job Reyes. What about the accused here, did you investigate the accused together
with the girl?

WITNESS:

Yes, we have interviewed the accused together with the girl but the accused availed of his constitutional right not to give any
written statement, sir. (TSN, October 8, 1987, p. 62; Original Records, p. 240)

The above testimony of the witness for the prosecution was not contradicted by the defense on cross-examination. As borne out by
the records, neither was there any proof by the defense that appellant gave uncounselled confession while being
investigated. What is more, we have examined the assailed judgment of the trial court and nowhere is there any reference made to
the testimony of appellant while under custodial investigation which was utilized in the finding of conviction. Appellant's second
assignment of error is therefore misplaced.

3. Coming now to appellant's third assignment of error, appellant would like us to believe that he was not the owner of the
packages which contained prohibited drugs but rather a certain Michael, a German national, whom appellant met in a pub along
Ermita, Manila: that in the course of their 30-minute conversation, Michael requested him to ship the packages and gave him
P2,000.00 for the cost of the shipment since the German national was about to leave the country the next day (October 15, 1987,
TSN, pp. 2-10).

Rather than give the appearance of veracity, we find appellant's disclaimer as incredulous, self-serving and contrary to human
experience. It can easily be fabricated. An acquaintance with a complete stranger struck in half an hour could not have pushed a
man to entrust the shipment of four (4) parcels and shell out P2,000.00 for the purpose and for appellant to readily accede to
comply with the undertaking without first ascertaining its contents. As stated by the trial court, "(a) person would not simply
entrust contraband and of considerable value at that as the marijuana flowering tops, and the cash amount of P2,000.00 to a
complete stranger like the Accused. The Accused, on the other hand, would not simply accept such undertaking to take custody of
the packages and ship the same from a complete stranger on his mere say-so" (Decision, p. 19, Rollo, p. 91). As to why he readily
agreed to do the errand, appellant failed to explain. Denials, if unsubstantiated by clear and convincing evidence, are negative
self-serving evidence which deserve no weight in law and cannot be given greater evidentiary weight than the testimony of
credible witnesses who testify on affirmative matters (People v. Esquillo, 171 SCRA 571 [1989]; People vs. Sariol, 174 SCRA
237 [1989]).

Appellant's bare denial is even made more suspect considering that, as per records of the Interpol, he was previously convicted of
possession of hashish by the Kleve Court in the Federal Republic of Germany on January 1, 1982 and that the consignee of the
frustrated shipment, Walter Fierz, also a Swiss national, was likewise convicted for drug abuse and is just about an hour's drive
from appellant's residence in Zurich, Switzerland (TSN, October 8, 1987, p. 66; Original Records, p. 244; Decision, p. 21; Rollo, p.
93).

Evidence to be believed, must not only proceed from the mouth of a credible witness, but it must be credible in itself such as the
common experience and observation of mankind can approve as probable under the circumstances (People v. Alto, 26 SCRA 342
[1968], citing Daggers v. Van Dyke, 37 N.J. Eg. 130; see also People v. Sarda, 172 SCRA 651 [1989]; People v. Sunga, 123
SCRA 327 [1983]); Castañares v. CA, 9
2 SCRA 567 [1979]). As records further show, appellant did not even bother to ask Michael's full name, his complete address or
passport number. Furthermore, if indeed, the German national was the owner of the merchandise, appellant should have so
indicated in the contract of shipment (Exh. "B", Original Records, p. 40). On the contrary, appellant signed the contract as the
owner and shipper thereof giving more weight to the presumption that things which a person possesses, or exercises acts of
ownership over, are owned by him (Sec. 5 [j], Rule 131). At this point, appellant is therefore estopped to claim otherwise.

Premises considered, we see no error committed by the trial court in rendering the assailed judgment.

WHEREFORE, the judgment of conviction finding appellant guilty beyond reasonable doubt of the crime charged is hereby
AFFIRMED. No costs.

SO ORDERED.

G.R. No. L-19550 June 19, 1967

HARRY S. STONEHILL, ROBERT P. BROOKS, JOHN J. BROOKS and KARL BECK, petitioners,
vs.
HON. JOSE W. DIOKNO, in his capacity as SECRETARY OF JUSTICE; JOSE LUKBAN, in his capacity as Acting
Director, National Bureau of Investigation; SPECIAL PROSECUTORS PEDRO D. CENZON, EFREN I. PLANA and
MANUEL VILLAREAL, JR. and ASST. FISCAL MANASES G. REYES; JUDGE AMADO ROAN, Municipal Court of
Manila; JUDGE ROMAN CANSINO, Municipal Court of Manila; JUDGE HERMOGENES CALUAG, Court of First
Instance of Rizal-Quezon City Branch, and JUDGE DAMIAN JIMENEZ, Municipal Court of Quezon City, respondents.

Paredes, Poblador, Cruz and Nazareno and Meer, Meer and Meer and Juan T. David for petitioners.
Office of the Solicitor General Arturo A. Alafriz, Assistant Solicitor General Pacifico P. de Castro, Assistant Solicitor General
Frine C. Zaballero, Solicitor Camilo D. Quiason and Solicitor C. Padua for respondents.

CONCEPCION, C.J.:

Upon application of the officers of the government named on the margin 1 — hereinafter referred to as Respondents-Prosecutors
— several judges2 — hereinafter referred to as Respondents-Judges — issued, on different dates,3 a total of 42 search warrants
against petitioners herein4 and/or the corporations of which they were officers, 5 directed to the any peace officer, to search the
persons above-named and/or the premises of their offices, warehouses and/or residences, and to seize and take possession of the
following personal property to wit:

Books of accounts, financial records, vouchers, correspondence, receipts, ledgers, journals, portfolios, credit journals, typewriters,
and other documents and/or papers showing all business transactions including disbursements receipts, balance sheets and profit
and loss statements and Bobbins (cigarette wrappers).

as "the subject of the offense; stolen or embezzled and proceeds or fruits of the offense," or "used or intended to be used as the
means of committing the offense," which is described in the applications adverted to above as "violation of Central Bank Laws,
Tariff and Customs Laws, Internal Revenue (Code) and the Revised Penal Code."

Alleging that the aforementioned search warrants are null and void, as contravening the Constitution and the Rules of Court —
because, inter alia: (1) they do not describe with particularity the documents, books and things to be seized; (2) cash money, not
mentioned in the warrants, were actually seized; (3) the warrants were issued to fish evidence against the aforementioned
petitioners in deportation cases filed against them; (4) the searches and seizures were made in an illegal manner; and (5) the
documents, papers and cash money seized were not delivered to the courts that issued the warrants, to be disposed of in
accordance with law — on March 20, 1962, said petitioners filed with the Supreme Court this original action for certiorari,
prohibition, mandamus and injunction, and prayed that, pending final disposition of the present case, a writ of preliminary
injunction be issued restraining Respondents-Prosecutors, their agents and /or representatives from using the effects seized as
aforementioned or any copies thereof, in the deportation cases already adverted to, and that, in due course, thereafter, decision be
rendered quashing the contested search warrants and declaring the same null and void, and commanding the respondents, their
agents or representatives to return to petitioners herein, in accordance with Section 3, Rule 67, of the Rules of Court, the
documents, papers, things and cash moneys seized or confiscated under the search warrants in question.

In their answer, respondents-prosecutors alleged, 6 (1) that the contested search warrants are valid and have been issued in
accordance with law; (2) that the defects of said warrants, if any, were cured by petitioners' consent; and (3) that, in any event, the
effects seized are admissible in evidence against herein petitioners, regardless of the alleged illegality of the aforementioned
searches and seizures.

On March 22, 1962, this Court issued the writ of preliminary injunction prayed for in the petition. However, by resolution dated
June 29, 1962, the writ was partially lifted or dissolved, insofar as the papers, documents and things seized from the offices of the
corporations above mentioned are concerned; but, the injunction was maintained as regards the papers, documents and things
found and seized in the residences of petitioners herein. 7

Thus, the documents, papers, and things seized under the alleged authority of the warrants in question may be split into two (2)
major groups, namely: (a) those found and seized in the offices of the aforementioned corporations, and (b) those found and seized
in the residences of petitioners herein.

As regards the first group, we hold that petitioners herein have no cause of action to assail the legality of the contested warrants
and of the seizures made in pursuance thereof, for the simple reason that said corporations have their respective personalities,
separate and distinct from the personality of herein petitioners, regardless of the amount of shares of stock or of the interest of
each of them in said corporations, and whatever the offices they hold therein may be. 8 Indeed, it is well settled that the legality of
a seizure can be contested only by the party whose rights have been impaired thereby, 9 and that the objection to an unlawful
search and seizure is purely personal and cannot be availed of by third parties. 10 Consequently, petitioners herein may not validly
object to the use in evidence against them of the documents, papers and things seized from the offices and premises of the
corporations adverted to above, since the right to object to the admission of said papers in evidence belongs exclusively to the
corporations, to whom the seized effects belong, and may not be invoked by the corporate officers in proceedings against them in
their individual capacity. 11 Indeed, it has been held:

. . . that the Government's action in gaining possession of papers belonging to the corporation did not relate to nor did it affect
the personal defendants. If these papers were unlawfully seized and thereby the constitutional rights of or any one were invaded,
they were the rights of the corporation and not the rights of the other defendants. Next, it is clear that a question of the lawfulness
of a seizure can be raised only by one whose rights have been invaded. Certainly, such a seizure, if unlawful, could not affect the
constitutional rights of defendants whose property had not been seized or the privacy of whose homes had not been disturbed; nor
could they claim for themselves the benefits of the Fourth Amendment, when its violation, if any, was with reference to the rights
of another. Remus vs. United States (C.C.A.)291 F. 501, 511. It follows, therefore, that the question of the admissibility of the
evidence based on an alleged unlawful search and seizure does not extend to the personal defendants but
embraces only the corporation whose property was taken. . . . (A Guckenheimer & Bros. Co. vs. United States, [1925] 3 F. 2d.
786, 789, Emphasis supplied.)

With respect to the documents, papers and things seized in the residences of petitioners herein, the aforementioned resolution of
June 29, 1962, lifted the writ of preliminary injunction previously issued by this Court, 12 thereby, in effect, restraining herein
Respondents-Prosecutors from using them in evidence against petitioners herein.

In connection with said documents, papers and things, two (2) important questions need be settled, namely: (1) whether the search
warrants in question, and the searches and seizures made under the authority thereof, are valid or not, and (2) if the answer to the
preceding question is in the negative, whether said documents, papers and things may be used in evidence against petitioners
herein.1äwphï1.ñët

Petitioners maintain that the aforementioned search warrants are in the nature of general warrants and that accordingly, the
seizures effected upon the authority there of are null and void. In this connection, the Constitution 13provides:

The right of the people to be secure in their persons, houses, papers, and effects against unreasonable searches and seizures shall
not be violated, and no warrants shall issue but upon probable cause, to be determined by the judge after examination under oath
or affirmation of the complainant and the witnesses he may produce, and particularly describing the place to be searched, and the
persons or things to be seized.

Two points must be stressed in connection with this constitutional mandate, namely: (1) that no warrant shall issue but upon
probable cause, to be determined by the judge in the manner set forth in said provision; and (2) that the warrant
shall particularly describe the things to be seized.
None of these requirements has been complied with in the contested warrants. Indeed, the same were issued upon applications
stating that the natural and juridical person therein named had committed a "violation of Central Ban Laws, Tariff and Customs
Laws, Internal Revenue (Code) and Revised Penal Code." In other words, no specific offense had been alleged in said applications.
The averments thereof with respect to the offense committed were abstract. As a consequence, it was impossible for the judges
who issued the warrants to have found the existence of probable cause, for the same presupposes the introduction of competent
proof that the party against whom it is sought has performed particular acts, or committed specific omissions, violating a given
provision of our criminal laws. As a matter of fact, the applications involved in this case do not allege any specific acts performed
by herein petitioners. It would be the legal heresy, of the highest order, to convict anybody of a "violation of Central Bank Laws,
Tariff and Customs Laws, Internal Revenue (Code) and Revised Penal Code," — as alleged in the aforementioned applications —
without reference to any determinate provision of said laws or

To uphold the validity of the warrants in question would be to wipe out completely one of the most fundamental rights guaranteed
in our Constitution, for it would place the sanctity of the domicile and the privacy of communication and correspondence at the
mercy of the whims caprice or passion of peace officers. This is precisely the evil sought to be remedied by the constitutional
provision above quoted — to outlaw the so-called general warrants. It is not difficult to imagine what would happen, in times of
keen political strife, when the party in power feels that the minority is likely to wrest it, even though by legal means.

Such is the seriousness of the irregularities committed in connection with the disputed search warrants, that this Court deemed it
fit to amend Section 3 of Rule 122 of the former Rules of Court 14 by providing in its counterpart, under the Revised Rules of
Court 15 that "a search warrant shall not issue but upon probable cause in connection with one specific offense." Not satisfied with
this qualification, the Court added thereto a paragraph, directing that "no search warrant shall issue for more than one specific
offense."

The grave violation of the Constitution made in the application for the contested search warrants was compounded by the
description therein made of the effects to be searched for and seized, to wit:

Books of accounts, financial records, vouchers, journals, correspondence, receipts, ledgers, portfolios, credit journals, typewriters,
and other documents and/or papers showing all business transactions including disbursement receipts, balance sheets and related
profit and loss statements.

Thus, the warrants authorized the search for and seizure of records pertaining to all business transactions of petitioners herein,
regardless of whether the transactions were legal or illegal. The warrants sanctioned the seizure of all records of the petitioners
and the aforementioned corporations, whatever their nature, thus openly contravening the explicit command of our Bill of Rights
— that the things to be seized be particularly described — as well as tending to defeat its major objective: the elimination
of general warrants.

Relying upon Moncado vs. People's Court (80 Phil. 1), Respondents-Prosecutors maintain that, even if the searches and seizures
under consideration were unconstitutional, the documents, papers and things thus seized are admissible in evidence against
petitioners herein. Upon mature deliberation, however, we are unanimously of the opinion that the position taken in the Moncado
case must be abandoned. Said position was in line with the American common law rule, that the criminal should not be allowed to
go free merely "because the constable has blundered," 16 upon the theory that the constitutional prohibition against unreasonable
searches and seizures is protected by means other than the exclusion of evidence unlawfully obtained, 17 such as the common-law
action for damages against the searching officer, against the party who procured the issuance of the search warrant and against
those assisting in the execution of an illegal search, their criminal punishment, resistance, without liability to an unlawful seizure,
and such other legal remedies as may be provided by other laws.

However, most common law jurisdictions have already given up this approach and eventually adopted the exclusionary rule,
realizing that this is the only practical means of enforcing the constitutional injunction against unreasonable searches and seizures.
In the language of Judge Learned Hand:

As we understand it, the reason for the exclusion of evidence competent as such, which has been unlawfully acquired, is that
exclusion is the only practical way of enforcing the constitutional privilege. In earlier times the action of trespass against the
offending official may have been protection enough; but that is true no longer. Only in case the prosecution which itself controls
the seizing officials, knows that it cannot profit by their wrong will that wrong be repressed.18

In fact, over thirty (30) years before, the Federal Supreme Court had already declared:

If letters and private documents can thus be seized and held and used in evidence against a citizen accused of an offense, the
protection of the 4th Amendment, declaring his rights to be secure against such searches and seizures, is of no value, and, so far as
those thus placed are concerned, might as well be stricken from the Constitution. The efforts of the courts and their officials to
bring the guilty to punishment, praiseworthy as they are, are not to be aided by the sacrifice of those great principles established
by years of endeavor and suffering which have resulted in their embodiment in the fundamental law of the land.19

This view was, not only reiterated, but, also, broadened in subsequent decisions on the same Federal Court. 20After reviewing
previous decisions thereon, said Court held, in Mapp vs. Ohio (supra.):

. . . Today we once again examine the Wolf's constitutional documentation of the right of privacy free from unreasonable state
intrusion, and after its dozen years on our books, are led by it to close the only courtroom door remaining open to evidence
secured by official lawlessness in flagrant abuse of that basic right, reserved to all persons as a specific guarantee against that very
same unlawful conduct. We hold that all evidence obtained by searches and seizures in violation of the Constitution is, by that
same authority, inadmissible in a State.

Since the Fourth Amendment's right of privacy has been declared enforceable against the States through the Due Process Clause
of the Fourteenth, it is enforceable against them by the same sanction of exclusion as it used against the Federal Government.
Were it otherwise, then just as without the Weeks rule the assurance against unreasonable federal searches and seizures would be
"a form of words," valueless and underserving of mention in a perpetual charter of inestimable human liberties, so too, without
that rule the freedom from state invasions of privacy would be so ephemeral and so neatly severed from its conceptual nexus with
the freedom from all brutish means of coercing evidence as not to permit this Court's high regard as a freedom "implicit in the
concept of ordered liberty." At the time that the Court held in Wolf that the amendment was applicable to the States through the
Due Process Clause, the cases of this Court as we have seen, had steadfastly held that as to federal officers the Fourth Amendment
included the exclusion of the evidence seized in violation of its provisions. Even Wolf "stoutly adhered" to that proposition. The
right to when conceded operatively enforceable against the States, was not susceptible of destruction by avulsion of the sanction
upon which its protection and enjoyment had always been deemed dependent under the Boyd, Weeks and Silverthorne Cases.
Therefore, in extending the substantive protections of due process to all constitutionally unreasonable searches — state or federal
— it was logically and constitutionally necessarily that the exclusion doctrine — an essential part of the right to privacy — be also
insisted upon as an essential ingredient of the right newly recognized by the Wolf Case. In short, the admission of the new
constitutional Right by Wolf could not tolerate denial of its most important constitutional privilege, namely, the exclusion of the
evidence which an accused had been forced to give by reason of the unlawful seizure. To hold otherwise is to grant the right but in
reality to withhold its privilege and enjoyment. Only last year the Court itself recognized that the purpose of the exclusionary rule
to "is to deter — to compel respect for the constitutional guaranty in the only effectively available way — by removing the
incentive to disregard it" . . . .

The ignoble shortcut to conviction left open to the State tends to destroy the entire system of constitutional restraints on which the
liberties of the people rest. Having once recognized that the right to privacy embodied in the Fourth Amendment is enforceable
against the States, and that the right to be secure against rude invasions of privacy by state officers is, therefore constitutional in
origin, we can no longer permit that right to remain an empty promise. Because it is enforceable in the same manner and to like
effect as other basic rights secured by its Due Process Clause, we can no longer permit it to be revocable at the whim of any police
officer who, in the name of law enforcement itself, chooses to suspend its enjoyment. Our decision, founded on reason and truth,
gives to the individual no more than that which the Constitution guarantees him to the police officer no less than that to which
honest law enforcement is entitled, and, to the courts, that judicial integrity so necessary in the true administration of justice.
(emphasis ours.)

Indeed, the non-exclusionary rule is contrary, not only to the letter, but also, to the spirit of the constitutional injunction against
unreasonable searches and seizures. To be sure, if the applicant for a search warrant has competent evidence to establish probable
cause of the commission of a given crime by the party against whom the warrant is intended, then there is no reason why the
applicant should not comply with the requirements of the fundamental law. Upon the other hand, if he has no such competent
evidence, then it is not possible for the Judge to find that there is probable cause, and, hence, no justification for the issuance of
the warrant. The only possible explanation (not justification) for its issuance is the necessity of fishing evidence of the commission
of a crime. But, then, this fishing expedition is indicative of the absence of evidence to establish a probable cause.

Moreover, the theory that the criminal prosecution of those who secure an illegal search warrant and/or make unreasonable
searches or seizures would suffice to protect the constitutional guarantee under consideration, overlooks the fact that violations
thereof are, in general, committed By agents of the party in power, for, certainly, those belonging to the minority could not
possibly abuse a power they do not have. Regardless of the handicap under which the minority usually — but, understandably —
finds itself in prosecuting agents of the majority, one must not lose sight of the fact that the psychological and moral effect of the
possibility 21 of securing their conviction, is watered down by the pardoning power of the party for whose benefit the illegality had
been committed.

In their Motion for Reconsideration and Amendment of the Resolution of this Court dated June 29, 1962, petitioners allege that
Rooms Nos. 81 and 91 of Carmen Apartments, House No. 2008, Dewey Boulevard, House No. 1436, Colorado Street, and Room
No. 304 of the Army-Navy Club, should be included among the premises considered in said Resolution as residences of herein
petitioners, Harry S. Stonehill, Robert P. Brook, John J. Brooks and Karl Beck, respectively, and that, furthermore, the records,
papers and other effects seized in the offices of the corporations above referred to include personal belongings of said petitioners
and other effects under their exclusive possession and control, for the exclusion of which they have a standing under the latest
rulings of the federal courts of federal courts of the United States. 22

We note, however, that petitioners' theory, regarding their alleged possession of and control over the aforementioned records,
papers and effects, and the alleged "personal" nature thereof, has Been Advanced, not in their petition or amended petition herein,
but in the Motion for Reconsideration and Amendment of the Resolution of June 29, 1962. In other words, said theory would
appear to be readjustment of that followed in said petitions, to suit the approach intimated in the Resolution sought to be
reconsidered and amended. Then, too, some of the affidavits or copies of alleged affidavits attached to said motion for
reconsideration, or submitted in support thereof, contain either inconsistent allegations, or allegations inconsistent with the theory
now advanced by petitioners herein.

Upon the other hand, we are not satisfied that the allegations of said petitions said motion for reconsideration, and the contents of
the aforementioned affidavits and other papers submitted in support of said motion, have sufficiently established the facts or
conditions contemplated in the cases relied upon by the petitioners; to warrant application of the views therein expressed, should
we agree thereto. At any rate, we do not deem it necessary to express our opinion thereon, it being best to leave the matter open
for determination in appropriate cases in the future.

We hold, therefore, that the doctrine adopted in the Moncado case must be, as it is hereby, abandoned; that the warrants for the
search of three (3) residences of herein petitioners, as specified in the Resolution of June 29, 1962, are null and void; that the
searches and seizures therein made are illegal; that the writ of preliminary injunction heretofore issued, in connection with the
documents, papers and other effects thus seized in said residences of herein petitioners is hereby made permanent; that the writs
prayed for are granted, insofar as the documents, papers and other effects so seized in the aforementioned residences are
concerned; that the aforementioned motion for Reconsideration and Amendment should be, as it is hereby, denied; and that the
petition herein is dismissed and the writs prayed for denied, as regards the documents, papers and other effects seized in the
twenty-nine (29) places, offices and other premises enumerated in the same Resolution, without special pronouncement as to
costs.

It is so ordered.

Reyes, J.B.L., Dizon, Makalintal, Bengzon, J.P., Zaldivar and Sanchez, JJ., concur.

CASTRO, J., concurring and dissenting:

From my analysis of the opinion written by Chief Justice Roberto Concepcion and from the import of the deliberations of the
Court on this case, I gather the following distinct conclusions:

1. All the search warrants served by the National Bureau of Investigation in this case are general warrants and are therefore
proscribed by, and in violation of, paragraph 3 of section 1 of Article III (Bill of Rights) of the Constitution;

2. All the searches and seizures conducted under the authority of the said search warrants were consequently illegal;

3. The non-exclusionary rule enunciated in Moncado vs. People, 80 Phil. 1, should be, and is declared, abandoned;

4. The search warrants served at the three residences of the petitioners are expressly declared null and void the searches and
seizures therein made are expressly declared illegal; and the writ of preliminary injunction heretofore issued against the use of the
documents, papers and effect seized in the said residences is made permanent; and

5. Reasoning that the petitioners have not in their pleadings satisfactorily demonstrated that they have legal standing to move for
the suppression of the documents, papers and effects seized in the places other than the three residences adverted to above, the
opinion written by the Chief Justice refrains from expressly declaring as null and void the such warrants served at such other
places and as illegal the searches and seizures made therein, and leaves "the matter open for determination in appropriate cases in
the future."

It is precisely the position taken by the Chief Justice summarized in the immediately preceding paragraph (numbered 5) with
which I am not in accord.

I do not share his reluctance or unwillingness to expressly declare, at this time, the nullity of the search warrants served at places
other than the three residences, and the illegibility of the searches and seizures conducted under the authority thereof. In my view
even the exacerbating passions and prejudices inordinately generated by the environmental political and moral developments of
this case should not deter this Court from forthrightly laying down the law not only for this case but as well for future cases and
future generations. All the search warrants, without exception, in this case are admittedly general, blanket and roving warrants and
are therefore admittedly and indisputably outlawed by the Constitution; and the searches and seizures made were therefore
unlawful. That the petitioners, let us assume in gratia argumente, have no legal standing to ask for the suppression of the papers,
things and effects seized from places other than their residences, to my mind, cannot in any manner affect, alter or otherwise
modify the intrinsic nullity of the search warrants and the intrinsic illegality of the searches and seizures made thereunder.
Whether or not the petitioners possess legal standing the said warrants are void and remain void, and the searches and seizures
were illegal and remain illegal. No inference can be drawn from the words of the Constitution that "legal standing" or the lack of it
is a determinant of the nullity or validity of a search warrant or of the lawfulness or illegality of a search or seizure.

On the question of legal standing, I am of the conviction that, upon the pleadings submitted to this Court the petitioners have the
requisite legal standing to move for the suppression and return of the documents, papers and effects that were seized from places
other than their family residences.

Our constitutional provision on searches and seizures was derived almost verbatim from the Fourth Amendment to the United
States Constitution. In the many years of judicial construction and interpretation of the said constitutional provision, our courts
have invariably regarded as doctrinal the pronouncement made on the Fourth Amendment by federal courts, especially the Federal
Supreme Court and the Federal Circuit Courts of Appeals.

The U.S. doctrines and pertinent cases on standing to move for the suppression or return of documents, papers and effects which
are the fruits of an unlawful search and seizure, may be summarized as follows; (a) ownership of documents, papers and effects
gives "standing;" (b) ownership and/or control or possession — actual or constructive — of premises searched gives "standing";
and (c) the "aggrieved person" doctrine where the search warrant and the sworn application for search warrant are "primarily"
directed solely and exclusively against the "aggrieved person," gives "standing."

An examination of the search warrants in this case will readily show that, excepting three, all were directed against the petitioners
personally. In some of them, the petitioners were named personally, followed by the designation, "the President and/or General
Manager" of the particular corporation. The three warrants excepted named three corporate defendants. But the
"office/house/warehouse/premises" mentioned in the said three warrants were also the same "office/house/warehouse/premises"
declared to be owned by or under the control of the petitioners in all the other search warrants directed against the petitioners
and/or "the President and/or General Manager" of the particular corporation. (see pages 5-24 of Petitioners' Reply of April 2,
1962). The searches and seizures were to be made, and were actually made, in the "office/house/warehouse/premises" owned by
or under the control of the petitioners.

Ownership of matters seized gives "standing."

Ownership of the properties seized alone entitles the petitioners to bring a motion to return and suppress, and gives them standing
as persons aggrieved by an unlawful search and seizure regardless of their location at the time of seizure. Jones vs. United States,
362 U.S. 257, 261 (1960) (narcotics stored in the apartment of a friend of the defendant); Henzel vs. United States, 296 F. 2d. 650,
652-53 (5th Cir. 1961), (personal and corporate papers of corporation of which the defendant was president), United States vs.
Jeffers, 342 U.S. 48 (1951) (narcotics seized in an apartment not belonging to the defendant); Pielow vs. United States, 8 F. 2d
492, 493 (9th Cir. 1925) (books seized from the defendant's sister but belonging to the defendant); Cf. Villano vs. United States,
310 F. 2d 680, 683 (10th Cir. 1962) (papers seized in desk neither owned by nor in exclusive possession of the defendant).

In a very recent case (decided by the U.S. Supreme Court on December 12, 1966), it was held that under the constitutional
provision against unlawful searches and seizures, a person places himself or his property within a constitutionally protected area,
be it his home or his office, his hotel room or his automobile:

Where the argument falls is in its misapprehension of the fundamental nature and scope of Fourth Amendment protection. What
the Fourth Amendment protects is the security a man relies upon when he places himself or his property within a constitutionally
protected area, be it his home or his office, his hotel room or his automobile. There he is protected from unwarranted
governmental intrusion. And when he puts some thing in his filing cabinet, in his desk drawer, or in his pocket, he has the right to
know it will be secure from an unreasonable search or an unreasonable seizure. So it was that the Fourth Amendment could not
tolerate the warrantless search of the hotel room in Jeffers, the purloining of the petitioner's private papers in Gouled, or the
surreptitious electronic surveilance in Silverman. Countless other cases which have come to this Court over the years have
involved a myriad of differing factual contexts in which the protections of the Fourth Amendment have been appropriately
invoked. No doubt, the future will bring countless others. By nothing we say here do we either foresee or foreclose factual
situations to which the Fourth Amendment may be applicable. (Hoffa vs. U.S., 87 S. Ct. 408 (December 12, 1966). See also U.S.
vs. Jeffers, 342 U.S. 48, 72 S. Ct. 93 (November 13, 1951). (Emphasis supplied).
Control of premises searched gives "standing."

Independent of ownership or other personal interest in the records and documents seized, the petitioners have standing to move for
return and suppression by virtue of their proprietary or leasehold interest in many of the premises searched. These proprietary and
leasehold interests have been sufficiently set forth in their motion for reconsideration and need not be recounted here, except to
emphasize that the petitioners paid rent, directly or indirectly, for practically all the premises searched (Room 91, 84 Carmen Apts;
Room 304, Army & Navy Club; Premises 2008, Dewey Boulevard; 1436 Colorado Street); maintained personal offices within the
corporate offices (IBMC, USTC); had made improvements or furnished such offices; or had paid for the filing cabinets in which
the papers were stored (Room 204, Army & Navy Club); and individually, or through their respective spouses, owned the
controlling stock of the corporations involved. The petitioners' proprietary interest in most, if not all, of the premises searched
therefore independently gives them standing to move for the return and suppression of the books, papers and affects seized
therefrom.

In Jones vs. United States, supra, the U.S. Supreme Court delineated the nature and extent of the interest in the searched premises
necessary to maintain a motion to suppress. After reviewing what it considered to be the unduly technical standard of the then
prevailing circuit court decisions, the Supreme Court said (362 U.S. 266):

We do not lightly depart from this course of decisions by the lower courts. We are persuaded, however, that it is unnecessarily and
ill-advised to import into the law surrounding the constitutional right to be free from unreasonable searches and seizures subtle
distinctions, developed and refined by the common law in evolving the body of private property law which, more than almost any
other branch of law, has been shaped by distinctions whose validity is largely historical. Even in the area from which they derive,
due consideration has led to the discarding of those distinctions in the homeland of the common law. See Occupiers' Liability Act,
1957, 5 and 6 Eliz. 2, c. 31, carrying out Law Reform Committee, Third Report, Cmd. 9305. Distinctions such as those between
"lessee", "licensee," "invitee," "guest," often only of gossamer strength, ought not be determinative in fashioning procedures
ultimately referable to constitutional safeguards. See also Chapman vs. United States, 354 U.S. 610, 616-17 (1961).

It has never been held that a person with requisite interest in the premises searched must own the property seized in order to have
standing in a motion to return and suppress. In Alioto vs. United States, 216 F. Supp. 48 (1963), a Bookkeeper for several
corporations from whose apartment the corporate records were seized successfully moved for their return. In United States vs.
Antonelli, Fireworks Co., 53 F. Supp. 870, 873 (W D. N. Y. 1943), the corporation's president successfully moved for the return
and suppression is to him of both personal and corporate documents seized from his home during the course of an illegal search:

The lawful possession by Antonelli of documents and property, "either his own or the corporation's was entitled to protection
against unreasonable search and seizure. Under the circumstances in the case at bar, the search and seizure were unreasonable and
unlawful. The motion for the return of seized article and the suppression of the evidence so obtained should be granted. (Emphasis
supplied).

Time was when only a person who had property in interest in either the place searched or the articles seize had the necessary
standing to invoke the protection of the exclusionary rule. But in MacDonald vs. Unite States, 335 U.S. 461 (1948), Justice Robert
Jackson joined by Justice Felix Frankfurter, advanced the view that "even a guest may expect the shelter of the rooftree he is
under against criminal intrusion." This view finally became the official view of the U.S. Supreme Court and was articulated
in United States vs. Jeffers, 432 U.S 48 (1951). Nine years later, in 1960, in Jones vs. Unite States, 362 U.S. 257, 267, the U.S.
Supreme Court went a step further. Jones was a mere guest in the apartment unlawfully searched but the Court nonetheless
declared that the exclusionary rule protected him as well. The concept of "person aggrieved by an unlawful search and seizure"
was enlarged to include "anyone legitimately on premise where the search occurs."

Shortly after the U.S. Supreme Court's Jones decision the U.S. Court of Appeals for the Fifth Circuit held that the defendant
organizer, sole stockholder and president of a corporation had standing in a mail fraud prosecution against him to demand the
return and suppression of corporate property. Henzel vs. United States, 296 F 2d 650, 652 (5th Cir. 1961), supra. The court
conclude that the defendant had standing on two independent grounds: First —he had a sufficient interest in the property seized,
and second — he had an adequate interest in the premises searched (just like in the case at bar). A postal inspector had unlawfully
searched the corporation' premises and had seized most of the corporation's book and records. Looking to Jones, the court
observed:

Jones clearly tells us, therefore, what is not required qualify one as a "person aggrieved by an unlawful search and seizure." It tells
us that appellant should not have been precluded from objecting to the Postal Inspector's search and seizure of the corporation's
books and records merely because the appellant did not show ownership or possession of the books and records or a substantial
possessory interest in the invade premises . . . (Henzel vs. United States, 296 F. 2d at 651). .
Henzel was soon followed by Villano vs. United States, 310 F. 2d 680, 683, (10th Cir. 1962). In Villano, police officers seized two
notebooks from a desk in the defendant's place of employment; the defendant did not claim ownership of either; he asserted that
several employees (including himself) used the notebooks. The Court held that the employee had a protected interest and that
there also was an invasion of privacy. Both Henzel and Villano considered also the fact that the search and seizure were "directed
at" the moving defendant. Henzel vs. United States, 296 F. 2d at 682; Villano vs. United States, 310 F. 2d at 683.

In a case in which an attorney closed his law office, placed his files in storage and went to Puerto Rico, the Court of Appeals for
the Eighth Circuit recognized his standing to move to quash as unreasonable search and seizure under the Fourth Amendment of
the U.S. Constitution a grand jury subpoena duces tecum directed to the custodian of his files. The Government contended that the
petitioner had no standing because the books and papers were physically in the possession of the custodian, and because the
subpoena was directed against the custodian. The court rejected the contention, holding that

Schwimmer legally had such possession, control and unrelinquished personal rights in the books and papers as not to enable the
question of unreasonable search and seizure to be escaped through the mere procedural device of compelling a third-party naked
possessor to produce and deliver them. Schwimmer vs. United States, 232 F. 2d 855, 861 (8th Cir. 1956).

Aggrieved person doctrine where the search warrant s primarily directed against said person gives "standing."

The latest United States decision squarely in point is United States vs. Birrell, 242 F. Supp. 191 (1965, U.S.D.C. S.D.N.Y.). The
defendant had stored with an attorney certain files and papers, which attorney, by the name of Dunn, was not, at the time of the
seizing of the records, Birrell's attorney. * Dunn, in turn, had stored most of the records at his home in the country and on a farm
which, according to Dunn's affidavit, was under his (Dunn's) "control and management." The papers turned out to be private,
personal and business papers together with corporate books and records of certain unnamed corporations in which Birrell did not
even claim ownership. (All of these type records were seized in the case at bar). Nevertheless, the search in Birrell was held
invalid by the court which held that even though Birrell did not own the premises where the records were stored, he had
"standing" to move for the return ofall the papers and properties seized. The court, relying on Jones vs. U.S., supra; U.S. vs.
Antonelli Fireworks Co., 53 F. Supp. 870, Aff'd 155 F. 2d 631: Henzel vs. U.S., supra; and Schwimmer vs. U.S., supra, pointed
out that

It is overwhelmingly established that the searches here in question were directed solely and exclusively against Birrell. The only
person suggested in the papers as having violated the law was Birrell. The first search warrant described the records as having
been used "in committing a violation of Title 18, United States Code, Section 1341, by the use of the mails by one Lowell M.
Birrell, . . ." The second search warrant was captioned: "United States of America vs. Lowell M. Birrell. (p. 198)

Possession (actual or constructive), no less than ownership, gives standing to move to suppress. Such was the rule even before
Jones. (p. 199)

If, as thus indicated Birrell had at least constructive possession of the records stored with Dunn, it matters not whether he had any
interest in the premises searched. See also Jeffers v. United States, 88 U.S. Appl. D.C. 58, 187 F. 2d 498 (1950), affirmed 432 U.S.
48, 72 S. Ct. 93, 96 L. Ed. 459 (1951).

The ruling in the Birrell case was reaffirmed on motion for reargument; the United States did not appeal from this decision. The
factual situation in Birrell is strikingly similar to the case of the present petitioners; as in Birrell, many personal and corporate
papers were seized from premises not petitioners' family residences; as in Birrell, the searches were "PRIMARILY DIRECTED
SOLETY AND EXCLUSIVELY" against the petitioners. Still both types of documents were suppressed in Birrell because of the
illegal search. In the case at bar, the petitioners connection with the premises raided is much closer than in Birrell.

Thus, the petitioners have full standing to move for the quashing of all the warrants regardless whether these were directed against
residences in the narrow sense of the word, as long as the documents were personal papers of the petitioners or (to the extent that
they were corporate papers) were held by them in a personal capacity or under their personal control.

Prescinding a from the foregoing, this Court, at all events, should order the return to the petitioners all personal and private papers
and effects seized, no matter where these were seized, whether from their residences or corporate offices or any other place or
places. The uncontradicted sworn statements of the petitioners in their, various pleadings submitted to this Court indisputably
show that amongst the things seized from the corporate offices and other places were personal and private papers and effects
belonging to the petitioners.

If there should be any categorization of the documents, papers and things which where the objects of the unlawful searches and
seizures, I submit that the grouping should be: (a) personal or private papers of the petitioners were they were unlawfully seized,
be it their family residences offices, warehouses and/or premises owned and/or possessed (actually or constructively) by them as
shown in all the search and in the sworn applications filed in securing the void search warrants and (b) purely corporate papers
belonging to corporations. Under such categorization or grouping, the determination of which unlawfully seized papers,
documents and things are personal/private of the petitioners or purely corporate papers will have to be left to the lower courts
which issued the void search warrants in ultimately effecting the suppression and/or return of the said documents.

And as unequivocally indicated by the authorities above cited, the petitioners likewise have clear legal standing to move for the
suppression of purely corporate papers as "President and/or General Manager" of the corporations involved as specifically
mentioned in the void search warrants.

Finally, I must articulate my persuasion that although the cases cited in my disquisition were criminal prosecutions, the great
clauses of the constitutional proscription on illegal searches and seizures do not withhold the mantle of their protection from cases
not criminal in origin or nature.

[A.M. No. RTJ-93-964. February 28, 1996]

LEOVIGILDO U. MANTARING, complainant, vs. JUDGE MANUEL A. ROMAN, JR., RTC, Branch 42, Pinamalayan,
Oriental Mindoro; and JUDGE IRENEO B. MOLATO, MTC, Bongabon, Oriental Mindoro, respondents.
SYLLABUS
1. REMEDIAL LAW; NEW TRIAL; REMEDY AVAILABLE TO PARTIES DENIED OF A FAIR AND IMPARTIAL
TRIAL. - There is a remedy available to the party seeking the disqualification of the judge. If he is denied a fair and
impartial trial, caused by the judges bias or prejudice, he can ask for a new trial in the interest of justice which will be
granted if that is really the case.
2. ID.; CRIMINAL PROCEDURE; SEARCH WARRANT; WARRANT OF ARREST; BASIS FOR ISSUANCE. - The
issuance of a search warrant and of a warrant of arrest requires the showing of probabilities as to different facts. In the case
of search warrants, the determination is based on the finding that (1) the articles to be seized are connected to a criminal
activity and (2) they are found in the place to be searched. It is not necessary that a particular person be implicated. On the
other hand, in arrest cases, the determination of probable cause is based on a finding that a crime has been committed and
that the person to be arrested has committed it.
3. ID.; ID.; WARRANT OF ARREST; REQUIREMENTS FOR ISSUANCE. - It is now settled that in issuing warrants of
arrest in preliminary investigations, the investigating judge must: (a) have examined in writing and under oath the
complainant and his witnesses by searching questions and answers; (b) be satisfied that probable cause exists; and (c) that
there is a need to place the respondent under immediate custody in order not to frustrate the ends of justice.

DECISION
MENDOZA, J.:
Respondent Judge Ireneo B. Molato is the presiding judge of the Municipal Trial Court of Bongabon, Oriental Mindoro. On
January 7, 1993, an administrative complaint was filed against him and Judge Manuel A. Roman, Jr., presiding judge of the
Regional Trial Court of Pinamalayan, Oriental Mindoro, Branch 42, by Leovigildo U. Mantaring, Sr., who charged them with
conduct unbecoming of members of the judiciary. On February 21, 1994, after the parties had filed their respective pleadings and
supporting documents, this Court dismissed the complaint against the two for lack of merit. The motion for reconsideration filed
by complainant was subsequently denied.
What is before us now is the Supplemental Complaint filed by Leovigildo U. Mantaring, Sr. against Judge Ireneo B. Molato,
which charges him with harassment. It is alleged that because of the filing of the first complaint against him, respondent Judge
Ireneo B. Molato should have inhibited himself from conducting the preliminary investigation of a criminal case considering that
the respondents in that case were complainant and his son. Instead, it is alleged, he took cognizance of the case and ordered the
arrest of complainant and his son, Leovigildo Mantaring, Jr., out of hatred and revenge for them because of the filing of the first
case by the complainant.
The Supplemental Complaint was referred to the Office of the Court Administrator which, in a Memorandum dated 25
November 1994, recommended the dismissal of the case for lack of merit. Nonetheless, the Court required the respondent Judge
Ireneo B. Molato to comment.
In his Comment dated July 6, 1995, respondent judge denies the allegations against him. He avers that on the application by
SPO4 Pacifico L. Fradejas, he issued a search warrant which resulted in the seizure from a certain Joel Gamo of a home-made gun,
a hand grenade, five live ammunitions for Cal. 38 and three live ammunitions for 12 gauge shotgun; that on August 25, 1993, a
complaint for Illegal Possession of Firearms and Ammunition was filed against Joel Gamo in which the herein complainant
Leovigildo, Sr. and his son, Leovigildo, Jr., were included; that finding that the house in which the firearms and ammunition had
been found was owned by complainant and his son, he concluded that there was probable cause to believe that complainant and
his son were guilty of illegal possession of firearms and ammunition and accordingly ordered their arrest. Respondent judge
claims that he inhibited himself from the case after he was ordered by the Executive Judge, RTC, Branch 41, Pinamalayan,
Oriental Mindoro.
In his Reply complainant contends that as the search warrant was issued only against Joel Gamo and Mantaring, Jr. it was
wrong for respondent judge to find probable cause against him on the theory that, as owners of the house in which the firearms
and ammunitions were found, they had constructive possession of the same. He likewise contends that respondent judge did not
inhibit himself until after the preliminary examination was terminated and the warrant of arrest issued, and only after complainant
had filed a petition for inhibition which the Executive Judge found to be well taken.
On October 16, 1995, this case was referred to the OCA for reevaluation, report and recommendation. On January 12, 1996,
the OCA submitted a Memorandum, recommending dismissal of the supplemental complaint for lack of merit, for the following
reasons:

(1) It is erroneous for herein complainant to equate the application for the issuance of search warrant with the institution and
prosecution of criminal action in a trial court. (Malaloan vs. Court of Appeals, 232 SCRA 249).Complainant cannot insist that
since his name was not included in the search warrant, the house designated to be searched did not belong to him, and that he was
not present at the preliminary investigation of witnesses preparatory to the issuance of the questioned warrant of arrest, there was
no basis for respondent judge to order his arrest.

(2) No taint of irregularity attended the issuance by respondent judge of the warrant of arrest against complainant and his
son. Neither was the charge that the warrant of arrest was issued by respondent judge in the spirit of anger, hatred or harassment
purposes substantiated.

To begin with, it cannot be contended that complainant Leovigildo Mantaring, Sr. could not be proceeded against simply
because he was not included in the search warrant issued against Gamo and Leovigildo Mantaring, Jr., who is apparently his
son. The determination of probable cause in preliminary investigations is based solely on the evidence presented by the
complainant, regardless of whether or not the respondent in that case is named in the proceedings for a search warrant. As
correctly pointed out by, the OCA,[1] the issuance of a search warrant and of a warrant of arrest requires the showing of
probabilities as to different facts. In the case of search warrants, the determination is based on the finding that (1) the articles to be
seized are connected to a criminal activity and (2) they are found in the place to be searched. It is not necessary that a particular
person be implicated. On the other hand, in arrest cases, the determination of probable cause is based on a finding that a crime has
been committed and that the person to be arrested has committed it.
In this case, the arrest of herein complainant and his son, together with Joel Gamo, was ordered on the basis of respondents
finding that the place from where the guns and ammunitions were seized belonged to complainant Leovigildo Mantaring, Sr. and
the testimonies of witnesses presented by SPO4 Fradejas. Of course complainant denies that the house in which the firearms and
ammunition were found belonged to him and claims that at the time of the search he was in Manila. The provincial prosecutor
subsequently dismissed the case against complainant on precisely these grounds, i.e., that the house did not belong to complainant
and he was in Manila at the time the search and seizure were conducted. But to say this is not to say that respondent acted
arbitrarily or that he abused his powers so as to give ground for administrative disciplinary action against him. It is only to say that
he committed an error of judgment for which complainants remedy is judicial.
What we think requires serious consideration is the contention by the complainant that respondent judge should have
inhibited himself from conducting the preliminary investigation of the criminal case, considering that the respondent was the
present complainant, who had earlier filed an administrative case against the judge and another one.
We are not unmindful of the cases in which it was stated that the mere filing of an administrative case against a judge by one
of the parties before him is not a ground for disqualifying him from hearing a case. [2] An examination of these cases reveals,
however, that the administrative cases were filed during the pendency of the cases, and it is evident that the administrative cases
were filed only to force the judge to inhibit himself from the consideration of the case before him. As this Court held, if on every
occasion the party apparently aggrieved were allowed to stop the proceedings in order to await the final decision on the desired
disqualification, or demand the immediate inhibition of the judge on the basis alone of his being so charged, many cases would
have to be kept pending or perhaps there would not be enough judges left to handle all the cases pending in all the courts. [3] On the
other hand, there is a remedy available to the party seeking the disqualification of the judge. If he is denied a fair and impartial
trial, caused by the judges bias or prejudice, he can ask for a new trial in the interest of justice which will be granted if that is
really the case.[4]
But, in the case at bar, an administrative complaint against respondent and Judge Manuel A. Roman, Jr. had previously been
filed and it was paramount that respondent was free from any appearance of bias against, or hostility toward, the complainant. The
impression could not be helped that his action in that case was dictated by a spirt of revenge against complainant for the latters
having filed an administrative disciplinary action against the judge. The situation called for sedulous regard on his part for the
principle that a party is entitled to nothing less than the cold neutrality of an impartial judge.
This circumstance should have underscored for respondent the need of steering clear of the case because he might be
perceived, rightly or wrongly, to be susceptible to bias and partiality. For his judgment must not be tainted by even the slightest
suspicion of improbity or preconceived interest in order to preserve at all times the faith and confidence in courts of justice by any
party to the litigation.[5]
Indeed prudence should have made respondent judge heed the admonition that a spotless dispensation of justice requires not
only that the decision rendered be intrinsically fair but that the judge rendering it must at all times maintain the appearance of
fairness and impartiality.[6]
Moreover, we think it was improper for respondent judge to have issued the warrants of arrest against complainant and his
son without any finding that it was necessary to place them in immediate custody in order to prevent a frustration of justice. It is
now settled[7] that in issuing warrants of arrest in preliminary investigations, the investigating judge must:

(a) have examined in writing and under oath the complainant and his witnesses by searching questions and answers;

(b) be satisfied that probable cause exists; and

(c) that there is a need to place the respondent under immediate custody in order not to frustrate the ends of justice.

In this case, respondent judge justified the issuance of the warrant of arrest on the following ground:

In view of the above considerations [referring to the antecedent facts], it is the honest belief and finding of the Court that there is
sufficient probable cause that the crime of Illegal Possession of Firearms and Ammunition was committed and that the named
three (3) accused Joel Gamo, Leovigildo Mantaring, Sr. and Leovigildo Mantaring Jr. are the ones probably guilty thereof for
which reason Warrant of Arrest was issued by undersigned against them.

He thus ordered the issuance of warrant of arrest solely on his finding of probable cause, totally omitting to consider the third
requirement that there must be a need to place the respondent under immediate custody in order not to frustrate the ends of justice.
The framers of the Constitution confined the determination of probable cause as basis for the issuance of warrants of arrest
and search warrants to judges the better to secure the people against unreasonable searches and seizures. Respondent judge failed
to live up to this expectation by refusing to inhibit himself even when his very impartiality was in question and worse by issuing a
warrant of arrest without determining whether or not it was justified by the need to prevent a frustration of the ends of
justice. Parenthetically, the records show that the criminal complaints against herein complainant and his son were eventually
dismissed by the Provincial Prosecutor, but not without the following parting words:
It cannot be gainsaid that respondents Mantarings were greatly prejudiced and suffered damages as a consequence of their
inclusion in the criminal complaint. The unfortunate incident could have been avoided had the Honorable Municipal Trial Judge
exercised the necessary prudence and judicial perpecuity [sic] expected of an impartial Judge in the conduct of preliminary
investigation before issuance of warrant of arrest.
WHEREFORE, respondent judge Ireneo B. Molato is REPRIMANDED and WARNED that commission of similar acts in
the future will be dealt with more severely. All other charges are dismissed for lack of merit.
SO ORDERED.

G.R. No. 82585 November 14, 1988

MAXIMO V. SOLIVEN, ANTONIO V. ROCES, FREDERICK K. AGCAOLI, and GODOFREDO L.


MANZANAS, petitioners,
vs.
THE HON. RAMON P. MAKASIAR, Presiding Judge of the Regional Trial Court of Manila, Branch 35,
UNDERSECRETARY SILVESTRE BELLO III, of the Department of Justice, LUIS C. VICTOR, THE CITY
FISCAL OF MANILA and PRESIDENT CORAZON C. AQUINO, respondents.

G.R. No. 82827 November 14, 1988

LUIS D. BELTRAN, petitioner,


vs.
THE HON. RAMON P. MAKASIAR, Presiding Judge of Branch 35 of the Regional Trial Court, at Manila, THE
HON. LUIS VICTOR, CITY FISCAL OF MANILA, PEOPLE OF THE PHILIPPINES, SUPERINTENDENT OF
THE WESTERN POLICE DISTRICT, and THE MEMBERS OF THE PROCESS SERVING UNIT AT THE
REGIONAL TRIAL COURT OF MANILA, respondents.

G.R. No. 83979 November 14, 1988.

LUIS D. BELTRAN, petitioner,


vs.
EXECUTIVE SECRETARY CATALINO MACARAIG, SECRETARY OF JUSTICE SEDFREY ORDOÑEZ,
UNDERSECRETARY OF JUSTICE SILVESTRE BELLO III, THE CITY FISCAL OF MANILA JESUS F.
GUERRERO, and JUDGE RAMON P. MAKASIAR, Presiding Judge of Branch 35 of the Regional Trial Court, at
Manila, respondents.

Angara, Abello, Concepcion, Regala and Cruz for petitioners in G.R. No. 82585.

Perfecto V. Fernandez, Jose P. Fernandez and Cristobal P. Fernandez for petitioner in G.R. Nos. 82827 and 83979.

RESOLUTION

PER CURIAM:

In these consolidated cases, three principal issues were raised: (1) whether or not petitioners were denied due process when
informations for libel were filed against them although the finding of the existence of a prima faciecase was still under
review by the Secretary of Justice and, subsequently, by the President; (2) whether or not the constitutional rights of Beltran
were violated when respondent RTC judge issued a warrant for his arrest without personally examining the complainant and
the witnesses, if any, to determine probable cause; and (3) whether or not the President of the Philippines, under the
Constitution, may initiate criminal proceedings against the petitioners through the filing of a complaint-affidavit.

Subsequent events have rendered the first issue moot and academic. On March 30, 1988, the Secretary of Justice denied
petitioners' motion for reconsideration and upheld the resolution of the Undersecretary of Justice sustaining the City Fiscal's
finding of a prima facie case against petitioners. A second motion for reconsideration filed by petitioner Beltran was denied
by the Secretary of Justice on April 7, 1988. On appeal, the President, through the Executive Secretary, affirmed the
resolution of the Secretary of Justice on May 2, 1988. The motion for reconsideration was denied by the Executive
Secretary on May 16, 1988. With these developments, petitioners' contention that they have been denied the administrative
remedies available under the law has lost factual support.

It may also be added that with respect to petitioner Beltran, the allegation of denial of due process of law in the preliminary
investigation is negated by the fact that instead of submitting his counter- affidavits, he filed a "Motion to Declare
Proceedings Closed," in effect waiving his right to refute the complaint by filing counter-affidavits. Due process of law does
not require that the respondent in a criminal case actually file his counter-affidavits before the preliminary investigation is
deemed completed. All that is required is that the respondent be given the opportunity to submit counter-affidavits if he is
so minded.

The second issue, raised by petitioner Beltran, calls for an interpretation of the constitutional provision on the issuance of
warrants of arrest. The pertinent provision reads:
Art. III, Sec. 2. The right of the people to be secure in their persons, houses, papers and effects against unreasonable
searches and seizures of whatever nature and for any purpose shall be inviolable, and no search warrant or warrant of arrest
shall issue except upon probable cause to be determined personally by the judge after examination nder oath or affirmation
of the complainant and the witnesses he may produce, and particularly describing the place to be searched and the persons
or things to be seized.

The addition of the word "personally" after the word "determined" and the deletion of the grant of authority by the 1973
Constitution to issue warrants to "other responsible officers as may be authorized by law," has apparently convinced
petitioner Beltran that the Constitution now requires the judge to personally examine the complainant and his witnesses in
his determination of probable cause for the issuance of warrants of arrest. This is not an accurate interpretation.

What the Constitution underscores is the exclusive and personal responsibility of the issuing judge to satisfy himself of the
existence of probable cause. In satisfying himself of the existence of probable cause for the issuance of a warrant of arrest,
the judge is not required to personally examine the complainant and his witnesses. Following established doctrine and
procedure, he shall: (1) personally evaluate the report and the supporting documents submitted by the fiscal regarding the
existence of probable cause and, on the basis thereof, issue a warrant of arrest; or (2) if on the basis thereof he finds no
probable cause, he may disregard the fiscal's report and require the submission of supporting affidavits of witnesses to aid
him in arriving at a conclusion as to the existence of probable cause.

Sound policy dictates this procedure, otherwise judges would be unduly laden with the preliminary examination and
investigation of criminal complaints instead of concentrating on hearing and deciding cases filed before their courts.

On June 30, 1987, the Supreme Court unanimously adopted Circular No. 12, setting down guidelines for the issuance of
warrants of arrest. The procedure therein provided is reiterated and clarified in this resolution.

It has not been shown that respondent judge has deviated from the prescribed procedure. Thus, with regard to the issuance
of the warrants of arrest, a finding of grave abuse of discretion amounting to lack or excess of jurisdiction cannot be
sustained.

Anent the third issue, petitioner Beltran argues that "the reasons which necessitate presidential immunity from suit impose a
correlative disability to file suit." He contends that if criminal proceedings ensue by virtue of the President's filing of her
complaint-affidavit, she may subsequently have to be a witness for the prosecution, bringing her under the trial court's
jurisdiction. This, continues Beltran, would in an indirect way defeat her privilege of immunity from suit, as by testifying on
the witness stand, she would be exposing herself to possible contempt of court or perjury.

The rationale for the grant to the President of the privilege of immunity from suit is to assure the exercise of Presidential
duties and functions free from any hindrance or distraction, considering that being the Chief Executive of the Government is
a job that, aside from requiring all of the office holder's time, also demands undivided attention.

But this privilege of immunity from suit, pertains to the President by virtue of the office and may be invoked only by the
holder of the office; not by any other person in the President's behalf. Thus, an accused in a criminal case in which the
President is complainant cannot raise the presidential privilege as a defense to prevent the case from proceeding against
such accused.

Moreover, there is nothing in our laws that would prevent the President from waiving the privilege. Thus, if so minded the
President may shed the protection afforded by the privilege and submit to the court's jurisdiction. The choice of whether to
exercise the privilege or to waive it is solely the President's prerogative. It is a decision that cannot be assumed and imposed
by any other person.

As regards the contention of petitioner Beltran that he could not be held liable for libel because of the privileged character
or the publication, the Court reiterates that it is not a trier of facts and that such a defense is best left to the trial court to
appreciate after receiving the evidence of the parties.

As to petitioner Beltran's claim that to allow the libel case to proceed would produce a "chilling effect" on press freedom,
the Court finds no basis at this stage to rule on the point.

The petitions fail to establish that public respondents, through their separate acts, gravely abused their discretion as to
amount to lack of jurisdiction. Hence, the writs of certiorari and prohibition prayed for cannot issue.
WHEREFORE, finding no grave abuse of discretion amounting to excess or lack of jurisdiction on the part of the public
respondents, the Court Resolved to DISMISS the petitions in G. R. Nos. 82585, 82827 and 83979. The Order to maintain
the status quo contained in the Resolution of the Court en banc dated April 7, 1988 and reiterated in the Resolution dated
April 26, 1988 is LIFTED.

Fernan, C.J., Narvasa, Melencio-Herrera, Cruz, Paras, Feliciano, Gancayco, Padilla, Bidin, Sarmiento, Cortes,
Griño-Aquino Medialdea and Regalado, JJ., concur.

Separate Opinions

GUTIERREZ, JR., J., concurring:

I concur with the majority opinion insofar as it involves the three principal issues mentioned in its opening statement.
However, as to the more important issue on whether or not the prosecution of the libel case would produce a "chilling
effect" on press freedom, I beg to reserve my vote. I believe this is the more important issue in these petitions and it should
be resolved now rather that later.

Consistent with our decision in Salonga v. Cruz Pano (134 SCRA 438 [1985]), the Court should not hesitate to quash a
criminal prosecution in the interest of more enlightened and substantial justice where it is not alone the criminal liability of
an accused in a seemingly minor libel case which is involved but broader considerations of governmental power versus a
preferred freedom.

We have in these four petitions the unusual situation where the highest official of the Republic and one who enjoys
unprecedented public support asks for the prosecution of a newspaper columnist, the publisher and chairman of the editorial
board, the managing editor and the business manager in a not too indubitable a case for alleged libel.

I am fully in accord with an all out prosecution if the effect will be limited to punishing a newspaperman who, instead of
observing accuracy and fairness, engages in unwarranted personal attacks, irresponsible twisting of facts, of malicious
distortions of half-truths which tend to cause dishonor, discredit, or contempt of the complainant. However, this case is not
a simple prosecution for libel. We have as complainant a powerful and popular President who heads the investigation and
prosecution service and appoints members of appellate courts but who feels so terribly maligned that she has taken the
unorthodox step of going to court inspite of the invocations of freedom of the press which would inevitably follow.

I believe that this Court should have acted on this issue now instead of leaving the matter to fiscals and defense lawyers to
argue before a trial judge.

There is always bound to be harassment inherent in any criminal prosecution. Where the harassment goes beyond the usual
difficulties encountered by any accused and results in an unwillingness of media to freely criticize government or to
question government handling of sensitive issues and public affairs, this Court and not a lower tribunal should draw the
demarcation line.

As early as March 8, 1918, the decision in United States v. Bustos (37 Phil. 731) stated that "(c)omplete liberty to comment
on the conduct of public men is a scalpel in the case of free speech. The sharp incision of its probe relieves the abscesses of
officialdom. Men in public life may suffer under a hostile and unjust accusation; the wound can be assuaged with the balm
of a clear conscience." The Court pointed out that while defamation is not authorized, criticism is to be expected and should
be borne for the common good.

In People v. Perfecto (43 Phil. 887 [1922]), the Court stated:

xxx xxx xxx

... No longer is there a Minister of the Crown own or a person in authority of such exalted position that the citizen must
speak of him only with bated breath. "In the eye of our Constitution and laws, every man is a sovereign, a ruler and a
freeman, and has equal rights with every other man." (at p. 900)
In fact, the Court observed that high official position, instead of affording immunity from slanderous and libelous charges,
would actually invite attacks by those who desire to create sensation. It would seem that what would ordinarily be slander if
directed at the typical person should be examined from various perspectives if directed at a high government official. Again,
the Supreme Court should draw this fine line instead of leaving it to lower tribunals.

This Court has stressed as authoritative doctrine in Elizalde v. Gutierrez (76 SCRA 448 [1977]) that a prosecution for libel
lacks justification if the offending words find sanctuary within the shelter of the free press guaranty. In other words, a
prosecution for libel should not be allowed to continue, where after discounting the possibility that the words may not be
really that libelous, there is likely to be a chilling effect, a patently inhibiting factor on the willingness of newspapermen,
especially editors and publishers to courageously perform their critical role in society. If, instead of merely reading more
carefully what a columnist writes in his daily column, the editors tell their people to lay off certain issues or certain officials,
the effect on a free press would be highly injurious.

Because many questions regarding press freedom are left unanswered by our resolution, I must call attention to our
decisions which caution that "no inroads on press freedom should be allowed in the guise of punitive action visited on what
otherwise should be characterized as libel." (Lopez v. Court of Appeals, 34 SCRA 117 [1970]; See also the citations in
Elizalde v. Gutierrez, supra).

The United States Supreme Court is even more emphatic, to wit:

In deciding the question now, we are compelled by neither precedent nor policy to give any more weight to the epithet
"libel" than we have to other "mere labels" of state law. N. A. A. C. P. v. Button, 371 US 415, 429, 9L ed 2d 405, 415, 83 S
Ct 328. Like insurrection, contempt, advocacy of unlawful acts, breach of the peace, obscenity, solicitation of legal business,
and the other various other formulae for the repression of expression that have been challenged in this Court, libel can claim
no talismanic immunity from constitutional limitations. It must be measured by standards that satisfy the First Amendment.

xxx xxx xxx

Those who won our independence believed ... that public discussion is a political duty; and that this should be a
fundamental principle of the American government. They recognized the risk to which all human institutions are subject.
But they knew that order cannot be secured merely through fear of punishment for its infraction; that it is hazardous to
discourage thought, hope and imagination; that fear breeds repression; that repression breeds hate; that hate menaces stable
government; that the path of safety lies in the opportunity to discuss freely supposed grievances and proposed remedies; and
that the fitting remedy for evil counsel is good ones. Believing in the power of reason as applied through public discussion,
they eschewed silence coerced by law—the argument of force in its worst form. ...

Thus we consider this case against the background of a profound national commitment to the principle that debate on public
issues should be uninhibited, robust, and wide open, and that it may well include vehement, caustic, and sometimes
unpleasantly sharp attacks on government and public officials. ... (at pp. 700-701)

Shunting aside the individual liability of Mr. Luis Beltran, is there a prima facie showing that Messrs. Maximo Soliven,
Antonio V. Roces, Frederick K. Agcaoili, and Godofredo L. Manzanas knowingly participated in a wilful purveying of
falsehood? Considering the free speech aspects of these petitions, should not a differentiated approach to their particular
liabilities be taken instead of lumping up everybody with the offending columnist? I realize that the law includes publishers
and editors but perhaps the "chilling effect" issue applies with singular effectivity to publishers and editors vis-a-vis
newspaper columnists. There is no question that, ordinarily, libel is not protected by the free speech clause but we have to
understand that some provocative words, which if taken literally may appear to shame or disparage a public figure, may
really be intended to provoke debate on public issues when uttered or written by a media personality. Will not a criminal
prosecution in the type of case now before us dampen the vigor and limit the variety of public debate? There are many other
questions arising from this unusual case which have not been considered.

I, of course, concur with the Court's opinion because it has decided to limit the issues to narrowly drawn ones. I see no
reason to disagree with the way the Court has resolved them. The first issue on prematurity is moot. The second issue
discusses a procedure now embodied in the recently amended Rules of Court on how a Judge should proceed before he
issues a warrant of arrest. Anent the third issue, considerations of public policy dictate that an incumbent President should
not be sued. At the same time, the President cannot stand by helplessly bereft of legal remedies if somebody vilifies or
maligns him or her.
The Court has decided to defer the "chilling effect" issue for a later day. To this, I take exception. I know that most of our
fiscals and judges are courageous individuals who would not allow any considerations of possible consequences to their
careers to stand in the way of public duty. But why should we subject them to this problem? And why should we allow the
possibility of the trial court treating and deciding the case as one for ordinary libel without bothering to fully explore the
more important areas of concern, the extremely difficult issues involving government power and freedom of expression.

However, since we have decided to defer the "chilling effect" issue for a later day, I limit myself to reiterating the dissenting
words of Mr. Justice Jackson in the American case of Beaurnhais v. Illinois (343 U. S. 250) when he said:

If one can claim to announce the judgment of legal history on any subject, it is that criminal libel laws are consistent with
the concept of ordered liberty only when applied with safeguards evolved to prevent their invasion of freedom of
expression.

In the trial of the libel case against the petitioners, the safeguards in the name of freedom of expression should be faithfully
applied.

Separate Opinions

GUTIERREZ, JR., J., concurring:

I concur with the majority opinion insofar as it involves the three principal issues mentioned in its opening statement.
However, as to the more important issue on whether or not the prosecution of the libel case would produce a "chilling
effect" on press freedom, I beg to reserve my vote. I believe this is the more important issue in these petitions and it should
be resolved now rather that later.

Consistent with our decision in Salonga v. Cruz Pano (134 SCRA 438 [1985]), the Court should not hesitate to quash a
criminal prosecution in the interest of more enlightened and substantial justice where it is not alone the criminal liability of
an accused in a seemingly minor libel case which is involved but broader considerations of governmental power versus a
preferred freedom.

We have in these four petitions the unusual situation where the highest official of the Republic and one who enjoys
unprecedented public support asks for the prosecution of a newspaper columnist, the publisher and chairman of the editorial
board, the managing editor and the business manager in a not too indubitable a case for alleged libel.

I am fully in accord with an all out prosecution if the effect will be limited to punishing a newspaperman who, instead of
observing accuracy and fairness, engages in unwarranted personal attacks, irresponsible twisting of facts, of malicious
distortions of half-truths which tend to cause dishonor, discredit, or contempt of the complainant. However, this case is not
a simple prosecution for libel. We have as complainant a powerful and popular President who heads the investigation and
prosecution service and appoints members of appellate courts but who feels so terribly maligned that she has taken the
unorthodox step of going to court inspite of the invocations of freedom of the press which would inevitably follow.

I believe that this Court should have acted on this issue now instead of leaving the matter to fiscals and defense lawyers to
argue before a trial judge.

There is always bound to be harassment inherent in any criminal prosecution. Where the harassment goes beyond the usual
difficulties encountered by any accused and results in an unwillingness of media to freely criticize government or to
question government handling of sensitive issues and public affairs, this Court and not a lower tribunal should draw the
demarcation line.

As early as March 8, 1918, the decision in United States v. Bustos (37 Phil. 731) stated that "(c)omplete liberty to comment
on the conduct of public men is a scalpel in the case of free speech. The sharp incision of its probe relieves the abscesses of
officialdom. Men in public life may suffer under a hostile and unjust accusation; the wound can be assuaged with the balm
of a clear conscience." The Court pointed out that while defamation is not authorized, criticism is to be expected and should
be borne for the common good.

In People v. Perfecto (43 Phil. 887 [1922]), the Court stated:

xxx xxx xxx


... No longer is there a Minister of the Crown own or a person in authority of such exalted position that the citizen must
speak of him only with bated breath. "In the eye of our Constitution and laws, every man is a sovereign, a ruler and a
freeman, and has equal rights with every other man." (at p. 900)

In fact, the Court observed that high official position, instead of affording immunity from slanderous and libelous charges,
would actually invite attacks by those who desire to create sensation. It would seem that what would ordinarily be slander if
directed at the typical person should be examined from various perspectives if directed at a high government official. Again,
the Supreme Court should draw this fine line instead of leaving it to lower tribunals.

This Court has stressed as authoritative doctrine in Elizalde v. Gutierrez (76 SCRA 448 [1977]) that a prosecution for libel
lacks justification if the offending words find sanctuary within the shelter of the free press guaranty. In other words, a
prosecution for libel should not be allowed to continue, where after discounting the possibility that the words may not be
really that libelous, there is likely to be a chilling effect, a patently inhibiting factor on the willingness of newspapermen,
especially editors and publishers to courageously perform their critical role in society. If, instead of merely reading more
carefully what a columnist writes in his daily column, the editors tell their people to lay off certain issues or certain officials,
the effect on a free press would be highly injurious.

Because many questions regarding press freedom are left unanswered by our resolution, I must call attention to our
decisions which caution that "no inroads on press freedom should be allowed in the guise of punitive action visited on what
otherwise should be characterized as libel." (Lopez v. Court of Appeals, 34 SCRA 117 [1970]; See also the citations in
Elizalde v. Gutierrez, supra).<äre||anº•1àw>

The United States Supreme Court is even more emphatic, to wit:

In deciding the question now, we are compelled by neither precedent nor policy to give any more weight to the epithet
"libel" than we have to other "mere labels" of state law. N. A. A. C. P. v. Button, 371 US 415, 429, 9L ed 2d 405, 415, 83 S
Ct 328. Like insurrection, contempt, advocacy of unlawful acts, breach of the peace, obscenity, solicitation of legal business,
and the other various other formulae for the repression of expression that have been challenged in this Court, libel can claim
no talismanic immunity from constitutional limitations. It must be measured by standards that satisfy the First Amendment.

xxx xxx xxx

Those who won our independence believed ... that public discussion is a political duty; and that this should be a
fundamental principle of the American government. They recognized the risk to which all human institutions are subject.
But they knew that order cannot be secured merely through fear of punishment for its infraction; that it is hazardous to
discourage thought, hope and imagination; that fear breeds repression; that repression breeds hate; that hate menaces stable
government; that the path of safety lies in the opportunity to discuss freely supposed grievances and proposed remedies; and
that the fitting remedy for evil counsel is good ones. Believing in the power of reason as applied through public discussion,
they eschewed silence coerced by law—the argument of force in its worst form. ...

Thus we consider this case against the background of a profound national commitment to the principle that debate on public
issues should be uninhibited, robust, and wide open, and that it may well include vehement, caustic, and sometimes
unpleasantly sharp attacks on government and public officials. ... (at pp. 700-701)

Shunting aside the individual liability of Mr. Luis Beltran, is there a prima facie showing that Messrs. Maximo Soliven,
Antonio V. Roces, Frederick K. Agcaoili, and Godofredo L. Manzanas knowingly participated in a wilful purveying of
falsehood? Considering the free speech aspects of these petitions, should not a differentiated approach to their particular
liabilities be taken instead of lumping up everybody with the offending columnist? I realize that the law includes publishers
and editors but perhaps the "chilling effect" issue applies with singular effectivity to publishers and editors vis-a-vis
newspaper columnists. There is no question that, ordinarily, libel is not protected by the free speech clause but we have to
understand that some provocative words, which if taken literally may appear to shame or disparage a public figure, may
really be intended to provoke debate on public issues when uttered or written by a media personality. Will not a criminal
prosecution in the type of case now before us dampen the vigor and limit the variety of public debate? There are many other
questions arising from this unusual case which have not been considered.

I, of course, concur with the Court's opinion because it has decided to limit the issues to narrowly drawn ones. I see no
reason to disagree with the way the Court has resolved them. The first issue on prematurity is moot. The second issue
discusses a procedure now embodied in the recently amended Rules of Court on how a Judge should proceed before he
issues a warrant of arrest. Anent the third issue, considerations of public policy dictate that an incumbent President should
not be sued. At the same time, the President cannot stand by helplessly bereft of legal remedies if somebody vilifies or
maligns him or her.

The Court has decided to defer the "chilling effect" issue for a later day. To this, I take exception. I know that most of our
fiscals and judges are courageous individuals who would not allow any considerations of possible consequences to their
careers to stand in the way of public duty. But why should we subject them to this problem? And why should we allow the
possibility of the trial court treating and deciding the case as one for ordinary libel without bothering to fully explore the
more important areas of concern, the extremely difficult issues involving government power and freedom of expression.

However, since we have decided to defer the "chilling effect" issue for a later day, I limit myself to reiterating the dissenting
words of Mr. Justice Jackson in the American case of Beaurnhais v. Illinois (343 U. S. 250) when he said:

If one can claim to announce the judgment of legal history on any subject, it is that criminal libel laws are consistent with
the concept of ordered liberty only when applied with safeguards evolved to prevent their invasion of freedom of
expression.

In the trial of the libel case against the petitioners, the safeguards in the name of freedom of expression should be faithfully
applied.

G.R. No. L-22196 June 30, 1967

ESTEBAN MORANO, CHAN SAU WAH and FU YAN FUN, petitioners-appellants,


vs.
HON. MARTINIANO VIVO in his capacity as Acting Commissioner of Immigration, respondent-appellant.

Engracio Fabre Law Office for petitioners-appellants.


Office of the Solicitor General Arturo A. Alafriz and Solicitor A. M. Amores for respondent-appellant.

SANCHEZ, J.:
Chan Sau Wah, a Chinese citizen born in Fukien, China on January 6, 1932, arrived in the Philippines on
November 23, 1961 to visit her cousin, Samuel Lee Malaps. She left in mainland China two of her children by a
first marriage: Fu Tse Haw and Fu Yan Kai With her was Fu Yan Fun, her minor son also by the first marriage,
born in Hongkong on September 11, 1957.

Chan Sau Wah and her minor son Fu Yan Fun were permitted only into the Philippines under a temporary visitor's
visa for two (2) months and after they posted a cash bond of P4,000.00.

On January 24, 1962, Chan Sau Wah married Esteban Morano, a native-born Filipino citizen. Born to this union
on September 16, 1962 was Esteban Morano, Jr.

To prolong their stay in the Philippines, Chan Sau Wah and Fu Yan Fun obtained several extensions. The last
extension expired on September 10, 1962. 1äwphï1. ñët

In a letter dated August 31, 1962, the Commissioner of Immigration ordered Chan Sau Wah and her son, Fu Yan
Fun, to leave the country on or before September 10, 1962 with a warning that upon failure so to do, he will issue
a warrant for their arrest and will cause the confiscation of their bond.

Instead of leaving the country, on September 10, 1962, Chan Sau Wah (with her husband Esteban Morano) and
Fu Yan Fun petitioned the Court of First Instance of Manila for mandamus to compel the Commissioner of
Immigration to cancel petitioners' Alien Certificates of Registration; prohibition to stop the Commissioner from
issuing a warrant for their arrest, and preliminary injunction to restrain the Commissioner from confiscating their
cash bond and from issuing warrants of arrest pending resolution of this case.1 The trial court, on November 3,
1962, issued the writ of preliminary injunction prayed for, upon a P2,000-bond. After trial and the stipulations of
facts filed by the parties, the Court of First Instance rendered judgment, viz:

IN VIEW OF ALL THE FOREGOING, judgment is hereby rendered as follows:

(a) Granting this petition for Mandamus and Prohibition with respect to petitioner CHAN SAU WAH, who is hereby
declared a citizen of the Philippines; ordering the respondent to cancel her Alien Certificate of Registration and
other immigration papers, upon the payment of proper dues; and declaring the preliminary injunction with respect
to her permanent, prohibiting the respondent, his representatives or subordinates from arresting and/or deporting
said petitioner;

(b) Dismissing this petition with respect to petitioner FU YAN FUN, and dissolving the writ of preliminary injunction
issued herein, restraining the respondent, his representatives or subordinates from arresting and/or deporting
said petitioner;

(c) Authorizing respondent Commissioner to forfeit the bond filed by herein petitioners CHAN SAU WAH and FU
YAN FUN in the amount of P4,000.00; and

(d) Denying, for lack of merit, the prayer to declare Sec. 37 (a) of the Philippine Immigration Act of 1940
unconstitutional;

Without pronouncement, as to costs.

Petitioners and respondent Commissioner both appealed.

We will deal with the claims of both appellants in their proper sequence.

1. The Solicitor General's brief assails the trial court's declaration that Chan Sau Wah is a citizen of the
Philippines. The court a quo took the position that "Chan Sau Wah became, by virtue of, and upon, her marriage
to Esteban Morano, a natural-born Filipino, a Filipino citizen.2

Placed to the fore is paragraph 1, Section 15 of Commonwealth Act 473 [Revised Naturalization Act], which
reads:
Sec. 15. Effect of the naturalization on wife children. — Any woman who is now or may hereafter be married to a
citizen of the Philippines, and who might herself be lawfully naturalized shall be deemed a citizen of the
Philippines.

To apply this provision, two requisites must concur: (a) valid marriage of an alien woman to a citizen of the
Philippines and (b) the alien woman herself might be lawfully naturalized.

We may concede that the first requisite has been properly met. The validity of the marriage is presumed.

But can the same be said of the second requisite? This question by all means is not new. In a series of cases, this
Court has declared that the marriage of an alien woman to a Filipino citizen does not ipso facto make her a
Filipino citizen. She must satisfactorily show that she has all the qualifications and none of the disqualifications
required by the Naturalization Law.3 Ly Giok Ha alias Wy Giok Ha et al. vs. Emilio Galang, L-21332, March 18,
1966,* clearly writes down the philosophy behind the rule in the following expressive language, viz:

Reflection will reveal why this must be so. The qualifications prescribed under section 2 of the Naturalization Act,
and the disqualifications enumerated in its section 4, are not mutually exclusive; and if all that were to be required
is that the wife of a Filipino be not disqualified under section 4, the result might well be that citizenship would be
conferred upon persons in violation of the policy of the statute. For example, section 4 disqualifies only —

"(c) Polygamists or believers in the practice of polygamy; and

(b) Persons convicted of crimes involving moral turpitude,"

so that a blackmailer, or a maintainer of gambling or bawdy houses, not previously convicted by a competent
court, would not be thereby disqualified; still it is certain that the law did not intend such a person to, be admitted
as a citizen in view of the requirement of section 2 that an applicant for citizenship "must be of good moral
character."

Similarly, the citizen's wife might be a convinced believer in racial supremacy, in government by certain selected
classes, in the right to vote exclusively by certain "herrenvolk," and thus disbelieve in the principles underlying the
Philippine Constitution; yet she would not be disqualified under section 4, as long as she is not "opposed to
organized government," nor affiliated to groups "upholding or teaching doctrines opposing all organized
governments," nor "defending or teaching the necessity or propriety of violence, personal assault or
assassination for the success or predominance of their ideas." Et sic de caeteris.

Upon the principle of selective citizenship, we cannot afford to depart from the wise precept affirmed and
reaffirmed in the cases heretofore noted.

In the additional stipulation of facts of July 3, 1963, petitioners admit that Chan Sau Wah is not possessed of all
the qualifications required by the Naturalization Law.

Because of all these we are left under no doubt that petitioner Chan Sau Wah did not become a Filipino citizen.

2. Squarely put in issue by petitioners is the constitutionality of Section 37 (a) of the Immigration Act of 1940,
which reads:

Sec. 37. (a) The following aliens shall be arrested upon the warrant of the Commissioner of Immigration or of any
other officer designated by him for the purpose and deported upon the warrant of the Commissioner of
Immigration after a determination by the Board of Commissioners of the existence of the ground for deportation
as charged against the alien:

xxx xxx xxx

(7) Any alien who remains in the Philippines in violation of any limitation or condition under which he was admitted
as a nonimmigrant.
Petitioners argue that the legal precept just quoted trenches upon the constitutional mandate in Section 1 (3),
Article III [Bill of Rights] of the Constitution, to wit:

(3) The right of the people to be secure in their persons, houses, papers, and effects against unreasonable
searches and seizures shall not be violated, and no warrants shall issue but upon probable cause, to be
determined by the judge after examination under oath or affirmation of the complainant and the witnesses he may
produce, and particularly describing the place to be searched, and the persons or things to be seized.

They say that the Constitution limits to judges the authority to issue warrants of arrest and that the legislative
delegation of such power to the Commissioner of Immigration is thus violative of the Bill of Rights.

Section 1 (3), Article III of the Constitution, we perceive, does not require judicial intervention in the execution of a
final order of deportation issued in accordance with law. The constitutional limitation contemplates an order of
arrest in the exercise of judicial power4 as a step preliminary or incidental to prosecution or proceedings for a
given offense or administrative action, not as a measure indispensable to carry out a valid decision by a
competent official, such as a legal order of deportation, issued by the Commissioner of Immigration, in pursuance
of a valid legislation.

The following from American Jurisprudence,5 is illuminating:

It is thoroughly established that Congress has power to order the deportation of aliens whose presence in the
country it deems hurtful. Owing to the nature of the proceeding, the deportation of an alien who is found in this
country in violation of law is not a deprivation of liberty without due process of law. This is so, although the inquiry
devolves upon executive officers, and their findings of fact, after a fair though summary hearing, are made
conclusive.

xxx xxx xxx

The determination of the propriety of deportation is not a prosecution for, or a conviction of, crime; nor is the
deportation a punishment, even though the facts underlying the decision may constitute a crime under local law.
The proceeding is in effect simply a refusal by the government to harbor persons whom it does not want. The
coincidence of local penal law with the policy of Congress is purely accidental, and, though supported by the
same facts, a criminal prosecution and a proceeding for deportation are separate and independent.

In consequence, the constitutional guarantee set forth in Section 1 (3), Article III of the Constitution aforesaid,
requiring that the issue of probable cause be determined by a judge, does not extend to deportation proceedings.6

The view we here express finds support in the discussions during the constitutional convention. The convention
recognized, as sanctioned by due process, possibilities and cases of deprivation of liberty, other than by order of
a competent court.7

Indeed, the power to deport or expel aliens is an attribute of sovereignty. Such power is planted on the "accepted
maxim of international law, that every sovereign nation has the power, as inherent in sovereignty, and essential to
self-preservation, to forbid the entrance of foreigners within its dominions."8 So it is, that this Court once aptly
remarked that there can be no controversy on the fact that where aliens are admitted as temporary visitors, "the
law is to the effect that temporary visitors who do not depart upon the expiration of the period of stay granted them
are subject to deportation by the Commissioner of Immigration, for having violated the limitation or condition
under which they were admitted as non-immigrants (Immigration Law, Sec. 37 (a), subsection (7); C.A. 613, as
amended)."9

And, in a case directly in point, where the power of the Commissioner to issue warrants of arrest was
challenged as unconstitutional, because "such power is only vested in a judge by Section 1, paragraph 3, Article
III of our Constitution," this Court declared —

This argument overlooks the fact that the stay of appellant Ng Hua To as temporary visitor is subject to certain
contractual stipulations as contained in the cash bond put up by him, among them, that in case of breach the
Commissioner may require the recommitment of the person in whose favor the bond has been filed. The
Commissioner did nothing but to enforce such condition. Such a step is necessary to enable the Commissioner to
prepare the ground for his deportation under section 37 (a) of Commonwealth Act 613. A contrary interpretation
would render such power nugatory to the detriment of the State.10

It is in this context that we rule that Section 37 (a) of the Immigration Act of 1940 is not constitutionally proscribed.

3. A sequel to the questions just discussed is the second error set forth in the government's brief. The Solicitor
General balks at the lower court's ruling that petitioner Chan Sau Wah is entitled to permanent residence in the
Philippines without first complying with the requirements of Sections 9 and 13 of the Immigration Act of 1940, as
amended by Republic Act 503.

We first go to the law, viz:

SEC. 9 [last paragraph]

An alien who is admitted as a nonimmigrant cannot remain in the Philippines permanently. To obtain permanent
admission, a nonimmigrant alien must depart voluntarily to some foreign country and procure from the
appropriate Philippine consul the proper visa and thereafter undergo examination by the officers of the Bureau of
Immigration at a Philippine port of entry for determination of his admissibility in accordance with the requirements
of this Act.

SEC. 13. Under the conditions set forth in this Act there may be admitted into the Philippines immigrants, termed
"quota immigrants" not in excess of fifty (50) of any one nationality or without nationality for any one calendar year,
except that the following immigrants, termed "nonquota immigrants," maybe admitted without regard to such
numerical limitations.

The corresponding Philippine Consular representative abroad shall investigate and certify the eligibility of a quota
immigrant previous to his admission into the Philippines. Qualified and desirable aliens who are in the Philippines
under temporary stay may be admitted within the quota, subject to the provisions of the last paragraph of section
9 of this Act.

(a) The wife or the husband or the unmarried child under twenty-one years of age of a Philippine citizen, if
accompanying or following to join such citizen;

(b) A child of alien parents born during the temporary visit abroad of the mother, the mother having been
previously lawfully admitted into the Philippine for permanent residence, if the child is accompanying or coming to
join a parent and applies for admission within five years from the date of its birth;

Concededly, Chan Sau Wah entered the Philippines on a tourist-temporary visitor's visa. She is a non-immigrant.
Under Section 13 just quoted, she may therefore be admitted if she were a qualified and desirable alien and
subject to the provisions of the last paragraph of Section 9. Therefore, first, she must depart voluntarily to some
foreign country; second, she must procure from the appropriate consul the proper visa; and third, she must
thereafter undergo examination by the officials of the Bureau of Immigration at the port of entry for determination
of her admissibility in accordance with the requirements of the immigration Act.

This Court in a number of cases has ruled, and consistently too, that an alien admitted as a temporary visitor
cannot change his or her status without first departing from the country and complying with the requirements of
Section 9 of the Immigration Act. 11

The gravamen of petitioners' argument is that Chan Sau Wah has, since her entry, married in Manila a
native-born Filipino, Esteban Morano. It will not particularly help analysis for petitioners to appeal to family
solidarity in an effort to thwart her deportation. Chan Sau Wah, seemingly is not one who has a high regard for
such solidarity. Proof: She left two of her children by the first marriage, both minors, in the care of neighbors in
Fukien, China.

Then, the wording of the statute heretofore adverted to is a forbidding obstacle which will prevent this Court from
writing into the law an additional provision that marriage of a temporary alien visitor to a Filipino would ipso
facto make her a permanent resident in his country. This is a field closed to judicial action. No breadth of
discretion is allowed us. We cannot insulate her from the State's power of deportation.
Really, it would be an easy matter for an alien woman to enter the Philippines as a temporary visitor, go through a
mock marriage, but actually live with another man as husband and wife, and thereby skirt the provisions of our
immigration law. Also, a woman of undesirable character may enter this country, ply a pernicious trade, marry a
Filipino, and again throw overboard Sections 9 and 13 of the Act. Such a flanking movement, we are confident, is
impermissible.

Recently we confirmed the rule that an alien wife of a Filipino may not stay permanently without first departing
from the Philippines. Reason: Discourage entry under false pretenses. 12

The ruling of the trial court on this score should be reversed.

4. It is petitioners' turn to point as error the dismissal of the petition for mandamus and prohibition with respect to
petitioner Fu Yan Fun.

Petitioners' line of thought is this: Fu Yan Fun follows the citizenship of his mother. They cite Section 15,
paragraph 3, Commonwealth Act 473, which says that:

A foreign-born minor child, if dwelling in the Philippines at the time of the naturalization of the parent, shall
automatically become a Philippine citizen. . . .

Petitioners' position is based on the assumption that Chan Sau Wah, the mother, is a Filipino citizen. We have
held that she is not. At best, Fu Yan Fun is a step-son of Esteban Morano, husband of Chan Sau Wah. A step-son
is not a foreign-born child of the step-father. The word child, we are certain, means legitimate child, not a
step-child. We are not wanting in precedents. Thus, when the Constitution provides that "[t]hose whose fathers
are citizens of the Philippines" are citizens thereof, 13 the fundamental charter intends "those" to apply to
legitimate children. 14 In another case, the term "minor children" or "minor child" in Section 15 of the Revised
Naturalization Law refers only to legitimate children of Filipino citizens. This Court, thru Mr. Chief Justice Roberto
Concepcion, there said: 15

It is claimed that the phrases "minor children" and "minor child," used in these provisions, include adopted
children. The argument is predicated upon the theory that an adopted child is, for all intents and purposes, a
legitimate child. Whenever, the word "children" or "child" is used in statutes, it is generally understood, however,
to refer to legitimate children, unless the context of the law and its spirit indicate clearly the contrary. Thus, for
instance, when the Constitution provides that "those whose fathers are citizens of the Philippines," and "those
whose mothers are citizens of the Philippines" who shall elect Philippine citizenship upon reaching the age of
majority, are citizens of the Philippines (Article IV, Section 1, subdivisions [3] and [4]), our fundamental law clearly
refers to legitimate children (Chiongbian vs. De Leon, 46 Off. Gaz., 3652-3654; Serra v. Republic, L-4223, May
12, 1952).

At any rate, Fu Yan Fun entered the Philippines as a temporary visitor. The status of a temporary visitor cannot
be converted into, that of a permanent resident, as we have heretofore held, without first complying with Section 9
of the Immigration Law.

5. Petitioners finally aver that the lower court erred in authorizing respondent Commissioner to forfeit the bond
filed by petitioners Chan Sau Wah and Fu Yan Fun in the amount of P4,000.00.

Here is petitioners' posture. They enjoyed their stay in the Philippines upon a bond. Now they come to court and
say that as the prescribed form of this bond was not expressly approved by the Secretary of Justice in
accordance with Section 3 of Commonwealth Act 613, which reads —

SEC. 3. . . . He [Commissioner of Immigration] shall issue, subject to the approval of the Department Head, such
rules and regulations and prescribes such forms of bond, reports, and other papers, and shall issue from time to
time such instruction, not inconsistent with law, as he shall deem best calculated to carry out the provisions of the
immigration laws. . . .

that bond is void.

Reasons there are which prevent us from giving our imprimatur to this argument.
The provision requiring official approval of a bond is merely directory. "Irregularity or entire failure in this respect
does not affect the validity of the bond. 16 The reason for the rule, is found in 9 C.J., p. 26 (footnote), which reads:

(a) Reason for rule. — "Statutes requiring bonds to be approved by certain officials are not for the purpose of
protecting the obligors in the bond, but are aimed to protect the public, to insure their solvency, and to create
evidence of an unimpeachable character of the fact of their execution. When they are executed for a legal
purpose, before a proper tribunal, and are in fact accepted and approved by the officer or body, whose duty it was
to approve them, it could serve no useful purpose of the law to hold them invalid, to release all the obligors
thereon, and to defeat every purpose of its execution, simply because the fact of approval was not indorsed
precisely as had been directed by the Legislature." American Book Co. vs. Wells, 83 SW 622, 627, 26 Ky L-1159.
(emphasis supplied)

And another. This bond was accepted by the government. It had been there. The form of the bond here used is of
long continued usage. If the government did not question the form of the bond at all, then we must assume that it
counted with the Secretary's approval. For the presumption is that official duty has been legally performed.

Surely enough, equitable considerations will stop petitioners from pleading invalidity of the bond. They offered
that bond to enable them to enter and stay in this country. They enjoyed benefits therefrom. They cannot, "in law,
and good conscience, be allowed to reap the fruits" of that bond, and then jettison the same. They are "precluded
from attacking the validity" of such bond. 17

Actually, to petitioners the bond was good while they sought entry into the Philippines; they offered it as security
for the undertaking; that they "will actually depart from the Philippines" when their term of stay expires. Now that
the bond is being confiscated because they overstayed, they make an about-face and say that such bond is null
and void. They shall not profit from this inconsistent position. Their bond should be confiscated.

Conformably to the foregoing, the judgment under review is hereby modified as follows:

(1) The portion thereof which reads:

(a) Granting their petition for Mandamus and Prohibition with respect to petitioner CHAN SAU WAH, who is
hereby declared a citizen of the Philippines; ordering the respondent to cancel her Alien Certificate of Registration
and other immigration papers, upon the payment of proper dues; and declaring preliminary injunction with respect
to her permanent, prohibiting the respondent, his representatives or subordinates from arresting and/or deporting
said petitioner;

is hereby reversed: and, in consequence —

The petition for mandamus and prohibition with respect to petitioner Chan Sau Wah is hereby denied; and the
judgment declaring her a citizen of the Philippines, directing respondent to cancel her Alien Certificate of
Registration and other immigration papers, and declaring the preliminary injunction with respect to her permanent,
are all hereby set aside; and

(2) In all other respects, the decision appealed from is hereby affirmed.

No costs. So ordered.
G.R. No. 82544 June 28, 1988

IN THE MATTER OF THE PETITION FOR HABEAS CORPUS OF: ANDREW HARVEY, JOHN SHERMAN and
ADRIAAN VAN DEL ELSHOUT, petitioners,
vs.
HONORABLE COMMISSIONER MIRIAM DEFENSOR SANTIAGO, COMMISSION ON IMMIGRATION AND
DEPORTATION, respondent.

MELENCIO-HERRERA, J.:

A petition for Habeas Corpus.

Petitioners Andrew Harvey and John Sherman, 52 and 72 years, respectively, are both American nationals
residing at Pagsanjan, Laguna, while Adriaan Van Elshout, 58 years old, is a Dutch citizen also residing at
Pagsanjan, Laguna.

The case stems from the apprehension of petitioners on 27 February 1988 from their respective residences by
agents of the Commission on Immigration and Deportation (CID) by virtue of Mission Orders issued by
respondent Commissioner Miriam Defensor Santiago of the CID. Petitioners are presently detained at the CID
Detention Center.

Petitioners were among the twenty-two (22) suspected alien pedophiles who were apprehended after three
months of close surveillance by CID agents in Pagsanjan, Laguna. Two (2) days after apprehension, or on 29
February 1988, seventeen (17) of the twenty-two (22) arrested aliens opted for self-deportation and have left the
country. One was released for lack of evidence; another was charged not for being a pedophile but for working
without a valid working visa. Thus, of the original twenty two (22), only the three petitioners have chosen to face
deportation.

Seized during petitioners apprehension were rolls of photo negatives and photos of the suspected child
prostitutes shown in salacious poses as well as boys and girls engaged in the sex act. There were also posters
and other literature advertising the child prostitutes.

The "Operation Report," on Andrew Harvey and Richard Sherman dated 29 February 1988 stated:

xxx xxx xxx

ANDREW MARK HARVEY was found together with two young boys.
RICHARD SHERMAN was found with two naked boys inside his room.

In respect of Van Den Elshout the "After Mission Report," dated 27 February 1988 read in part:

Noted:

There were two (2) children ages 14 & 16 which subject readily accepted having been in his care and live-in for
quite sometime.

On 4 March 1988, deportation proceedings were instituted against petitioners for being undesirable aliens under
Section 69 of the Revised Administrative Code (Deportation Case No. 88-13). The "Charge Sheet" read inter alia:

Wherefore, this Office charges the respondents for deportation, as undesirable aliens, in that: they, being
pedophiles, are inimical to public morals, public health and public safety as provided in Section 69 of the Revised
Administrative Code.

On 7 March 1988, Warrants of Arrest were issued by respondent against petitioners for violation of Sections 37,
45 and 46 of the Immigration Act and Section 69 of the Revised Administrative Code On the same date, the
Board of Special Inquiry III commenced trial against petitioners.

On 14 March 1988, petitioners filed an Urgent Petition for Release Under Bond alleging that their health was
being seriously affected by their continuous detention. Upon recommendation of the Board of Commissioners for
their provisional release, respondent ordered the CID doctor to examine petitioners, who certified that petitioners
were healthy.

On 22 March 1988, petitioners filed a Petition for Bail which, however, respondent denied considering the
certification by the CID physician that petitioners were healthy. To avoid congestion, respondent ordered
petitioners' transfer to the CID detention cell at Fort Bonifacio, but the transfer was deferred pending trial due to
the difficulty of transporting them to and from the CID where trial was on-going.

On 4 April 1988 petitioner Andrew Harvey filed a Manifestation/Motion stating that he had "finally agreed to a
self-deportation" and praying that he be "provisionally released for at least 15 days and placed under the custody
of Atty. Asinas before he voluntarily departs the country." On 7 April 1988, the Board of Special Inquiry — III
allowed provisional release of five (5) days only under certain conditions. However, it appears that on the same
date that the aforesaid Manifestation/ Motion was filed, Harvey and his co-petitioners had already filed the
present petition.

On 4 April 1988, as heretofore stated, petitioners availed of this Petition for a Writ of Habeas Corpus. A Return of
the Writ was filed by the Solicitor General and the Court heard the case on oral argument on 20 April 1988. A
Traverse to the Writ was presented by petitioners to which a Reply was filed by the Solicitor General.

Petitioners question the validity of their detention on the following grounds:

1) There is no provision in the Philippine Immigration Act of 1940 nor under Section 69 of the Revised
Administrative Code, which legally clothes the Commissioner with any authority to arrest and detain petitioners
pending determination of the existence of a probable cause leading to an administrative investigation.

2) Respondent violated Section 2, Article III of the 1987 Constitution prohibiting unreasonable searches and
seizures since the CID agents were not clothed with valid Warrants of arrest, search and seizure as required by
the said provision.

3) Mere confidential information made to the CID agents and their suspicion of the activities of petitioners that
they are pedophiles, coupled with their association with other suspected pedophiles, are not valid legal grounds
for their arrest and detention unless they are caught in the act. They further allege that being a pedophile is not
punishable by any Philippine Law nor is it a crime to be a pedophile.

We reject petitioners' contentions and uphold respondent's official acts ably defended by the Solicitor General.
There can be no question that the right against unreasonable searches and seizures guaranteed by Article III,
Section 2 of the 1987 Constitution, is available to all persons, including aliens, whether accused of crime or not
(Moncado vs. People's Court, 80 Phil. 1 [1948]. One of the constitutional requirements of a valid search warrant
or warrant of arrest is that it must be based upon probable cause. Probable cause has been defined as referring
to "such facts and circumstances antecedent to the issuance of the warrant that in themselves are sufficient to
induce a cautious man to rely on them and act in pursuance thereof." (People vs. Syjuco 64 Phil. 667 [1937];
Alverez vs. CFI, 64 Phil. 33 [1937]).

The 1985 Rules on Criminal Procedure also provide that an arrest wit a warrant may be effected by a peace
officer or even a private person (1) when such person has committed, actually committing, or is attempting to
commit an offense in his presence; and (2) when an offense has, in fact, been committed and he has personal
knowledge of facts indicating that the person to be arrested has committed it (Rule 113, Section 5).

In this case, the arrest of petitioners was based on probable cause determined after close surveillance for three (3)
months during which period their activities were monitored. The existence of probable cause justified the arrest
and the seizure of the photo negatives, photographs and posters without warrant (See Papa vs. Mago, L-27360,
February 28, 1968,22 SCRA 857; People vs. Court of First Instance of Rizal, L-41686, November 17, 1980, 101
SCRA 86, cited in CRUZ, Constitutional Law, 1987 ed., p. 143). Those articles were seized as an incident to a
lawful arrest and, are therefore, admissible in evidence (Section 12, Rule 126,1985 Rules on criminal Procedure).

But even assuming arguendo that the arrest of petitioners was not valid at its inception, the records show that
formal deportation charges have been filed against them, as undesirable aliens, on 4 March 1988. Warrants of
arrest were issued against them on 7 March 1988 "for violation of Section 37, 45 and 46 of the Immigration Act
and Section 69 of the Administrative Code." A hearing is presently being conducted by a Board of Special Inquiry.
The restraint against their persons, therefore, has become legal. The Writ has served its purpose. The process of
the law is being followed (Cruz vs. Montoya, L-39823, February 25, 1975, 62 SCRA 543). "were a person's
detention was later made by virtue of a judicial order in relation to criminal cases subsequently filed against the
detainee, his petition for hebeas corpus becomes moot and academic" (Beltran vs. Garcia, L-49014, April 30,
1979, 89 SCRA 717). "It is a fumdamental rule that a writ of habeas corpus will not be granted when the
confinement is or has become legal, although such confinement was illegal at the beginning" (Matsura vs.
Director of Prisons, 77 Phil. 1050 [1947]).

That petitioners were not "caught in the act" does not make their arrest illegal. Petitioners were found with young
boys in their respective rooms, the ones with John Sherman being naked. Under those circumstances the CID
agents had reasonable grounds to believe that petitioners had committed "pedophilia" defined as "psychosexual
perversion involving children" (Kraft-Ebbing Psychopatia Sexualis p. 555; Paraphilia (or unusual sexual activity) in
which children are the preferred sexual object" (Webster's Third New International Dictionary, 1971 ed., p. 1665)
[Solicitor General's Return of the Writ, on p. 101. While not a crime under the Revised Penal Code, it is behavior
offensive to public morals and violative of the declared policy of the State to promote and protect the physical,
moral, spiritual, and social well-being of our youth (Article II, Section 13, 1987 Constitution).

At any rate, the filing by petitioners of a petition to be released on bail should be considered as a waiver of any
irregularity attending their arrest and estops them from questioning its validity (Callanta v. Villanueva, L-24646 &
L-24674, June 20, 1977, 77 SCRA 377; Bagcal vs. Villaraza, L-61770, January 31, 1983, 120 SCRA 525).

The deportation charges instituted by respondent Commissioner are in accordance with Section 37(a) of the
Philippine Immigration Act of 1940, in relation to Section 69 of the Revised Administrative Code. Section 37(a)
provides in part:

(a) The following aliens shall be arrested upon the warrant of the Commissioner of Immigration and Deportation
or any other officer designated by him for the purpose and deported upon the warrant of the Commissioner of
Immigration and Deportation after a determination by the Board of Commissioners of the existence of the ground
for deportation as charged against the alien;

xxx xxx xxx

The foregoing provision should be construed in its entirety in view of the summary and indivisible nature of a
deportation proceeding, otherwise, the very purpose of deportation proceeding would be defeated.
Section 37(a) is not constitutionally proscribed (Morano vs. Vivo, L-22196, June 30, 1967, 20 SCRA 562). The
specific constraints in both the 1935 and 1987 Constitutions, which are substantially Identical, contemplate
1 2

prosecutions essentially criminal in nature. Deportation proceedings, on the other hand, are administrative in
character. An order of deportation is never construed as a punishment. It is preventive, not a penal process. It
need not be conducted strictly in accordance with ordinary Court proceedings.

It is of course well-settled that deportation proceedings do not constitute a criminal action. The order of
deportation is not a punishment, (Maliler vs. Eby, 264 U.S., 32), it being merely the return to his country of an
alien who has broken the conditions upon which he could continue to reside within our borders (U.S. vs. De los
Santos, 33 Phil., 397). The deportation proceedings are administrative in character, (Kessler vs. Stracker 307
U.S., 22) summary in nature, and need not be conducted strictly in accordance with the ordinary court
proceedings (Murdock vs. Clark, 53 F. [2d], 155). It is essential, however, that the warrant of arrest shall give the
alien sufficient information about the charges against him, relating the facts relied upon. (U.S. vs. Uhl 211 F., 628.)
It is also essential that he be given a fair hearing with the assistance of counsel, if he so desires, before
unprejudiced investigators (Strench vs. Pedaris, 55 F. [2d], 597; Ex parte Jew You On, 16 F. [2d], 153). However,
all the strict rules of evidence governing judicial controversies do not need to be observed; only such as are
fumdamental and essential like the right of cross-examination. (U.S. vs. Hughes, 104 F. [2d], 14; Murdock vs.
Clark, 53 F. [2d], 155.) Hearsay evidence may even be admitted, provided the alien is given the opportunity to
explain or rebut it (Morrell vs. Baker, 270 F., 577; Sercerchi vs. Ward, 27 F. Supp., 437). (Lao Tang Bun vs. Fabre
81 Phil. 682 [1948]).

The ruling in Vivo vs. Montesa (G. R. No. 24576, July 29, 1968, 24 SCRA 155) that "the issuance of warrants of
arrest by the Commissioner of Immigration, solely for purposes of investigation and before a final order of
deportation is issued, conflicts with paragraph 3, Section I of Article III of the Constitution" (referring to the 1935
Constitution) is not invocable herein. Respondent Commissioner's Warrant of Arrest issued on 7 March 1988 did
3

not order petitioners to appear and show cause why they should not be deported. They were issued specifically
"for violation of Sections 37, 45 and 46 of the Immigration Act and Section 69 of the Revised Administrative
Code." Before that, deportation proceedings had been commenced against them as undesirable aliens on 4
March 1988 and the arrest was a step preliminary to their possible deportation.

Section 37 of the Immigration Law, which empowers the Commissioner of Immigration to issue warrants for the
arrest of overstaying aliens is constitutional. The arrest is a stop preliminary to the deportation of the aliens who
had violated the condition of their stay in this country. (Morano vs. Vivo, L-22196, June 30, 1967, 20 SCRA 562).

To rule otherwise would be to render the authority given the Commissioner nugatory to the detriment of the State.

The pertinent provision of Commonwealth Act No. 613, as amended, which gives authority to the Commissioner
of Immigration to order the arrest of an alien temporary visitor preparatory to his deportation for failure to put up
new bonds required for the stay, is not unconstitutional.

xxx xxx xxx

... Such a step is necessary to enable the Commissioner to prepare the ground for his deportation under Section
37[al of Commonwealth Act 613. A contrary interpretation would render such power nugatory to the detriment of
the State. (Ng Hua To vs. Galang, G. R. No. 10145, February 29, 1964, 10 SCRA 411).

"The requirement of probable cause, to be determined by a Judge, does not extend to deportation proceedings."
(Morano vs. Vivo, supra, citing Tiu Chun Hai vs. Commissioner, infra). There need be no "truncated" recourse to
both judicial and administrative warrants in a single deportation proceedings.

The foregoing does not deviate from the ruling in Qua Chee Gan vs. Deportation Board (G. R. No. 10280,
September 30, 1963, 9 SCRA 27 [1963]) reiterated in Vivo vs. Montesa, supra, that "under the express terms of
our Constitution (the 1935 Constitution), it is therefore even doubtful whether the arrest of an individual may be
ordered by any authority other than a judge if the purpose is merely to determine the existence of a probable
cause, leading to an administrative investigation." For, as heretofore stated, probable cause had already been
shown to exist before the warrants of arrest were issued.
What is essential is that there should be a specific charge against the alien intended to be arrested and deported,
that a fair hearing be conducted (Section 37[c]) with the assistance of counsel, if desired, and that the charge be
substantiated by competent evidence. Thus, Section 69 of the Revised Administrative Code explicitly provides:

Sec. 69. Deportation of subject of foreign power. A subject of a foreign power residing in the Philippines shall not
be deported, expelled, or excluded from said Islands or repatriated to his own country by the President of the
Philippines except upon prior investigation, conducted by said Executive or his authorized agent, of the ground
upon which such action is contemplated. In such a case the person concerned shall be informed of the charge or
charges against him and he shall be allowed not less than 3 days for the preparation of his defense. He shall also
have the right to be heard by himself or counsel, to produce witnesses in his own behalf, and to cross-examine
the opposing witnesses.

The denial by respondent Commissioner of petitioners' release on bail, also challenged by them, was in order
because in deportation proceedings, the right to bail is not a matter of right but a matter of discretion on the part of
the Commissioner of Immigration and Deportation. Thus, Section 37(e) of the Philippine Immigration Act of 1940
provides that "any alien under arrest in a deportation proceeding may be released under bond or under such
other conditions as may be imposed by the Commissioner of Immigration." The use of the word "may" in said
provision indicates that the grant of bail is merely permissive and not mandatory on the part of the Commissioner.
The exercise of the power is wholly discretionary (Ong Hee Sang vs. Commissioner of Immigration, L-9700,
February 28,1962, 4 SCRA 442). "Neither the Constitution nor Section 69 of the Revised Administrative Code
guarantees the right of aliens facing deportation to provisional liberty on bail." (Tiu Chun Hai et al vs. Deportation
Board, 104 Phil. 949 [1958]). As deportation proceedings do not partake of the nature of a criminal action, the
constitutional guarantee to bail may not be invoked by aliens in said proceedings (Ong Hee Sang vs.
Commissioner of Immigration, supra).

Every sovereign power has the inherent power to exclude aliens from its territory upon such grounds as it may
deem proper for its self-preservation or public interest (Lao Tan Bun vs. Fabre 81 Phil. 682 [1948]). The power to
deport aliens is an act of State, an act done by or under the authority of the sovereign power (In re McCulloch
Dick, 38 Phil. 41 [1918]). It is a police measure against undesirable aliens whose continued presence in the
country is found to be injurious to the public good and the domestic tranquility of the people (Forbes vs. Chuoco
Tiaco et al., 16 Phil. 534 [1910]). Particularly so in this case where the State has expressly committed itself to
defend the tight of children to assistance and special protection from all forms of neglect, abuse, cruelty,
exploitation, and other conditions prejudicial to their development (Article XV, Section 3[2]). Respondent
Commissioner of Immigration and Deportation, in instituting deportation proceedings against petitioners, acted in
the interests of the State.

WHEREFORE, the Petition is dismissed and the Writ of Habeas Corpus is hereby denied.

SO ORDERED.
G.R. No. 81510 March 14, 1990

HORTENCIA SALAZAR, petitioner,


vs.
HON. TOMAS D. ACHACOSO, in his capacity as Administrator of the Philippine Overseas Employment
Administration, and FERDIE MARQUEZ, respondents.

Gutierrez & Alo Law Offices for petitioner.

SARMIENTO, J.:

This concerns the validity of the power of the Secretary of Labor to issue warrants of arrest and seizure under
Article 38 of the Labor Code, prohibiting illegal recruitment.

The facts are as follows:

xxx xxx xxx

1. On October 21, 1987, Rosalie Tesoro of 177 Tupaz Street, Leveriza, Pasay City, in a sworn statement filed
with the Philippine Overseas Employment Administration (POEA for brevity) charged petitioner Hortencia
Salazar, viz:
04. T: Ano ba ang dahilan at ikaw ngayon ay narito at
nagbibigay ng salaysay.

S: Upang ireklamo sa dahilan ang aking PECC Card ay


ayaw ibigay sa akin ng dati kong manager. — Horty
Salazar — 615 R.O. Santos, Mandaluyong, Mla.

05. T: Kailan at saan naganap and ginawang panloloko sa


iyo ng tao/mga taong inireklamo mo?

S. Sa bahay ni Horty Salazar.

06. T: Paano naman naganap ang pangyayari?

S. Pagkagaling ko sa Japan ipinatawag niya ako. Kinuha


ang PECC Card ko at sinabing hahanapan ako ng
booking sa Japan. Mag 9 month's na ako sa Phils. ay
hindi pa niya ako napa-alis. So lumipat ako ng ibang
company pero ayaw niyang ibigay and PECC Card
ko.

2. On November 3, 1987, public respondent Atty. Ferdinand Marquez to whom said complaint was assigned, sent
to the petitioner the following telegram:

YOU ARE HEREBY DIRECTED TO APPEAR BEFORE FERDIE MARQUEZ POEA ANTI ILLEGAL
RECRUITMENT UNIT 6TH FLR. POEA BLDG. EDSA COR. ORTIGAS AVE. MANDALUYONG MM ON
NOVEMBER 6, 1987 AT 10 AM RE CASE FILED AGAINST YOU. FAIL NOT UNDER PENALTY OF LAW.

4. On the same day, having ascertained that the petitioner had no license to operate a recruitment agency, public
respondent Administrator Tomas D. Achacoso issued his challenged CLOSURE AND SEIZURE ORDER NO.
1205 which reads:

HORTY SALAZAR
No. 615 R.O. Santos St.
Mandaluyong, Metro Manila

Pursuant to the powers vested in me under Presidential Decree No. 1920 and Executive Order No. 1022, I hereby
order the CLOSURE of your recruitment agency being operated at No. 615 R.O. Santos St., Mandaluyong, Metro
Manila and the seizure of the documents and paraphernalia being used or intended to be used as the means of
committing illegal recruitment, it having verified that you have —

(1) No valid license or authority from the Department of Labor and Employment to recruit and deploy workers for
overseas employment;

(2) Committed/are committing acts prohibited under Article 34 of the New Labor Code in relation to Article 38 of
the same code.

This ORDER is without prejudice to your criminal prosecution under existing laws.

Done in the City of Manila, this 3th day of November, 1987.

5. On January 26, 1988 POEA Director on Licensing and Regulation Atty. Estelita B. Espiritu issued an office
order designating respondents Atty. Marquez, Atty. Jovencio Abara and Atty. Ernesto Vistro as members of a
team tasked to implement Closure and Seizure Order No. 1205. Doing so, the group assisted by Mandaluyong
policemen and mediamen Lito Castillo of the People's Journal and Ernie Baluyot of News Today proceeded to the
residence of the petitioner at 615 R.O. Santos St., Mandaluyong, Metro Manila. There it was found that petitioner
was operating Hannalie Dance Studio. Before entering the place, the team served said Closure and Seizure order
on a certain Mrs. Flora Salazar who voluntarily allowed them entry into the premises. Mrs. Flora Salazar informed
the team that Hannalie Dance Studio was accredited with Moreman Development (Phil.). However, when
required to show credentials, she was unable to produce any. Inside the studio, the team chanced upon twelve
talent performers — practicing a dance number and saw about twenty more waiting outside, The team
confiscated assorted costumes which were duly receipted for by Mrs. Asuncion Maguelan and witnessed by Mrs.
Flora Salazar.

6. On January 28, 1988, petitioner filed with POEA the following letter:

Gentlemen:

On behalf of Ms. Horty Salazar of 615 R.O. Santos, Mandaluyong, Metro Manila, we respectfully request that the
personal properties seized at her residence last January 26, 1988 be immediately returned on the ground that
said seizure was contrary to law and against the will of the owner thereof. Among our reasons are the following:

1. Our client has not been given any prior notice or hearing, hence the Closure and Seizure Order No. 1205 dated
November 3, 1987 violates "due process of law" guaranteed under Sec. 1, Art. III, of the Philippine Constitution.

2. Your acts also violate Sec. 2, Art. III of the Philippine Constitution which guarantees right of the people "to be
secure in their persons, houses, papers, and effects against unreasonable searches and seizures of whatever
nature and for any purpose."

3. The premises invaded by your Mr. Ferdi Marquez and five (5) others (including 2 policemen) are the private
residence of the Salazar family, and the entry, search as well as the seizure of the personal properties belonging
to our client were without her consent and were done with unreasonable force and intimidation, together with
grave abuse of the color of authority, and constitute robbery and violation of domicile under Arts. 293 and 128 of
the Revised Penal Code.

Unless said personal properties worth around TEN THOUSAND PESOS (P10,000.00) in all (and which were
already due for shipment to Japan) are returned within twenty-four (24) hours from your receipt hereof, we shall
feel free to take all legal action, civil and criminal, to protect our client's interests.

We trust that you will give due attention to these important matters.

7. On February 2, 1988, before POEA could answer the letter, petitioner filed the instant petition; on even date,
POEA filed a criminal complaint against her with the Pasig Provincial Fiscal, docketed as IS-88-836. 1

On February 2, 1988, the petitioner filed this suit for prohibition. Although the acts sought to be barred are
alreadyfait accompli, thereby making prohibition too late, we consider the petition as one for certiorari in view of
the grave public interest involved.

The Court finds that a lone issue confronts it: May the Philippine Overseas Employment Administration (or the
Secretary of Labor) validly issue warrants of search and seizure (or arrest) under Article 38 of the Labor Code? It
is also an issue squarely raised by the petitioner for the Court's resolution.

Under the new Constitution, which states:

. . . no search warrant or warrant of arrest shall issue except upon probable cause to be determined personally by
the judge after examination under oath or affirmation of the complainant and the witnesses he may produce, and
particularly describing the place to be searched and the persons or things to be seized. 2

it is only a judge who may issue warrants of search and arrest. In one case, it was declared that mayors may not
3

exercise this power:

xxx xxx xxx

But it must be emphasized here and now that what has just been described is the state of the law as it was in
September, 1985. The law has since been altered. No longer does the mayor have at this time the power to
conduct preliminary investigations, much less issue orders of arrest. Section 143 of the Local Government Code,
conferring this power on the mayor has been abrogated, rendered functus officio by the 1987 Constitution which
took effect on February 2, 1987, the date of its ratification by the Filipino people. Section 2, Article III of the 1987
Constitution pertinently provides that "no search warrant or warrant of arrest shall issue except upon probable
cause to be determined personally by the judge after examination under oath or affirmation of the complainant
and the witnesses he may produce, and particularly describing the place to be searched and the person or things
to be seized." The constitutional proscription has thereby been manifested that thenceforth, the function of
determining probable cause and issuing, on the basis thereof, warrants of arrest or search warrants, may be
validly exercised only by judges, this being evidenced by the elimination in the present Constitution of the phrase,
"such other responsible officer as may be authorized by law" found in the counterpart provision of said 1973
Constitution, who, aside from judges, might conduct preliminary investigations and issue warrants of arrest or
search warrants. 4

Neither may it be done by a mere prosecuting body:

We agree that the Presidential Anti-Dollar Salting Task Force exercises, or was meant to exercise, prosecutorial
powers, and on that ground, it cannot be said to be a neutral and detached "judge" to determine the existence of
probable cause for purposes of arrest or search. Unlike a magistrate, a prosecutor is naturally interested in the
success of his case. Although his office "is to see that justice is done and not necessarily to secure the conviction
of the person accused," he stands, invariably, as the accused's adversary and his accuser. To permit him to issue
search warrants and indeed, warrants of arrest, is to make him both judge and jury in his own right, when he is
neither. That makes, to our mind and to that extent, Presidential Decree No. 1936 as amended by Presidential
Decree No. 2002, unconstitutional. 5

Section 38, paragraph (c), of the Labor Code, as now written, was entered as an amendment by Presidential
Decrees Nos. 1920 and 2018 of the late President Ferdinand Marcos, to Presidential Decree No. 1693, in the
exercise of his legislative powers under Amendment No. 6 of the 1973 Constitution. Under the latter, the then
Minister of Labor merely exercised recommendatory powers:

(c) The Minister of Labor or his duly authorized representative shall have the power to recommend the arrest and
detention of any person engaged in illegal recruitment. 6

On May 1, 1984, Mr. Marcos promulgated Presidential Decree No. 1920, with the avowed purpose of giving more
teeth to the campaign against illegal recruitment. The Decree gave the Minister of Labor arrest and closure
powers:

(b) The Minister of Labor and Employment shall have the power to cause the arrest and detention of such
non-licensee or non-holder of authority if after proper investigation it is determined that his activities constitute a
danger to national security and public order or will lead to further exploitation of job-seekers. The Minister shall
order the closure of companies, establishment and entities found to be engaged in the recruitment of workers for
overseas employment, without having been licensed or authorized to do so. 7

On January 26, 1986, he, Mr. Marcos, promulgated Presidential Decree No. 2018, giving the Labor Minister
search and seizure powers as well:

(c) The Minister of Labor and Employment or his duly authorized representatives shall have the power to cause
the arrest and detention of such non-licensee or non-holder of authority if after investigation it is determined that
his activities constitute a danger to national security and public order or will lead to further exploitation of
job-seekers. The Minister shall order the search of the office or premises and seizure of documents,
paraphernalia, properties and other implements used in illegal recruitment activities and the closure of companies,
establishment and entities found to be engaged in the recruitment of workers for overseas employment, without
having been licensed or authorized to do so. 8

The above has now been etched as Article 38, paragraph (c) of the Labor Code.

The decrees in question, it is well to note, stand as the dying vestiges of authoritarian rule in its twilight moments.
We reiterate that the Secretary of Labor, not being a judge, may no longer issue search or arrest warrants. Hence,
the authorities must go through the judicial process. To that extent, we declare Article 38, paragraph (c), of the
Labor Code, unconstitutional and of no force and effect.

The Solicitor General's reliance on the case of Morano v. Vivo is not well-taken. Vivo involved a deportation
9

case, governed by Section 69 of the defunct Revised Administrative Code and by Section 37 of the Immigration
Law. We have ruled that in deportation cases, an arrest (of an undesirable alien) ordered by the President or his
duly authorized representatives, in order to carry out a final decision of deportation is valid. It is valid, however,
10

because of the recognized supremacy of the Executive in matters involving foreign affairs. We have held: 11

xxx xxx xxx

The State has the inherent power to deport undesirable aliens (Chuoco Tiaco vs. Forbes, 228 U.S. 549, 57 L. Ed.
960, 40 Phil. 1122, 1125). That power may be exercised by the Chief Executive "when he deems such action
necessary for the peace and domestic tranquility of the nation." Justice Johnson's opinion is that when the Chief
Executive finds that there are aliens whose continued presence in the country is injurious to the public interest,
"he may, even in the absence of express law, deport them". (Forbes vs. Chuoco Tiaco and Crossfield, 16 Phil.
534, 568, 569; In re McCulloch Dick, 38 Phil. 41).

The right of a country to expel or deport aliens because their continued presence is detrimental to public welfare
is absolute and unqualified (Tiu Chun Hai and Go Tam vs. Commissioner of Immigration and the Director of NBI,
104 Phil. 949, 956). 12

The power of the President to order the arrest of aliens for deportation is, obviously, exceptional. It (the power to
order arrests) can not be made to extend to other cases, like the one at bar. Under the Constitution, it is the sole
domain of the courts.

Moreover, the search and seizure order in question, assuming, ex gratia argumenti, that it was validly issued, is
clearly in the nature of a general warrant:

Pursuant to the powers vested in me under Presidential Decree No. 1920 and Executive Order No. 1022, I hereby
order the CLOSURE of your recruitment agency being operated at No. 615 R.O. Santos St., Mandaluyong, Metro
Manila and the seizure of the documents and paraphernalia being used or intended to be used as the means of
committing illegal recruitment, it having verified that you have —

(1) No valid license or authority from the Department of Labor and Employment to recruit and deploy workers for
overseas employment;

(2) Committed/are committing acts prohibited under Article 34 of the New Labor Code in relation to Article 38 of
the same code.

This ORDER is without prejudice to your criminal prosecution under existing laws. 13

We have held that a warrant must identify clearly the things to be seized, otherwise, it is null and void, thus:

xxx xxx xxx

Another factor which makes the search warrants under consideration constitutionally objectionable is that they
are in the nature of general warrants. The search warrants describe the articles sought to be seized in this wise:

1) All printing equipment, paraphernalia, paper, ink, photo equipment, typewriters, cabinets, tables,
communications/ recording equipment, tape recorders, dictaphone and the like used and/or connected in the
printing of the "WE FORUM" newspaper and any and all documents/communications, letters and facsimile of
prints related to the "WE FORUM" newspaper.

2) Subversive documents, pamphlets, leaflets, books, and other publications to promote the objectives and
purposes of the subversive organizations known as Movement for Free Philippines, Light-a-Fire Movement and
April 6 Movement; and
3) Motor vehicles used in the distribution/circulation of the "WE FORUM" and other subversive materials and
propaganda, more particularly,

1) Toyota-Corolla, colored yellow with Plate No. NKA 892;

2) DATSUN, pick-up colored white with Plate No. NKV 969;

3) A delivery truck with Plate No. NBS 542;

4) TOYOTA-TAMARAW, colored white with Plate No. PBP 665; and

5) TOYOTA Hi-Lux, pick-up truck with Plate No. NGV 472 with marking "Bagong Silang."

In Stanford v. State of Texas, the search warrant which authorized the search for "books, records, pamphlets,
cards, receipts, lists, memoranda, pictures, recordings and other written instruments concerning the Communist
Parties of Texas, and the operations of the Community Party in Texas," was declared void by the U.S. Supreme
Court for being too general. In like manner, directions to "seize any evidence in connection with the violation of
SDC 13-3703 or otherwise" have been held too general, and that portion of a search warrant which authorized the
seizure of any "paraphernalia which could be used to violate Sec. 54-197 of the Connecticut General Statutes
(the statute dealing with the crime of conspiracy)" was held to be a general warrant, and therefore invalid. The
description of the articles sought to be seized under the search warrants in question cannot be characterized
differently.

In the Stanford case, the U.S. Supreme court calls to mind a notable chapter in English history; the era of
disaccord between the Tudor Government and the English Press, when "Officers of the Crown were given roving
commissions to search where they pleased in order to suppress and destroy the literature of dissent both Catholic
and Puritan." Reference herein to such historical episode would not be relevant for it is not the policy of our
government to suppress any newspaper or publication that speaks with "the voice of non-conformity" but poses
no clear and imminent danger to state security. 14

For the guidance of the bench and the bar, we reaffirm the following principles:

1. Under Article III, Section 2, of the l987 Constitution, it is only judges, and no other, who may issue warrants of
arrest and search:

2. The exception is in cases of deportation of illegal and undesirable aliens, whom the President or the
Commissioner of Immigration may order arrested, following a final order of deportation, for the purpose of
deportation.

WHEREFORE, the petition is GRANTED. Article 38, paragraph (c) of the Labor Code is declared
UNCONSTITUTIONAL and null and void. The respondents are ORDERED to return all materials seized as a
result of the implementation of Search and Seizure Order No. 1205.

No costs.

SO ORDERED.

Fernan, C.J., Narvasa, Melencio-Herrera, Gutierrez, Jr., Cruz, Paras, Feliciano, Gancayco, Padilla, Bidin, Cortes,
Griño-Aquino, Medialdea and Regalado, JJ., concur.

G.R. No. L-45358 January 29, 1937

NARCISO ALVAREZ, petitioner,


vs.
THE COURT OF FIRST INSTANCE OF TAYABAS and THE ANTI-USURY BOARD, respondents.
Godofredo Reyes for petitioner.
Adolfo N. Feliciano for respondents Anti-Usury Board.
No appearance for other respondent.

IMPERIAL, J.:

The petitioner asks that the warrant of June 3, 1936, issued by the Court of First Instance of Tayabas, ordering
the search of his house and the seizure, at any time of the day or night, of certain accounting books, documents
and papers belonging to him in his residence situated in Infanta, Province of Tayabas, as well as the order of a
later date, authorizing the agents of the Anti-Usury Board to retain the articles seized, be declared illegal and set
aside, and prays that all the articles in question be returned to him.

On the date above-mentioned, the chief of the secret service of the Anti-Usury Board, of the Department of
Justice, presented to Judge Eduardo Gutierrez David then presiding over the Court of First Instance of Tayabas,
an affidavit alleging that according to reliable information, the petitioner kept in his house in Infanta, Tayabas,
books, documents, receipts, lists, chits and other papers used by him in connection with his activities as a
money-lender charging usurious rates of interest in violation of the law. In his oath at the and of the affidavit, the
chief of the secret service stated that his answers to the questions were correct to the best of his knowledge and
belief. He did not swear to the truth of his statements upon his own knowledge of the facts but upon the
information received by him from a reliable person. Upon the affidavit in question the Judge, on said date, issued
the warrant which is the subject matter of the petition, ordering the search of the petitioner's house at nay time of
the day or night, the seizure of the books and documents above-mentioned and the immediate delivery thereof to
him to be disposed of in accordance with the law. With said warrant, several agents of the Anti-Usury Board
entered the petitioner's store and residence at seven o'clock on the night of June 4, 1936, and seized and took
possession of the following articles: internal revenue licenses for the years 1933 to 1936, one ledger, two journals,
two cashbooks, nine order books, four notebooks, four checks stubs, two memorandums, three bankbooks, two
contracts, four stubs, forty-eight stubs of purchases of copra, two inventories, two bundles of bills of lading, one
bundle of credit receipts, one bundle of stubs of purchases of copra, two packages of correspondence, one
receipt book belonging to Luis Fernandez, fourteen bundles of invoices and other papers many documents and
loan contracts with security and promissory notes, 504 chits, promissory notes and stubs of used checks of the
Hongkong & Shanghai Banking Corporation. The search for and a seizure of said articles were made with the
opposition of the petitioner who stated his protest below the inventories on the ground that the agents seized
even the originals of the documents. As the articles had not been brought immediately to the judge who issued
the search warrant, the petitioner, through his attorney, filed a motion on June 8, 1936, praying that the agent
Emilio L. Siongco, or any other agent, be ordered immediately to deposit all the seized articles in the office of the
clerk of court and that said agent be declared guilty of contempt for having disobeyed the order of the court. On
said date the court issued an order directing Emilio L. Siongco to deposit all the articles seized within twenty-four
hours from the receipt of notice thereof and giving him a period of five (5) days within which to show cause why he
should not be punished for contempt of court. On June 10th, Attorney Arsenio Rodriguez, representing the
Anti-Usury Board, filed a motion praying that the order of the 8th of said month be set aside and that the
Anti-Usury Board be authorized to retain the articles seized for a period of thirty (30) days for the necessary
investigation. The attorney for the petitioner, on June 20th, filed another motion alleging that, notwithstanding the
order of the 8th of said month, the officials of the Anti-Usury Board had failed to deposit the articles seized by
them and praying that a search warrant be issued, that the sheriff be ordered to take all the articles into his
custody and deposit of the Anti-Usury Board be punished for contempt of court. Said attorney, on June 24th, filed
an ex parte petition alleging that while agent Emilio L. Siongco had deposited some documents and papers in the
office of the clerk of court, he had so far failed to file an inventory duly verified by oath of all the documents seized
by him, to return the search warrant together with the affidavit it presented in support thereof, or to present the
report of the proceedings taken by him; and prayed that said agent be directed to filed the documents in question
immediately. On the 25th of said month the court issued an order requiring agent Emilio L. Siongco forthwith to
file the search warrant and the affidavit in the court, together with the proceedings taken by him, and to present an
inventory duly verified by oath of all the articles seized. On July 2d of said year, the attorney for the petitioner filed
another petition alleging that the search warrant issue was illegal and that it had nit yet been returned to date
together with the proceedings taken in connection therewith, and praying that said warrant be cancelled, that an
order be issued directing the return of all the articles seized to the petitioner, that the agent who seized them be
declared guilty of contempt of court, and that charges be filed against him for abuse of authority. On September
10, 1936, the court issued an order holding: that the search warrant was obtained and issued in accordance with
the law, that it had been duly complied with and, consequently, should not be cancelled, and that agent Emilio L.
Siongco did not commit any contempt of court and must, therefore, be exonerated, and ordering the chief of the
Anti-Usury Board in Manila to show case, if any, within the unextendible period of two (2) days from the date of
notice of said order, why all the articles seized appearing in the inventory, Exhibit 1, should not be returned to the
petitioner. The assistant chief of the Anti-Usury Board of the Department of Justice filed a motion praying, for the
reasons stated therein, that the articles seized be ordered retained for the purpose of conducting an investigation
of the violation of the Anti-Usury Law committed by the petitioner. In view of the opposition of the attorney for the
petitioner, the court, on September 25th, issued an order requiring the Anti-Usury Board to specify the time
needed by it to examine the documents and papers seized and which of them should be retained, granting it a
period of five (5) days for said purpose. On the 30th of said month the assistant chief of the Anti-Usury Board filed
a motion praying that he be granted ten (10) days to comply with the order of September 25th and that the clerk of
court be ordered to return to him all the documents and papers together with the inventory thereof. The court, in
an order of October 2d of said year, granted him the additional period of ten(10) days and ordered the clerk of
court to send him a copy of the inventory. On October 10th, said official again filed another motion alleging that he
needed sixty (60) days to examine the documents and papers seized, which are designated on pages 1 to 4 of
the inventory by Nos. 5, 1016, 23, 25, 26, 27, 30, 31, 34, 36, 37, 38, 39, 40, 41, 42, 43 and 45, and praying that he
be granted said period of sixty (60) days. In an order of October 16th, the court granted him the period of sixty (60)
days to investigate said nineteen (19) documents. The petitioner alleges, and it is not denied by the respondents,
that these nineteen (19)documents continue in the possession of the court, the rest having been returned to said
petitioner.

I. A search warrant is an order in writing, issued in the name of the People of the Philippine Islands, signed by a
judge or a justice of the peace, and directed to a peace officer, commanding him to search for personal property
and bring it before the court (section 95, General Orders. No. 58, as amended by section 6 of Act No. 2886). Of all
the rights of a citizen, few are of greater importance or more essential to his peace and happiness than the right of
personal security, and that involves the exemption of his private affairs, books, and papers from the inspection
and scrutiny of others (In re Pacific Railways Commission, 32 Fed., 241; Interstate Commerce Commission vs
Brimson, 38 Law. ed., 1047; Broyd vs. U. S., 29 Law. ed., 746; Caroll vs. U. S., 69 Law. ed., 543, 549). While the
power to search and seize is necessary to the public welfare, still it must be exercised and the law enforced
without transgressing the constitutional rights or citizen, for the enforcement of no statue is of sufficient
importance to justify indifference to the basis principles of government (People vs.Elias, 147 N. E., 472).

II. As the protection of the citizen and the maintenance of his constitutional right is one of the highest duties and
privileges of the court, these constitutional guaranties should be given a liberal construction or a strict
construction in favor of the individual, to prevent stealthy encroachment upon, or gradual depreciation on, the
rights secured by them(State vs. Custer County, 198 Pac., 362; State vs. McDaniel, 231 Pac., 965; 237 Pac.,
373). Since the proceeding is a drastic one, it is the general rule that statutes authorizing searches and seizure or
search warrants must be strictly construed (Rose vs. St. Clair, 28 Fed., [2d], 189; Leonard vs. U. S., 6 Fed. [2d],
353; Perry vs. U. S. 14 Fed. [2d],88; Cofer vs. State, 118 So., 613).

III. The petitioner claims that the search warrant issued by the court is illegal because it has been based upon the
affidavit of agent Mariano G. Almeda in whose oath he declared that he had no personal knowledge of the facts
which were to serve as a basis for the issuance of the warrant but that he had knowledge thereof through mere
information secured from a person whom he considered reliable. To the question "What are your reason for
applying for this search warrant", appearing in the affidavit, the agent answered: "It has been reported to me by a
person whom I consider to be reliable that there are being kept in said premises, books, documents, receipts, lists,
chits, and other papers used by him in connection with his activities as a money-lender, charging a usurious rate
of interest, in violation of the law" and in attesting the truth of his statements contained in the affidavit, the said
agent states that he found them to be correct and true to the best of his knowledge and belief.

Section 1, paragraph 3, of Article III of the Constitution, relative to the bill of rights, provides that "The right of the
people to be secure in their persons, houses, papers, and effects against unreasonable searches and seizures
shall not be violated, and no warrants shall issue but upon probable cause, to be determined by the judge after
examination under oath or affirmation of the complainant and the witnesses he may produce, and particularly
describing the place top be searched, and the persons or things to be seized." Section 97 of General Orders, No.
58 provides that "A search warrant shall not issue except for probable cause and upon application supported by
oath particularly describing the place to be searched and the person or thing to be seized." It will be noted that
both provisions require that there be not only probable cause before the issuance of a search warrant but that the
search warrant must be based upon an application supported by oath of the applicant ands the witnesses he may
produce. In its broadest sense, an oath includes any form of attestation by which a party signifies that he is bound
in conscience to perform an act faithfully and truthfully; and it is sometimes defined asan outward pledge given by
the person taking it that his attestation or promise is made under an immediate sense of his responsibility to God
(Bouvier's Law Dictionary; State vs. Jackson, 137 N. W., 1034; In re Sage, 24 Oh. Cir. Ct. [N. S.], 7;
Pumphery vs. State, 122 N. W., 19; Priest vs. State, 6 N. W., 468; State vs. Jones, 154 Pac., 378;
Atwood vs. State, 111 So., 865). The oath required must refer to the truth of the facts within the personal
knowledge of the petitioner or his witnesses, because the purpose thereof is to convince the committing
magistrate, not the individual making the affidavit and seeking the issuance of the warrant, of the existence of
probable cause (U. S. vs. Tureaud, 20 Fed., 621; U. S. vs. Michalski, 265 Fed., 8349; U. S. vs. Pitotto, 267 Fed.,
603; U. S. vs. Lai Chew, 298 Fed., 652). The true test of sufficiency of an affidavit to warrant issuance of a search
warrant is whether it has been drawn in such a manner that perjury could be charged thereon and affiant be held
liable for damages caused (State vs. Roosevelt Country 20th Jud. Dis. Ct., 244 Pac., 280; State vs. Quartier, 236
Pac., 746).

It will likewise be noted that section 1, paragraph 3, of Article III of the Constitution prohibits unreasonable
searches and seizure. Unreasonable searches and seizures are a menace against which the constitutional
guarantee afford full protection. The term "unreasonable search and seizure" is not defined in the Constitution or
in General Orders No. 58, and it is said to have no fixed, absolute or unchangeable meaning, although the term
has been defined in general language. All illegal searches and seizure are unreasonable while lawful ones are
reasonable. What constitutes a reasonable or unreasonable search or seizure in any particular case is purely a
judicial question, determinable from a consideration of the circumstances involved, including the purpose of the
search, the presence or absence or probable cause, the manner in which the search and seizure was made, the
place or thing searched, and the character of the articles procured (Go-Bart Importing Co. vs. U. S. 75 Law. ed.,
374; Peru vs. U. S., 4 Fed., [2d], 881;U. S. vs. Vatune, 292 Fed., 497; Angelo vs. U. S. 70 Law, ed., 145;
Lambert vs. U. S. 282 Fed., 413; U. S. vs. Bateman, 278 Fed., 231; Mason vs. Rollins, 16 Fed. Cas. [No. 9252], 2
Biss., 99).

In view of the foregoing and under the above-cited authorities, it appears that the affidavit, which served as the
exclusive basis of the search warrant, is insufficient and fatally defective by reason of the manner in which the
oath was made, and therefore, it is hereby held that the search warrant in question and the subsequent seizure of
the books, documents and other papers are illegal and do not in any way warrant the deprivation to which the
petitioner was subjected.

IV. Another ground alleged by the petitioner in asking that the search warrant be declared illegal and cancelled is
that it was not supported by other affidavits aside from that made by the applicant. In other words, it is contended
that the search warrant cannot be issued unless it be supported by affidavits made by the applicant and the
witnesses to be presented necessity by him. Section 1, paragraph 3, of Article III of the Constitution provides that
no warrants shall issue but upon probable cause, to be determined by the judge after examination under oath or
affirmation of the complainant and the witnesses he may produce. Section 98 of General Orders, No. 58 provides
that the judge or justice must, before issuing the warrant, examine under oath the complainant and any witnesses
he may produce and take their depositions in writing. It is the practice in this jurisdiction to attach the affidavit of at
least the applicant or complainant to the application. It is admitted that the judge who issued the search warrant in
this case, relied exclusively upon the affidavit made by agent Mariano G. Almeda and that he did not require nor
take the deposition of any other witness. Neither the Constitution nor General Orders. No. 58 provides that it is of
imperative necessity to take the deposition of the witnesses to be presented by the applicant or complainant in
addition to the affidavit of the latter. The purpose of both in requiring the presentation of depositions is nothing
more than to satisfy the committing magistrate of the existence of probable cause. Therefore, if the affidavit of the
applicant or complainant is sufficient, the judge may dispense with that of other witnesses. Inasmuch as the
affidavit of the agent in this case was insufficient because his knowledge of the facts was not personal but merely
hearsay, it is the duty of the judge to require the affidavit of one or more witnesses for the purpose of determining
the existence of probable cause to warrant the issuance of the search warrant. When the affidavit of the applicant
of the complaint contains sufficient facts within his personal and direct knowledge, it is sufficient if the judge is
satisfied that there exist probable cause; when the applicant's knowledge of the facts is mere hearsay, the
affidavit of one or more witnesses having a personal knowledge of the fact is necessary. We conclude, therefore,
that the warrant issued is likewise illegal because it was based only on the affidavit of the agent who had no
personal knowledge of the facts.
V. The petitioner alleged as another ground for the declaration of the illegality of the search warrant and the
cancellation thereof, the fact that it authorized its execution at night. Section 101 of General Orders, No. 58
authorizes that the search be made at night when it is positively asserted in the affidavits that the property is on
the person or in the place ordered to be searched. As we have declared the affidavits insufficient and the warrant
issued exclusively upon it illegal, our conclusion is that the contention is equally well founded and that the search
could not legally be made at night.

VI. One of the grounds alleged by the petitioner in support of his contention that the warrant was issued illegally is
the lack of an adequate description of the books and documents to be seized. Section 1, paragraphs 3, of Article
III of the Constitution, and section 97 of General Orders, No. 58 provide that the affidavit to be presented, which
shall serve as the basis for determining whether probable cause exist and whether the warrant should be issued,
must contain a particular description of the place to be searched and the person or thing to be seized. These
provisions are mandatory and must be strictly complied with (Munch vs. U. S., 24 Fed. [2d], 518; U. S. vs. Boyd, 1
Fed. [2d], 1019; U. S. vs. Carlson, 292 Fed., 463; U. S. vs.Borkowski, 268 Fed., 408; In re Tri-State Coal & Coke
Co., 253 Fed., 605; People vs. Mayen, 188 Cal., 237; People vs. Kahn, 256 Ill. App., 4125); but where, by the
nature of the goods to be seized, their description must be rather generally, it is not required that a technical
description be given, as this would mean that no warrant could issue (People vs. Rubio, 57 Phil., 284;
People vs. Kahn, supra). The only description of the articles given in the affidavit presented to the judge was as
follows: "that there are being kept in said premises books, documents, receipts, lists, chits and other papers used
by him in connection with his activities as money-lender, charging a usurious rate of interest, in violation of the
law." Taking into consideration the nature of the article so described, it is clear that no other more adequate and
detailed description could have been given, particularly because it is difficult to give a particular description of the
contents thereof. The description so made substantially complies with the legal provisions because the officer of
the law who executed the warrant was thereby placed in a position enabling him to identify the articles, which he
did.

VII. The last ground alleged by the petitioner, in support of his claim that the search warrant was obtained illegally,
is that the articles were seized in order that the Anti-Usury Board might provide itself with evidence to be used by
it in the criminal case or cases which might be filed against him for violation of the Anti-usury Law. At the hearing
of the incidents of the case raised before the court it clearly appeared that the books and documents had really
been seized to enable the Anti-Usury Board to conduct an investigation and later use all or some of the articles in
question as evidence against the petitioner in the criminal cases that may be filed against him. The seizure of
books and documents by means of a search warrant, for the purpose of using them as evidence in a criminal
case against the person in whose possession they were found, is unconstitutional because it makes the warrant
unreasonable, and it is equivalent to a violation of the constitutional provision prohibiting the compulsion of an
accused to testify against himself (Uy Kheytin vs.Villareal, 42 Phil,, 886; Brady vs. U. S., 266 U. S., 620;
Temperani vs. U. S., 299 Fed., 365; U. S. vs. Madden, 297 Fed., 679; Boyd vs. U. S.,116 U. S., 116; Caroll vs. U.
S., 267 U. S., 132). Therefore, it appearing that at least nineteen of the documents in question were seized for the
purpose of using them as evidence against the petitioner in the criminal proceeding or proceedings for violation
against him, we hold that the search warrant issued is illegal and that the documents should be returned to him.

The Anti-Usury Board insinuates in its answer that the petitioner cannot now question the validity of the search
warrant or the proceedings had subsequent to the issuance thereof, because he has waived his constitutional
rights in proposing a compromise whereby he agreed to pay a fine of P200 for the purpose of evading the criminal
proceeding or proceedings. We are of the opinion that there was no such waiver, first, because the petitioner has
emphatically denied the offer of compromise and, second, because if there was a compromise it reffered but to
the institution of criminal proceedings fro violation of the Anti-Usury Law. The waiver would have been a good
defense for the respondents had the petitioner voluntarily consented to the search and seizure of the articles in
question, but such was not the case because the petitioner protested from the beginning and stated his protest in
writing in the insufficient inventory furnished him by the agents.

Said board alleges as another defense that the remedy sought by the petitioner does not lie because he can
appeal from the orders which prejudiced him and are the subject matter of his petition. Section 222 of the Code of
Civil Procedure in fact provides that mandamus will not issue when there is another plain, speedy and adequate
remedy in the ordinary course of law. We are of the opinion, however, that an appeal from said orders would have
to lapse before he recovers possession of the documents and before the rights, of which he has been unlawfully
deprived, are restored to him (Fajardo vs. Llorente, 6 Phil., 426; Manotoc vs. McMicking and Trinidad, 10 Phil.,
119; Cruz Herrera de Lukban vs. McMicking, 14 Phil., 641; Lamb vs. Phipps, 22 Phil., 456).
Summarizing the foregoing conclusions, we hold:

1. That the provisions of the Constitution and General Orders, No. 58, relative to search and seizure, should be
given a liberal construction in favor of the individual in order to maintain the constitutional guaranties whole and in
their full force;

2. That since the provisions in question are drastic in their form and fundamentally restrict the enjoyment of the
ownership, possession and use of the personal property of the individual, they should be strictly construed;

3. That the search and seizure made are illegal for the following reasons: (a) Because the warrant was based
solely upon the affidavit of the petitioner who had no personal knowledge of the facts of probable cause, and (b)
because the warrant was issued for the sole purpose of seizing evidence which would later be used in the
criminal proceedings that might be instituted against the petitioner, for violation of the Anti-Usury Law;

4. That as the warrant had been issued unreasonably, and as it does not appear positively in the affidavit that the
articles were in the possession of the petitioner and in the place indicated, neither could the search and seizure
be made at night;

5. That although it is not mandatory to present affidavits of witnesses to corroborate the applicant or a
complainant in cases where the latter has personal knowledge of the facts, when the applicant's or complainant's
knowledge of the facts is merely hearsay, it is the duty of the judge to require affidavits of other witnesses so that
he may determine whether probable cause exists;

6. That a detailed description of the person and place to be searched and the articles to be seized is necessary,
but whereby, by the nature of the articles to be seized, their description must be rather general, but is not required
that a technical description be given, as this would mean that no warrant could issue;

7. That the petitioner did not waive his constitutional rights because the offer of compromise or settlement
attributed to him, does not mean, if so made, that he voluntarily tolerated the search and seizure; and

8. That an appeal from the orders questioned by the petitioner, if taken by him, would not be an effective, speedy
or adequate remedy in the ordinary course of law, and, consequently, the petition for mandamus filed by him, lies.

For the foregoing considerations, the search warrant and the seizure of June 3, 1936, and the orders of the
respondent court authorizing the relation of the books and documents, are declared illegal and are set aside, and
it is ordered that the judge presiding over the Court of First Instance of Tayabas direct the immediate return to the
petitioner of the nineteen (19) documents designated on pages 1 to 4 of the inventory by Nos. 5, 10, 16, 23, 25,26,
27, 30, 31, 34, 36, 37, 38, 39, 40, 41, 42, 43 and 45, without special pronouncement as to costs. So ordered.

Avanceña, C.J., Villa-Real, Diaz and Concepcion, JJ., concur.


[G.R. No. 50720. March 26, 1984.]

SORIANO MATA, Petitioner, v. HON. JOSEPHINE K. BAYONA, in her capacity as Presiding Judge of the City Court of
Ormoc, BERNARDO GOLES and REYNALDO MAYOTE, Respondents.

Valeriano R. Ocubillo for Petitioner.

The Solicitor General for Respondents.

SYLLABUS

1. CONSTITUTIONAL LAW; BILL OF RIGHTS; RIGHT AGAINST UNLAWFUL SEARCH AND SEIZURE; REQUISITES FOR ISSUANCE OF
SEARCH WARRANT. — Under the Constitution "no search warrant shall issue but upon probable cause to be determined by the Judge
or such other responsible officer as may be authorized by law after examination under oath or affirmation of the complainant and the
witnesses he may produce." More emphatic and detailed is the implementing rule of the constitutional injunction, Section 4 of Rule
126 which provides that the judge must before issuing the warrant personally examine on oath or affirmation the complainant and
any witnesses he may produce and take their depositions in writing, and attach them to the record, in addition to any affidavits
presented to him.

2. ID.; ID.; ID.; ID.; INSUFFICIENCY OF AFFIDAVITS OF COMPLAINANT AND HIS WITNESSES IN THE CASE AT BAR. — Before
issuing a search warrant, the examining Judge has to take depositions in writing of the complainant and the witnesses he may
produce and to attach them to the record. Such written deposition is necessary in order that the Judge may be able to properly
determine the existence or non-existence of the probable cause, and to hold liable for perjury the person giving it if it will be found
later that his declarations are false. Mere affidavits of the complainant and his witnesses are thus not sufficient.

3. ID.; ID.; ID.; ID.; NO "DEPOSITION IN WRITING" ATTACHED TO RECORDS OF CASE IN CASE AT BAR. — The judge’s insistence
that she examined the complainants under oath has become dubious by petitioner’s claim that at the particular time when he
examined all the relevant papers connected with the issuance of the questioned search warrant, after he demanded the same from
the lower court since they were not attached to the records, he did not find any certification at the back of the joint affidavit of the
complainants. Before he filed his motion to quash the search warrant and for the return of the articles seized, he was furnished, upon
his request, certified true copies of the said affidavits by the Clerk of Court but which certified true copies do not bear any
certification at the back. Petitioner likewise claims that his xerox copy of the said joint affidavit obtained at the outset of this case
does not show also the certification of respondent judge. This doubt becomes more confirmed by respondent Judge’s own admission,
while insisting that she did examine thoroughly the applicants, that "she did not take the deposition of Mayote and Goles because
to have done so would be to hold a judicial proceeding which will be open and public", such that, according to her, the persons
subject of the intended raid will just disappear and move his illegal operations somewhere else. Could it be that the certification was
made belatedly to cure the defect of the warrant? Be that as it may, there was no "deposition in writing" attached to the records of
the case in palpable disregard of the statutory prohibition heretofore quoted.

4. ID.; ID.; ID.; ID.; DEPOSITIONS, HOW TAKEN. — The searching questions propounded to the applicants of the search warrant
and his witnesses must depend to a large extent upon the discretion of the Judge just as long as the answers establish a reasonable
ground to believe the commission of a specific offense and that the applicant is one authorized by law, and said answers particularly
describe with certainty the place to be searched and the persons or things to be seized. The examination or investigation which must
be under oath may not be in public. It may even be held in the secrecy of his chambers. Far more important is that the examination
or investigation is not merely routinary but one that is thorough and elicit the required information. To repeat, it must be under oath
and must be in writing.

5. ID.; ID.; ID.; ID.; MUST BE STRICTLY COMPLIED WITH; CASE AT BAR. — Nothing can justify the issuance of the search warrant
but the fulfillment of the legal requisites. Thus, in issuing a search warrant the Judge must strictly comply with the requirements of
the Constitution and the statutory provisions. In the case at bar, the search warrant is tainted with illegality by the failure of the
Judge to conform with essential requisites of taking the depositions in writing and attaching them to record, rendering the search
warrant invalid.
6. ID.; ID.; ID.; ALTHOUGH ILLEGAL, THINGS SEIZED CANNOT BE RETURNED; CASE AT BAR. — While the search warrant is illegal,
the return of the things seized cannot be ordered. In Castro v. Pabalan (70 SCRA 478), it was held that the illegality of the search
warrant does not call for the return of the things seized, the possession of which is prohibited.

DECISION

DE CASTRO, J.:

The validity of the search warrant issued by respondent Judge (not reappointed) is challenged by petitioner for its alleged failure to
comply with the requisites of the Constitution and the Rules of Court.

Specifically, the contention is that the search warrant issued by respondent Judge was based merely on the application for search
warrant and a joint affidavit of private respondents which were wrongfully it is alleged subscribed, and sworn to before the Clerk of
Court of respondent Judge. Furthermore, there was allegedly a failure on the part of respondent Judge to attach the necessary
papers pertinent to the issuance of the search warrant to the records of Criminal Case No. 4298-CC wherein petitioner is accused
under PD 810, as amended by PD 1306, the information against him alleging that Soriano Mata offered, took and arranged bets on
the Jai Alai game by "selling illegal tickets known as ‘Masiao tickets’ without any authority from the Philippine Jai Alai & Amusement
Corporation or from the government authorities concerned." 1

Petitioner claims that during the hearing of the case, he discovered that nowhere from the records of the said case could be found
the search warrant and other pertinent papers connected to the issuance of the same, so that he had to inquire from the City Fiscal
its whereabouts, and to which inquiry respondent Judge replied, "it is with the court." The Judge then handed the records to the
Fiscal who attached them to the records. chanrob les.co m : virt ual law li bra ry

This led petitioner to file a motion to quash and annul the search warrant and for the return of the articles seized, citing and invoking,
among others, Section 4 of Rule 126 of the Revised Rules of Court. The motion was denied by respondent Judge on March 1, 1979,
stating that the court has made a thorough investigation and examination under oath of Bernardo U. Goles and Reynaldo T. Mayote,
members of the Intelligence Section of 352nd PC Co./Police District II INP; that in fact the court made a certification to that effect;
and that the fact that documents relating to the search warrant were not attached immediately to the record of the criminal case is
of no moment, considering that the rule does not specify when these documents are to be attached to the records. 2 Petitioner’s
motion for reconsideration of the aforesaid order having been denied, he came to this Court, with the instant petition, praying,
among others, that this Court declare the search warrant to be invalid and all the articles confiscated under such warrant as
inadmissible as evidence in the case, or in any proceedings on the matter.

We hold that the search warrant is tainted with illegality for being violative of the Constitution and the Rules of Court.

Under the Constitution "no search warrant shall issue but upon probable cause to be determined by the Judge or such other
responsible officer as may be authorized by law after examination under oath or affirmation of the complainant and the witnesses
he may produce." More emphatic and detailed is the implementing rule of the constitutional injunction, Section 4 of Rule 126 which
provides that the judge must before issuing the warrant personally examine on oath or affirmation the complainant and any
witnesses he may produce and take their depositions in writing, and attach them to the record, in addition to any affidavits
presented to him.

Mere affidavits of the complainant and his witnesses are thus not sufficient. The examining Judge has to take depositions in writing
of the complainant and the witnesses he may produce and to attach them to the record. Such written deposition is necessary in order
that the Judge may be able to properly determine the existence or non-existence of the probable cause, to hold liable for perjury the
person giving it if it will be found later that his declarations are false.

We, therefore, hold that the search warrant is tainted with illegality by the failure of the Judge to conform with the essential
requisites of taking the depositions in writing and attaching them to the record, rendering the search warrant invalid. chanroblesv irtualawl ibrary

The judge’s insistence that she examined the complainants under oath has become dubious by petitioner’s claim that at the
particular time when he examined all the relevant papers connected with the issuance of the questioned search warrant, after he
demanded the same from the lower court since they were not attached to the records, he did not find any certification at the back
of the joint affidavit of the complainants. As stated earlier, before he filed his motion to quash the search warrant and for the return
of the articles seized, he was furnished, upon his request, certified true copies of the said affidavits by the Clerk of Court but which
certified true copies do not bear any certification at the back. Petitioner likewise claims that his xerox copy of the said joint affidavit
obtained at the outset of this case does not show also the certification of respondent judge. This doubt becomes more confirmed by
respondent Judge’s own admission, while insisting that she did examine thoroughly the applicants, that "she did not take the
deposition of Mayote and Goles because to have done so would be to hold a judicial proceeding which will be open and public", 3 such
that, according to her, the persons subject of the intended raid will just disappear and move his illegal operations somewhere else.

Could it be that the certification was made belatedly to cure the defect of the warrant? Be that as it may, there was no "deposition
in writing" attached to the records of the case in palpable disregard of the statutory prohibition heretofore quoted.

Respondent Judge impresses this Court that the urgency to stop the illegal gambling that lures every man, woman and child, and
even the lowliest laborer who could hardly make both ends meet justifies her action. She claims that in order to abate the
proliferation of this illegal "masiao" lottery, she thought it more prudent not to conduct the taking of deposition which is done usually
and publicly in the court room.

Two points must be made clear. The term "depositions" is sometimes used in a broad sense to describe any written statement
verified by oath; but in its more technical and appropriate sense the meaning of the word is limited to written testimony of a witness
given in the course of a judicial proceeding in advance of the trial or hearing upon oral examination. 4 A deposition is the testimony
of a witness, put or taken in writing, under oath or affirmation before a commissioner, examiner or other judicial officer, in answer
to interlocutory and cross interlocutory, and usually subscribed by the witnesses. 5 The searching questions propounded to the
applicants of the search warrant and his witnesses must depend to a large extent upon the discretion of the Judge just as long as the
answers establish a reasonable ground to believe the commission of a specific offense and that the applicant is one authorized by law,
and said answers particularly describe with certainty the place to be searched and the persons or things to be seized. The
examination or investigation which must be under oath may not be in public. It may even be held in the secrecy of his chambers. Far
more important is that the examination or investigation is not merely routinary but one that is thorough and elicit the required
information. To repeat, it must be under oath and must be in writing. cralawnad

The other point is that nothing can justify the issuance of the search warrant but the fulfillment of the legal requisites. It might be
well to point out what has been said in Asian Surety & Insurance Co., Inc. v. Herrera: jgc:chanroble s.com.p h

"It has been said that of all the rights of a citizen, few are of greater importance or more essential to his peace and happiness than
the right of personal security, and that involves the exemption of his private affairs, books, and papers from inspection and scrutiny
of others. While the power to search and seize is necessary to the public welfare, still it must be exercised and the law enforced
without transgressing the constitutional rights of the citizens, for the enforcement of no statute is of sufficient importance to justify
indifference to the basic principles of government." 6

Thus, in issuing a search warrant the Judge must strictly comply with the requirements of the Constitution and the statutory
provisions. A liberal construction should be given in favor of the individual to prevent stealthy encroachment upon, or gradual
depreciation of the rights secured by the Constitution. 7 No presumption of regularity are to be invoked in aid of the process when
an officer undertakes to justify it. 8

While We hold that the search warrant is illegal, the return of the things seized cannot be ordered. In Castro v. Pabalan, 9 it was held
that the illegality of the search warrant does not call for the return of the things seized, the possession of which is prohibited.

WHEREFORE, the writ of certiorari is granted and the order of March 1, 1979 denying the motion to annul the search warrant as well
as the order of March 21, 1979 denying the motion for reconsideration are hereby reversed, the search warrant, being declared
herein as illegal. Notwithstanding such illegality, the things seized under such warrant, such as stock of "masiao" tickets; "masiao"
issue tickets; bet money; control pad or "masiao" numbers; stamping pad with rubber stamp marked Ormoc City Jai-Alai," cannot
be returned as sought by petitioner. No costs.

SO ORDERED.
EN BANC

[G.R. No. 127755. April 14, 1999]

PEOPLE OF THE PHILIPPINES, plaintiff-appellee, vs. JOSELITO DEL ROSARIO


y PASCUAL, accused-appellant.

DECISION
BELLOSILLO, J.:

ON AUTOMATIC REVIEW is the decision of the court a quo finding accused Joselito del
Rosario y Pascual guilty as co-principal in the crime of Robbery with Homicide and sentencing him
to death and to pay the heirs of the victim Virginia Bernas P550,000.00 as actual damages
and P100,000.00 as moral and exemplary damages.[1]
Joselito del Rosario y Pascual, Ernesto Marquez alias Jun, Virgilio Santos alias Boy Santos
and John Doe alias Dodong were charged with special complex crime of Robbery with Homicide for
having robbed Virginia Bernas, a 66-year old businesswoman, of P200,000.00 in cash and jewelry
and on the occasion thereof shot and killed her.[2]
While accused Joselito del Rosario pleaded not guilty,[3] Virgilio Boy Santos and John Doe alias
Dodong remained at large. Ernesto Jun Marquez was killed in a police encounter. Only Joselito del
Rosario was tried.
These facts were established by the prosecution from the eyewitness account of tricycle driver
Paul Vincent Alonzo: On 13 May 1996 between 6:00 and 6:30 in the evening, Alonzo stopped his
tricycle by the side of Nitas Drugstore, General Luna St., Cabanatuan City, when three women
flagged him. Parked at a distance of about one and a-half (1) meters in front of him was a tricycle
driven by accused Joselito del Rosario. At that point, Alonzo saw two (2) men and a woman
grappling for possession of a bag. After taking hold of the bag one of the two men armed with a gun
started chasing a man who was trying to help the woman, while the other snatcher kicked the
woman sending her to the ground. Soon after, the armed man returned and while the woman was
still on the ground he shot her on the head. The bag taken by the man was brought to the tricycle of
accused del Rosario where someone inside received the bag. The armed man then sat behind the
driver while his companion entered the sidecar. When the tricycle sped away Alonzo gave chase
and was able to get the plate number of the tricycle. He also recognized the driver, after which he
went to the nearest police headquarters and reported the incident. [4]
Accused Joselito del Rosario gave his own version of the incident: At around 5:30 in the
afternoon he was hired for P120.00[5] by a certain Boy Santos,[6] his co-accused. Their original
agreement was that he would drive him to cockpit at the Blas Edward Coliseum. [7] However, despite
their earlier arrangement boy Santos directed him to proceed to the market place to fetch Jun
Marquez and Dodong Bisaya. He (del Rosario) acceded.[8] Marquez and Bisaya boarded in front of
the parking lot of Merced Drugstore at the public market.[9] Subsequently, he was asked to proceed
and stop at the corner of Burgos and General Luna Sts. where Bisaya alighted on the pretest of
buying a cigarette. The latter then accosted the victim Virginia Bernas and grappled with her for the
possession of her bag. Jun Marquez alighted from the tricycle to help Dodong Bisaya.[10] Accused
del Rosario tried to leave and seek help but Boy Santos who stayed inside the tricycle prevented
him from leaving and threatened in fact to shoot him.
Meanwhile, Dodong Bisaya succeeded in taking the victims bag, but before boarding the
tricycle Jun Marquez mercilessly shot the victim on the head while she was lying prone on the
ground. After the shooting, Dodong Bisaya boarded the sidecar of the tricycle while Jun Marquez
rode behind del Rosario and ordered him to start the engine and drive towards Dicarma. While
inside his tricycle, del Rosario overheard his passengers saying that they would throw the bag at
Zulueta St. where there were cogon grasses.[11] Upon arriving at Dicarma, the three (3) men
alighted and warned del Rosario not to inform the police authorities about the incident otherwise he
and his family would be harmed.[12] Del Rosario then went home.[13] Because of the threat, however,
he did not report the matter to the owner of the tricycle nor to the barangay captain and the
police.[14]
As earlier stated, the court a quo found accused Joselito del Rosario guilty as charged and
sentenced him to death. He now contends in this automatic review that the court a quo erred in: (1)
Not finding the presence of threat and irresistible force employed upon him by his co-accused
Virgilio Boy Santos, Ernesto Jun Marquez and Dodong Bisaya; (2) Not considering his defense that
he was not part of the conspiracy among co-accused "Boy" Santos, "Jun" Marquez and "Dodong"
Bisaya to commit the crime of Robbery with Homicide; (3) Not considering the violations on his
constitutional rights as an accused; and, (4) Not considering that there was no lawful warrantless
arrest within the meaning of Sec. 5, Rule 113, of the Rules of Court. [15]
The conviction of del Rosario must be set aside. His claim for exemption from criminal liability
under Art. 12, par. 5, Revised Penal Code as he acted under the compulsion of an irresistible force
must be sustained. He was then unarmed and unable to protect himself when he was prevented at
gunpoint by his co-accused from leaving the crime scene during the perpetration of the robbery and
killing, and was only forced to help them escape after the commission of the crime.[16]
But the trial court ruled that his fear was merely speculative, fanciful and remote, hence, could
not be considered uncontrollable; and that a gun pointed at him did not constitute irresistible force
because it fell short of the test required by law and jurisprudence. [17]
We disagree. A person who acts under the compulsion of an irresistible force, like one who
acts under the impulse of an uncontrollable fear of equal or greater injury, is exempt from criminal
liability because he does not act with freedom. Actus me invito factus non est meus actus. An act
done by me against my will is not my act. The force contemplated must be so formidable as to
reduce the actor to a mere instrument who acts not only without will but against his will. The duress,
force, fear or intimidation must be present, imminent and impending, and of such nature as to
induce a well-grounded apprehension of death or serious bodily harm if the act be done. A threat of
future injury is not enough. The compulsion must be of such a character as to leave no opportunity
for the accused for escape or self-defense in equal combat.[18]
As a rule, it is natural for people to be seized by fear when threatened with weapons, even
those less powerful that a gun, such as knives and clubs. People will normally, usually and
probably do what an armed man asks them to do, nothing more, nothing less. In the instant case,
del Rosario was threatened with a gun. He could not therefore be expected to flee nor risk his life to
help a stranger. A person under the same circumstances would be more concerned with his
personal welfare and security rather than the safety of a person whom he only saw for the first time
that day.[19]
Corollary with defense of del Rosario, we hold that the trial court erred when it said that it was
Boy Santos who left the tricycle to chase the companion of the victim and then shot the victim on
the head, instantly killing her.[20] A careful and meticulous scrutiny of the transcripts and records of
the case, particularly the testimonies of the witness Alonzo and del Rosario himself, reveals that it
was Jun Marquez who ran after the victims helper and fired at the victim. Witness Alonzo testified
on direct examination -
Q: What was that unusual incident that transpired in that place at that time?
A: I saw two men and a lady grappling for the possession of a bag, sir x x x x
Q: What happened after the bag of the lady was grabbed by the two men?
A: One helper of the lady was chased by the other man, sir.
Q: Who was that man who chased the helper of the lady?
A: He was the one holding the gun, sir x x x x
Q: What happened when the bag of the woman was already taken by the two men who grappled the
same from her?
A: The man who chased the helper of the lady returned to the scene while the other man was then kicking
the lady who in turn fell to the ground, sir.
Q: What happened to the lady who to the ground?
A: The man who chased the helper of the lady returned and then shot the woman who was then lying on
the ground, sir x x x x
Q: What about the bag, what happened to the bag?
A: The bag was taken to a motorcycle, sir.
Q: Will you please state before the Court what you noticed from the tricycle which was at a distance of
about one and a half meter?
A: There was a passenger inside the tricycle, sir x x x x
Q: What happened to that woman that was shot by the man who grappled for the possession of the bag?
A: She was no longer moving and lying down, sir.
Q: After the shooting by one of the two men of the woman what else happened?
A: They went away, sir x x x x
Q: Will you please tell the Court in what portion of the tricycle did these men sit in the tricycle?
A: The man who was holding the gun sat himself behind the driver while the other
man entered the sidecar, sir.[21]
On the continuation of his direct examination, after an ocular inspection on the crime scene
conducted by the trial court, witness Alonzo categorically stated
Q: Will you please tell us where in particular did you see the accused who was then holding the gun fired
at the victim?
A: At the time one man was kicking the victim it was then his other companion holding the gun chased
the helper of the deceased going towards Burgos Avenue, sir.
Q: What happen (sic) afterwards?
A: The man with the gun returned and then while the victim was lying down in this spot the man holding
the gun shot the victim, sir.[22]
On cross-examination, the same witness further clarified
Q: So, you saw the two other accused returned back to the tricycle?
A: Yes, sir.
Q: And one of their companion was already inside the tricycle?
xxxx
Court: There was somebody inside the tricycle where the handbag was given.
xxxx
A: Yes, sir.
Q: And the one who sat at the back of the tricycle driver was the person with the gun?
A: Yes, sir.[23]
On the other hand, accused Del Rosario declared during the direct examination that
Q: x x x x On the evening of May 13, 1996 you were the driver of the tricycle as testified to by Eduardo
Nalagon?
A: Yes, sir.
Q: Now, you also heard that there was a shoot out near the Cathedral and the Nitas Drugstore at Gen.
Tinio St.?
A: Yes, sir.
xxxx
Court: At that time you were seated at the tricycle, which tricycle was used by the assailants?
A: Yes, sir.
Q: Then what did you do?
A: I tried to escape, sir, but I was stopped by them.
Q: When you said they to whom are you referring?
A: Boy Santos and Jun Marquez, sir.
Q: And at that time where was Boy Santos?
A: He was inside the tricycle, sir.
Q: And what about Jun Marquez?
A: He alighted from the tricycle and helped him grabbed (sic) the bag of the victim.
Q: And was the bag grabbed and by whom?
A: Yes, sir, by Dodong Visaya was able to grab the bag.
Q: And after that what happened?
A: Both of them rode inside my tricycle, sir.
Court: Did you not see any shooting?
A: There was, sir.
Q: Who was shot?
A: Jun Marquez shot the woman, sir x x x x
Q: When the bag of the woman was being grabbed you know that what was transpiring was wrong and
illegal?
A: Yes, sir.
Q: But you did not try to leave?
A: I tried to leave but Boy Santos who was inside my tricycle prevented me.
Q: During that time before you leave (sic) how many firearms did you see?
A: Two firearms, sir, one in the possession of Boy (Jun?) Marquez and one in the possession of Boy
Santos x x x x
Q: And at the time when the shooting took place where was Boy Santos?
A: He was still inside my tricycle, sir.
Q: And during the shooting when Boy Santos was inside the tricycle and when you tried to escape that
was the time when Boy Santos threatened you if you will escape something will happen to your
family?
A: Yes, sir.
Q: After the shooting who first boarded the tricycle, Boy (Jun?) Marquez or Dodong Visaya?
A: Dodong Visaya, sir.
Q: And immediately thereafter Jun Marquez boarded your tricycle sitting at your back?
A: Yes, sir.[24]
On cross-examination, accused further stated
Q: After shopping in that place for one minute what else happened?
A: I saw Dodong Bisaya grabbing the bag of the woman, sir.
Q: How about your two companions, what are (sic) they doing while Dodong Bisaya was grabbing the
bag of the woman?
A: Jun Marquez was helping Dodong Bisaya, sir.
Q: What happened after Jun Marquez helped Dodong Bisaya?
A: I heard a gunshot and I saw the woman lying down x x x x
Q: You could have ran away to seek the help of the police or any private persons?
A: I was not able to ask for help because Boy Santos pointed his gun to me, sir.
Q: Was the gun being carried by Boy Santos, is the one that is used in shooting the old woman?
A: No, sir x x x x.
Q: Where was Boy Santos when Dodong Bisaya and Jun Marquez were grappling for the possession of
the handbag?
A: He was then inside the tricycle, sir x x x x[25]
Q: Mr. Witness, you testified that the reason why you just cannot leave the area where the incident
occurred is because a gun was pointed to you by Boy Santos and he was telling you that you should
not do anything against their will, they will kill you and your family will be killed also, is that correct?
A: Yes, sir.
Q: Now, is it not a fact that at the time you stop (sic) your tricycle which was loaded by your other three
co-accused in this case, all of them alighted and that Boy Santos ran after a helper of the victim
going towards the public market along Burgos Street?
A: He did not alight from the tricycle, sir.
Court: Are you quite sure of that?
A: Yes, sir.[26]
Del Rosario maintains that Boy Santos never left the tricycle and that the latter pointed his gun
at him and threatened to shoot if he tried to escape. He also asserted that it was Jun Marquez who
shot the victim and sat behind him in the tricycle.
From the narration of witness Alonzo, these events stood out: that after the bag of the victim
was grabbed, her male helper was chased by a man holding a gun; that the gunwielder returned
and shot the victim and then sat behind the driver of the tricycle; and, the bag was given to a person
who was inside the tricycle. Taking the testimony of witness Alonzo in juxtaposition with the
testimony of del Rosario, it can be deduced that Jun Marquez was the person witness Alonzo was
referring to when he mentioned that a helper of the lady was chased by the other man and that
this other man could not be Boy Santos who stayed inside the tricycle and to whom the bag was
handed over. This conclusion gives credence to the claim of del Rosario that Boy Santos never left
the tricycle, and to his allegation that Boy Santos stayed inside the tricycle precisely to threaten him
with violence and prevent him from fleeing; that there could have been no other plausible reason
for Boy Santos to stay in the tricycle if the accused was indeed a conspirator; that Boy Santos could
have just left the tricycle and helped in the commission of the crime, particularly when he saw the
victim grappling with Dodong Bisaya and resisting the attempts to grab her bag; and, that Boy
Santos opted to remain inside the tricycle to fulfill his preordained role of threatening del Rosario
and insuring that he would not escape and leave them behind. [27]
Even if the tricycle of del Rosario was only parked one meter and a half (1) in front of the
tricycle of witness Alonzo, the latter still could not have totally seen and was not privy to events that
were transpiring inside the vehicle, i.e., the pointing of the gun by Boy Santos at del Rosario
simultaneously with the robbing and shooting of the victim. From the exhibits submitted by the
prosecution panel the back of the sidecar of del Rosario tricycle was not transparent. [28]
There is no doubt that the fear entertained by del Rosario because of the gun directly pointed at
him was real and imminent. Such fear rendered him immobile and subject to the will of Boy Santos,
making him for the moment of automaton without a will of his own. In other words, in effect, he
could not be any more than a mere instrument acting involuntarily an against his will. He is
therefore exempt from criminal liability since by reason of fear of bodily harm he was compelled
against his will to transport his co-accused away from the crime scene.
On the issue of conspiracy, the trial court anchored del Rosarios conviction on his participation
in the orchestrated acts of Boy Santos, Jun Marquez and Dodong Bisaya. According to the trial
court, del Rosario facilitated the escape of the other malefactors from the crime scene and
conspiracy between accused and his passengers was evident because while the grappling of the
bag, the chasing of the helper of the victim and the shooting that led to the death of Virginia Bernas
were happening, accused Joselito del Rosario was riding on his tricycle and the engine of the motor
was running;[29] that the accused did not deny that the tricycle driven by him and under his control
was hired and used by his co-accused in the commission of the crime; neither did he deny his
failure to report to the authorities the incident of robbery, killing and fleeing away from the scene of
the crime.[30]
We disagree with the trial court. A conspiracy in the statutory language exists when two or
more persons come to an agreement concerning the commission of a felony and decide to commit
it. The objective of the conspirators is to perform an act or omission punishable by law. That must
be their intent. There is need for concurrence of wills or unity of action and purpose or for common
and joint purpose and design. Its manifestation could be shown by united and concerted action. [31]
Admittedly, direct proof is not essential to establish conspiracy. Since by its nature conspiracy
is planned in utmost secrecy, it can rarely be proved by direct evidence. Consequently, the
presence of the concurrence of minds which is involved in conspiracy may be inferred from proof of
facts and circumstances which, taken together, apparently indicate that they are merely parts of
some complete whole. If it is proved that two or more persons aimed by their acts towards the
accomplishment of the same unlawful object, each doing a part so that their combined acts, though
apparently independent, were in fact connected and cooperative, indicating a closeness of
personal association and a concurrence of sentiment, a conspiracy may be inferred though no
actual meeting among them to concert means is proved.That would be termed an implied
conspiracy.[32] Nevertheless, mere knowledge, acquiescence or approval of the act, without the
cooperation or agreement to cooperate, is not enough to constitute one a party to a conspiracy, but
that there must be intentional participation in the transaction with a view to the furtherance of the
common design and purpose. Conspiracy must be established, not by conjectures, but by positive
and conclusive evidence. In fact, the same degree of proof necessary to establish the crime is
required to support a finding of the presence of a criminal conspiracy, which is, proof beyond
reasonable doubt.[33]
In the instant case, while del Rosario admits that he was at the locus criminis as he was the
driver of the getaway vehicle, he nonetheless rebuts the imputation of guilt against him by asserting
that he had no inkling of the malevolent design of his co-accused to rob and kill since he was not
given any briefing thereof. He was merely hired by Boy Santos to drive to an agreed destination
and he was prevented at gunpoint from leaving the scene of the crime since he was ordered to help
them escape.
In this case, the trial court stated that "there is no evidence that the accused came to an
agreement concerning the commission of the felony and decided to commit the same." [34] Therefore,
in order to convict the accused, the presence of an implied conspiracy is required to be proved
beyond reasonable doubt. However, the fact that del Rosario was with the other accused when the
crime was committed is insufficient proof to show cabal. Mere companionship does not establish
conspiracy.[35] The only incriminating evidence against del Rosario is that he was at the scene of the
crime but he has amply explained the reason for his presence and the same has not been
successfully refuted by the prosecution. As stated earlier, he feared for his safety and security
because of the threat made by his co-accused that he would, be killed should he shout for help. No
complicity can be deduced where there is absolutely no showing that the accused directly
participated in the overt act of robbing and shooting although he was with the persons who robbed
and killed the victim.[36]
That del Rosario did not disclose what he knew about the incident to the authorities, to his
employer or to the barangay captain does not affect his credibility. The natural hesitance of most
people to get involved in a criminal case is of judicial notice. [37] It must be recalled that del Rosario
was merely a tricycle driver with a family to look after. Given his quite limited means, del Rosario
understandably did not want to get involved in the case so he chose to keep his silence. Besides,
he was threatened with physical harm should he squeal.
Del Rosario further contends that there was violation of his right to remain silent, right to have
competent and independent counsel preferably of his own choice, and right to be informed of these
rights as enshrined and guaranteed in the Bill of Rights. [38] As testified to by SP04 Geronimo de
Leon, the prosecution witness who was the team leader of the policemen who investigated the 13
May incident, during his cross-examination -

Upon finding the name of the owner of the tricycle, they proceeded to Bakod Bayan in
the house of the barangay captain where the owner of the tricycle was summoned and
who in turn revealed the driver's name and was invited for interview. The driver was
accused Joselito del Rosario who volunteered to name his passengers on May 13,
1996. On the way to the police station, accused informed them of the bag and lunch kit's
location and the place where the hold-uppers may be found and they reported these
findings to their officers, Capt. Biag and Capt. Cruz. After lunch, they proceeded to
Brgy. Dicarma composed of 15 armed men where a shoot-out transpired that lasted
from 1:00 to 4:00 o'clock in the afternoon. After a brief encounter, they went inside the
house where they found Marquez dead holding a magazine and a gun. While all of
these were happening, accused del Rosario was at the back of the school, after which
they went back to the police station. The investigator took the statement of the accused
on May 14,1996, and was only subscribed on May 22,1996. All the while, he was
detained in the police station as ordered by the Fiscal. His statements were only signed
on May 16, 1996. He also executed a waiver of his detention. His Sinumpaang Salaysay
was done with the assistance of Ex-Judge Talavera.[39]

A further perusal of the transcript reveals that during the encounter at Brgy. Dicarma, del Rosario
was handcuffed by the police because allegedly they had already gathered enough evidence
against him and they were afraid that he might attempt to escape. [40]
Custodial investigation is the stage where the police investigation is no longer a general inquiry
into an unsolved crime but has begun to focus on a particular suspect taken into custody by the
police who carry out a process of interrogation that lends itself to elicit incriminating statements. It is
well-settled that it encompasses any question initiated by law enforcers after a person has been
taken into custody or otherwise deprived of his freedom of action in any significant way. [41] This
concept of custodial investigation has been broadened by RA 7438 [42] to include "the Practice of
issuing an 'invitation' to a person who is investigated in connection with an offense he is suspected
to have committed." Section 2 of the same Act further provides that -

x x x x Any public officer or employee, or anyone acting under his order or in his place,
who arrests, detains or investigates any person for the commission of an offense shall
inform the latter, in a language known and understood by him of his right to remain silent
and to have competent and independent counsel, preferably of his own choice, who
shall at all times be allowed to confer privately with the person arrested, detained or
under custodial investigation. If such person cannot afford the services of his own
counsel, he must be provided with a competent and independent counsel by the
investigating officer.

From the foregoing, it is clear that del Rosario was deprived of his rights during custodial
investigation. From the time he was "invited" for questioning at the house of the barangay captain,
he was already under effective custodial investigation, but he was not apprised nor made aware
thereof by the investigating officers. The police already knew the name of the tricycle driver and the
latter was already a suspect in the robbing and senseless slaying of Virginia Bernas. Since the
prosecution failed to establish that del Rosario had waived his right to remain silent, his verbal
admissions on his participation in the crime even before his actual arrest were inadmissible against
him, as the same transgressed the safeguards provided by law and the Bill of Rights.
Del Rosario also avers that his arrest was unlawful since there was no warrant
therefor. Section 5, Rule 113 of the Rules of Court provides:[43]

Sec. 5. Arrest without warrant; when lawful. - A peace officer or a private person may,
without a warrant, arrest a person: (a) When, in his presence, the person to be arrested
has committed, is actually committing, or is attempting to commit an offense; (b) When
an offense has in fact been committed and he has personal knowledge of facts
indicating that the person to be arrested has committed it; and, (c) When the person to
be arrested is a prisoner who has escaped from penal establishment or place where he
is serving final judgment or temporarily confined while his case is pending, or has
escaped while being transferred from one confinement to another.

It must be recalled that del Rosario was arrested by SPO4 De Leon during the police raid at the
place of "Jun" Marquez at Brgy. Dicarma on 14 May 1996. In People vs Sucro[44] we held that when
a police officer sees the offense, although at a distance, or hears the disturbances created thereby,
and proceeds at once to the scene thereof, he may effect an arrest without a warrant on the basis
of Sec. 5, par. (a), Rule 113, since the offense is deemed committed in his presence or within his
view. In essence, Sec. 5, par. (a), Rule 113, requires that the accused be caught in flagrante
delicto or caught immediately after the consummation of the act. The arrest of del Rosario is
obviously outside the purview of the aforequoted rule since he was arrested on the day following
the commission of the robbery with homicide.
On the other hand, Sec. 5, par. (b), Rule 113, necessitates two (2) stringent requirements
before a warrantless arrest can be effected: (1) an offense has just been committed; and (2) the
person making the arrest has personal knowledge of facts indicating that the person to be arrested
had committed it. Hence, there must be a large measure of immediacy between the time the
offense was committed and the time of the arrest, and if there was an appreciable lapse of time
between the arrest and the commission of the crime, a warrant of arrest must be secured. Aside
from the sense of immediacy, it is also mandatory that the person making the arrest must
have personal knowledge of certain facts indicating that the person to be taken into custody has
committed the crime.[45] Again, the arrest of del Rosario does not comply with these requirements
since, as earlier explained, the arrest came a day after the consummation of the crime and not
immediately thereafter. As such, the crime had not been "just committed" at the time the accused
was arrested. Likewise, the arresting officers had no personal knowledge of facts indicating that the
person to be arrested had committed the offense since they were not present and were not actual
eyewitnesses to the crime, and they became aware of his identity as the driver of the getaway
tricycle only during the custodial investigation.
However the conspicuous illegality of del Rosario's arrest cannot affect the jurisdiction of the
court a quo because even in instances not allowed by law, a warrantless arrest is not a
jurisdictional defect and any objection thereto is waived when the person arrested submits to
arraignment without any objection, as in this case.[46]
A transgression of the law has occurred. Unfortunately, an innocent person lost her life and
property in the process. Someone therefore must be held accountable, but it will not be accused
Joselito del Rosario; we must acquit him. Like victim Virginia Bernas, he too was a hapless victim
who was forcibly used by other persons with nefarious designs to perpetrate a dastardly act. Del
Rosario's defense of "irresistible force" has been substantiated by clear and convincing
evidence. On the other hand, conspiracy between him and his co-accused was not proved beyond
a whimper of a doubt by the prosecution, thus clearing del Rosario of any complicity in the crime
charged.
WHEREFORE, the decision of the Regional Trial Court of Cabanatuan City convicting accused
JOSELITO DEL ROSARIO Y PASCUAL of Robbery with Homicide and sentencing him to death, is
REVERSED and SET ASIDE, and the accused is ACQUITTED of the crime charged. His
immediate RELEASE from confinement is ordered unless held for some other lawful cause. In this
regard, the Director of Prisons is directed to report to the Court his compliance herewith within five
(5) days from receipt hereof.
SO ORDERED.
G.R. No. 95847-48. March 10, 1993.

PEOPLE OF THE PHILIPPINES, plaintiff-appellee, vs. GABRIEL GERENTE y BULLO, accused-appellant.

The Solicitor General for plaintiff-appellee.

Public Attorney's Office for accused-appellant.

SYLLABUS

1. REMEDIAL LAW; CRIMINAL PROCEDURE; ARREST WITHOUT WARRANT; LAWFUL WHEN ARRESTING
OFFICER HAS PERSONAL KNOWLEDGE THAT THE PERSON TO BE ARRESTED HAS COMMITTED THE
CRIME; CASE AT BAR. — The policemen arrested Gerente only some three (3) hours after Gerente and his
companions had killed Blace. They saw Blace dead in the hospital and when they inspected the scene of the
crime, they found the instruments of death: a piece of wood and a concrete hollow block which the killers had
used to bludgeon him to death. The eye-witness, Edna Edwina Reyes, reported the happening to the policemen
and pinpointed her neighbor, Gerente, as one of the killers. Under those circumstances, since the policemen had
personal knowledge of the violent death of Blace and of facts indicating that Gerente and two others had killed
him, they could lawfully arrest Gerente without a warrant. If they had postponed his arrest until they could obtain a
warrant, he would have fled the law as his two companions did.

2. ID.; ID.; SEARCH AND SEIZURE; VALID EVEN WITHOUT A WARRANT WHEN MADE AS AN INCIDENT TO
LAWFUL ARREST; RATIONALE. — The search conducted on Gerente's person was likewise lawful because it
was made as an incident to a valid arrest. This is in accordance with Section 12, Rule 126 of the Revised Rules of
Court which provides: "Section 12. Search incident to lawful arrest. — A person lawfully arrested may be
searched for dangerous weapons or anything which may be used as proof of the commission of an offense,
without a search warrant." The frisk and search of appellant's person upon his arrest was a permissible
precautionary measure of arresting officers to protect themselves, for the person who is about to be arrested may
be armed and might attack them unless he is first disarmed. In Adams vs. Williams, 47 U.S. 143, cited in Justice
Isagani A. Cruz's Constitutional Law, 1991 Edition, p. 150, it was ruled that "the individual being arrested may be
frisked for concealed weapons that may be used against the arresting officer and all unlawful articles found his
person, or within his immediate control may be seized."

3. CRIMINAL LAW; CONSPIRACY; LIABILITY OF CONSPIRATORS; RULE; CASE AT BAR. — There is no


merit in appellant's allegation that the trial court erred in convicting him of having conspired and cooperated with
Fredo and Totoy Echigoren to kill Blace despite the testimony of Dr. Valentin Bernales that the fracture on the
back of the victim's skull could have been inflicted by one person only. what Dr. Bernales stated was a mere
possibility that only one person dropped the concrete hollow block on the head of the victim, smashing it. That
circumstance, even if true, does not absolve the other two co-conspirators in the murder of Blace for when there
is a conspiracy to commit a crime, the act of one conspirator is the act of all. The conspiracy was proven by the
eyewitness-testimony of Edna Edwina Reyes, that she overheard the appellant and his companions conspire to
kill Blace, that acting in concert, they attacked their victim with a piece of wood and a hollow block and caused his
death. "When there is no evidence indicating that the principal witness for the prosecution was moved by
improper motive, the presumption is that he was not so moved and his testimony is entitled to full faith and credit"
(People vs. Belibet, 199 SCRA 587, 588). Hence, the trial court did not err in giving full credit to Edna Reyes'
testimony.

4. ID.; CIVIL INDEMNITY FOR DEATH; INCREASED TO P50,000.00. — The Solicitor General correctly pointed
out in the appellee's brief that the award of P30,000.00 as civil indemnity for the death of Clarito Blace should be
increased to P50,000.00 in accordance with our ruling in People vs. Sison, 189 SCRA 643.

DECISION

GRIÑO-AQUINO, J p:
This is an appeal from the decision of the Regional Trial Court of Valenzuela, Metro Manila, Branch 172, which
found the appellant guilty of Violation of Section 8 of Republic Act 6425 (Dangerous Drugs Act of 1972) and
sentenced him to suffer the penalty of imprisonment for a term of twelve (12) years and one (1) day, as minimum,
to twenty (20) years, as maximum; and also found him guilty of Murder for which crime he was sentenced to
suffer the penalty of reclusion perpetua. The dispositive portion of the appealed decision reads:

"WHEREFORE, in view of the foregoing the Court finds the accused Gabriel Gerente in Criminal Case No.
10255-V-90 guilty beyond reasonable doubt of Violation of Section 8 of R.A. 6425 and hereby sentences him to
suffer the penalty of imprisonment of twelve years and one day as minimum to twenty years as maximum, and a
fine of twelve thousand, without subsidiary imprisonment in case of insolvency, and to pay the costs.

"In Criminal Case No. 10256-V-90, the Court finds the accused Gabriel Gerente guilty beyond reasonable doubt
of the crime of Murder, and there by (sic) no aggravating circumstances nor mitigating circumstances, is hereby
sentenced to suffer the penalty of reclusion perpetua; to indemnify the heirs of the victim in the sum of P30,000.00,
and in the amount of P17,609.00 as funeral expenses, without subsidiary imprisonment in case of insolvency, and
to pay the costs. The accused Gabriel Gerente shall be credited with the full term of his preventive imprisonment."
(p. 25, Rollo.)

Appellant Gabriel Gerente y Bullo was charged with Violation of Section 8, Art. II of R.A. 6425, which was
docketed as Criminal Case No. 10255-V-90 of the Regional Trial Court of Valenzuela, Metro Manila. The
Information reads:

"That on or about the 30th day of April, 1990, in the municipality of Valenzuela, Metro Manila, Philippines, and
within the jurisdiction of this Honorable Court, the above-named accused, without justification, did then and there
wilfully, unlawfully and feloniously have in his possession and control dried flowering tops wrapped in foil with
markings and place in a transparent plastic bag which are considered prohibited drugs." (p. 2, Rollo.)

The same accused, together with Totoy and Fredo Echigoren who are both at large, was charged with Murder in
Criminal Case No. 10256-V-90 in an information of the same date and signed by the same Assistant Provincial
Prosecutor, as follows:

"That on or about the 30th day of April, 1990, in the municipality of Valenzuela, Metro Manila, Philippines, and
within the jurisdiction of this Honorable Court, the above-named accused together with two (2) others who are still
at large and against whom the preliminary investigation has not yet been terminated by the Office of the
Provincial Prosecutor of Bulacan, conspiring, confederating together and mutually helping one another, armed
with a piece of wood and hallow (sic) block and with intent to kill one Clarito B. Blace, did then and there wilfully,
unlawfully and feloniously, with evident premeditation and treachery, attack, assault and hit with the said piece of
wood and hollow block the said Clarito B. Blace, hitting the latter on the different parts of his body, thereby
inflicting serious physical injuries which directly caused the death of the said victim." (p. 3, Rollo.)

Edna Edwina Reyes testified that at about 7:00 a.m. of April 30, 1990, appellant Gabriel Gerente, together with
Fredo Echigoren and Totoy Echigoren, started drinking liquor and smoking marijuana in the house of the
appellant which is about six (6) meters away from the house of the prosecution witness who was in her house on
that day. She overheard the three men talking about their intention to kill Clarito Blace. She testified that she
heard Fredo Echigoren saying, "Gabriel, papatayin natin si Clarito Blace," and Totoy Echigoren allegedly
seconded Fredo's suggestion saying: "Papatayin natin 'yan mamaya." Appellant allegedly agreed: "Sigue,
papatayin natin mamaya." (pp. 3-4, tsn, August 24, 1990.)

Fredo and Totoy Echigoren and Gerente carried out their plan to kill Clarito Blace at about 2:00 p.m. of the same
day. The prosecution witness, Edna Edwina Reyes, testified that she witnessed the killing. Fredo Echigoren
struck the first blow against Clarito Blace, followed by Totoy Echigoren and Gabriel Gerente who hit him twice
with a piece of wood in the head and when he fell, Totoy Echigoren dropped a hollow block on the victim's head.
Thereafter, the three men dragged Blace to a place behind the house of Gerente.

At about 4:00 p.m. of the same day, Patrolman Jaime Urrutia of the Valenzuela Police Station received a report
from the Palo Police Detachment about a mauling incident. He went to the Valenzuela District Hospital where the
victim was brought. He was informed by the hospital officials that the victim died on arrival. The cause of death
was massive fracture of the skull caused by a hard and heavy object. Right away, Patrolman Urrutia, together
with Police Corporal Romeo Lima and Patrolman Alex Umali, proceeded to Paseo de Blas where the mauling
incident took place. There they found a piece of wood with blood stains, a hollow block and two roaches of
marijuana. They were informed by the prosecution witness, Edna Edwina Reyes, that she saw the killing and she
pointed to Gabriel Gerente as one of the three men who killed Clarito.

The policemen proceeded to the house of the appellant who was then sleeping. They told him to come out of the
house and they introduced themselves as policemen. Patrolman Urrutia frisked appellant and found a coin purse
in his pocket which contained dried leaves wrapped in cigarette foil. The dried leaves were sent to the National
Bureau of Investigation for examination. The Forensic Chemist found them to be marijuana.

Only the appellant, Gabriel Gerente, was apprehended by the police. The other suspects, Fredo and Totoy
Echigoren, are still at large.

On May 2, 1990, two separate informations were filed by Assistant Provincial Prosecutor Benjamin Caraig against
him for Violation of Section 8, Art. II, of R.A. 6425, and for Murder.

When arraigned on May 16, 1990, the appellant pleaded not guilty to both charges. A joint trial of the two cases
was held. On September 24, 1990, the trial court rendered a decision convicting him of Violation of Section 8 of
R.A. 6425 and of Murder.

In this appeal of the appellant, the following errors are ascribed to the trial court:

1. the court a quo gravely erred in admitting the marijuana leaves adduced in evidence by the prosecution; and

2. the court a quo gravely erred in convicting the accused-appellant of the crimes charged despite the absence of
evidence required to prove his guilt beyond reasonable doubt.

The appellant contends that the trial court erred in admitting the marijuana leaves as evidence in violation of his
constitutional right not to be subjected to illegal search and seizure, for the dried marijuana leaves were seized
from him in the course of a warrantless arrest by the police officers. We do not agree.

The search of appellant's person and the seizure of the marijuana leaves in his possession were valid because
they were incident to a lawful warrantless arrest.

Paragraphs (a) and (b), Section 5, Rule 113 of the Revised Rules of Court provide:

'SECTION 5. Arrest without warrant; when lawful. — A peace officer or a private person may, without a warrant,
arrest a person:

"(a) When, in his presence, the person to be arrested has committed, is actually committing, or is attempting to
commit an offense;"

"(b) When an offense has in fact just been committed, and he has personal knowledge of facts indicating that the
person to be arrested has committed it; . . .'

The policemen arrested Gerente only some three (3) hours after Gerente and his companions had killed Blace.
They saw Blace dead in the hospital and when they inspected the scene of the crime, they found the instruments
of death: a piece of wood and a concrete hollow block which the killers had used to bludgeon him to death. The
eye-witness, Edna Edwina Reyes, reported the happening to the policemen and pinpointed her neighbor,
Gerente, as one of the killers. Under those circumstances, since the policemen had personal knowledge of the
violent death of Blace and of facts indicating that Gerente and two others had killed him, they could lawfully arrest
Gerente without a warrant. If they had postponed his arrest until they could obtain a warrant, he would have fled
the law as his two companions did.

In Umil vs. Ramos, 187 SCRA 311, the arrest of the accused without a warrant was effected one (1) day after he
had shot to death two Capcom soldiers. The arrest was held lawful by this Court upon the rationale stated by us in
People vs. Malasugui, 63 Phil. 221, 228, thus:
"To hold that no criminal can, in any case, be arrested and searched for the evidence and tokens of his crime
without a warrant, would be to leave society, to a large extent, at the mercy of the shrewdest, the most expert, and
the most depraved of criminals, facilitating their escape in many instances."

The search conducted on Gerente's person was likewise lawful because it was made as an incident to a valid
arrest. This is in accordance with Section 12, Rule 126 of the Revised Rules of Court which provides:

"SECTION 12. Search incident to lawful arrest. — A person lawfully arrested may be searched for dangerous
weapons or anything which may be used as proof of the commission of an offense, without a search warrant."

The frisk and search of appellant's person upon his arrest was a permissible precautionary measure of arresting
officers to protect themselves, for the person who is about to be arrested may be armed and might attack them
unless he is first disarmed. In Adams vs. Williams, 47 U.S. 143, cited in Justice Isagani A. Cruz's Constitutional
Law, 1991 Edition, p. 150, it was ruled that "the individual being arrested may be frisked for concealed weapons
that may be used against the arresting officer and all unlawful articles found in his person, or within his immediate
control may be seized."

There is no merit in appellant's allegation that the trial court erred in convicting him of having conspired and
cooperated with Fredo and Totoy Echigoren to kill Blace despite the testimony of Dr. Valentin Bernales that the
fracture on the back of the victim's skull could have been inflicted by one person only.

What Dr. Bernales stated was a mere possibility that only one person dropped the concrete hollow block on the
head of the victim, smashing it. That circumstance, even if true, does not absolve the other two co-conspirators in
the murder of Blace for when there is a conspiracy to commit a crime, the act of one conspirator is the act of all.
The conspiracy was proven by the eyewitness-testimony of Edna Edwina Reyes, that she overheard the
appellant and his companions conspire to kill Blace, that acting in concert, they attacked their victim with a piece
of wood and a hollow block and caused his death. "When there is no evidence indicating that the principal witness
for the prosecution was moved by improper motive, the presumption is that he was not so moved and his
testimony is entitled to full faith and credit" (People vs. Belibet, 199 SCRA 587, 588). Hence, the trial court did not
err in giving full credit to Edna Reyes' testimony.

Appellant's failure to escape (because he was very drunk) is no indicium of his innocence.

The Solicitor General correctly pointed out in the appellee's brief that the award of P30,000.00 as civil indemnity
for the death of Clarito Blace should be increased to P50,000.00 in accordance with our ruling in People vs. Sison,
189 SCRA 643.

WHEREFORE, the appealed decision is hereby AFFIRMED, with modification of the civil indemnity awarded to
the heirs of the victim, Clarito Blace, which is hereby increased to P50,000.00.

SO ORDERED.
G.R. No. 93239 March 18, 1991

PEOPLE OF THE PHILIPPINES, plaintiff-appellee,


vs.
EDISON SUCRO, accused-appellant.

The Solicitor General for plaintiff-appellee.


Fidencio S. Raz for accused-appellant.

GUTIERREZ, JR., J.:

Edison Sucro was charged with and convicted of violation of Section 4, Article II of the Dangerous Drugs Act,
under an Information which reads:

That on or about the 21st day of March, 1989, in the evening, in the Poblacion, Municipality of Kalibo, Province of
Aklan, Republic of the Philippines, and within the jurisdiction of this Honorable Court, the above-named accused,
acting as a pusher or broker in the business of selling, administering, delivery, giving away to another and/or
distributing prohibited drugs, did then and there wilfully, unlawfully and feloniously and without authority of law
have in his possession and control nineteen (19) pieces of marijuana cigarette sticks and four (4) tea bags of
dried marijuana leaves which were confiscated from him by the police authorities of Kalibo, Aklan, shortly after
having sold one tea bag of dried marijuana leaves to a customer. (Rollo, p. 9)

Upon arraignment, the accused-appellant, assisted by counsel, entered a plea of "not guilty" to the offense
charged. Trial ensued and a judgment of conviction was rendered, the pertinent portion of which reads:

WHEREFORE, judgment is rendered finding the accused Edison Sucro guilty of the sale of prohibited drug under
Section 4, Article II of the Dangerous Drug Act, as amended, and sentencing him to suffer the penalty of life
imprisonment, and pay a fine of P20,000, and costs. He shall be entitled to full credit in the service of his sentence
with the period for which he has undergone preventive imprisonment to the date of promulgation of this judgment.
All the items of marijuana confiscated in this case are declared forfeited in favor of the State. (Rollo, p. 41)

From the foregoing judgment of conviction, accused-appellant interposes this appeal, assigning the following as
errors allegedly committed by the court a quo, to wit:

THE LOWER COURT ERRED IN ADMITTING AS EVIDENCE FOR THE PROSECUTION EXHIBITS "E"-"E-4",
TEA BAGS OF ALLEGED MARIJUANA, TO BE THE CORPUS DELICTI; FURTHERMORE, THAT THE SAME
WERE TAKEN WITHOUT THE REQUIRED WARRANT OF SEARCH AND ARREST SINCE THE ACCUSED
WAS NOT IN THE ACT OF COMMITTING ANY OFFENSE AT THE TIME OF HIS ARREST.
II

THE LOWER COURT ERRED IN FINDING THE ACCUSED EDISON SUCRO GUILTY OF THE SALE OF
PROHIBITED DRUGS UNDER SECTION 4, ARTICLE II, OF THE DANGEROUS DRUGS ACT AND
SENTENCING HIM TO SUFFER A PENALTY OF LIFE IMPRISONMENT AND TO PAY A FINE OF P 20,000.00.
(Appellant's Brief, p. 1)

The antecedent facts of the case as summarized by the Solicitor General are as follows:

On March 21, 1989, Pat. Roy Fulgencio, a member of the INP, Kalibo, Aklan, was instructed by P/Lt. Vicente
Seraspi, Jr. (Station Commander of the INP Kalibo, Aklan) to monitor the activities of appellant Edison Sucro,
because of information gathered by Seraspi that Sucro was selling marijuana. (p. 6, TSN, May 2,1989).

As planned, at about 5:00 P.M. on said date, Pat. Fulgencio Positioned himself under the house of a certain Arlie
Regalado at C. Quimpo Street. Adjacent to the house of Regalado, about 2 meters away, was a chapel.
Thereafter, Pat. Fulgencio saw appellant enter the chapel, taking something which turned out later to be
marijuana from the compartment of a cart found inside the chapel, and then return to the street where he handed
the same to a buyer, Aldie Borromeo. After a while appellant went back to the chapel and again came out with
marijuana which he gave to a group of persons. (pp. 6-8, 15-18, Ibid). It was at this instance that Pat. Fulgencio
radioed P/Lt. Seraspi and reported the activity going on. P/Lt. Seraspi instructed Pat. Fulgencio to continue
monitoring developments. At about 6:30 P.M., Pat. Fulgencio again called up Seraspi to report that a third buyer
later Identified as Ronnie Macabante, was transacting with appellant. (pp. 18-19, Ibid)

At that point, the team of P/Lt. Seraspi proceeded to the area and while the police officers were at the Youth
Hostel at Maagma St., Pat. Fulgencio told P/Lt. Seraspi to intercept Macabante and appellant. P/Lt. Seraspi and
his team caught up with Macabante at the crossing of Mabini and Maagma Sts. in front of the Aklan Medical
Center. Upon seeing the police, Macabante threw something to the ground which turned out to be a tea bag of
marijuana. (pp. 6-8, TSN, June 19, 1989) When confronted, Macabante readily admitted that he bought the same
from appellant (Edison Sucro) in front of the chapel. (p. 6, TSN, May 24, 1989) The police team was able to
overtake and arrest appellant at the corner of C. Quimpo and Veterans Sts. The police recovered 19 sticks and 4
teabags of marijuana from the cart inside the chapel and another teabag from Macabante, The teabags of
marijuana were sent to the PC-INP Crime Laboratory Service, at Camp Delgado, Iloilo City for analysis. The
specimens (Exhibits "G" to "G-18", Exhibits "E" to "E-4") were all found positive of marijuana. (pp. 47, TSN, Sept.
4, 1989)" (Appellee's Brief, pp. 3-6)

As can be seen from the facts, the issue hinges mainly on whether or not the arrest without warrant of the
accused is lawful and consequently, whether or not the evidence resulting from such arrest is admissible.

We rule in the affirmative.

The accused-appellant contends that his arrest was illegal, being a violation of his rights granted under Section 2,
Article III of the 1987 Constitution. He stresses that there was sufficient time for the police officers to apply for a
search and arrest warrants considering that Fulgencio informed his Station Commander of the activities of the
accused two days before March 21, 1989, the date of his arrest.

This contention is without merit.

Section 5, Rule 113 of the Rules on Criminal Procedure provides for the instances where arrest without warrant is
considered lawful. The rule states:

Arrest without warrant, when lawful. — A peace officer or private person may, without warrant, arrest a person:

(a) When in his presence, the person to be arrested has committed, is actually committing, or is attempting to
commit an offense;

(b) When an offense has in fact just been committed, and he has personal knowledge of facts indicating that the
person to be arrested has committed it; (Emphasis supplied)
An offense is committed in the presence or within the view of an officer, within the meaning of the rule authorizing
an arrest without a warrant, when the officer sees the offense, although at a distance, or hears the disturbances
created thereby and proceeds at once to the scene thereof. (U.S. v. Fortaleza, 12 Phil. 472 [1909]; and U.S. v.
Samonte, 16 Phil. 516 [1910])

The records show that Fulgencio went to Arlie Regalado's house at C. Quimpo Street to monitor the activities of
the accused who was earlier reported to be selling marijuana at a chapel two (2) meters away from Regalado's
house.

Fulgencio, within a distance of two meters saw Sucro conduct his nefarious activity. He saw Sucro talk to some
persons, go inside the chapel, and return to them and exchange some things. These, Sucro did three times
during the time that he was being monitored. Fulgencio would then relay the on-going transaction to P/Lt. Seraspi.

Anent the second requirement, the fact that Macabante, when intercepted by the police, was caught throwing the
marijuana stick and when confronted, readily admitted that he bought the same from accused-appellant clearly
indicates that Sucro had just sold the marijuana stick to Macabante, and therefore, had just committed an illegal
act of which the police officers had personal knowledge, being members of the team which monitored Sucro's
nefarious activity.

The court earlier indicated in the case of People v. Bati (G.R. No. 87429, August 27, 1990) that police officers
have personal knowledge of the actual commission of the crime when it had earlier conducted surveillance
activities of the accused. Thus, it stated:

When Luciano and Caraan reached the place where the alleged transaction would take place and while
positioned at a street comer, they saw appellant Regalado Bati and Warner Marquez by the side of the street
about forty to fifty meters away from them (the public officers). They saw Marquez giving something to Bati, who,
thereafter handed a wrapped object to Marquez who then inserted the object inside the front of his pants in front
of his abdomen while Bati, on his part, placed the thing given to him inside his pocket. (p. 2)

xxx xxx xxx

. . . Both Patrolman Luciano and Caraan actually witnessed the same and their testimonies were based on their
actual and personal knowledge of the events that took place leading to appellant's arrest. They may not have
been within hearing distance, specially since conversation would expectedly be carried on in hushed tones, but
they were certainly near enough to observe the movements of the appellant and the buyer. Moreover, these
prosecution witnesses are all law enforcers and are, therefore, presumed to have regularly performed their duties
in the absence of proof to the contrary (People v. Bati, supra citing People v. Agapito, G.R. No. 73786, October
12, 1987)

The accused questions the failure of the police officers to secure a warrant considering that Fulgencio himself
knew of Sucro's activities even prior to the former's joining the police force. Fulgencio reported Sucro's activities
only three days before the incident.

As the records reveal, Fulgencio and Sucro had known each other since their childhood years and that after
Fulgencio joined the police force, he told the accused-appellant not to sell drugs in their locality. Hence, it is
possible that because of this friendship, Fulgencio hesitated to report his childhood friend and merely advised him
not to engage in such activity. However, because of reliable information given by some informants that selling
was going on everyday, he was constrained to report the matter to the Station Commander.

On the other hand, the failure of the police officers to secure a warrant stems from the fact that their knowledge
acquired from the surveillance was insufficient to fulfill the requirements for the issuance of a search warrant.
What is paramount is that probable cause existed. Thus, it has been held in the case of People v. Lo Ho Wing, et
al. (G.R. No. 88017, January 21, 1991):

In the instant case, it was firmly established from the factual findings of the trial court that the authorities had
reasonable ground to believe that appellant would attempt to bring in contraband and transport it within the
country. The belief was based on intelligence reports gathered from surveillance activities on the suspected
syndicate, of which appellant was touted to be a member. Aside from this, they were also certain as to the
expected date and time of arrival of the accused from China. But such knowledge was clearly insufficient to
enable them to fulfill the requirements for the issuance of a search warrant. Still and all, the important thing is that
there was probable cause to conduct the warrantless search, which must still be present in such a case.

As the Solicitor General has pointed out:

There are several instances when a warrantless search and seizure can be effected without necessarily being
preceded by an arrest provided the same is effected on the basis of probable cause (e.g. stop and search without
warrant at checkpoints). Between warrantless searches and seizures at checkpoints and in the case at bar the
latter is more reasonable considering that unlike in the former, it was effected on the basis of probable cause.
Under the circumstances (monitoring of transactions) there existed probable cause for the arresting officers, to
arrest appellant who was in fact selling marijuana and to seize the contraband.

That searches and seizures must be supported by a valid warrant is not an absolute rule (Manipon, Jr. v.
Sandiganbayan, 143 SCRA 267 [1986]). Among the exceptions granted by law is a search incidental to a lawful
arrest under Sec. 12, Rule 126 of the Rules on Criminal Procedure, which provides that a person lawfully arrested
may be searched for dangerous weapons or anything which may be used as proof of the commission of an
offense, without a search warrant. (People v. Castiller, G.R. No. 87783, August 6, 1990)

The accused-appellant claims that the arrest having been done without warrant, it follows that the evidence
obtained therefrom is inadmissible.

As earlier discussed, there is nothing unlawful about the arrest considering its compliance with the requirements
of a warrantless arrest. Ergo, the fruits obtained from such lawful arrest are admissible in evidence.

Edison Sucro assails the trial court's reliance on the statement of Macabante whose reason for testifying could be
merely to escape prosecution.

We quote the trial court's finding as to the testimony of Macabante:

The non-filing of a complaint against him for possession of marijuana may have been the reason of (sic) his
willingness to testify in court against the accused. But this does not necessarily taint the evidence that proceeds
from his lips. As explained by Lt. Seraspi, the best sources of information against drug pushers are usually their
customers, especially if as in this case, there is no other direct evidence of the selling except the testimony of the
buyer. We accept this observation as a realistic appraisal of a situation in which drug users are, and should be
employed by law enforcement authorities to bolster the drive against pushers who are the real felons in our
society. We have observed the demeanor of the witness in court, and found him to be straightforward,
unhesitating, and spontaneous in his declarations, so that we are satisfied as to his intention and disposition to
tell the truth (Rollo, p. 40)

Time and again it has been held that the findings of the trial court are entitled to great weight and should not be
disturbed on appeal unless it is shown that the trial court had overlooked certain facts of weight and importance, it
being acknowledged. that the court below, having seen and heard the witnesses during the trial, is in a better
position to evaluate their testimonies (People v. Umali, et al., G.R. No. 84450, February 4, 1991 citing People v.
Alvarez, 163 SCRA 745 [1988]; People v. Dorado, 30 SCRA 53 [1969]; and People v. Espejo, 36 SCRA 400
[1970]).

Furthermore, the testimony of Macabante was corroborated on material points by public officers Fulgencio and
Seraspi.

There is nothing in the record to suggest that the police officers were compelled by any motive than to accomplish
their mission to capture a drug pusher in the execution of the crime, the presumption being that police officers
perform their duties regularly in the absence of any evidence to the contrary (Rule 131, Sec. 3(m), Revised Rules
on Evidence; People v. Castiller, supra citing People v. Natipravat, 145 SCRA 483 [1986]).

The prosecution evidence was further bolstered by the findings of the Forensic Chemist that the items seized
were all positive for marijuana.
In contrast to the evidence presented by the prosecution, accused-appellant's defense is alibi which is unavailing
considering that he was positively identified by Macabante to be the person from whom he bought marijuana.

Sucro alleges that he could not have committed the crime since he was with his uncle and cousin distributing
handbills for his Auntie's candidacy. The fact, however, remains that it does not preclude the possibility that he
was present in the vicinity as established by his admission that he moved a lot and even had the occasion to meet
Macabante on the street.

It is well-settled that mere denials cannot prevail against the positive identification of the appellant as the seller of
the prohibited substances. (People v. Khan, 161 SCRA 406 [1988]; and People v. Paco, 170 SCRA 681 [1989])

Premises considered, this Court is convinced that appellant Edison Sucro had indeed committed the offense
charged. The trial court's decision must be upheld.

WHEREFORE, the decision appealed from is hereby AFFIRMED.

SO ORDERED.

G.R. No. 95902 February 4, 1992

PEOPLE OF THE PHILIPPINES, plaintiff-appellee,


vs.
DON RODRIGUEZA, accused-appellant.

The Solicitor General for plaintiff-appellee.

Public Attorney's Office for accused-appellant.

REGALADO, J.:

On appeal before us is the decision of the Regional Trial Court of Legaspi City, Branch 10, finding
accused-appellant Don Rodrigueza guilty beyond reasonable doubt of violating Section 4, Article II of the
Dangerous Drugs Act of 1972 (Republic Act No. 6425, as amended) and sentencing him to suffer the penalty of
life imprisonment and to pay a fine of P20,000.00 and costs. 1

However, the Solicitor General, deviating from his conventional stance in the prosecution of criminal cases,
recommends the acquittal of appellant for the reasons stated in his Manifestation for Acquittal (In Lieu of
Appellee's Brief) filed with the Court. We have reviewed and analyzed the testimonial and documentary evidence
in this case and we find said recommendation to be well taken.

The information, dated July 10, 1987, charges Don Rodrigueza and his co-accused, Samuel Segovia and Antonio
Lonceras, with allegedly having in their custody and possession 100 grams of marijuana leaves and for selling, in
a buy-bust operation, said 100 grams of dried marijuana leaves for a consideration of P200.00. 2
During the arraignment, all the accused pleaded not guilty to the charge against them. At the trial, the prosecution
and the defense presented several witnesses after which the court a quo rendered judgment acquitting Samuel
Segovia and Antonio Lonceras but convicting and penalizing herein appellant as hereinbefore stated.

The following facts are culled from the decision of the trial court and the evidence presented by the prosecution.

At around 5:00 o'clock in the afternoon of July 1, 1987, CIC Ciriaco Taduran was in their headquarters at the
Office of the Narcotics Regional Unit at Camp Bagong Ibalon, Legaspi City, together with S/Sgt. Elpidio Molinawe,
CIC Leonardo B. Galutan and their commanding officer, Major Crisostomo M. Zeidem, when a confidential
informer arrived and told them that there was an ongoing illegal traffic of prohibited drugs in Tagas, Daraga, Albay.
Major Zeidem formed a team to conduct a buy-bust operation, which team was given P200.00 in different
denominations to buy marijuana. These bills were treated with ultraviolet powder at the Philippine Constabulary
Crime Laboratory (PCCL). Sgt. Molinawe gave the money to Taduran who acted as the poseur buyer. He was
told to look for a certain Don, the alleged seller of prohibited drugs. Taduran went to Tagas alone and, while along
the road, he met Samuel Segovia. He asked Segovia where be could find Don and where he could buy marijuana.
Segovia left for a while and when be returned, he was accompanied by a man who was later on introduced to him
as Don, herein appellant. 3

After agreeing on the price of P200.00 for 100 grams of marijuana, Don halted a passing tricycle driven by
Antonio Lonceras. He boarded it and left Taduran and Segovia. When he came back, Don gave Taduran "a
certain object wrapped in a plastic" which was later identified as marijuana, and received payment therefor.
Thereafter, Taduran returned to the headquarters and made a report regarding his said purchase of marijuana. 4

Based on that information, Major Zeidem ordered a team to conduct an operation to apprehend the suspects. In
the evening of the same date, CIC Galutan and S/Sgt. Molinawe proceeded to Regidor Street, Daraga, Albay and
arrested appellant, Antonio Lonceras and Samuel Segovia. The constables were not, however, armed with a
warrant of arrest when they apprehended the three accused. The arrestees were brought to the headquarters for
investigation. 5

Thereafter, agents of the Narcotics Command (NARCOM) conducted a raid in the house of Jovencio Rodrigueza,
father of appellant. Taduran did not go with them. During the raid, they were able to confiscate dried marijuana
leaves and a plastic syringe, among others. The search, however, was not authorized by any search warrant. 6

The next day, July 2, 1987, Jovencio Rodrigueza was released from detention but appellant was detained. An
affidavit, allegedly taken from and executed by him, was sworn to by him before the assistant city prosecutor.
Appellant had no counsel when his sworn statement was taken during that custodial investigation. The arrestees
were also examined by personnel of the PCCL and were found positive for ultraviolet powder. 7

The three accused presented different versions of their alleged participations.

Samuel Segovia testified that he was in their house in the evening of July 1, 1987 listening to the radio. Later, he
ate his merienda and then went out to buy cigarettes from the store. While he was at the store, a jeep stopped
behind him. Several armed men alighted therefrom and ordered him to get inside the jeep. He refused but he was
forced to board the vehicle. He was even hit by the butt of a gun. 8

He was thereafter brought to Camp Bagong Ibalon where he was investigated and was repeatedly asked regarding the whereabouts of Rodrigueza. He was
manhandled by the NARCOM agents and was detained while inside the camp. He was then made to hold a P10.00 bill treated with ultraviolet powder. When
he was taken to the PCCL and examined he was found positive of the ultraviolet powder. He was also made to sign some papers but he did not know what they
were all about. 9

Appellant, on the other hand, testified that on said date he was in the house of his aunt in San Roque, Legaspi
City. He stayed there overnight and did not leave the place until the next day when his brother arrived and told
him that their father was taken by some military men the preceding night. Appellant went to Camp Bagong Ibalon
and arrived there at around 8:00 o'clock in the morning of July 2, 1987. When he arrived, he was asked if he knew
anything about the marijuana incident, to which question he answered in the negative. Like Segovia, he was
made to hold a P10.00 bill and was brought to the crime laboratory for examination. From that time on, he was not
allowed to go home and was detained inside the camp. He was also tortured in order to make him admit his
complicity in the alleged sale of marijuana. 10
In the assignment of errors in his brief, appellant contends that the trial court erred in (1) admitting in evidence the
sworn statement of appellant which was obtained in violation of his constitutional rights; (2) convicting appellant of
the crime charged despite the fact that the 100 grams of dried marijuana leaves allegedly bought from him were
not properly identified; (3) convicting appellant of the crime charged despite the fact that the evidence for the
prosecution is weak and not convincing; and (4) finding appellant guilty beyond reasonable doubt of selling or at
least acting as broker in the sale of the 100 grams of marijuana to CIC Taduran late in the afternoon of July 1,
1987, despite the failure of the prosecution to prove his guilt beyond reasonable doubt. 11

We rule for the appellant and approve the recommendation for his acquittal. In disposing of this case, however,
we feel that the issues raised by appellant should properly be discussed seriatim.

1. A buy-bust operation is a form of entrapment employed by peace officers to trap and catch a malefactor in
flagrante delicto. Applied to the case at bar, the term in flagrante delicto requires that the suspected drug dealer
12

must be caught redhanded in the act of selling marijuana or any prohibited drug to a person acting or posing as a
buyer.

In the instant case, however, the procedure adopted by the NARCOM agents failed to meet this qualification.
Based on the very evidence of the prosecution, after the alleged consummation of the sale of dried marijuana
leaves, CIC Taduran immediately released appellant Rodrigueza instead of arresting and taking him into his
custody. This act of CIC Taduran, assuming arguendo that the supposed sale of marijuana did take place, is
decidedly contrary to the natural course of things and inconsistent with the aforestated purpose of a buy-bust
operation. It is rather absurd on his part to let appellant escape without having been subjected to the sanctions
imposed by law. It is, in fact, a dereliction of duty by an agent of the law.

2. The admissibility of the sworn statement allegedly executed by appellant was squarely placed in issue and, as
correctly pointed out by the defense, said sworn statement is inadmissible in evidence against appellant.

We have once again to reiterate and emphasize that Article III of the 1987 Constitution provides:

Sec. 12 (1). Any person under investigation for the commission of an offense shall have the right to be informed of
his right to remain silent and to have a competent and independent counsel preferably of his own choice. If the
person cannot afford the services of counsel, he must be provided with one. These rights cannot be waived
except in writing and in the presence of counsel.

xxx xxx xxx

(3) Any confession or admission obtained in violation of this or section 17 hereof shall be inadmissible in evidence
against him.

An examination of said sworn statement shows that appellant was informed of his constitutional right to remain
silent and to be assisted by counsel during custodial examination. He was also asked if he was waiving his right
to be assisted by counsel and he answered in the affirmative. However, while the rights of a person under
custodial investigation may be waived, such waiver must be made not only voluntarily, knowingly and intelligently
but also in the presence and with the assistance of counsel. In the present case, the waiver made by appellant
13

being without the assistance of counsel, this omission alone is sufficient to invalidate said sworn statement. 14

3. Corollary to this, we take cognizance of the error of the trial court in admitting in evidence against appellant the
articles allegedly confiscated during the raid conducted in the house of Jovencio Rodrigueza.

As provided in the present Constitution, a search, to be valid, must generally be authorized by a search warrant
duly issued by the proper government authority. True, in some instances, this Court has allowed government
15

authorities to conduct searches and seizures even without a search warrant. Thus, when the owner of the
premises waives his right against such incursion; when the search is incidental to a lawful arrest; when it is
16 17

made on vessels and aircraft for violation of customs laws; when it is made on automobiles for the purpose of
18

preventing violations of smuggling or immigration laws; when it involves prohibited articles in plain view; or in
19 20

cases of inspection of buildings and other premises for the enforcement of fire, sanitary and building
regulations, a search may be validly made even without a search warrant.
21
In the case at bar, however, the raid conducted by the NARCOM agents in the house of Jovencio Rodrigueza was
not authorized by any search warrant. It does not appear, either, that the situation falls under any of the
aforementioned cases. Hence, appellant's right against unreasonable search and seizure was clearly violated.
The NARCOM agents could not have justified their act by invoking the urgency and necessity of the situation
because the testimonies of the prosecution witnesses reveal that the place had already been put under
surveillance for quite some time. Had it been their intention to conduct the raid, then they should, because they
easily could, have first secured a search warrant during that time.

4. The Court further notes the confusion and ambiguity in the identification of the confiscated marijuana leaves
and other prohibited drug paraphernalia presented as evidence against appellant.

CIC Taduran, who acted as the poseur buyer, testified that appellant sold him 100 grams of dried marijuana
leaves wrapped in a plastic bag. Surprisingly, and no plausible explanation has been advanced therefor, what
were submitted to and examined by the PCCL and thereafter utilized as evidence against the appellant were the
following items:

One (1) red and white colored plastic bag containing the following:

Exh. "A"—Thirty (30) grams of suspected dried marijuana fruiting tops contained inside a transparent plastic bag.

Exh. "B"— Fifty (50) grams of suspected dried marijuana leaves and seeds contained inside a white colored
plastic labelled "Robertson".

Exh. "C"— Four (4) aluminum foils each containing suspected dried marijuana fruiting tops having a total weight
of seven grams then further wrapped with a piece of aluminum foil.

Exh. "D"— Five (5) small transparent plastic bags each containing suspected dried marijuana fruiting tops having
a total weight of seventeen grams.

Exh. "E"— One plastic syringe. 22

Evidently, these prohibited articles were among those confiscated during the so-called follow-up raid in the house
of Jovencio Rodrigueza. The unanswered question then arises as to the identity of the marijuana leaves that
became the basis of appellant's conviction. In People vs. Rubio, this Court had the occasion to rule that the
23 24

plastic bag and the dried marijuana leaves contained therein constitute the corpus delicti of the crime. As such,
the existence thereof must be proved with certainty and conclusiveness. Failure to do so would be fatal to the
cause of the prosecution.

5. It is accepted that, as a rule, minor inconsistencies in the testimony of a witness will not affect his credibility. It
even enhances such credibility because it only shows that he has not been rehearsed. However, when the
25

inconsistencies pertain to material and crucial points, the same detract from his overall credibility.

The exception, rather than the rule, applies in the case at bar. As correctly pointed out by the Solicitor General,
the testimonies of the prosecution witnesses are tainted with serious flaws and material inconsistencies rendering
the same incredible. 26

CIC Taduran, in his testimony, said that they had already been conducting surveillance of the place where the
buy-bust operation was to take place. It turned out, however, that he did not even know the exact place and the
identity of the person from whom he was to buy marijuana leaves. Thus:

FISCAL TOLOSA

Q What place in Tagas were you able to go (to)?

WITNESS

A I am not actually familiar in (sic) that place, in Tagas, although we occasionally passed there.
Q Now, upon your arrival in Tagas, what did you do that afternoon?

A I waited for the suspect because previously, we have already been conducted (sic) surveylance (sic) in the
vicinity.

Q Upon arrival in Tagas, were you able to see the suspect?

A By the road, sir.

Q Who was the first person did you see (sic) when you arrived at Tagas?

A The first person whom I saw is Samuel Segovia.

Q Were you able to talk with this Samuel Segovia?

A According to him, we could get some. 27

The same findings go for the testimony of witness Galutan. In his direct examination, he declared that they
arrested the three accused all at the same time on the fateful night of July 1, 1987. But, in his cross-examination
and as corroborated by the Joint Affidavit of Arrest submitted by him and Molinawe, it appeared that Lonceras
28

and Segovia were arrested on different times and that appellant Don Rodrigueza was not among those who were
arrested. Instead, it was Jovencio Rodrigueza, Don's father, who was picked up at a much later time.

With said inconsistencies in sharp focus, we are constrained to give more credibility to the testimony of appellant
Rodrigueza. While it is true that appellant's defense amounts to an alibi, and as such is the weakest defense in a
criminal prosecution, there are, nonetheless, some evidentiary aspects pointing to the truth in his testimony.
Firstly, the Joint Affidavit of Arrest corroborates his testimony that he was not among those who were arrested on
the night of July 1, 1987. His co-accused Segovia also testified that appellant Rodrigueza was not with them
when they were apprehended by the NARCOM agents.

Secondly, the apparent motive of the NARCOM agents in prosecuting the accused was also revealed during the
trial of the case. Rebuttal witnesses Gracita Bahillo, sister of appellant, and Hospicio Segovia, father of Samuel
Segovia, testified that Sgt. Molinawe, who has since been reportedly dismissed from the service, asked for
P10,000.00 from each of them in exchange for the liberty of the accused. This allegation was never refuted by
29

the prosecution. Hence, the rule laid down by this Court that the statements of prosecution witnesses are entitled
to full faith and credit has no application in the case at bar.
30

Finally, the Court has repeatedly ruled that to sustain the conviction of the accused, the prosecution must rely on
the strength of its own evidence and not on the weakness of the defense. As clearly shown by the evidence, the
31

prosecution has failed to establish its cause. It has not overcome the presumption of innocence accorded to
appellant. This being the case, appellant should not be allowed to suffer for unwarranted and imaginary
imputations against him.

WHEREFORE, the judgment of conviction of the court below is hereby REVERSED and SET ASIDE and
accused-appellant Don Rodrigueza is hereby ACQUITTED of the crime charged. It is hereby ordered that he be
immediately released from custody unless he is otherwise detained for some other lawful cause.

SO ORDERED.
G.R. No. 101837 February 11, 1992

ROLITO GO y TAMBUNTING, petitioner,


vs.
THE COURT OF APPEALS, THE HON. BENJAMIN V. PELAYO, Presiding Judge, Branch 168, Regional
Trial Court, NCJR Pasig, M.M., and PEOPLE OF THE PHILIPPINES, respondents.

FELICIANO, J.:

According to the findings of the San Juan Police in their Investigation Report, on 2 July 1991, Eldon Maguan was
1

driving his car along Wilson St., San Juan, Metro Manila, heading towards P. Guevarra St. Petitioner entered
Wilson St., where it is a one-way street and started travelling in the opposite or "wrong" direction. At the corner of
Wilson and J. Abad Santos Sts., petitioner's and Maguan's cars nearly bumped each other. Petitioner alighted
from his car, walked over and shot Maguan inside his car. Petitioner then boarded his car and left the scene. A
security guard at a nearby restaurant was able to take down petitioner's car plate number. The police arrived
shortly thereafter at the scene of the shooting and there retrieved an empty shell and one round of live
ammunition for a 9 mm caliber pistol. Verification at the Land Transportation Office showed that the car was
registered to one Elsa Ang Go.

The following day, the police returned to the scene of the shooting to find out where the suspect had come from;
they were informed that petitioner had dined at Cravings Bake Shop shortly before the shooting. The police
obtained a facsimile or impression of the credit card used by petitioner from the cashier of the bake shop. The
security guard of the bake shop was shown a picture of petitioner and he positively identified him as the same
person who had shot Maguan. Having established that the assailant was probably the petitioner, the police
launched a manhunt for petitioner.

On 8 July 1991, petitioner presented himself before the San Juan Police Station to verify news reports that he
was being hunted by the police; he was accompanied by two (2) lawyers. The police forthwith detained him. An
eyewitness to the shooting, who was at the police station at that time, positively identified petitioner as the
gunman. That same day, the police promptly filed a complaint for frustrated homicide against petitioner with the
2

Office of the Provincial Prosecutor of Rizal. First Assistant Provincial Prosecutor Dennis Villa Ignacio
("Prosecutor") informed petitioner, in the presence of his lawyers, that he could avail himself of his right to
preliminary investigation but that he must first sign a waiver of the provisions of Article 125 of the Revised Penal
Code. Petitioner refused to execute any such waiver.

On 9 July 1991, while the complaint was still with the Prosecutor, and before an information could be filed in court,
the victim, Eldon Maguan, died of his gunshot wound(s).

Accordingly, on 11 July 1991, the Prosecutor, instead of filing an information for frustrated homicide, filed an
information for murder before the Regional Trial Court. No bail was recommended. At the bottom of the
3

information, the Prosecutor certified that no preliminary investigation had been conducted because the accused
did not execute and sign a waiver of the provisions of Article 125 of the Revised Penal Code.

In the afternoon of the same day, 11 July 1991, counsel for petitioner filed with the Prosecutor an omnibus motion
for immediate release and proper preliminary investigation, alleging that the warrantless arrest of petitioner was
4

unlawful and that no preliminary investigation had been conducted before the information was filed. Petitioner
also prayed that he be released on recognizance or on bail. Provincial Prosecutor Mauro Castro, acting on the
omnibus motion, wrote on the last page of the motion itself that he interposed no objection to petitioner being
granted provisional liberty on a cash bond of P100,000.00.

On 12 July 1991, petitioner filed an urgent ex-parte motion for special raffle in order to expedite action on the
5

Prosecutor's bail recommendation. The case was raffled to the sala of respondent Judge, who, on the same date,
approved the cash bond posted by petitioner and ordered his release. Petitioner was in fact released that same
6 7

day.

On 16 July 1991, the Prosecutor filed with the Regional Trial Court a motion for leave to conduct preliminary
investigation and prayed that in the meantime all proceedings in the court be suspended. He stated that
8

petitioner had filed before the Office of the Provincial Prosecutor of Rizal an omnibus motion for immediate
release and preliminary investigation, which motion had been granted by Provincial Prosecutor Mauro Castro,
who also agreed to recommend cash bail of P100,000.00. The Prosecutor attached to the motion for leave a copy
of petitioner's omnibus motion of 11 July 1991.

Also on 16 July 1991, the trial court issued an Order granting leave to conduct preliminary investigation and
9

cancelling the arraignment set for 15 August 1991 until after the prosecution shall have concluded its preliminary
investigation.

On 17 July 1991, however, respondent Judge motu proprio issued an Order, embodying the following: (1) the
10

12 July 1991 Order which granted bail was recalled; petitioner was given 48 hours from receipt of the Order to
surrender himself; (2) the 16 July 1991 Order which granted leave to the prosecutor to conduct preliminary
investigation was recalled and cancelled; (3) petitioner's omnibus motion for immediate release and preliminary
investigation dated 11 July 1991 was treated as a petition for bail and set for hearing on 23 July 1991.
On 19 July 1991, petitioner filed a petition for certiorari, prohibition and mandamus before the Supreme Court
assailing the 17 July 1991 Order, contending that the information was null and void because no preliminary
investigation had been previously conducted, in violation of his right to due process. Petitioner also moved for
suspension of all proceedings in the case pending resolution by the Supreme Court of his petition; this motion
was, however, denied by respondent Judge.

On 23 July 1991, petitioner surrendered to the police.

By a Resolution dated 24 July 1991, this Court remanded the petition for certiorari, prohibition and mandamus to
the Court of Appeals.

On 16 August 1991, respondent Judge issued an order in open court setting the arraignment of petitioner on 23
August 1991.

On 19 August 1991, petitioner filed with the Court of Appeals a motion to restrain his arraignment.

On 23 August 1991, respondent judge issued a Commitment Order directing the Provincial Warden of Rizal to
admit petitioner into his custody at the Rizal Provincial Jail. On the same date, petitioner was arraigned. In view,
however, of his refusal to enter a plea, the trial court entered for him a plea of not guilty. The Trial court then set
the criminal case for continuous hearings on 19, 24 and 26 September; on 2, 3, 11 and 17 October; and on 7, 8,
14, 15, 21 and 22 November 1991. 11

On 27 August 1991, petitioner filed a petition for habeas corpus 12 in the Court of Appeals. He alleged that in view of public
respondent's failure to join issues in the petition for certiorari earlier filed by him, after the lapse of more than a month, thus prolonging his detention, he was
entitled to be released on habeas corpus.

On 30 August 1991, the Court of Appeals issued the writ of habeas corpus. The petition for certiorari, prohibition
13

and mandamus, on the one hand, and the petition for habeas corpus, upon the other, were subsequently
consolidated in the Court of Appeals.

The Court of Appeals, on 2 September 1991, issued a resolution denying petitioner's motion to restrain his
arraignment on the ground that that motion had become moot and academic.

On 19 September 1991, trial of the criminal case commenced and the prosecution presented its first witness.

On 23 September 1991, the Court of Appeals rendered a consolidated decision dismissing the two (2) petitions, 14

on the following grounds:

a. Petitioner's warrantless arrest was valid because the offense for which he was arrested and charged had been
"freshly committed." His identity had been established through investigation. At the time he showed up at the
police station, there had been an existing manhunt for him. During the confrontation at the San Juan Police
Station, one witness positively identified petitioner as the culprit.

b. Petitioner's act of posting bail constituted waiver of any irregularity attending his arrest. He waived his right to
preliminary investigation by not invoking it properly and seasonably under the Rules.

c. The trial court did not abuse its discretion when it issued the 17 July 1991 Order because the trial court had the
inherent power to amend and control its processes so as to make them conformable to law and justice.

d. Since there was a valid information for murder against petitioner and a valid commitment order (issued by the
trial judge after petitioner surrendered to the authorities whereby petitioner was given to the custody of the
Provincial Warden), the petition for habeas corpus could not be granted.

On 3 October 1991, the prosecution presented three (3) more witnesses at the trial. Counsel for petitioner also
filed a "Withdrawal of Appearance" with the trial court, with petitioner's conformity.
15
On 4 October 1991, the present Petition for Review on Certiorari was filed. On 14 October 1991, the Court issued
a Resolution directing respondent Judge to hold in abeyance the hearing of the criminal case below until further
orders from this Court.

In this Petition for Review, two (2) principal issues need to be addressed: first, whether or not a lawful warrantless
arrest had been effected by the San Juan Police in respect of petitioner Go; and second, whether petitioner had
effectively waived his right to preliminary investigation. We consider these issues seriatim.

In respect of the first issue, the Solicitor General argues that under the facts of the case, petitioner had been
validly arrested without warrant. Since petitioner's identity as the gunman who had shot Eldon Maguan on 2 July
1991 had been sufficiently established by police work, petitioner was validly arrested six (6) days later at the San
Juan Police Station. The Solicitor General invokes Nazareno v. Station Commander, etc., et al., one of the
16

seven (7) cases consolidated with In the Matter of the Petition for Habeas Corpus of Roberto Umil, etc., v. Ramos,
et al. where a majority of the Court upheld a warrantees arrest as valid although effected fourteen (14) days
17

after the killing in connection with which Nazareno had been arrested. Accordingly, in the view of the Solicitor
General, the provisions of Section 7, Rule 112 of the Rules of Court were applicable and because petitioner had
declined to waive the provisions of Article 125 of the Revised Penal Code, the Prosecutor was legally justified in
filing the information for murder even without preliminary investigation.

On the other hand, petitioner argues that he was not lawfully arrested without warrant because he went to the
police station six (6) days after the shooting which he had allegedly perpetrated. Thus, petitioner argues, the
crime had not been "just committed" at the time that he was arrested. Moreover, none of the police officers who
arrested him had been an eyewitness to the shooting of Maguan and accordingly none had the "personal
knowledge" required for the lawfulness of a warrantees arrest. Since there had been no lawful warrantless arrest.
Section 7, Rule 112 of the Rules of Court which establishes the only exception to the right to preliminary
investigation, could not apply in respect of petitioner.

The reliance of both petitioner and the Solicitor General upon Umil v. Ramos is, in the circumstances of this case,
misplaced. In Umil v. Ramos, by an eight-to-six vote, the Court sustained the legality of the warrantless arrests of
petitioners made from one (1) to fourteen days after the actual commission of the offenses, upon the ground that
such offenses constituted "continuing crimes." Those offenses were subversion, membership in an outlawed
organization like the New People's Army, etc. In the instant case, the offense for which petitioner was arrested
was murder, an offense which was obviously commenced and completed at one definite location in time and
space. No one had pretended that the fatal shooting of Maguan was a "continuing crime."

Secondly, we do not believe that the warrantees "arrest" or detention of petitioner in the instant case falls within
the terms of Section 5 of Rule 113 of the 1985 Rules on Criminal Procedure which provides as follows:

Sec. 5 Arrest without warrant; when lawful. — A peace officer or a private person may, without warrant, arrest a
person:

(a) When, in his presence, the person to be arrested has committed, is actually committing, or is attempting to
commit an offense;

(b) When an offense has in fact just been committed, and he has personal knowledge of facts indicating that the
person to be arrested has committed it; and

(c) When the person to be arrested is a prisoner who has escaped from a penal establishment or place where he
is serving final judgment or temporarily confined while his case is pending, or has escaped while being transferred
from one confinement to another.

In cases falling under paragraphs (a) and (b) hereof, the person arrested without a warrant shall be forthwith
delivered to the nearest police station or jail, and he shall be proceed against in accordance with Rule 112,
Section 7.

Petitioner's "arrest" took place six (6) days after the shooting of Maguan. The "arresting" officers obviously were
not present, within the meaning of Section 5(a), at the time petitioner had allegedly shot Maguan. Neither could
the "arrest" effected six (6) days after the shooting be reasonably regarded as effected "when [the shooting had]
in fact just been committed" within the meaning of Section 5(b). Moreover, none of the "arresting" officers had any
"personal knowledge" of facts indicating that petitioner was the gunman who had shot Maguan. The information
upon which the police acted had been derived from statements made by alleged eyewitnesses to the shooting —
one stated that petitioner was the gunman; another was able to take down the alleged gunman's car's plate
number which turned out to be registered in petitioner's wife's name. That information did not, however, constitute
"personal knowledge." 18

It is thus clear to the Court that there was no lawful warrantless arrest of petitioner within the meaning of Section 5
of Rule 113. It is clear too that Section 7 of Rule 112, which provides:

Sec. 7 When accused lawfully arrested without warrant. — When a person is lawfully arrested without a warrant
for an offense cognizable by the Regional Trial Court the complaint or information may be filed by the offended
party, peace officer or fiscal without a preliminary investigation having been first conducted, on the basis of the
affidavit of the offended party or arresting office or person

However, before the filing of such complaint or information, the person arrested may ask for a preliminary
investigation by a proper officer in accordance with this Rule, but he must sign a waiver of the provisions of Article
125 of the Revised Penal Code, as amended, with the assistance of a lawyer and in case of non-availability of a
lawyer, a responsible person of his choice. Notwithstanding such waiver, he may apply for bail as provided in the
corresponding rule and the investigation must be terminated within fifteen (15) days from its inception.

If the case has been filed in court without a preliminary investigation having been first conducted, the accused
may within five (5) days from the time he learns of the filing of the information, ask for a preliminary
investigation with the same right to adduce evidence in his favor in the manner prescribed in this Rule. (Emphasis
supplied)

is also not applicable. Indeed, petitioner was not arrested at all. When he walked into San Juan Police Station,
accompanied by two (2) lawyers, he in fact placed himself at the disposal of the police authorities. He did not
state that he was "surrendering" himself, in all probability to avoid the implication he was admitting that he had
slain Eldon Maguan or that he was otherwise guilty of a crime. When the police filed a complaint for frustrated
homicide with the Prosecutor, the latter should have immediately scheduled a preliminary investigation to
determine whether there was probable cause for charging petitioner in court for the killing of Eldon Maguan.
Instead, as noted earlier, the Prosecutor proceed under the erroneous supposition that Section 7 of Rule 112 was
applicable and required petitioner to waive the provisions of Article 125 of the Revised Penal Code as a condition
for carrying out a preliminary investigation. This was substantive error, for petitioner was entitled to a preliminary
investigation and that right should have been accorded him without any conditions. Moreover, since petitioner had
not been arrested, with or without a warrant, he was also entitled to be released forthwith subject only to his
appearing at the preliminary investigation.

Turning to the second issue of whether or not petitioner had waived his right to preliminary investigation, we note
that petitioner had from the very beginning demanded that a preliminary investigation be conducted. As earlier
pointed out, on the same day that the information for murder was filed with the Regional Trial Court, petitioner
filed with the Prosecutor an omnibus motion for immediate release and preliminary investigation. The Solicitor
General contends that that omnibus motion should have been filed with the trial court and not with the Prosecutor,
and that the petitioner should accordingly be held to have waived his right to preliminary investigation. We do not
believe that waiver of petitioner's statutory right to preliminary investigation may be predicated on such a slim
basis. The preliminary investigation was to be conducted by the Prosecutor, not by the Regional Trial Court. It is
true that at the time of filing of petitioner's omnibus motion, the information for murder had already been filed with
the Regional Trial Court: it is not clear from the record whether petitioner was aware of this fact at the time his
omnibus motion was actually filed with the Prosecutor. In Crespo v. Mogul, this Court held:
19

The preliminary investigation conducted by the fiscal for the purpose of determining whether a prima facie case
exists to warranting the prosecution of the accused is terminated upon the filing of the information in the proper
court. In turn, as above stated, the filing of said information sets in motion the criminal action against the accused
in Court. Should the fiscal find it proper to conduct a reinvestigation of the case, at such stage, the permission of
the Court must be secured. After such reinvestigation the finding and recommendations of the fiscal should be
submitted to the Court for appropriate action.While it is true that the fiscal has the quasi-judicial discretion to
determine whether or not a criminal case should be filed in court or not, once the case had already been brought
to Court whatever disposition the fiscal may feel should be proper in the case thereafter should be addressed for
the consideration of the Court. The only qualification is that the action of the Court must not impair the substantial
rights of the accused., or the right of the People to due process of law.

xxx xxx xxx

The rule therefore in this jurisdiction is that once a complaint or information is filed in Court any disposition of the
case [such] as its dismissal or the conviction or acquittal of the accused rests in the sound discretion of the
Court. Although the fiscal retains the direction and control of the prosecution of criminal cases even while the
case is already in Court he cannot impose his opinion on the trial court. The Court is the best and sole judge on
what to do with the case before it. . . . (Citations omitted; emphasis supplied)
20

Nonetheless, since petitioner in his omnibus motion was asking for preliminary investigation and not for a
re-investigation (Crespo v. Mogul involved a re-investigation), and since the Prosecutor himself did file with the
trial court, on the 5th day after filing the information for murder, a motion for leave to conduct preliminary
investigation (attaching to his motion a copy of petitioner's omnibus motion), we conclude that petitioner's
omnibus motion was in effect filed with the trial court. What was crystal clear was that petitioner did ask for a
preliminary investigation on the very day that the information was filed without such preliminary investigation, and
that the trial court was five (5) days later apprised of the desire of the petitioner for such preliminary investigation.
Finally, the trial court did in fact grant the Prosecutor's prayer for leave to conduct preliminary investigation. Thus,
even on the (mistaken) supposition apparently made by the Prosecutor that Section 7 of Rule 112 of the Revised
Court was applicable, the 5-day reglementary period in Section 7, Rule 112 must be held to have been
substantially complied with.

We believe and so hold that petitioner did not waive his right to a preliminary investigation. While that right is
statutory rather than constitutional in its fundament, since it has in fact been established by statute, it is a
component part of due process in criminal justice. The right to have a preliminary investigation conducted
21

before being bound over to trial for a criminal offense and hence formally at risk of incarceration or some other
penalty, is not a mere formal or technical right; it is a substantive right. The accused in a criminal trial is inevitably
exposed to prolonged anxiety, aggravation, humiliation, not to speak of expense; the right to an opportunity to
avoid a process painful to any one save, perhaps, to hardened criminals, is a valuable right. To deny petitioner's
claim to a preliminary investigation would be to deprive him the full measure of his right to due process.

The question may be raised whether petitioner still retains his right to a preliminary investigation in the instant
case considering that he was already arraigned on 23 August 1991. The rule is that the right to preliminary
investigation is waived when the accused fails to invoke it before or at the time of entering a plea at
arraignment. In the instant case, petitioner Go had vigorously insisted on his right to preliminary
22

investigation before his arraignment. At the time of his arraignment, petitioner was already before the Court of
Appeals on certiorari, prohibition and mandamusprecisely asking for a preliminary investigation before being
forced to stand trial.

Again, in the circumstances of this case, we do not believe that by posting bail petitioner had waived his right to
preliminary investigation. In People v. Selfaison, we did hold that appellants there had waived their right to
23

preliminary investigation because immediately after their arrest, they filed bail and proceeded to trial "without
previously claiming that they did not have the benefit of a preliminary investigation." In the instant case,
24

petitioner Go asked for release on recognizance or on bail and for preliminary investigation in one omnibus
motion. He had thus claimed his right to preliminary investigation before respondent Judge approved the cash
bond posted by petitioner and ordered his release on 12 July 1991. Accordingly, we cannot reasonably imply
waiver of preliminary investigation on the part of petitioner. In fact, when the Prosecutor filed a motion in court
asking for leave to conduct preliminary investigation, he clearly if impliedly recognized that petitioner's claim to
preliminary investigation was a legitimate one.

We would clarify, however, that contrary to petitioner's contention the failure to accord preliminary investigation,
while constituting a denial of the appropriate and full measure of the statutory process of criminal justice, did not
impair the validity of the information for murder nor affect the jurisdiction of the trial court.
25

It must also be recalled that the Prosecutor had actually agreed that petitioner was entitled to bail. This was
equivalent to an acknowledgment on the part of the Prosecutor that the evidence of guilt then in his hands was
not strong. Accordingly, we consider that the 17 July 1991 order of respondent Judge recalling his own order
granting bail and requiring petitioner to surrender himself within forty-eight (48) hours from notice, was plainly
arbitrary considering that no evidence at all — and certainly no new or additional evidence — had been submitted
to respondent Judge that could have justified the recall of his order issued just five (5) days before. It follows that
petitioner was entitled to be released on bail as a matter of right.

The final question which the Court must face is this: how does the fact that, in the instant case, trial on the merits
has already commenced, the Prosecutor having already presented four (4) witnesses, impact upon, firstly,
petitioner's right to a preliminary investigation and, secondly, petitioner's right to be released on bail? Does he
continue to be entitled to have a preliminary investigation conducted in respect of the charge against him? Does
petitioner remain entitled to be released on bail?

Turning first to the matter of preliminary investigation, we consider that petitioner remains entitled to a preliminary
investigation although trial on the merits has already began. Trial on the merits should be suspended or held in
abeyance and a preliminary investigation forthwith accorded to petitioner. It is true that the Prosecutor might, in
26

view of the evidence that he may at this time have on hand, conclude that probable cause exists; upon the other
hand, the Prosecutor conceivably could reach the conclusion that the evidence on hand does not warrant a
finding of probable cause. In any event, the constitutional point is that petitioner was not accorded what he was
entitled to by way of procedural due process. Petitioner was forced to undergo arraignment and literally pushed
27

to trial without preliminary investigation, with extraordinary haste, to the applause from the audience that filled the
courtroom. If he submitted to arraignment at trial, petitioner did so "kicking and screaming," in a manner of
speaking . During the proceedings held before the trial court on 23 August 1991, the date set for arraignment of
petitioner, and just before arraignment, counsel made very clear petitioner's vigorous protest and objection to the
arraignment precisely because of the denial of preliminary investigation. So energetic and determined were
28

petitioner's counsel's protests and objections that an obviously angered court and prosecutor dared him to
withdraw or walkout, promising to replace him with counsel de oficio. During the trial, before the prosecution
called its first witness, petitioner through counsel once again reiterated his objection to going to trial without
preliminary investigation: petitioner's counsel made of record his "continuing objection." Petitioner had promptly
29

gone to the appellate court on certiorari and prohibition to challenge the lawfulness of the procedure he was being
forced to undergo and the lawfulness of his detention. If he did not walk out on the trial, and if he cross-examined
30

the prosecution's witnesses, it was because he was extremely loath to be represented by counsel de
oficio selected by the trial judge, and to run the risk of being held to have waived also his right to use what is
frequently the only test of truth in the judicial process.

In respect of the matter of bail, we similarly believe and so hold that petitioner remains entitled to be released on
bail as a matter of right. Should the evidence already of record concerning petitioner's guilt be, in the reasonable
belief of the Prosecutor, strong, the Prosecutor may move in the trial court for cancellation of petitioner's bail. It
would then be up to the trial court, after a careful and objective assessment of the evidence on record, to grant or
deny the motion for cancellation of bail.

To reach any other conclusions here, that is, to hold that petitioner's rights to a preliminary investigation and to
bail were effectively obliterated by evidence subsequently admitted into the record would be to legitimize the
deprivation of due process and to permit the Government to benefit from its own wrong or culpable omission and
effectively to dilute important rights of accused persons well-nigh to the vanishing point. It may be that to require
the State to accord petitioner his rights to a preliminary investigation and to bail at this point, could turn out
ultimately to be largely a ceremonial exercise. But the Court is not compelled to speculate. And, in any case, it
would not be idleceremony; rather, it would be a celebration by the State of the rights and liberties of its own
people and a re-affirmation of its obligation and determination to respect those rights and liberties.

ACCORDINGLY, the Court resolved to GRANT the Petition for Review on Certiorari. The Order of the trial court
dated 17 July 1991 is hereby SET ASIDE and NULLIFIED, and the Decision of the Court of Appeals dated 23
September 1991 hereby REVERSED.

The Office of the Provincial Prosecutor is hereby ORDERED to conduct forthwith a preliminary investigation of
the charge of murder against petitioner Go, and to complete such preliminary investigation within a period of
fifteen (15) days from commencement thereof. The trial on the merits of the criminal case in the Regional Trial
Court shall be SUSPENDED to await the conclusion of the preliminary investigation.
Meantime, petitioner is hereby ORDERED released forthwith upon posting of a cash bail bond of One Hundred
Thousand Pesos (P100,000.00). This release shall be without prejudice to any lawful order that the trial court may
issue, should the Office of the Provincial Prosecutor move for cancellation of bail at the conclusion of the
preliminary investigation.

No pronouncement as to costs. This Decision is immediately executory.

SO ORDERED.

G.R. No. 89139 August 2, 1990

ROMEO POSADAS y ZAMORA, petitioner,


vs.
THE HONORABLE COURT OF APPEALS and THE PEOPLE OF THE PHILIPPINES, respondents.

Rudy G. Agravate for petitioner.

GANCAYCO, J.:

The validity of a warrantless search on the person of petitioner is put into issue in this case.

On October 16, 1986 at about 10:00 o'clock in the morning Pat. Ursicio Ungab and Pat. Umbra Umpar, both
members of the Integrated National Police (INP) of the Davao Metrodiscom assigned with the Intelligence Task
Force, were conducting a surveillance along Magallanes Street, Davao City. While they were within the premises
of the Rizal Memorial Colleges they spotted petitioner carrying a "buri" bag and they noticed him to be acting
suspiciously.

They approached the petitioner and identified themselves as members of the INP. Petitioner attempted to flee but
his attempt to get away was thwarted by the two notwithstanding his resistance.

They then checked the "buri" bag of the petitioner where they found one (1) caliber .38 Smith & Wesson revolver
with Serial No. 770196 two (2) rounds of live ammunition for a .38 caliber gun a smoke (tear gas) grenade, and
1 2 3

two (2) live ammunitions for a .22 caliber gun. They brought the petitioner to the police station for further
4

investigation. In the course of the same, the petitioner was asked to show the necessary license or authority to
possess firearms and ammunitions found in his possession but he failed to do so. He was then taken to the
Davao Metrodiscom office and the prohibited articles recovered from him were indorsed to M/Sgt. Didoy the
officer then on duty. He was prosecuted for illegal possession of firearms and ammunitions in the Regional Trial
Court of Davao City wherein after a plea of not guilty and trial on the merits a decision was rendered on October 8,
1987 finding petitioner guilty of the offense charged as follows:

WHEREFORE, in view of all the foregoing, this Court , finds the accused guilty beyond reasonable doubt of the
offense charged.

It appearing that the accuse d was below eighteen (18) years old at the time of the commission of the offense (Art.
68, par. 2), he is hereby sentenced to an indeterminate penalty ranging from TEN (10) YEARS and ONE (1) DAY
of prision mayor to TWELVE (12) Years, FIVE (5) months and Eleven (11) days of Reclusion Temporal, and to
pay the costs.

The firearm, ammunitions and smoke grenade are forfeited in favor of the government and the Branch Clerk of
Court is hereby directed to turn over said items to the Chief, Davao Metrodiscom, Davao City. 5

Not satisfied therewith the petitioner interposed an appeal to the Court of Appeals wherein in due course a
decision was rendered on February 23, 1989 affirming in toto the appealed decision with costs against the
petitioner. 6
Hence, the herein petition for review, the main thrust of which is that there being no lawful arrest or search and
seizure, the items which were confiscated from the possession of the petitioner are inadmissible in evidence
against him.

The Solicitor General, in justifying the warrantless search of the buri bag then carried by the petitioner, argues
that under Section 12, Rule 136 of the Rules of Court a person lawfully arrested may be searched for dangerous
weapons or anything used as proof of a commission of an offense without a search warrant. It is further alleged
that the arrest without a warrant of the petitioner was lawful under the circumstances.

Section 5, Rule 113 of the 1985 Rules on Criminal Procedure provides as follows:

SEC. 5. Arrest without warrant; when lawful — A peace officer or a private person may, without a warrant, arrest
a person:

(a) When in his presence, the person to be arrested has committed is actually committing, or is attempting to
commit an offense;

(b) When an offense has in fact just been committed, and he has personal knowledge of facts indicating that the
person to be arrested has committed it; and

(c) When the person to be arrested is a prisoner who has escaped from a penal establishment or place where he
is serving final judgment or temporarily confined while his case is pending, or has escaped while being transferred
from one confinement to another.

In cases falling under paragraphs (a) and (b) hereof, the person arrested without a warrant shall be forthwith
delivered to the nearest police station or jail, and he shall be proceeded against in accordance with Rule 112,
Section 7. (6a, 17a)

From the foregoing provision of law it is clear that an arrest without a warrant may be effected by a peace officer
or private person, among others, when in his presence the person to be arrested has committed, is actually
committing, or is attempting to commit an offense; or when an offense has in fact just been committed, and he
has personal knowledge of the facts indicating that the person arrested has committed it.

The Solicitor General argues that when the two policemen approached the petitioner, he was actually committing
or had just committed the offense of illegal possession of firearms and ammunitions in the presence of the police
officers and consequently the search and seizure of the contraband was incidental to the lawful arrest in
accordance with Section 12, Rule 126 of the 1985 Rules on Criminal Procedure. We disagree.

At the time the peace officers in this case identified themselves and apprehended the petitioner as he attempted
to flee they did not know that he had committed, or was actually committing the offense of illegal possession of
firearms and ammunitions. They just suspected that he was hiding something in the buri bag. They did now know
what its contents were. The said circumstances did not justify an arrest without a warrant.

However, there are many instances where a warrant and seizure can be effected without necessarily being
preceded by an arrest, foremost of which is the "stop and search" without a search warrant at military or police
checkpoints, the constitutionality or validity of which has been upheld by this Court in Valmonte vs. de Villa, as7

follows:

Petitioner Valmonte's general allegation to the effect that he had been stopped and searched without a search
warrant by the military manning the checkpoints, without more, i.e., without stating the details of the incidents
which amount to a violation of his light against unlawful search and seizure, is not sufficient to enable the Court to
determine whether there was a violation of Valmonte's right against unlawful search and seizure. Not all searches
and seizures are prohibited. Those which are reasonable are not forbidden. A reasonable search is not to be
determined by any fixed formula but is to be resolved according to the facts of each case.

Where, for example, the officer merely draws aside the curtain of a vacant vehicle which is parked on the public
fair grounds, or simply looks into a vehicle or flashes a light therein, these do not constitute unreasonable search.
The setting up of the questioned checkpoints in Valenzuela (and probably in other areas) may be considered as a
security measure to enable the NCRDC to pursue its mission of establishing effective territorial defense and
maintaining peace and order for the benefit of the public. Checkpoints may also be regarded as measures to
thwart plots to destabilize the government in the interest of public security. In this connection, the Court may take
judicial notice of the shift to urban centers and their suburbs of the insurgency movement, so clearly reflected in
the increased killings in cities of police and military men by NPA "sparrow units," not to mention the abundance of
unlicensed firearms and the alarming rise in lawlessness and violence in such urban centers, not all of which are
reported in media, most likely brought about by deteriorating economic conditions — which all sum up to what
one can rightly consider, at the very least, as abnormal times. Between the inherent right of the state to protect its
existence and promote public welfare and an individual's right against a warrantless search which is however
reasonably conducted, the former should prevail.

True, the manning of checkpoints by the military is susceptible of abuse by the men in uniform in the same
manner that all governmental power is susceptible of abuse. But, at the cost of occasional inconvenience,
discomfort and even irritation to the citizen, the checkpoints during these abnormal times, when conducted within
reasonable limits, are part of the price we pay for an orderly society and a peaceful community. (Emphasis
supplied).

Thus, as between a warrantless search and seizure conducted at military or police checkpoints and the search
thereat in the case at bar, there is no question that, indeed, the latter is more reasonable considering that unlike in
the former, it was effected on the basis of a probable cause. The probable cause is that when the petitioner acted
suspiciously and attempted to flee with the buri bag there was a probable cause that he was concealing
something illegal in the bag and it was the right and duty of the police officers to inspect the same.

It is too much indeed to require the police officers to search the bag in the possession of the petitioner only after
they shall have obtained a search warrant for the purpose. Such an exercise may prove to be useless, futile and
much too late.

In People vs. CFI of Rizal, this Court held as follows:


8

. . . In the ordinary cases where warrant is indispensably necessary, the mechanics prescribed by the Constitution
and reiterated in the Rules of Court must be followed and satisfied. But We need not argue that there are
exceptions. Thus in the extraordinary events where warrant is not necessary to effect a valid search or seizure, or
when the latter cannot be performed except without warrant, what constitutes a reasonable or unreasonable
search or seizure becomes purely a judicial question, determinable from the uniqueness of the circumstances
involved, including the purpose of the search or seizure, the presence or absence of probable cause, the manner
in which the search and seizure was made, the place or thing searched and the character of the articles procured.

The Court reproduces with approval the following disquisition of the Solicitor General:

The assailed search and seizure may still be justified as akin to a "stop and frisk" situation whose object is either
to determine the identity of a suspicious individual or to maintain the status quo momentarily while the police
officer seeks to obtain more information. This is illustrated in the case of Terry vs. Ohio, 392 U.S. 1 (1968). In this
case, two men repeatedly walked past a store window and returned to a spot where they apparently conferred
with a third man. This aroused the suspicion of a police officer. To the experienced officer, the behaviour of the
men indicated that they were sizing up the store for an armed robbery. When the police officer approached the
men and asked them for their names, they mumbled a reply. Whereupon, the officer grabbed one of them, spun
him around and frisked him. Finding a concealed weapon in one, he did the same to the other two and found
another weapon. In the prosecution for the offense of carrying a concealed weapon, the defense of illegal search
and seizure was put up. The United States Supreme Court held that "a police officer may in appropriate
circumstances and in an appropriate manner approach a person for the purpose of investigating possible criminal
behaviour even though there is no probable cause to make an arrest." In such a situation, it is reasonable for an
officer rather than simply to shrug his shoulder and allow a crime to occur, to stop a suspicious individual briefly in
order to determine his identity or maintain thestatus quo while obtaining more information. . . .

Clearly, the search in the case at bar can be sustained under the exceptions heretofore discussed, and hence,
the constitutional guarantee against unreasonable searches and seizures has not been violated. 9
WHEREFORE, the petition is DENIED with costs against petitioner.

SO ORDERED.

G.R. No. 87059 June 22, 1992

THE PEOPLE OF THE PHILIPPINES, plaintiff-appellee,


vs.
ROGELIO MENGOTE y TEJAS, accused-appellant.

CRUZ, J.:

Accused-appellant Rogelio Mengote was convicted of illegal possession of firearms on the strength mainly of the
stolen pistol found on his person at the moment of his warrantless arrest. In this appeal, he pleads that the
weapon was not admissible as evidence against him because it had been illegally seized and was therefore the
fruit of the poisonous tree. The Government disagrees. It insists that the revolver was validly received in evidence
by the trial judge because its seizure was incidental to an arrest that was doubtless lawful even if admittedly
without warrant.

The incident occurred shortly before noon of August 8, 1987, after the Western Police District received a
telephone call from an informer that there were three suspicious-looking persons at the corner of Juan Luna and
North Bay Boulevard in Tondo, Manila. A surveillance team of plainclothesmen was forthwith dispatched to the
place. As later narrated at the trial by Patrolmen Rolando Mercado and Alberto Juan, they there saw two men
1

"looking from side to side," one of whom was holding his abdomen. They approached these persons and
identified themselves as policemen, whereupon the two tried to run away but were unable to escape because the
other lawmen had surrounded them. The suspects were then searched. One of them, who turned out to be the
accused-appellant, was found with a .38 caliber Smith and Wesson revolver with six live bullets in the chamber.
His companion, later identified as Nicanor Morellos, had a fan knife secreted in his front right pants pocket. The
weapons were taken from them. Mengote and Morellos were then turned over to police headquarters for
investigation by the Intelligence Division.

On August 11, 1987, the following information was filed against the accused-appellant before the Regional Trial
Court of Manila:

The undersigned accuses ROGELIO MENGOTE y TEJAS of a violation of Presidential Decree No. 1866,
committed as follows:
That on or about August 8, 1987, in the City of Manila, Philippines, the said accused did then and there wilfully,
unlawfully and knowingly have in his possession and under his custody and control a firearm, to wit:

one (1) cal. 38 "S & W" bearing


Serial No. 8720-T

without first having secured the necessary license or permit therefor from the proper authorities.

Besides the police officers, one other witness presented by the prosecution was Rigoberto Danganan, who
identified the subject weapon as among the articles stolen from him during the robbery in his house in Malabon on
June 13, 1987. He pointed to Mengote as one of the robbers. He had duly reported the robbery to the police,
indicating the articles stolen from him, including the revolver. For his part, Mengote made no effort to prove that
2

he owned the firearm or that he was licensed to possess it and claimed instead that the weapon had been
"Planted" on him at the time of his arrest. 3

The gun, together with the live bullets and its holster, were offered as Exhibits A, B, and C and admitted over the
objection of the defense. As previously stated, the weapon was the principal evidence that led to Mengote's
conviction for violation of P.D. 1866. He was sentenced to reclusion
perpetua. 4

It is submitted in the Appellant's Brief that the revolver should not have been admitted in evidence because of its
illegal seizure. no warrant therefor having been previously obtained. Neither could it have been seized as an
incident of a lawful arrest because the arrest of Mengote was itself unlawful, having been also effected without a
warrant. The defense also contends that the testimony regarding the alleged robbery in Danganan's house was
irrelevant and should also have been disregarded by the trial court.

The following are the pertinent provision of the Bill of Rights:

Sec. 2. The right of the people to be secure in their persons, houses, papers, and effects against unreasonable
searches and seizures of whatever nature and for any purpose shall be inviolable, and no search warrant or
warrant of arrest shall issue except upon probable cause to be determined personally by the judge after
examination under oath or affirmation of the complainant and the witnesses he may produce, and particularly
describing the place to be searched and the persons or things to be seized.

Sec. 3 (1). The privacy of communication and correspondence shall be inviolable except upon lawful order of the
court, or when public safety or order requires otherwise as prescribed by law.

(2) Any evidence obtained in violation of this or the preceding section shall be inadmissible for any purpose in any
proceeding.

There is no question that evidence obtained as a result of an illegal search or seizure is inadmissible in any
proceeding for any purpose. That is the absolute prohibition of Article III, Section 3(2), of the Constitution. This is
the celebrated exclusionary rule based on the justification given by Judge Learned Hand that "only in case the
prosecution, which itself controls the seizing officials, knows that it cannot profit by their wrong will the wrong be
repressed." The Solicitor General, while conceding the rule, maintains that it is not applicable in the case at bar.
His reason is that the arrest and search of Mengote and the seizure of the revolver from him were lawful under
Rule 113, Section 5, of the Rules of Court reading as follows:

Sec. 5. Arrest without warrant when lawful. — A peace officer or private person may, without a warrant, arrest a
person;

(a) When, in his presence, the person to be arrested has committed, is actually committing, or is attempting to
commit an offense;

(b) When an offense has in fact just been committed, and he has personal knowledge of facts indicating that the
person to be arrested has committed it; and
(c) When the person to be arrested is a prisoner who has escaped from a penal establishment or place where he
is serving final judgment or temporarily confined while his case is pending, or has escaped while being transferred
from one confinement to another.

In cases failing under paragraphs (a) and (b) hereof, the person arrested without a warrant shall be forthwith
delivered to the nearest police station or jail, and he shall be proceeded against in accordance with Rule 112,
Section 7.

We have carefully examined the wording of this Rule and cannot see how we can agree with the prosecution.

Par. (c) of Section 5 is obviously inapplicable as Mengote was not an escapee from a penal institution when he
was arrested. We therefore confine ourselves to determining the lawfulness of his arrest under either Par. (a) or
Par. (b) of this section.

Par. (a) requires that the person be arrested (1) after he has committed or while he is actually committing or is at
least attempting to commit an offense, (2) in the presence of the arresting officer.

These requirements have not been established in the case at bar. At the time of the arrest in question, the
accused-appellant was merely "looking from side to side" and "holding his abdomen," according to the arresting
officers themselves. There was apparently no offense that had just been committed or was being actually
committed or at least being attempted by Mengote in their presence.

The Solicitor General submits that the actual existence of an offense was not necessary as long as Mengote's
acts "created a reasonable suspicion on the part of the arresting officers and induced in them the belief that an
offense had been committed and that the accused-appellant had committed it." The question is, What offense?
What offense could possibly have been suggested by a person "looking from side to side" and "holding his
abdomen" and in a place not exactly forsaken?

These are certainly not sinister acts. And the setting of the arrest made them less so, if at all. It might have been
different if Mengote bad been apprehended at an ungodly hour and in a place where he had no reason to be, like
a darkened alley at 3 o'clock in the morning. But he was arrested at 11:30 in the morning and in a crowded street
shortly after alighting from a passenger jeep with I his companion. He was not skulking in the shadows but
walking in the clear light of day. There was nothing clandestine about his being on that street at that busy hour in
the blaze of the noonday sun.

On the other hand, there could have been a number of reasons, all of them innocent, why his eyes were darting
from side to side and be was holding his abdomen. If they excited suspicion in the minds of the arresting officers,
as the prosecution suggests, it has nevertheless not been shown what their suspicion was all about. In fact, the
policemen themselves testified that they were dispatched to that place only because of the telephone call from
the informer that there were "suspicious-looking" persons in that vicinity who were about to commit a robbery at
North Bay Boulevard. The caller did not explain why he thought the men looked suspicious nor did he elaborate
on the impending crime.

In the recent case of People v. Malmstedt, the Court sustained the warrantless arrest of the accused because
5

there was a bulge in his waist that excited the suspicion of the arresting officer and, upon inspection, turned out to
be a pouch containing hashish. In People v. Claudio, the accused boarded a bus and placed the buri bag she
6

was carrying behind the seat of the arresting officer while she herself sat in the seat before him. His suspicion
aroused, be surreptitiously examined the bag, which he found to contain marijuana. He then and there made the
warrantless arrest and seizure that we subsequently upheld on the ground that probable cause had been
sufficiently established.

The case before us is different because there was nothing to support the arresting officers' suspicion other than
Mengote's darting eyes and his hand on his abdomen. By no stretch of the imagination could it have been inferred
from these acts that an offense had just been committed, or was actually being committed, or was at least being
attempted in their presence.

This case is similar to People v. Aminnudin, where the Court held that the warrantless arrest of the accused was
7

unconstitutional. This was effected while be was coming down a vessel, to all appearances no less innocent than
the other disembarking passengers. He had not committed nor was be actually committing or attempting to
commit an offense in the presence of the arresting officers. He was not even acting suspiciously. In short, there
was no probable cause that, as the prosecution incorrectly suggested, dispensed with the constitutional
requirement of a warrant.

Par. (b) is no less applicable because its no less stringent requirements have also not been satisfied. The
prosecution has not shown that at the time of Mengote's arrest an offense had in fact just been committed and
that the arresting officers had personal knowledge of facts indicating that Mengote had committed it. All they had
was hearsay information from the telephone caller, and about a crime that had yet to be committed.

The truth is that they did not know then what offense, if at all, had been committed and neither were they aware of
the participation therein of the accused-appellant. It was only later, after Danganan had appeared at the Police
headquarters, that they learned of the robbery in his house and of Mengote's supposed involvement therein. 8 As
for the illegal possession of the firearm found on Mengote's person, the policemen discovered this only after he had been searched and the investigation
conducted later revealed that he was not its owners nor was he licensed to possess it.

Before these events, the Peace officers had no knowledge even of Mengote' identity, let alone the fact (or
suspicion) that he was unlawfully carrying a firearm or that he was involved in the robbery of Danganan's house.

In the landmark case of People v. Burgos, this Court declared:


9

Under Section 6(a) of Rule 113, the officer arresting a person who has just committed, is committing, or is about
to commit an offense must have personal knowledge of the fact. The offense must also be committed in his
presence or within his view. (Sayo v. Chief of Police, 80 Phil. 859). (Emphasis supplied)

xxx xxx xxx

In arrests without a warrant under Section 6(b), however, it is not enough that there is reasonable ground to
believe that the person to be arrested has committed a crime. A crime must in fact or actually have been
committed first. That a crime has actually been committed is an essential precondition. It is not enough to suspect
that a crime may have been committed. The fact of the commission of the offense must be undisputed. The test of
reasonable ground applies only to the identity of the perpetrator. (Emphasis supplied)

This doctrine was affirmed in Alih v. Castro, thus: 10

If the arrest was made under Rule 113, Section 5, of the Rules of Court in connection with a crime about to be
committed, being committed, or just committed, what was that crime? There is no allegation in the record of such
a falsification. Parenthetically, it may be observed that under the Revised Rule 113, Section 5(b), the officer
making the arrest must have personal knowledge of the ground therefor as stressed in the recent case of People
v. Burgos. (Emphasis supplied)

It would be a sad day, indeed, if any person could be summarily arrested and searched just because he is holding
his abdomen, even if it be possibly because of a stomach-ache, or if a peace officer could clamp handcuffs on
any person with a shifty look on suspicion that he may have committed a criminal act or is actually committing or
attempting it. This simply cannot be done in a free society. This is not a police state where order is exalted over
liberty or, worse, personal malice on the part of the arresting officer may be justified in the name of security.

There is no need to discuss the other issues raised by the accused-appellant as the ruling we here make is
sufficient to sustain his exoneration. Without the evidence of the firearm taken from him at the time of his illegal
arrest, the prosecution has lost its most important exhibit and must therefore fail. The testimonial evidence
against Mengote (which is based on the said firearm) is not sufficient to prove his guilt beyond reasonable doubt
of the crime imputed to him.

We commend Atty. Violeta Calvo-Drilon for her able and spirited defense of the accused-appellant not only in the
brief but also in the reply brief, which she did not have to file but did so just the same to stress the constitutional
rights of her client. The fact that she was acting only as a counsel de oficio with no expectation of material reward
makes her representation even more commendable.
The Court feels that if the peace officers had been more mindful of the provisions of the Bill of Rights, the
prosecution of the accused-appellant might have succeeded. As it happened, they allowed their
over-zealousness to get the better of them, resulting in their disregard of the requirements of a valid search and
seizure that rendered inadmissible the vital evidence they had invalidly seized.

This should be a lesson to other peace officers. Their impulsiveness may be the very cause of the acquittal of
persons who deserve to be convicted, escaping the clutches of the law because, ironically enough, it has not
been observed by those who are supposed to enforce it.

WHEREFORE, the appealed decision is REVERSED and SET ASIDE. The accused-appellant is ACQUITTED
and ordered released immediately unless he is validly detained for other offenses. No costs.

SO ORDERED.

[G.R. No. 123595. December 12, 1997]

SAMMY MALACAT y MANDAR, petitioner, vs. COURT OF APPEALS, and


PEOPLE OF THE PHILIPPINES, respondents.

DECISION
DAVIDE, JR., J.:

In an Information filed on 30 August 1990, in Criminal Case No. 90-86748 before


[1]

the Regional Trial Court (RTC) of Manila, Branch 5, petitioner Sammy Malacat y Mandar
was charged with violating Section 3 of Presidential Decree No. 1866, as follows: [2]
That on or about August 27, 1990, in the City of Manila, Philippines, the said accused did then
and there willfully, unlawfully and knowingly keep, possess and/or acquire a hand grenade,
without first securing the necessary license and/or permit therefor from the proper authorities.

At arraignment on 9 October 1990, petitioner, assisted by counsel de oficio, entered


[3]

a plea of not guilty.


At pre-trial on 11 March 1991, petitioner admitted the existence of Exhibits A, A-1,
and A-2, while the prosecution admitted that the police authorities were not armed with
[4]

a search warrant nor warrant of arrest at the time they arrested petitioner. [5]

At trial on the merits, the prosecution presented the following police officers as its
witnesses: Rodolfo Yu, the arresting officer; Josefino G. Serapio, the investigating
officer; and Orlando Ramilo, who examined the grenade.
Rodolfo Yu of the Western Police District, Metropolitan Police Force of the
Integrated National Police, Police Station No. 3, Quiapo, Manila, testified that on 27
August 1990, at about 6:30 p.m., in response to bomb threats reported seven days
earlier, he was on foot patrol with three other police officers (all of them in uniform) along
Quezon Boulevard, Quiapo, Manila, near the Mercury Drug store at Plaza Miranda.They
chanced upon two groups of Muslim-looking men, with each group, comprised of three
to four men, posted at opposite sides of the corner of Quezon Boulevard near the
Mercury Drug Store. These men were acting suspiciously with [t]heir eyes moving very
fast.
[6]

Yu and his companions positioned themselves at strategic points and observed both
groups for about thirty minutes. The police officers then approached one group of men,
who then fled in different directions. As the policemen gave chase, Yu caught up with
and apprehended petitioner. Upon searching petitioner, Yu found a fragmentation
grenade tucked inside petitioners front waist line. Yus companion, police officer
[7]

Rogelio Malibiran, apprehended Abdul Casan from whom a .38 caliber revolver was
recovered. Petitioner and Casan were then brought to Police Station No. 3 where Yu
placed an X mark at the bottom of the grenade and thereafter gave it to his
commander. [8]

On cross-examination, Yu declared that they conducted the foot patrol due to a


report that a group of Muslims was going to explode a grenade somewhere in the
vicinity of Plaza Miranda. Yu recognized petitioner as the previous Saturday, 25 August
1990, likewise at Plaza Miranda, Yu saw petitioner and 2 others attempt to detonate a
grenade. The attempt was aborted when Yu and other policemen chased petitioner and
his companions; however, the former were unable to catch any of the latter. Yu further
admitted that petitioner and Casan were merely standing on the corner of Quezon
Boulevard when Yu saw them on 27 August 1990. Although they were not creating a
commotion, since they were supposedly acting suspiciously, Yu and his companions
approached them. Yu did not issue any receipt for the grenade he allegedly recovered
from petitioner. [9]

Josefino G. Serapio declared that at about 9:00 a.m. of 28 August 1990, petitioner
and a certain Abdul Casan were brought in by Sgt. Saquilla for investigation. Forthwith,
[10]
Serapio conducted the inquest of the two suspects, informing them of their rights to
remain silent and to be assisted by competent and independent counsel. Despite
Serapios advice, petitioner and Casan manifested their willingness to answer questions
even without the assistance of a lawyer. Serapio then took petitioners uncounselled
confession (Exh. E), there being no PAO lawyer available, wherein petitioner admitted
possession of the grenade. Thereafter, Serapio prepared the affidavit of arrest and
booking sheet of petitioner and Casan. Later, Serapio turned over the grenade to the
Intelligence and Special Action Division (ISAD) of the Explosive Ordnance Disposal Unit
for examination. [11]

On cross-examination, Serapio admitted that he took petitioners confession knowing


it was inadmissible in evidence. [12]

Orlando Ramilo, a member of the Bomb Disposal Unit, whose principal duties
included, among other things, the examination of explosive devices, testified that on 22
March 1991, he received a request dated 19 March 1991 from Lt. Eduardo Cabrera and
PO Diosdado Diotoy for examination of a grenade. Ramilo then affixed an orange tag on
the subject grenade detailing his name, the date and time he received the specimen.
During the preliminary examination of the grenade, he [f]ound that [the] major
components consisting of [a] high filler and fuse assembly [were] all present, and
concluded that the grenade was [l]ive and capable of exploding. On even date, he
issued a certification stating his findings, a copy of which he forwarded to Diotoy on 11
August 1991. [13]

Petitioner was the lone defense witness. He declared that he arrived in Manila on 22
July 1990 and resided at the Muslim Center in Quiapo, Manila. At around 6:30 in the
evening of 27 August 1990, he went to Plaza Miranda to catch a breath of fresh
air. Shortly after, several policemen arrived and ordered all males to stand aside. The
policemen searched petitioner and two other men, but found nothing in their
possession. However, he was arrested with two others, brought to and detained at
Precinct No. 3, where he was accused of having shot a police officer. The officer
showed the gunshot wounds he allegedly sustained and shouted at petitioner [i]to ang
tama mo sa akin. This officer then inserted the muzzle of his gun into petitioners mouth
and said, [y]ou are the one who shot me. Petitioner denied the charges and explained
that he only recently arrived in Manila. However, several other police officers mauled
him, hitting him with benches and guns. Petitioner was once again searched, but
nothing was found on him. He saw the grenade only in court when it was presented. [14]

The trial court ruled that the warrantless search and seizure of petitioner was akin to
a stop and frisk, where a warrant and seizure can be effected without necessarily being
preceded by an arrest and whose object is either to maintain the status quo momentarily
while the police officer seeks to obtain more information. Probable cause was not
[15]

required as it was not certain that a crime had been committed, however, the situation
called for an investigation, hence to require probable cause would have been
premature. The RTC emphasized that Yu and his companions were [c]onfronted with
[16]

an emergency, in which the delay necessary to obtain a warrant, threatens the


destruction of evidence and the officers [h]ad to act in haste, as petitioner and his
[17]

companions were acting suspiciously, considering the time, place and reported cases of
bombing. Further, petitioners group suddenly ran away in different directions as they
saw the arresting officers approach, thus [i]t is reasonable for an officer to conduct a
limited search, the purpose of which is not necessarily to discover evidence of a crime,
but to allow the officer to pursue his investigation without fear of violence. [18]

The trial court then ruled that the seizure of the grenade from petitioner was
incidental to a lawful arrest, and since petitioner [l]ater voluntarily admitted such fact to
the police investigator for the purpose of bombing the Mercury Drug Store, concluded
that sufficient evidence existed to establish petitioners guilt beyond reasonable doubt.
In its decision dated 10 February 1994 but promulgated on 15 February 1994, the
[19]

trial court thus found petitioner guilty of the crime of illegal possession of explosives
under Section 3 of P.D. No. 1866, and sentenced him to suffer:

[T]he penalty of not less than SEVENTEEN (17) YEARS, FOUR (4) MONTHS AND ONE (1)
DAY OF RECLUSION TEMPORAL, as minimum, and not more than THIRTY (30) YEARS
OF RECLUSION PERPETUA, as maximum.

On 18 February 1994, petitioner filed a notice of appeal indicating that he was


[20]

appealing to this Court. However, the record of the case was forwarded to the Court of
Appeals which docketed it as CA-G.R. CR No. 15988 and issued a notice to file briefs. [21]

In his Appellants Brief [22]


filed with the Court of Appeals, petitioner asserted that:
1. THE LOWER COURT ERRED IN HOLDING THAT THE SEARCH UPON THE PERSON OF
ACCUSED-APPELLANT AND THE SEIZURE OF THE ALLEGED HANDGRENADE FROM HIM
WAS AN APPROPRIATE INCIDENT TO HIS ARREST.
2. THE LOWER COURT ERRED IN ADMITTING AS EVIDENCE AGAINST
ACCUSED-APPELLANT THE HANDGRENADE ALLEGEDLY SEIZED FROM HIM AS IT WAS
A PRODUCT OF AN UNREASONABLE AND ILLEGAL SEARCH.

In sum, petitioner argued that the warrantless arrest was invalid due to absence of
any of the conditions provided for in Section 5 of Rule 113 of the Rules of Court,
citing People vs. Mengote. As such, the search was illegal, and the hand grenade
[23]

seized, inadmissible in evidence.


In its Brief for the Appellee, the Office of the Solicitor General agreed with the trial
court and prayed that its decision be affirmed in toto. [24]

In its decision of 24 January 1996, the Court of Appeals affirmed the trial court,
[25]

noting, first, that petitioner abandoned his original theory before the court a quo that the
grenade was planted by the police officers; and second, the factual finding of the trial
court that the grenade was seized from petitioners possession was not raised as an
issue. Further, respondent court focused on the admissibility in evidence of Exhibit D,
the hand grenade seized from petitioner. Meeting the issue squarely, the Court of
Appeals ruled that the arrest was lawful on the ground that there was probable cause for
the arrest as petitioner was attempting to commit an offense, thus:

We are at a loss to understand how a man, who was in possession of a live grenade and in the
company of other suspicious character[s] with unlicensed firearm[s] lurking in Plaza Miranda at
a time when political tension ha[d] been enkindling a series of terroristic activities, [can] claim
that he was not attempting to commit an offense. We need not mention that Plaza Miranda is
historically notorious for being a favorite bomb site especially during times of political
upheaval. As the mere possession of an unlicensed grenade is by itself an offense, Malacats
posture is simply too preposterous to inspire belief.

In so doing, the Court of Appeals took into account petitioners failure to rebut the
testimony of the prosecution witnesses that they received intelligence reports of a bomb
threat at Plaza Miranda; the fact that PO Yu chased petitioner two days prior to the
latters arrest, or on 27 August 1990; and that petitioner and his companions acted
suspiciously, the accumulation of which was more than sufficient to convince a
reasonable man that an offense was about to be committed. Moreover, the Court of
Appeals observed:

The police officers in such a volatile situation would be guilty of gross negligence and
dereliction of duty, not to mention of gross incompetence, if they [would] first wait for Malacat
to hurl the grenade, and kill several innocent persons while maiming numerous others, before
arriving at what would then be an assured but moot conclusion that there was indeed probable
cause for an arrest. We are in agreement with the lower court in saying that the probable cause
in such a situation should not be the kind of proof necessary to convict, but rather the practical
considerations of everyday life on which a reasonable and prudent mind, and not legal
technicians, will ordinarily act.

Finally, the Court of Appeals held that the rule laid down in People v.
Mengote, which petitioner relied upon, was inapplicable in light of [c]rucial differences,
[26]

to wit:

[In Mengote] the police officers never received any intelligence report that someone [at] the
corner of a busy street [would] be in possession of a prohibited article. Here the police officers
were responding to a [sic] public clamor to put a check on the series of terroristic bombings in
the Metropolis, and, after receiving intelligence reports about a bomb threat aimed at the vicinity
of the historically notorious Plaza Miranda, they conducted foot patrols for about seven days to
observe suspicious movements in the area. Furthermore, in Mengote, the police officers [had] no
personal knowledge that the person arrested has committed, is actually committing, or is
attempting to commit an offense. Here, PO3 Yu [had] personal knowledge of the fact that he
chased Malacat in Plaza Miranda two days before he finally succeeded in apprehending him.

Unable to accept his conviction, petitioner forthwith filed the instant petition and
assigns the following errors:
1. THE RESPONDENT COURT ERRED IN AFFIRMING THE FINDING OF THE TRIAL COURT
THAT THE WARRANTLESS ARREST OF PETITIONER WAS VALID AND LEGAL.
2. THE RESPONDENT COURT ERRED IN HOLDING THAT THE RULING IN PEOPLE VS.
MENGOTE DOES NOT FIND APPLICATION IN THE INSTANT CASE.

In support thereof, petitioner merely restates his arguments below regarding the validity
of the warrantless arrest and search, then disagrees with the finding of the Court of
Appeals that he was attempting to commit a crime, as the evidence for the prosecution
merely disclosed that he was standing at the corner of Plaza Miranda and Quezon
Boulevard with his eyes moving very fast and looking at every person that come (sic)
nearer (sic) to them. Finally, petitioner points out the factual similarities between his
case and that of People v. Mengote to demonstrate that the Court of Appeals
miscomprehended the latter.
In its Comment, the Office of the Solicitor General prays that we affirm the
challenged decision.
For being impressed with merit, we resolved to give due course to the petition.
The challenged decision must immediately fall on jurisdictional grounds. To repeat,
the penalty imposed by the trial court was:

[N]ot less than SEVENTEEN (17) YEARS, FOUR (4) MONTHS AND ONE (1) DAY
OF RECLUSION TEMPORAL, as minimum, and not more than THIRTY (30) YEARS
OF RECLUSION PERPETUA, as maximum.

The penalty provided by Section 3 of P.D. No. 1866 upon any person who shall
unlawfully possess grenades is reclusion temporal in its maximum period to reclusion
perpetua.
For purposes of determining appellate jurisdiction in criminal cases, the maximum of
the penalty, and not the minimum, is taken into account. Since the maximum of the
penalty is reclusion perpetua, the appeal therefrom should have been to us, and not the
Court of Appeals, pursuant to Section 9(3) of the Judiciary Reorganization Act of 1980
(B.P. Blg. 129), in relation to Section 17 of the Judiciary Act of 1948, Section 5(2) of
[27] [28]

Article VIII of the Constitution and Section 3(c) of Rule 122 of the Rules of Court. The
[29] [30]

term life imprisonment as used in Section 9 of B.P. Blg. 129, the Judiciary Act of 1948,
and Section 3 of Rule 122 must be deemed to include reclusion perpetua in view of
Section 5(2) of Article VIII of the Constitution.
Petitioners Notice of Appeal indicated that he was appealing from the trial courts
decision to this Court, yet the trial court transmitted the record to the Court of Appeals
and the latter proceeded to resolve the appeal.
We then set aside the decision of the Court of Appeals for having been rendered
without jurisdiction, and consider the appeal as having been directly brought to us, with
the petition for review as petitioners Brief for the Appellant, the comment thereon by the
Office of the Solicitor General as the Brief for the Appellee and the memoranda of the
parties as their Supplemental Briefs.
Deliberating on the foregoing pleadings, we find ourselves convinced that the
prosecution failed to establish petitioners guilt with moral certainty.
First, serious doubt surrounds the story of police officer Yu that a grenade was found
in and seized from petitioners possession. Notably, Yu did not identify, in court, the
grenade he allegedly seized.According to him, he turned it over to his commander after
putting an X mark at its bottom; however, the commander was not presented to
corroborate this claim. On the other hand, the grenade presented in court and identified
by police officer Ramilo referred to what the latter received from Lt. Eduardo Cabrera
and police officer Diotoy not immediately after petitioners arrest, but nearly seven (7)
months later, or on 19 March 1991; further, there was no evidence whatsoever that what
Ramilo received was the very same grenade seized from petitioner. In his testimony, Yu
never declared that the grenade passed on to Ramilo was the grenade the former
confiscated from petitioner. Yu did not, and was not made to, identify the grenade
examined by Ramilo, and the latter did not claim that the grenade he examined was that
seized from petitioner. Plainly, the law enforcement authorities failed to safeguard and
preserve the chain of evidence so crucial in cases such as these.
Second, if indeed petitioner had a grenade with him, and that two days earlier he
was with a group about to detonate an explosive at Plaza Miranda, and Yu and his
fellow officers chased, but failed to arrest them, then considering that Yu and his three
fellow officers were in uniform and therefore easily cognizable as police officers, it was
then unnatural and against common experience that petitioner simply stood there in
proximity to the police officers. Note that Yu observed petitioner for thirty minutes and
must have been close enough to petitioner in order to discern petitioners eyes moving
very fast.
Finally, even assuming that petitioner admitted possession of the grenade during his
custodial investigation by police officer Serapio, such admission was inadmissible in
evidence for it was taken in palpable violation of Section 12(1) and (3) of Article III of the
Constitution, which provide as follows:

SEC. 12 (1). Any person under investigation for the commission of an offense shall have the
right to be informed of his right to remain silent and to have competent and independent counsel
preferably of his own choice. If the person cannot afford the services of counsel, he must be
provided with one. These rights cannot be waived except in writing and in the presence of
counsel.

xxx
(3) Any confession or admission obtained in violation of this or Section 17 hereof shall be
inadmissible in evidence against him.

Serapio conducted the custodial investigation on petitioner the day following his arrest.
No lawyer was present and Serapio could not have requested a lawyer to assist
petitioner as no PAO lawyer was then available. Thus, even if petitioner consented to
the investigation and waived his rights to remain silent and to counsel, the waiver was
invalid as it was not in writing, neither was it executed in the presence of counsel.
Even granting ex gratia that petitioner was in possession of a grenade, the arrest
and search of petitioner were invalid, as will be discussed below.
The general rule as regards arrests, searches and seizures is that a warrant is
needed in order to validly effect the same. The Constitutional prohibition against
[31]

unreasonable arrests, searches and seizures refers to those effected without a validly
issued warrant, subject to certain exceptions. As regards valid warrantless arrests,
[32]

these are found in Section 5, Rule 113 of the Rules of Court, which reads, in part:
Sec. 5. -- Arrest, without warrant; when lawful -- A peace officer or a private person may,
without a warrant, arrest a person:

(a) When, in his presence, the person to be arrested has committed, is actually committing, or is
attempting to commit an offense;
(b) When an offense has in fact just been committed, and he has personal knowledge of facts
indicating that the person to be arrested has committed it; and
(c) When the person to be arrested is a prisoner who has escaped ***

A warrantless arrest under the circumstances contemplated under Section 5(a) has
been denominated as one "in flagrante delicto," while that under Section 5(b) has been
described as a "hot pursuit" arrest.
Turning to valid warrantless searches, they are limited to the following: (1) customs
searches; (2) search of moving vehicles; (3) seizure of evidence in plain view; (4)
consent searches; (5) a search incidental to a lawful arrest; and (6) a "stop and
[33] [34]

frisk."
[35]

In the instant petition, the trial court validated the warrantless search as a stop and
frisk with the seizure of the grenade from the accused [as] an appropriate incident to his
arrest, hence necessitating a brief discussion on the nature of these exceptions to the
warrant requirement.
At the outset, we note that the trial court confused the concepts of a "stop-and-frisk"
and of a search incidental to a lawful arrest. These two types of warrantless searches
differ in terms of the requisite quantum of proof before they may be validly effected and
in their allowable scope.
In a search incidental to a lawful arrest, as the precedent arrest determines the
validity of the incidental search, the legality of the arrest is questioned in a large majority
of these cases, e.g., whether an arrest was merely used as a pretext for conducting a
search. In this instance, the law requires that there first be a lawful arrest before a
[36]

search can be made -- the process cannot be reversed. At bottom, assuming a valid
[37]

arrest, the arresting officer may search the person of the arrestee and the area within
which the latter may reach for a weapon or for evidence to destroy, and seize any
money or property found which was used in the commission of the crime, or the fruit of
the crime, or that which may be used as evidence, or which might furnish the arrestee
with the means of escaping or committing violence. [38]

Here, there could have been no valid in flagrante delicto or hot pursuit arrest
preceding the search in light of the lack of personal knowledge on the part of Yu, the
arresting officer, or an overt physical act, on the part of petitioner, indicating that a crime
had just been committed, was being committed or was going to be committed.
Having thus shown the invalidity of the warrantless arrest in this case, plainly, the
search conducted on petitioner could not have been one incidental to a lawful arrest.
We now proceed to the justification for and allowable scope of a "stop-and-frisk" as a
"limited protective search of outer clothing for weapons," as laid down in Terry, thus:
We merely hold today that where a police officer observes unusual conduct which leads him
reasonably to conclude in light of his experience that criminal activity may be afoot and that
the persons with whom he is dealing may be armed and presently dangerous, where in the
course of investigating this behavior he identifies himself as a policeman and makes
reasonable inquiries, and where nothing in the initial stages of the encounter serves to dispel
his reasonable fear for his own or others' safety, he is entitled for the protection of himself
and others in the area to conduct a carefully limited search of the outer clothing of such
persons in an attempt to discover weapons which might be used to assault him. Such a
search is a reasonable search under the Fourth Amendment *** [39]

Other notable points of Terry are that while probable cause is not required to conduct a
"stop and frisk," it nevertheless holds that mere suspicion or a hunch will not validate a
[40]

"stop and frisk." A genuine reason must exist, in light of the police officer's experience
and surrounding conditions, to warrant the belief that the person detained has weapons
concealed about him. Finally, a "stop-and-frisk" serves a two-fold interest: (1) the
[41]

general interest of effective crime prevention and detection, which underlies the
recognition that a police officer may, under appropriate circumstances and in an
appropriate manner, approach a person for purposes of investigating possible criminal
behavior even without probable cause; and (2) the more pressing interest of safety and
self-preservation which permit the police officer to take steps to assure himself that the
person with whom he deals is not armed with a deadly weapon that could unexpectedly
and fatally be used against the police officer.
Here, here are at least three (3) reasons why the stop-and-frisk was invalid:
First, we harbor grave doubts as to Yus claim that petitioner was a member of the
group which attempted to bomb Plaza Miranda two days earlier. This claim is neither
supported by any police report or record nor corroborated by any other police officer
who allegedly chased that group. Aside from impairing Yu's credibility as a witness, this
likewise diminishes the probability that a genuine reason existed so as to arrest and
search petitioner. If only to further tarnish the credibility of Yu's testimony, contrary to his
claim that petitioner and his companions had to be chased before being apprehended,
the affidavit of arrest (Exh. "A") expressly declares otherwise, i.e., upon arrival of five (5)
other police officers, petitioner and his companions were "immediately collared."
Second, there was nothing in petitioners behavior or conduct which could have
reasonably elicited even mere suspicion other than that his eyes were moving very fast
an observation which leaves us incredulous since Yu and his teammates were nowhere
near petitioner and it was already 6:30 p.m., thus presumably dusk. Petitioner and his
companions were merely standing at the corner and were not creating any commotion
or trouble, as Yu explicitly declared on cross-examination:
Q And what were they doing?
A They were merely standing.
Q You are sure of that?
A Yes, sir.
Q And when you saw them standing, there were nothing or they did not create any commotion?
A None, sir.
Q Neither did you see them create commotion?
A None, sir.[42]

Third, there was at all no ground, probable or otherwise, to believe that petitioner
was armed with a deadly weapon. None was visible to Yu, for as he admitted, the
alleged grenade was discovered inside the front waistline of petitioner, and from all
indications as to the distance between Yu and petitioner, any telltale bulge, assuming
that petitioner was indeed hiding a grenade, could not have been visible to Yu.In fact, as
noted by the trial court:

When the policemen approached the accused and his companions, they were not yet aware that a
handgrenade was tucked inside his waistline. They did not see any bulging object in [sic] his
person.[43]

What is unequivocal then in this case are blatant violations of petitioners rights
solemnly guaranteed in Sections 2 and 12(1) of Article III of the Constitution.
WHEREFORE, the challenged decision of the Seventeenth Division of the Court of
Appeals in CA-G.R. CR No. 15988 is SET ASIDE for lack of jurisdiction on the part of
said Court and, on ground of reasonable doubt, the decision of 10 February 1994 of
Branch 5 of the Regional Trial Court of Manila is REVERSED and petitioner SAMMY
MALACAT y MANDAR is hereby ACQUITTED and ORDERED immediately released
from detention, unless his further detention is justified for any other lawful cause.
Costs de oficio.
SO ORDERED.
G.R.No. 74869 July 6, 1988

PEOPLE OF THE PHILIPPINES, plaintiff-appellee,


vs.
IDEL AMINNUDIN y AHNI, defendant-appellant.

The Solicitor General for plaintiff-appellee.

Herminio T. Llariza counsel de-officio for defendant-appellant.

CRUZ, J.:

The accused-appellant claimed his business was selling watches but he was nonetheless arrested, tried and found guilty of illegally transporting marijuana.
The trial court, disbelieving him, held it was high time to put him away and sentenced him to life imprisonment plus a fine of P20,000.00. 1

Idel Aminnudin was arrested on June 25, 1984, shortly after disembarking from the M/V Wilcon 9 at about 8:30 in
the evening, in Iloilo City. The PC officers who were in fact waiting for him simply accosted him, inspected his bag
and finding what looked liked marijuana leaves took him to their headquarters for investigation. The two bundles
of suspect articles were confiscated from him and later taken to the NBI laboratory for examination. When they
were verified as marijuana leaves, an information for violation of the Dangerous Drugs Act was filed against
him. Later, the information was amended to include Farida Ali y Hassen, who had also been arrested with him
2

that same evening and likewise investigated. Both were arraigned and pleaded not guilty. Subsequently, the
3 4

fiscal filed a motion to dismiss the charge against Ali on the basis of a sworn statement of the arresting officers
absolving her after a 'thorough investigation." The motion was granted, and trial proceeded only against the
5

accused-appellant, who was eventually convicted . 6

According to the prosecution, the PC officers had earlier received a tip from one of their informers that the
accused-appellant was on board a vessel bound for Iloilo City and was carrying marijuana. He was Identified by 7

name. Acting on this tip, they waited for him in the evening of June 25, 1984, and approached him as he
8

descended from the gangplank after the informer had pointed to him. They detained him and inspected the bag
9

he was carrying. It was found to contain three kilos of what were later analyzed as marijuana leaves by an NBI
forensic examiner, who testified that she conducted microscopic, chemical and chromatographic tests on them.
10

On the basis of this finding, the corresponding charge was then filed against Aminnudin.

In his defense, Aminnudin disclaimed the marijuana, averring that all he had in his bag was his clothing consisting
of a jacket, two shirts and two pairs of pants. He alleged that he was arbitrarily arrested and immediately
11

handcuffed. His bag was confiscated without a search warrant. At the PC headquarters, he was manhandled to
force him to admit he was carrying the marijuana, the investigator hitting him with a piece of wood in the chest and
arms even as he parried the blows while he was still handcuffed. He insisted he did not even know what
12

marijuana looked like and that his business was selling watches and sometimes cigarettes. He also argued that 13

the marijuana he was alleged to have been carrying was not properly Identified and could have been any of
several bundles kept in the stock room of the PC headquarters. 14

The trial court was unconvinced, noting from its own examination of the accused that he claimed to have come to
Iloilo City to sell watches but carried only two watches at the time, traveling from Jolo for that purpose and
spending P107.00 for fare, not to mention his other expenses. Aminnudin testified that he kept the two watches
15

in a secret pocket below his belt but, strangely, they were not discovered when he was bodily searched by the
arresting officers nor were they damaged as a result of his manhandling. He also said he sold one of the 16

watches for P400.00 and gave away the other, although the watches belonged not to him but to his cousin, to a 17
friend whose full name he said did not even know. The trial court also rejected his allegations of maltreatment,
18

observing that he had not sufficiently proved the injuries sustained by him. 19

There is no justification to reverse these factual findings, considering that it was the trial judge who had immediate
access to the testimony of the witnesses and had the opportunity to weigh their credibility on the stand. Nuances
of tone or voice, meaningful pauses and hesitation, flush of face and dart of eyes, which may reveal the truth or
expose the lie, are not described in the impersonal record. But the trial judge sees all of this, discovering for
himself the truant fact amidst the falsities.

The only exception we may make in this case is the trial court's conclusion that the accused-appellant was not
really beaten up because he did not complain about it later nor did he submit to a medical examination. That is
hardly fair or realistic. It is possible Aminnudin never had that opportunity as he was at that time under detention
by the PC authorities and in fact has never been set free since he was arrested in 1984 and up to the present. No
bail has been allowed for his release.

There is one point that deserves closer examination, however, and it is Aminnudin's claim that he was arrested
and searched without warrant, making the marijuana allegedly found in his possession inadmissible in evidence
against him under the Bill of Rights. The decision did not even discuss this point. For his part, the Solicitor
General dismissed this after an all-too-short argument that the arrest of Aminnudin was valid because it came
under Rule 113, Section 6(b) of the Rules of Court on warrantless arrests. This made the search also valid as
incidental to a lawful arrest.

It is not disputed, and in fact it is admitted by the PC officers who testified for the prosecution, that they had no
warrant when they arrested Aminnudin and seized the bag he was carrying. Their only justification was the tip
they had earlier received from a reliable and regular informer who reported to them that Aminnudin was arriving in
Iloilo by boat with marijuana. Their testimony varies as to the time they received the tip, one saying it was two
days before the arrest, another two weeks and a third "weeks before June 25." On this matter, we may prefer
20 21 22

the declaration of the chief of the arresting team, Lt. Cipriano Querol, Jr., who testified as follows:

Q You mentioned an intelligence report, you mean with respect to the coming of Idel Aminnudin on June 25,
1984?

A Yes, sir.

Q When did you receive this intelligence report?

A Two days before June 25, 1984 and it was supported by reliable sources.

Q Were you informed of the coming of the Wilcon 9 and the possible trafficking of marijuana leaves on that date?

A Yes, sir, two days before June 25, 1984 when we received this information from that particular informer, prior to
June 25, 1984 we have already reports of the particular operation which was being participated by Idel
Aminnudin.

Q You said you received an intelligence report two days before June 25, 1984 with respect to the coming of
Wilcon 9?

A Yes, sir.

Q Did you receive any other report aside from this intelligence report?

A Well, I have received also other reports but not pertaining to the coming of Wilcon 9. For instance, report of
illegal gambling operation.

COURT:
Q Previous to that particular information which you said two days before June 25, 1984, did you also receive daily
report regarding the activities of Idel Aminnudin

A Previous to June 25, 1984 we received reports on the activities of Idel Aminnudin.

Q What were those activities?

A Purely marijuana trafficking.

Q From whom did you get that information?

A It came to my hand which was written in a required sheet of information, maybe for security reason and we
cannot Identify the person.

Q But you received it from your regular informer?

A Yes, sir.

ATTY. LLARIZA:

Q Previous to June 25, 1984, you were more or less sure that Idel Aminnudin is coming with drugs?

A Marijuana, sir.

Q And this information respecting Idel Aminnudin's coming to Iloilo with marijuana was received by you many
days before you received the intelligence report in writing?

A Not a report of the particular coming of Aminnudin but his activities.

Q You only knew that he was coming on June 25,1984 two days before?

A Yes, sir.

Q You mean that before June 23, 1984 you did not know that minnudin was coming?

A Before June 23,1984, I, in my capacity, did not know that he was coming but on June 23, 1984 that was the
time when I received the information that he was coming. Regarding the reports on his activities, we have reports
that he was already consummated the act of selling and shipping marijuana stuff.

COURT:

Q And as a result of that report, you put him under surveillance?

A Yes, sir.

Q In the intelligence report, only the name of Idel Aminnudin was mentioned?

A Yes, sir.

Q Are you sure of that?

A On the 23rd he will be coming with the woman.

Q So that even before you received the official report on June 23, 1984, you had already gathered information to
the effect that Idel Aminnudin was coming to Iloilo on June 25, 1984?

A Only on the 23rd of June.


Q You did not try to secure a search warrant for the seizure or search of the subject mentioned in your intelligence
report?

A No, more.

Q Why not?

A Because we were very very sure that our operation will yield positive result.

Q Is that your procedure that whenever it will yield positive result you do not need a search warrant anymore?

A Search warrant is not necessary. 23

That last answer is a cavalier pronouncement, especially as it comes from a mere lieutenant of the PC. The
Supreme Court cannot countenance such a statement. This is still a government of laws and not of men.

The mandate of the Bill of Rights is clear:

Sec. 2. The right of the people to be secure in their persons, houses, papers and effects against unreasonable
searches and seizures of whatever nature and for any purpose shall be inviolable, and no search warrant or
warrant of arrest shall issue except upon probable cause to be determined personally by the judge after
examination under oath or affirmation of the complainant and the witnesses he may produce, and particularly
describing the place to be searched and the persons or things to be seized.

In the case at bar, there was no warrant of arrest or search warrant issued by a judge after personal
determination by him of the existence of probable cause. Contrary to the averments of the government, the
accused-appellant was not caught in flagrante nor was a crime about to be committed or had just been committed
to justify the warrantless arrest allowed under Rule 113 of the Rules of Court. Even expediency could not be
invoked to dispense with the obtention of the warrant as in the case of Roldan v. Arca, for example. Here it was
24

held that vessels and aircraft are subject to warrantless searches and seizures for violation of the customs law
because these vehicles may be quickly moved out of the locality or jurisdiction before the warrant can be secured.

The present case presented no such urgency. From the conflicting declarations of the PC witnesses, it is clear
that they had at least two days within which they could have obtained a warrant to arrest and search Aminnudin
who was coming to Iloilo on the M/V Wilcon 9. His name was known. The vehicle was Identified. The date of its
arrival was certain. And from the information they had received, they could have persuaded a judge that there
was probable cause, indeed, to justify the issuance of a warrant. Yet they did nothing. No effort was made to
comply with the law. The Bill of Rights was ignored altogether because the PC lieutenant who was the head of the
arresting team, had determined on his own authority that a "search warrant was not necessary."

In the many cases where this Court has sustained the warrantless arrest of violators of the Dangerous Drugs Act,
it has always been shown that they were caught red-handed, as a result of what are popularly called "buy-bust"
operations of the narcotics agents. Rule 113 was clearly applicable because at the precise time of arrest the
25

accused was in the act of selling the prohibited drug.

In the case at bar, the accused-appellant was not, at the moment of his arrest, committing a crime nor was it
shown that he was about to do so or that he had just done so. What he was doing was descending the gangplank
of the M/V Wilcon 9 and there was no outward indication that called for his arrest. To all appearances, he was like
any of the other passengers innocently disembarking from the vessel. It was only when the informer pointed to
him as the carrier of the marijuana that he suddenly became suspect and so subject to apprehension. It was the
furtive finger that triggered his arrest. The Identification by the informer was the probable cause as determined by
the officers (and not a judge) that authorized them to pounce upon Aminnudin and immediately arrest him.

Now that we have succeeded in restoring democracy in our country after fourteen years of the despised
dictatorship, when any one could be picked up at will, detained without charges and punished without trial, we will
have only ourselves to blame if that kind of arbitrariness is allowed to return, to once more flaunt its disdain of the
Constitution and the individual liberties its Bill of Rights guarantees.
While this is not to say that the accused-appellant is innocent, for indeed his very own words suggest that he is
lying, that fact alone does not justify a finding that he is guilty. The constitutional presumption is that he is
innocent, and he will be so declared even if his defense is weak as long as the prosecution is not strong enough
to convict him.

Without the evidence of the marijuana allegedly seized from Aminnudin, the case of the prosecution must fall.
That evidence cannot be admitted, and should never have been considered by the trial court for the simple fact is
that the marijuana was seized illegally. It is the fruit of the poisonous tree, to use Justice Holmes' felicitous phrase.
The search was not an incident of a lawful arrest because there was no warrant of arrest and the warrantless
arrest did not come under the exceptions allowed by the Rules of Court. Hence, the warrantless search was also
illegal and the evidence obtained thereby was inadmissible.

The Court strongly supports the campaign of the government against drug addiction and commends the efforts of
our law-enforcement officers against those who would inflict this malediction upon our people, especially the
susceptible youth. But as demanding as this campaign may be, it cannot be more so than the compulsions of the
Bill of Rights for the protection of the liberty of every individual in the realm, including the basest of criminals. The
Constitution covers with the mantle of its protection the innocent and the guilty alike against any manner of high-
handedness from the authorities, however praiseworthy their intentions.

Those who are supposed to enforce the law are not justified in disregarding the rights of the individual in the
name of order. Order is too high a price for the loss of liberty. As Justice Holmes, again, said, "I think it a less evil
that some criminals should escape than that the government should play an ignoble part." It is simply not allowed
in the free society to violate a law to enforce another, especially if the law violated is the Constitution itself.

We find that with the exclusion of the illegally seized marijuana as evidence against the accused-appellant, his
guilt has not been proved beyond reasonable doubt and he must therefore be discharged on the presumption that
he is innocent.

ACCORDINGLY, the decision of the trial court is REVERSED and the accused-appellant is ACQUITTED. It is so
ordered.

Narvasa, Gancayco and Medialdea, JJ., concur.


G.R. No. 91107 June 19, 1991
THE PEOPLE OF THE PHILIPPINES, plaintiff-appellee,
vs.
MIKAEL MALMSTEDT, *defendant-appellant.

The Solicitor General for plaintiff-appellee.


Romulo, Mabanta, Buenaventura, Sayoc & De los Angeles for defendant-appellant.

PADILLA, J.:

In an information dated 15 June 1989, accused-appellant Mikael Malmstedt (hereinafter referred to as the
accused) was charged before the Regional Trial Court (RTC) of La Trinidad, Benguet, Branch 10, in Criminal
Case No. 89-CR-0663, for violation of Section 4, Art. II of Republic Act 6425, as amended, otherwise known as
the Dangerous Drugs Act of 1972, as amended. The factual background of the case is as follows:

Accused Mikael Malmstedt, a Swedish national, entered the Philippines for the third time in December 1988 as a
tourist. He had visited the country sometime in 1982 and 1985.

In the evening of 7 May 1989, accused left for Baguio City. Upon his arrival thereat in the morning of the following
day, he took a bus to Sagada and stayed in that place for two (2) days.

At around 7:00 o'clock in the morning of 11 May 1989, accused went to the Nangonogan bus stop in Sagada to
catch the first available trip to Baguio City. From Baguio City, accused planned to take a late afternoon trip to
Angeles City, then proceed to Manila to catch his flight out of the country, scheduled on 13 May 1989. From
Sagada, accused took a Skyline bus with body number 8005 and Plate number AVC 902. 1

At about 8: 00 o'clock in the morning of that same day (11 May 1989), Captain Alen Vasco, the Commanding
Officer of the First Regional Command (NARCOM) stationed at Camp Dangwa, ordered his men to set up a
temporary checkpoint at Kilometer 14, Acop, Tublay, Mountain Province, for the purpose of checking all vehicles
coming from the Cordillera Region. The order to establish a checkpoint in the said area was prompted by
persistent reports that vehicles coming from Sagada were transporting marijuana and other prohibited drugs.
Moreover, information was received by the Commanding Officer of NARCOM, that same morning, that a
Caucasian coming from Sagada had in his possession prohibited drugs. 2

The group composed of seven (7) NARCOM officers, in coordination with Tublay Police Station, set up a
checkpoint at the designated area at about 10:00 o'clock in the morning and inspected all vehicles coming from
the Cordillera Region.

At about 1:30 o'clock in the afternoon, the bus where accused was riding was stopped. Sgt. Fider and CIC
Galutan boarded the bus and announced that they were members of the NARCOM and that they would conduct
an inspection. The two (2) NARCOM officers started their inspection from the front going towards the rear of the
bus. Accused who was the sole foreigner riding the bus was seated at the rear thereof.

During the inspection, CIC Galutan noticed a bulge on accused's waist. Suspecting the bulge on accused's waist
to be a gun, the officer asked for accused's passport and other identification papers. When accused failed to
comply, the officer required him to bring out whatever it was that was bulging on his waist. The bulging object
turned out to be a pouch bag and when accused opened the same bag, as ordered, the officer noticed four (4)
suspicious-looking objects wrapped in brown packing tape, prompting the officer to open one of the wrapped
objects. The wrapped objects turned out to contain hashish, a derivative of marijuana.

Thereafter, accused was invited outside the bus for questioning. But before he alighted from the bus, accused
stopped to get two (2) travelling bags from the luggage carrier.

Upon stepping out of the bus, the officers got the bags and opened them. A teddy bear was found in each bag.
Feeling the teddy bears, the officer noticed that there were bulges inside the same which did not feel like foam
stuffing. It was only after the officers had opened the bags that accused finally presented his passport.
Accused was then brought to the headquarters of the NARCOM at Camp Dangwa, La Trinidad, Benguet for
further investigation. At the investigation room, the officers opened the teddy bears and they were found to also
contain hashish. Representative samples were taken from the hashish found among the personal effects of
accused and the same were brought to the PC Crime Laboratory for chemical analysis.

In the chemistry report, it was established that the objects examined were hashish. a prohibited drug which is a
derivative of marijuana. Thus, an information was filed against accused for violation of the Dangerous Drugs Act.

During the arraignment, accused entered a plea of "not guilty." For his defense, he raised the issue of illegal
search of his personal effects. He also claimed that the hashish was planted by the NARCOM officers in his
pouch bag and that the two (2) travelling bags were not owned by him, but were merely entrusted to him by an
Australian couple whom he met in Sagada. He further claimed that the Australian couple intended to take the
same bus with him but because there were no more seats available in said bus, they decided to take the next ride
and asked accused to take charge of the bags, and that they would meet each other at the Dangwa Station.

Likewise, accused alleged that when the NARCOM officers demanded for his passport and other Identification
papers, he handed to one of the officers his pouch bag which was hanging on his neck containing, among others,
his passport, return ticket to Sweden and other papers. The officer in turn handed it to his companion who brought
the bag outside the bus. When said officer came back, he charged the accused that there was hashish in the bag.
He was told to get off the bus and his picture was taken with the pouch bag placed around his neck. The trial court
did not give credence to accused's defense.

The claim of the accused that the hashish was planted by the NARCOM officers, was belied by his failure to raise
such defense at the earliest opportunity. When accused was investigated at the Provincial Fiscal's Office, he did
not inform the Fiscal or his lawyer that the hashish was planted by the NARCOM officers in his bag. It was only
two (2) months after said investigation when he told his lawyer about said claim, denying ownership of the two (2)
travelling bags as well as having hashish in his pouch bag.

In a decision dated 12 October 1989, the trial court found accused guilty beyond reasonable doubt for violation of
the Dangerous Drugs Act, specifically Section 4, Art. II of RA 6425, as amended. The dispositive portion of the
3

decision reads as follows:

WHEREFORE, finding the guilt of the accused Mikael Malmstedt established beyond reasonable doubt, this
Court finds him GUILTY of violation of Section 4, Article 11 of Republic Act 6425, as amended, and hereby
sentences him to suffer the penalty of life imprisonment and to pay a fine of Twenty Thousand Pesos
(P20,000.00), with subsidiary imprisonment in case of insolvency and to pay the costs.

Let the hashish subject of this case be turned over to the First Narcotics Regional Unit at Camp Bado; Dangwa,
La Trinidad Benguet for proper disposition under Section 20, Article IV of Republic Act 6425, as amended.

SO ORDERED. 4

Seeking the reversal of the decision of the trial court finding him guilty of the crime charged, accused argues that
the search of his personal effects was illegal because it was made without a search warrant and, therefore, the
prohibited drugs which were discovered during the illegal search are not admissible as evidence against him.

The Constitution guarantees the right of the people to be secure in their persons, houses, papers and effects
against unreasonable searches and seizures. However, where the search is made pursuant to a lawful arrest,
5

there is no need to obtain a search warrant. A lawful arrest without a warrant may be made by a peace officer or a
private person under the following circumstances. 6

Sec. 5 Arrest without warrant; when lawful. –– A peace officer or a private person may, without a warrant, arrest a
person:

(a) When, in his presence, the person to be arrested has committed is actually committing, or is attempting to
commit an offense;
(b) When an offense has in fact just been committed, and he has personal knowledge of facts indicating that the
person to be arrested has committed it; and

(c) When the person to be arrested is a prisoner who has escaped from a penal establishment or place where he
is serving final judgment or temporarily confined while his case is pending, or has escaped while being transferred
from one confinement to another.

In cases falling under paragraphs (a) and (b) hereof, the person arrested without a warrant shall be forthwith
delivered to the nearest police station or jail, and he shall be proceeded against in accordance with Rule 112,
Section 7. (6a 17a).

Accused was searched and arrested while transporting prohibited drugs (hashish). A crime was actually being
committed by the accused and he was caught in flagrante delicto. Thus, the search made upon his personal
effects falls squarely under paragraph (1) of the foregoing provisions of law, which allow a warrantless search
incident to a lawful arrest.
7

While it is true that the NARCOM officers were not armed with a search warrant when the search was made over
the personal effects of accused, however, under the circumstances of the case, there was sufficient probable
cause for said officers to believe that accused was then and there committing a crime.

Probable cause has been defined as such facts and circumstances which could lead a reasonable, discreet and
prudent man to believe that an offense has been committed, and that the objects sought in connection with the
offense are in the place sought to be searched. The required probable cause that will justify a warrantless search
8

and seizure is not determined by any fixed formula but is resolved according to the facts of each case. 9

Warrantless search of the personal effects of an accused has been declared by this Court as valid, because of
existence of probable cause, where the smell of marijuana emanated from a plastic bag owned by the
accused, or where the accused was acting suspiciously, and attempted to flee.
10 11 12

Aside from the persistent reports received by the NARCOM that vehicles coming from Sagada were transporting
marijuana and other prohibited drugs, their Commanding Officer also received information that a Caucasian
coming from Sagada on that particular day had prohibited drugs in his possession. Said information was received
by the Commanding Officer of NARCOM the very same morning that accused came down by bus from Sagada
on his way to Baguio City.

When NARCOM received the information, a few hours before the apprehension of herein accused, that a
Caucasian travelling from Sagada to Baguio City was carrying with him prohibited drugs, there was no time to
obtain a search warrant. In the Tangliben case, the police authorities conducted a surveillance at the Victory
13

Liner Terminal located at Bgy. San Nicolas, San Fernando Pampanga, against persons engaged in the traffic of
dangerous drugs, based on information supplied by some informers. Accused Tangliben who was acting
suspiciously and pointed out by an informer was apprehended and searched by the police authorities. It was held
that when faced with on-the-spot information, the police officers had to act quickly and there was no time to
secure a search warrant.

It must be observed that, at first, the NARCOM officers merely conducted a routine check of the bus (where
accused was riding) and the passengers therein, and no extensive search was initially made. It was only when
one of the officers noticed a bulge on the waist of accused, during the course of the inspection, that accused was
required to present his passport. The failure of accused to present his identification papers, when ordered to do
so, only managed to arouse the suspicion of the officer that accused was trying to hide his identity. For is it not a
regular norm for an innocent man, who has nothing to hide from the authorities, to readily present his
identification papers when required to do so?

The receipt of information by NARCOM that a Caucasian coming from Sagada had prohibited drugs in his
possession, plus the suspicious failure of the accused to produce his passport, taken together as a whole, led the
NARCOM officers to reasonably believe that the accused was trying to hide something illegal from the authorities.
From these circumstances arose a probable cause which justified the warrantless search that was made on the
personal effects of the accused. In other words, the acts of the NARCOM officers in requiring the accused to open
his pouch bag and in opening one of the wrapped objects found inside said bag (which was discovered to contain
hashish) as well as the two (2) travelling bags containing two (2) teddy bears with hashish stuffed inside them,
were prompted by accused's own attempt to hide his identity by refusing to present his passport, and by the
information received by the NARCOM that a Caucasian coming from Sagada had prohibited drugs in his
possession. To deprive the NARCOM agents of the ability and facility to act accordingly, including, to search even
without warrant, in the light of such circumstances, would be to sanction impotence and ineffectiveness in law
enforcement, to the detriment of society.

WHEREFORE, premises considered, the appealed judgment of conviction by the trial court is hereby AFFIRMED.
Costs against the accused-appellant.

SO ORDERED.

Melencio-Herrera, Paras, Feliciano, Bidin, Griño-Aquino, Medialdea, Regalado and Davide, Jr., JJ., concur.
Sarmiento, J., is on leave.

Separate Opinions

NARVASA, J., concurring and dissenting:

The ancient tradition that a man's home is his castle, safe from intrusion even by the king, has not only found its
niche in all our charters, from 1935 to the present; it has also received unvarying recognition and acceptance in
our case law. The present Constitution declares that —
1 2

The right of the people to be secure in their persons, houses, papers, and effects against unreasonable searches
and seizures of whatever nature and for any purpose, shall be inviolable, and no search warrant or warrant of
arrest shall issue except upon probable cause to be determined personally by the judge after examination under
oath or affirmation of the complainant and the witnesses he may produce, and particularly describing the place to
be searched, and the persons or things to be seized.

It further ordains that any evidence obtained in violation of said right, among others, "shall be inadmissible for any
purpose in any proceeding." 3

The rule is that no person may be subjected by the police or other government authority to a search of his body,
or his personal effects or belongings, or his residence except by virtue of a search warrant or on the occasion of a
legitimate arrest.4

An arrest is legitimate, of course, if effected by virtue of a warrant of arrest. Even without a warrant, an arrest may
also be lawfully made by a peace officer or a private person: 5

(a) when, in his presence, the person to be arrested has committed is actually committing, or is attempting to
commit an offense;

(b) When an offense has in fact just been committed, and he has personal knowledge of facts indicating that the
person to be arrested has committed it; and

(c) When the person to be arrested is a prisoner who has escaped from a penal establishment or place where he
is serving final judgment or temporarily confined while his case is pending, or has escaped while being transferred
from one confinement to another.

In cases falling under paragraphs (a) and (b) hereof, the person arrested without a warrant shall be forthwith
delivered to the nearest police station or jail, and he shall be proceeded against in accordance with Rule 112,
Section 7.

In any of these instances of a lawful arrest, the person arrested "may be searched for dangerous weapons or
anything which may be used as proof of the commission of an offense, without a search warrant." And it has6
been held that the search may extend to the area "within his immediate control," i.e., the area from which said
person arrested might gain possession of a weapon or destructible evidence. 7

Apart from "search incidental to an arrest," a warrantless search has also been held to be proper in cases of
"search of a moving vehicle, and "seizure of evidence in plain view." This was the pronouncement in Manipon, Jr.
8 9

v. Sandiganbayan, 143 SCRA 267, 276, which drew attention to Moreno v. Ago Chi; Alvero v. Dizon, Papa v.
10 11

Mago, and an American precedent, Harris v. U.S.


12 13

If, on the other, a person is searched without a warrant, or under circumstances other than those justifying an
arrest without warrant in accordance with law, supra, merely on suspicion that he is engaged in some felonious
enterprise, and in order to discover if he has indeed committed a crime, it is not only the arrest which is illegal but
also, the search on the occasion thereof, as being "the fruit of the poisonous tree. In that event, any evidence
14

taken, even if confirmatory of the initial suspicion, is inadmissible "for any purpose in any proceeding." But the
15

right against an unreasonable search and seizure may be waived by the person arrested, provided he knew of
such right and knowingly decided not to invoke it. 16

There is unanimity among the members of the Court upon the continuing validity of these established principles.
However, the Court is divided as regards the ultimate conclusions which may properly be derived from the proven
facts and consequently, the manner in which the principles just cited should apply thereto.

The proofs of the prosecution and those of the defense are diametrically at odds. What is certain, however, is that
the soldiers had no warrant of arrest when they conducted a search of Malmstedt's person and the things in his
possession at the time. Indeed, the Court a quo acknowledged that the soldiers could "not be expected to be
armed with a warrant or arrest nor a search warrant everytime they establish a temporary checkpoint . . . (and) no
judge would issue them one considering that searching questions have to be asked before a warrant could be
issued." Equally plain is that prior to the search, a warrantless arrest of Malmstedt could not validly have been in
accordance with the norms of the law. For Malmstedt had not committed, nor was he actually committing or
attempting to commit a crime, in the soldiers' presence, nor did said soldiers have personal and competent
knowledge that Malmstedt had in fact just committed a crime. All they had was a suspicion that Malmstedt might
have some prohibited drug on him or in his bags; all they had was, in the words of the Trial Court, "the hope of
intercepting any dangerous drug being transported," or, as the Office of the Solicitor General asserts, "information
that most of the buses coming . . . (from the Cordillera) were transporting marijuana and other prohibited drugs."

This case, is remarkably similar to Peo. v. Aminnudin, decided on July 6, 1988 also by the First Division. There, 17

Aminnudin was arrested without a warrant by PC officers as he was disembarking from an inter-island vessel.
The officers were waiting for him because he was, according to an informer's report, then transporting marijuana.
The search of Aminnudin's bag confirmed the informer's report; the bag indeed contained marijuana. The Court
nevertheless held that since the PC officers had failed to procure a search warrant although they had sufficient
time (two days) to do so and therefore, the case presented no such urgency as to justify a warrantless search, the
search of Aminnudin's person and bag, the seizure of the marijuana and his subsequent arrest were illegal; and
the marijuana was inadmissible in evidence in the criminal action subsequently instituted against Aminnudin for
violating the Dangerous Drugs Act.

There are, on the other hand, other cases adjudicated by this Court in which apparently different conclusions
were reached. It is needful to devote a few words to them so that the relevant constitutional and legal propositions
are not misunderstood.

In People v. Claudio (decision promulgated on April 15, 1988), the accused boarded a "Victory Liner" passenger
18

bus going to Olongapo from Baguio City. She placed the plastic bag she was carrying at the back of the seat then
occupied by Obiña, an INP member "on Detached Service with the Anti-Narcotics Unit." This avowedly aroused
Obiña's suspicion, and at the first opportunity, and without Claudio's knowledge, he surreptitiously looked into the
plastic bag and noted that it contained camote tops as well as a package, and that there emanated from the
package the smell of marijuana with which he had become familiar on account of his work. So when the bus
stopped at Sta. Rita, and Claudio alighted, Obiña accosted her, showed her his ID, identified himself as a
policeman, and announced his intention to search her bag which he said contained marijuana because of the
distinctive odor detected by him. Ignoring her plea — "Please go with me, let us settle this at home" — he brought
her to the police headquarters., where examination of the package in Claudio's bag confirmed his suspicion that it
indeed contained marijuana. The Court held the warrantless arrest under the circumstances to be lawful, the
search justified, and the evidence thus discovered admissible in evidence against the accused.

In People v. Tangliben (decision promulgated on April 6, 1990), two police officers and a barangay tanod were
19

conducting a "surveillance mission" at the Victory Liner Terminal at San Nicolas, San Fernando, Pampanga,
"aimed not only against persons who may commit misdemeanors . . . (there) but also on persons who may be
engaging in the traffic of dangerous drugs based on information supplied by informers; . . . they noticed a person
carrying a red travelling bag . . who was acting suspiciously;" they asked him to open the bag; the person did so
only after they identified themselves as peace officers; found in the bag were marijuana leaves wrapped in plastic
weighing one kilogram, more or less; the person was then taken to the police headquarters at San Fernando,
Pampanga, where he was investigated; and an information was thereafter filed against that person, Tangliben,
charging him with a violation of the Dangerous Drugs Act of 1972 (RA 6425), as amended. Upon these facts it
was ruled, citing Claudio, supra, that there was a valid warrantless arrest and a proper warrantless search
incident thereto.

The facts in Tangliben were pronounced to be different from those in People v. Aminnudin, supra. "In contrast"
to Aminnudin where the Court perceived no urgency as to preclude the application for and obtention of a search
warrant, it was declared that the Tangliben case —

. . . presented urgency. . . (The evidence revealed) that there was an informer who pointed to the
accused-appellant as carrying marijuana . . . Faced with such on-the-spot information, the police officers had to
act quickly. There was not enough time to secure a search warrant . . . To require search warrants during
on-the-spot apprehensions of drug pushers, illegal possessors of firearms, jueteng collectors, smugglers of
contraband goods, robber, etc. would make it extremely difficult, if not impossible to contain the crimes with which
these persons are associated.

In Tangliben, therefore, there was in the Court's view sufficient evidence on hand to enable the PC officers to
secure a search warrant, had there been time. But because there was actually no time to get the warrant, and
there were "on-the-spot" indications that Tangliben was then actually committing a crime, the search of his person
and his effects was considered valid.

Two other decisions presented substantially similar circumstance instances: Posadas v. C.A., et al., decided on
August 2, 1990, and People v. Moises Maspil, Jr., et al., decided on August 20, 1990.
20 21

In the first case, Posadas was seen to be acting suspiciously by two members of the INP, Davao Metrodiscom,
and when he was accosted by the two, who identified themselves as police officers, he suddenly fled. He was
pursued, overtaken and, notwithstanding his resistance, placed in custody. The buri bag Posadas was then
carrying was found to contain a revolver, for which he could produce no license or authority to possess, four
rounds of live ammunition, and a tear gas grenade. He was prosecuted for illegal possession of firearms and
ammunition and convicted after trial. This Court affirmed Posadas' conviction, holding that there was, in the
premises, probable cause for a search without warrant, i.e., the appellant was acting suspiciously and attempted
to flee with the buri bag he had with him at the time. The Court cited with approval the ruling of the U.S. Federal
Supreme Court in John W. Terry v. State of Ohio, a 1968 case, which the Solicitor General had invoked to justify
22

the search.

In the case of Maspil, et al., a checkpoint was set up by elements of the First Narcotics Regional Unit of the
Narcotics Command at Sayangan, Atok, Benguet, to monitor, inspect and scrutinize vehicles on the highway
going towards Baguio City. This was done because of a confidential report by informers that Maspil and another
person, Bagking, would be transporting a large quantity of marijuana to Baguio City. In fact, the informers were
with the policemen manning the checkpoint. As expected, at about 2 o'clock in the early morning of November 1,
1986, a jeepney approached the checkpoint, driven by Maspil, with Bagking as passenger. The officers stopped
the vehicle and saw that on it were loaded 2 plastic sacks, a jute sack, and 3 big round tin cans. When opened,
the sacks and cans were seen to contain what appeared to be marijuana leaves. The policemen thereupon
placed Maspil and Bagking under arrest, and confiscated the leaves which, upon scientific examination, were
verified to be marijuana leaves. The Court upheld the validity of the search thus conducted, as being incidental to
a lawful warrantless arrest, and declared that, as in Tangliben, supra, Maspil and Bagking had been caught in
23

flagrante delictotransporting prohibited drugs at the time of their arrest. Again, the Court took occasion to
distinguish the case from Aminnudin in which, as aforestated, it appeared that the police officers were aware of
24
Aminnudin's identity, his projected criminal enterprise and the vessel on which he would be arriving, and, equally
as importantly, had sufficient time and opportunity to obtain a search warrant. In the case of Maspil and Bagking,
the Court found that the officers concerned had no exact description of the vehicle the former would be using to
transport marijuana, and no inkling of the definite time of the suspects' arrival, and pointed out that a jeepney on
the road is not the same as a passenger boat on the high seas whose route and time of arrival are more or less
certain, and which ordinarily cannot deviate from or otherwise alter its course, or select another destination. 25

The most recent decision treating of warrantless search and seizure appears to be People v. Lo Ho Wing; et al.,
G.R. No. 88017, decided on January 21, 1991 (per Gancayco, J.). In that case, an undercover or "deep
penetration" agent, Tia, managed somehow to gain acceptance into a group of suspected drug smugglers, which
included Peter Lo and Lim Ching Huat. Tia accompanied Peter Lo to Guangzhou, China, where he saw him and
other person empty the contents of six (6) tins of tea and replace them with white powder. On their return to
Manila with the cans of substituted "tea," they were met at the airport by Lim. As they were leaving the airport in
separate vehicles, they were intercepted by officers and operatives of the Narcotics Command (NARCOM), who
had earlier been tipped off by Tia, and placed under arrest. As search of the luggage brought in by Tia and Peter
Lo, loaded on the group's vehicles, quickly disclosed the six (6) tin cans containing fifty-six (56) bags of white
crystalline powder which, upon analysis, was identified as metamphetamine. Tia, Lo and Lim were indicted for
violation of the Dangerous Drugs Act of 1972. Tia was discharged as state witness. Lo and Lim were
subsequently convicted and sentenced to life imprisonment. One of the questions raised by them in this Court on
appeal was whether the warrantless search of their vehicles and personal effects was legal. The
Court, citing Manipon, Jr. v. Sandiganbayan, 143 SCRA 267 (1986), held legal the search of the appellants'
26

moving vehicles and the seizure therefrom of the dangerous drug, considering that there was intelligence
information, including clandestine reports by a planted spy actually participating in the activity, that the appellants
were bringing prohibited drugs into the country; that the requirement of obtaining a search warrant "borders on
the impossible in the case of smuggling effected by the use of a moving vehicle that can transport contraband
from one place to another with impunity," and "it is not practicable to secure a warrant because the vehicle can be
quickly moved out of the locality or jurisdiction in which the warrant must be sought. 27

In all five cases, Claudio, Tangliben, Posadas, Maspil, and Lo Ho Wing, facts existed which were found by the
Court as justifying warantless arrests. In Claudio, the arresting officer had secretly ascertained that the woman he
was arresting was in fact in possession of marijuana; he had personally seen that her bag contained not only
vegetables but also a package emitting the odor of marijuana. In Tangliben, the person arrested and searched
was acting suspiciously, and had been positively pointed to as carrying marijuana. And in both cases, the
accused were about to board passenger buses, making it urgent for the police officers concerned to take quick
and decisive action. In Posadas, the person arrested and searched was acting suspiciously, too, and when
accosted had attempted to flee from the police officers. And in Maspil and Lo Ho Wing, there was definite
information of the precise identity of the persons engaged in transporting prohibited drugs at a particular time and
place.

Now, as regards the precise issue at hand, whether or not the facts in the case at bar make out a legitimate
instance of a warrantless search and seizure, there is, as earlier pointed out, a regrettable divergence of views
among the members of the Court.

Contrary to the conclusion reached by the majority, I believe that the appellant should be absolved on reasonable
doubt. There was in this case no confidential report from, or positive identification by an informer; no attempt to
flee; no bag or package emitting tell-tale odors; no other reasonably persuasive indications that Malmstedt was at
the time in process of perpetrating the offense for which he was subsequently prosecuted. Hence, when the
soldiers searched Malmstedt's pouch and the bags in his possession, they were simply "fishing" for evidence. It
matters not that the search disclosed that the bags contained prohibited substances, confirming their initial
information and suspicion. The search was not made by virtue of a warrant or as an incident of a lawful
warrantless arrest, i.e., under circumstances sufficient to engender a reasonable belief that some crime was
being or about to be committed, or adjust been committed. There was no intelligent and intentional waiver of the
right against unreasonable searches and seizure. The search was therefore illegal, since the law requires that
there first be a lawful arrest of an individual before a search of his body and his belongings may licitly be made.
The process cannot be reversed, i.e., a search be first undertaken, and then an arrest effected, on the strength of
the evidence yielded by the search. An arrest made in that case would be unlawful, and the search undertaken as
an incident of such an unlawful arrest, also unlawful.
The fact that when investigated at the headquarters of the Narcotic Command at Camp Dangwa, La Trinidad,
Malmstedt had, it is said, willingly admitted that there were was hashish inside the "teddy bears" in the luggage
found in his possession — an admission subsequently confirmed by laboratory examination — does not help the
cause of the prosecution one bit. Nothing in the record even remotely suggests that Malmstedt was accorded the
rights guaranteed by the Constitution to all persons under custodial investigation. He was not informed, prior to
28

being interrogated, that he had the "right to remain silent and to have competent and independent counsel
preferably of his own choice," and that if he could not afford the services of counsel, he would be provided with
one; not does it appear at all that he waived those rights "in writing and in the presence of counsel." The soldiers
and the police officers simply went ahead with the investigation of Malmstedt, without counsel. The admissions
elicited from Malmstedt under these circumstances, as the Constitution clearly states, are "inadmissible in
evidence against him. 29

The prohibited drugs supposedly discovered in Malmstedt's bags, having been taken in violation of the
constitutional right against unreasonable searches and seizures, are inadmissible against him "for any purpose in
any proceeding." Also pronounced as incompetent evidence against him are the admissions supposedly made by
him without his first being accorded the constitutional rights of persons under custodial investigation. Without
such object evidence and admissions, nothing remains of the case against Malmstedt.

It may be conceded that, as the Trial Court points out, the evidence presented by Malmstedt in his defense is
feeble, unworthy of credence. This is beside the point; for conformably to the familiar axiom, the State must rely
on the strength of its evidence and not on the weakness of the defense. The unfortunate fact is that although the
existence of the hashish is an objective physical reality that cannot but be conceded, there is in law no evidence
to demonstrate with any degree of persuasion, much less beyond reasonable doubt, that Malmstedt was engaged
in a criminal activity. This is the paradox created by the disregard of the applicable constitutional safeguards. The
tangible benefit is that the hashish in question has been correctly confiscated and thus effectively withdrawn from
private use.

What is here said should not by any means be taken as a disapproval or a disparagement of the efforts of the
police and military authorities to deter and detect offenses, whether they be possession of and traffic in prohibited
drugs, or some other. Those efforts obviously merit the support and commendation of the Courts and indeed of
every responsible citizen. But those efforts must take account of the basic rights granted by the Constitution and
the law to persons who may fall under suspicion of engaging in criminal acts. Disregard of those rights may not be
justified by the objective of ferreting out and punishing crime, no matter how eminently desirable attainment of
that objective might be. Disregard of those rights, as this Court has earlier stressed, may result in the escape of
the guilty, and all because the "constable has blundered," rendering the evidence inadmissible even if truthful or
otherwise credible.30

I therefore vote to reverse the Trial Court's judgment of October 12, 1989 and to acquit the appellant on
reasonable doubt.

CRUZ, J., dissenting:

I join Mr. Justice Andres R. Narvasa in his dissent, which I believe represents the correct application to the facts
of this case of the provisions of the Bill of Rights and the Rules of Court on searches and seizures. It is consistent
with my ponencia in People v. Aminnudin, 163 SCRA 402, and also with Alih v. Castro, 151 SCRA 279, the latter
being a unanimous decision of the Court en banc, and my dissents in Umil v. Ramos (on warrantless arrests, 187
SCRA 311, Valmonte v. De Villa (on checkpoints), 178, SCRA 211, 185 SCRA 665, and Guazon v. De Villa (on
"zonas"), 181 SCRA 623.

I write this separate opinion merely to remark on an observation made during the deliberation on this case that
some members of the Court seem to be coddling criminals instead of extending its protection to society, which
deserves our higher concern. The inference is that because of our wrong priorities, criminals are being
imprudently let free, to violate our laws again; and it is all our fault.

Believing myself to be among those alluded to, I will say without apology that I do not consider a person a criminal,
until he is convicted by final judgment after a fair trial by a competent and impartial court. Until then, the
Constitution bids us to presume him innocent. He may seem boorish or speak crudely or sport tattoos or dress
weirdly or otherwise fall short of our own standards of propriety and decorum. None of these makes him a criminal
although he may look like a criminal.

It is so easy to condemn a person on the basis of his appearance but it is also so wrong.

On the question before us, it seems to be the inclination of some judges to wink at an illegal search and seizure
as long as the suspect has been actually found in possession of a prohibited article That fact will retroactively
validate the violation of the Bill of Rights for after all, as they would rationalize, the suspect is a criminal. What
matters to them is the fact of illegal possession, not the fact of illegal search and seizure.

This kind of thinking takes us back to the intolerant days of Moncado v. People's Court, 80 Phil. 1, which was
discredited in Stonehill v. Diokno, 20 SCRA 383, even before it was definitely rejected by an express provision in
the 1973 Constitution. That provision, which has been retained in the present Constitution, again explicitly
declares that any evidence illegally obtained "shall be inadmissible for any purpose in any proceeding."

The fruit of the poisonous tree should not be allowed to poison our system of criminal justice. In the case at bar,
1âwphi1

the search was made at a checkpoint established for the preposterous reason that the route was being used by
marijuana dealers and on an individual who had something bulging at his waist that excited the soldier's suspicion.
Was that probable cause? The ponencia notes that the military had advance information that a Caucasian was
coming from the Sagada with prohibited drugs in his possession. This is what the military says now, after the fact,
to justify the warrantless search. It is so easy to make such a claim, and I am surprised that the majority should
readily accept it.

The conclusion that there was probable cause may have been influenced by the subsequent discovery that the
accused was carrying a prohibited drug. This is supposed to justify the soldier's suspicion. In other words, it was
the fact of illegal possession that retroactively established the probable cause that validated the illegal search and
seizure. It was the fruit of the poisonous tree that washed clean the tree itself.

In Olmstead v. U.S., 277 U.S. 438, Justice Holmes said sixty-four years ago:

. . . It is desirable that criminals should be detected, and to that end that all available evidence should be used. It
1avvphi1

is also desirable that the government should not itself foster and pay for other crimes, when they are the means
by which the evidence is to be obtained. If it pays its officers for having got evidence by crime, I do not see why it
may not as well pay them for getting it in the same way, and I can attach no importance to protestations of
disapproval if it knowingly accepts and pays and announces that in the future it will pay for the fruits. We have to
choose, and for my part I think it a less evil that some criminals should escape than that the government should
play an ignoble part.

If by deterring the government from playing "an ignoble part," I am "coddling criminals," I welcome the accusation
and take pride in it. I would rather err in favor of the accused who is impaled with outlawed evidence than exalt
order at the price of liberty.
RODEL LUZ y ONG, G. R. No. 197788
Petitioner,

Present:

CARPIO, J., Chairperson,


- versus - BRION,
PEREZ,
SERENO, and
REYES, JJ.

PEOPLE OF THE Promulgated:


[1]
PHILIPPINES,
Respondent. February 29, 2012
x--------------------------------------------------x

DECISION

SERENO, J.:

This is a Petition for Review on Certiorari under Rule 45 seeking to set aside the Court
of Appeals (CA) Decision in CA-G.R. CR No. 32516 dated 18 February 2011[2] and
Resolution dated 8 July 2011.

Statement of the Facts and of the Case

The facts, as found by the Regional Trial Court (RTC), which sustained the version of
the prosecution, are as follows:

PO2 Emmanuel L. Alteza, who was then assigned at the Sub-Station 1 of the Naga
City Police Station as a traffic enforcer, substantially testified that on March 10, 2003 at
around 3:00 oclock in the morning, he saw the accused, who was coming from the direction
of Panganiban Drive and going to Diversion Road, Naga City, driving a motorcycle without a
helmet; that this prompted him to flag down the accused for violating a municipal ordinance
which requires all motorcycle drivers to wear helmet (sic) while driving said motor vehicle;
that he invited the accused to come inside their sub-station since the place where he flagged
down the accused is almost in front of the said sub-station; that while he and SPO1 Rayford
Brillante were issuing a citation ticket for violation of municipal ordinance, he noticed that
the accused was uneasy and kept on getting something from his jacket; that he was alerted
and so, he told the accused to take out the contents of the pocket of his jacket as the latter
may have a weapon inside it; that the accused obliged and slowly put out the contents of the
pocket of his jacket which was a nickel-like tin or metal container about two (2) to three (3)
inches in size, including two (2) cellphones, one (1) pair of scissors and one (1) Swiss knife;
that upon seeing the said container, he asked the accused to open it; that after the accused
opened the container, he noticed a cartoon cover and something beneath it; and that upon his
instruction, the accused spilled out the contents of the container on the table which turned out
to be four (4) plastic sachets, the two (2) of which were empty while the other two (2)
contained suspected shabu.[3]

Arraigned on 2 July 2003, petitioner, assisted by counsel, entered a plea of Not guilty
to the charge of illegal possession of dangerous drugs. Pretrial was terminated on 24
September 2003, after which, trial ensued.

During trial, Police Officer 3 (PO3) Emmanuel Alteza and a forensic chemist testified
for the prosecution. On the other hand, petitioner testified for himself and raised the defense
of planting of evidence and extortion.

In its 19 February 2009 Decision,[4] the RTC convicted petitioner of illegal possession
of dangerous drugs[5] committed on 10 March 2003. It found the prosecution evidence
sufficient to show that he had been lawfully arrested for a traffic violation and then subjected
to a valid search, which led to the discovery on his person of two plastic sachets later found
to contain shabu. The RTC also found his defense of frame-up and extortion to be weak,
self-serving and unsubstantiated. The dispositive portion of its Decision held:

WHEREFORE, judgment is hereby rendered, finding accused RODEL LUZ y


ONG GUILTY beyond reasonable doubt for the crime of violation of Section 11, Article II of
Republic Act No. 9165 and sentencing him to suffer the indeterminate penalty of
imprisonment ranging from twelve (12) years and (1) day, as minimum, to thirteen (13) years,
as maximum, and to pay a fine of Three Hundred Thousand Pesos (₱300,000.00).

The subject shabu is hereby confiscated for turn over to the Philippine Drug
Enforcement Agency for its proper disposition and destruction in accordance with law.

SO ORDERED.[6]

Upon review, the CA affirmed the RTCs Decision.

On 12 September 2011, petitioner filed under Rule 45 the instant Petition for Review
on Certiorari dated 1 September 2011. In a Resolution dated 12 October 2011, this Court
required respondent to file a comment on the Petition. On 4 January 2012, the latter filed its
Comment dated 3 January 2012.

Petitioner raised the following grounds in support of his Petition:

(i) THE SEARCH AND SEIZURE OF THE ALLEGED SUBJECT SHABU IS


INVALID.

(ii) THE PRESUMPTION OF REGULARITY IN THE PERFORMANCE OF


DUTY OF THE POLICE OFFICER CANNOT BE RELIED UPON IN THIS
CASE.

(iii) THE INTEGRITY AND EVIDENTIARY VALUE OF THE ALLEGED


SUBJECT SPECIMEN HAS BEEN COMPROMISED.
(iv) THE GUILT OF THE ACCUSED-PETITIONER WAS NOT PROVEN
BEYOND THE REASONABLE DOUBT (sic).[7]

Petitioner claims that there was no lawful search and seizure, because there was no lawful
arrest. He claims that the finding that there was a lawful arrest was erroneous, since he was
not even issued a citation ticket or charged with violation of the city ordinance. Even
assuming there was a valid arrest, he claims that he had never consented to the search
conducted upon him.

On the other hand, finding that petitioner had been lawfully arrested, the RTC held thus:

It is beyond dispute that the accused was flagged down and apprehended in this case by
Police Officers Alteza and Brillante for violation of City Ordinance No. 98-012, an ordinance
requiring the use of crash helmet by motorcycle drivers and riders thereon in the City of Naga
and prescribing penalties for violation thereof. The accused himself admitted that he was not
wearing a helmet at the time when he was flagged down by the said police officers, albeit he
had a helmet in his possession. Obviously, there is legal basis on the part of the apprehending
officers to flag down and arrest the accused because the latter was actually committing a
crime in their presence, that is, a violation of City Ordinance No. 98-012. In other words, the
accused, being caught in flagrante delicto violating the said Ordinance, he could therefore be
lawfully stopped or arrested by the apprehending officers. x x x.[8]

We find the Petition to be impressed with merit, but not for the particular reasons alleged. In
criminal cases, an appeal throws the entire case wide open for review and the reviewing
tribunal can correct errors, though unassigned in the appealed judgment, or even reverse the
trial courts decision based on grounds other than those that the parties raised as errors.[9]

First, there was no valid arrest of petitioner. When he was flagged down for committing a
traffic violation, he was not, ipso facto and solely for this reason, arrested.

Arrest is the taking of a person into custody in order that he or she may be bound to
answer for the commission of an offense.[10] It is effected by an actual restraint of the person
to be arrested or by that persons voluntary submission to the custody of the one making the
arrest. Neither the application of actual force, manual touching of the body, or physical
restraint, nor a formal declaration of arrest, is required. It is enough that there be an intention
on the part of one of the parties to arrest the other, and that there be an intent on the part of
the other to submit, under the belief and impression that submission is necessary. [11]

Under R.A. 4136, or the Land Transportation and Traffic Code, the general procedure
for dealing with a traffic violation is not the arrest of the offender, but the confiscation of the
drivers license of the latter:

SECTION 29. Confiscation of Driver's License. Law enforcement and peace officers
of other agencies duly deputized by the Director shall, in apprehending a driver for any
violation of this Act or any regulations issued pursuant thereto, or of local traffic rules and
regulations not contrary to any provisions of this Act, confiscate the license of the driver
concerned and issue a receipt prescribed and issued by the Bureau therefor which shall
authorize the driver to operate a motor vehicle for a period not exceeding seventy-two hours
from the time and date of issue of said receipt. The period so fixed in the receipt shall not be
extended, and shall become invalid thereafter. Failure of the driver to settle his case within
fifteen days from the date of apprehension will be a ground for the suspension and/or
revocation of his license.

Similarly, the Philippine National Police (PNP) Operations Manual[12] provides the following
procedure for flagging down vehicles during the conduct of checkpoints:
SECTION 7. Procedure in Flagging Down or Accosting Vehicles While in Mobile
Car. This rule is a general concept and will not apply in hot pursuit operations. The mobile
car crew shall undertake the following, when applicable: x x x

m. If it concerns traffic violations, immediately issue a Traffic Citation Ticket (TCT) or


Traffic Violation Report (TVR). Never indulge in prolonged, unnecessary
conversation or argument with the driver or any of the vehicles occupants;

At the time that he was waiting for PO3 Alteza to write his citation ticket, petitioner
could not be said to have been under arrest. There was no intention on the part of PO3 Alteza
to arrest him, deprive him of his liberty, or take him into custody. Prior to the issuance of the
ticket, the period during which petitioner was at the police station may be characterized
merely as waiting time. In fact, as found by the trial court, PO3 Alteza himself testified that
the only reason they went to the police sub-station was that petitioner had been flagged down
almost in front of that place. Hence, it was only for the sake of convenience that they were
waiting there. There was no intention to take petitioner into custody.

In Berkemer v. McCarty,[13] the United States (U.S.) Supreme Court discussed at


length whether the roadside questioning of a motorist detained pursuant to a routine traffic
stop should be considered custodial interrogation. The Court held that, such questioning does
not fall under custodial interrogation, nor can it be considered a formal arrest, by virtue of
the nature of the questioning, the expectations of the motorist and the officer, and the length
of time the procedure is conducted. It ruled as follows:

It must be acknowledged at the outset that a traffic stop significantly curtails the
freedom of action of the driver and the passengers, if any, of the detained vehicle. Under the
law of most States, it is a crime either to ignore a policemans signal to stop ones car or, once
having stopped, to drive away without permission. x x x

However, we decline to accord talismanic power to the phrase in the Miranda opinion
emphasized by respondent. Fidelity to the doctrine announced in Miranda requires that it be
enforced strictly, but only in those types of situations in which the concerns that powered the
decision are implicated. Thus, we must decide whether a traffic stop exerts upon a detained
person pressures that sufficiently impair his free exercise of his privilege against
self-incrimination to require that he be warned of his constitutional rights.

Two features of an ordinary traffic stop mitigate the danger that a person questioned
will be induced to speak where he would not otherwise do so freely, Miranda v. Arizona, 384
U. S., at 467. First, detention of a motorist pursuant to a traffic stop is presumptively
temporary and brief. The vast majority of roadside detentions last only a few minutes. A
motorists expectations, when he sees a policemans light flashing behind him, are that he will
be obliged to spend a short period of time answering questions and waiting while the officer
checks his license and registration, that he may then be given a citation, but that in the end he
most likely will be allowed to continue on his way. In this respect, questioning incident to an
ordinary traffic stop is quite different from stationhouse interrogation, which frequently is
prolonged, and in which the detainee often is aware that questioning will continue until he
provides his interrogators the answers they seek. See id., at 451.

Second, circumstances associated with the typical traffic stop are not such that
the motorist feels completely at the mercy of the police. To be sure, the aura of authority
surrounding an armed, uniformed officer and the knowledge that the officer has some
discretion in deciding whether to issue a citation, in combination, exert some pressure on the
detainee to respond to questions. But other aspects of the situation substantially offset these
forces. Perhaps most importantly, the typical traffic stop is public, at least to some degree. x x
x

In both of these respects, the usual traffic stop is more analogous to a so-called
Terry stop, see Terry v. Ohio, 392 U. S. 1 (1968), than to a formal arrest. x x x The
comparatively nonthreatening character of detentions of this sort explains the absence of any
suggestion in our opinions that Terry stops are subject to the dictates of Miranda. The
similarly noncoercive aspect of ordinary traffic stops prompts us to hold that persons
temporarily detained pursuant to such stops are not in custody for the purposes of Miranda.

xxxxxxxxx

We are confident that the state of affairs projected by respondent will not come to
pass. It is settled that the safeguards prescribed by Miranda become applicable as soon as a
suspects freedom of action is curtailed to a degree associated with formal arrest. California v.
Beheler, 463 U. S. 1121, 1125 (1983) (per curiam). If a motorist who has been detained
pursuant to a traffic stop thereafter is subjected to treatment that renders him in custody for
practical purposes, he will be entitled to the full panoply of protections prescribed by
Miranda. See Oregon v. Mathiason, 429 U. S. 492, 495 (1977) (per curiam). (Emphasis
supplied.)

The U.S. Court in Berkemer thus ruled that, since the motorist therein was only subjected to
modest questions while still at the scene of the traffic stop, he was not at that moment placed
under custody (such that he should have been apprised of his Miranda rights), and neither
can treatment of this sort be fairly characterized as the functional equivalent of a formal
arrest. Similarly, neither can petitioner here be considered under arrest at the time that his
traffic citation was being made.

It also appears that, according to City Ordinance No. 98-012, which was violated by
petitioner, the failure to wear a crash helmet while riding a motorcycle is penalized by a fine
only. Under the Rules of Court, a warrant of arrest need not be issued if the information or
charge was filed for an offense penalized by a fine only. It may be stated as a corollary that
neither can a warrantless arrest be made for such an offense.

This ruling does not imply that there can be no arrest for a traffic violation. Certainly, when
there is an intent on the part of the police officer to deprive the motorist of liberty, or to take
the latter into custody, the former may be deemed to have arrested the motorist. In this case,
however, the officers issuance (or intent to issue) a traffic citation ticket negates the
possibility of an arrest for the same violation.
Even if one were to work under the assumption that petitioner was deemed arrested
upon being flagged down for a traffic violation and while awaiting the issuance of his
ticket, then the requirements for a valid arrest were not complied with.

This Court has held that at the time a person is arrested, it shall be the duty of the
arresting officer to inform the latter of the reason for the arrest and must show that person the
warrant of arrest, if any. Persons shall be informed of their constitutional rights to remain
silent and to counsel, and that any statement they might make could be used against
them.[14] It may also be noted that in this case, these constitutional requirements were
complied with by the police officers only after petitioner had been arrested for illegal
possession of dangerous drugs.

In Berkemer, the U.S. Court also noted that the Miranda warnings must also be given
to a person apprehended due to a traffic violation:

The purposes of the safeguards prescribed by Miranda are to ensure that the police do
not coerce or trick captive suspects into confessing, to relieve the inherently compelling
pressures generated by the custodial setting itself, which work to undermine the individuals
will to resist, and as much as possible to free courts from the task of scrutinizing individual
cases to try to determine, after the fact, whether particular confessions were voluntary. Those
purposes are implicated as much by in-custody questioning of persons suspected of
misdemeanors as they are by questioning of persons suspected of felonies.

If it were true that petitioner was already deemed arrested when he was flagged down for a
traffic violation and while he waiting for his ticket, then there would have been no need for
him to be arrested for a second timeafter the police officers allegedly discovered the drugsas
he was already in their custody.

Second, there being no valid arrest, the warrantless search that resulted from it was
likewise illegal.

The following are the instances when a warrantless search is allowed: (i) a warrantless search
incidental to a lawful arrest; (ii) search of evidence in plain view; (iii) search of a moving
vehicle; (iv) consented warrantless search; (v) customs search; (vi) a stop and frisk search;
and (vii) exigent and emergency circumstances.[15] None of the above-mentioned instances,
especially a search incident to a lawful arrest, are applicable to this case.

It must be noted that the evidence seized, although alleged to be inadvertently discovered,
was not in plain view. It was actually concealed inside a metal container inside petitioners
pocket. Clearly, the evidence was not immediately apparent.[16]

Neither was there a consented warrantless search. Consent to a search is not to be lightly
inferred, but shown by clear and convincing evidence.[17] It must be voluntary in order to
validate an otherwise illegal search; that is, the consent must be unequivocal, specific,
intelligently given and uncontaminated by any duress or coercion. While the prosecution
claims that petitioner acceded to the instruction of PO3 Alteza, this alleged accession does
not suffice to prove valid and intelligent consent. In fact, the RTC found that petitioner was
merely told to take out the contents of his pocket.[18]

Whether consent to the search was in fact voluntary is a question of fact to be


determined from the totality of all the circumstances. Relevant to this determination are the
following characteristics of the person giving consent and the environment in which consent
is given: (1) the age of the defendant; (2) whether the defendant was in a public or a secluded
location; (3) whether the defendant objected to the search or passively looked on; (4) the
education and intelligence of the defendant; (5) the presence of coercive police procedures;
(6) the defendants belief that no incriminating evidence would be found; (7) the nature of the
police questioning; (8) the environment in which the questioning took place; and (9) the
possibly vulnerable subjective state of the person consenting. It is the State that has the
burden of proving, by clear and positive testimony, that the necessary consent was obtained,
and was freely and voluntarily given.[19] In this case, all that was alleged was that petitioner
was alone at the police station at three in the morning, accompanied by several police
officers. These circumstances weigh heavily against a finding of valid consent to a
warrantless search.

Neither does the search qualify under the stop and frisk rule. While the rule normally applies
when a police officer observes suspicious or unusual conduct, which may lead him to believe
that a criminal act may be afoot, the stop and frisk is merely a limited protective search of
outer clothing for weapons.[20]

In Knowles v. Iowa,[21] the U.S. Supreme Court held that when a police officer stops a person
for speeding and correspondingly issues a citation instead of arresting the latter, this
procedure does not authorize the officer to conduct a full search of the car. The Court therein
held that there was no justification for a full-blown search when the officer does not arrest
the motorist. Instead, police officers may only conduct minimal intrusions, such as ordering
the motorist to alight from the car or doing a patdown:

In Robinson, supra, we noted the two historical rationales for the search incident to
arrest exception: (1) the need to disarm the suspect in order to take him into custody, and (2)
the need to preserve evidence for later use at trial. x x x But neither of these underlying
rationales for the search incident to arrest exception is sufficient to justify the search in the
present case.

We have recognized that the first rationaleofficer safetyis both legitimate and weighty,
x x x The threat to officer safety from issuing a traffic citation, however, is a good deal less
than in the case of a custodial arrest. In Robinson, we stated that a custodial arrest involves
danger to an officer because of the extended exposure which follows the taking of a suspect
into custody and transporting him to the police station. 414 U. S., at 234-235. We recognized
that [t]he danger to the police officer flows from the fact of the arrest, and its attendant
proximity, stress, and uncertainty, and not from the grounds for arrest. Id., at 234, n. 5. A
routine traffic stop, on the other hand, is a relatively brief encounter and is more
analogous to a so-called Terry stop . . . than to a formal arrest. Berkemer v. McCarty, 468
U. S. 420, 439 (1984). See also Cupp v. Murphy, 412 U. S. 291, 296 (1973) (Where there is
no formal arrest . . . a person might well be less hostile to the police and less likely to take
conspicuous, immediate steps to destroy incriminating evidence).
This is not to say that the concern for officer safety is absent in the case of a
routine traffic stop. It plainly is not. See Mimms, supra, at 110; Wilson, supra, at
413-414. But while the concern for officer safety in this context may justify the minimal
additional intrusion of ordering a driver and passengers out of the car, it does not by
itself justify the often considerably greater intrusion attending a full fieldtype
search. Even without the search authority Iowa urges, officers have other, independent bases
to search for weapons and protect themselves from danger. For example, they may order out
of a vehicle both the driver, Mimms, supra, at 111, and any passengers, Wilson, supra, at 414;
perform a patdown of a driver and any passengers upon reasonable suspicion that they may
be armed and dangerous, Terry v. Ohio, 392 U. S. 1 (1968); conduct a Terry patdown of the
passenger compartment of a vehicle upon reasonable suspicion that an occupant is dangerous
and may gain immediate control of a weapon, Michigan v. Long, 463 U. S. 1032, 1049
(1983); and even conduct a full search of the passenger compartment, including any
containers therein, pursuant to a custodial arrest, New York v. Belton, 453 U. S. 454, 460
(1981).

Nor has Iowa shown the second justification for the authority to search incident to arrestthe
need to discover and preserve evidence. Once Knowles was stopped for speeding and issued
a citation, all the evidence necessary to prosecute that offense had been obtained. No further
evidence of excessive speed was going to be found either on the person of the offender or in
the passenger compartment of the car. (Emphasis supplied.)

The foregoing considered, petitioner must be acquitted. While he may have failed to object
to the illegality of his arrest at the earliest opportunity, a waiver of an illegal warrantless
arrest does not, however, mean a waiver of the inadmissibility of evidence seized during the
illegal warrantless arrest.[22]

The Constitution guarantees the right of the people to be secure in their persons,
houses, papers and effects against unreasonable searches and seizures.[23] Any evidence
obtained in violation of said right shall be inadmissible for any purpose in any proceeding.
While the power to search and seize may at times be necessary to the public welfare, still it
must be exercised and the law implemented without contravening the constitutional rights of
citizens, for the enforcement of no statute is of sufficient importance to justify indifference to
the basic principles of government.[24]

The subject items seized during the illegal arrest are inadmissible.[25] The drugs are the
very corpus delicti of the crime of illegal possession of dangerous drugs. Thus, their
inadmissibility precludes conviction and calls for the acquittal of the accused.[26]

WHEREFORE, the Petition is GRANTED. The 18 February 2011 Decision of the


Court of Appeals in CA-G.R. CR No. 32516 affirming the judgment of conviction dated 19
February 2009 of the Regional Trial Court, 5 th Judicial Region, Naga City, Branch 21, in
Criminal Case No. RTC 2003-0087, is hereby REVERSED and SET ASIDE. Petitioner
Rodel Luz y Ong is hereby ACQUITTED and ordered immediately released from detention,
unless his continued confinement is warranted by some other cause or ground.

SO ORDERED.
[G.R. No. 120431. April 1, 1998]

RODOLFO ESPANO, accused-petitioner, vs. COURT OF APPEALS and


PEOPLE OF THE PHILIPPINES, respondents.

DECISION
ROMERO, J.:

This is a petition for review of the decision of the Court of Appeals in CA-G.R. CR No.
13976 dated January 16, 1995, which affirmed in toto the judgment of the Regional
[1]

Trial Court of Manila, Branch 1, convicting petitioner Rodolfo Espano for violation of
Article II, Section 8 of Republic Act No. 6425, as amended, otherwise known as the
Dangerous Drugs Act.
Petitioner was charged under the following information:

That on or about July 14, 1991, in the City of Manila, Philippines, the said accused, not being
authorized by law to possess or use any prohibited drug, did then and there wilfully, unlawfully
and knowingly have in his possession and under his custody and control twelve (12) plastic
cellophane (bags) containing crushed flowering tops, marijuana weighing 5.5 grams which is a
prohibited drug.

Contrary to law.[2]

The evidence for the prosecution, based on the testimony of Pat. Romeo Pagilagan,
shows that on July 14, 1991, at about 12:30 a.m., he and other police officers, namely,
Pat. Wilfredo Aquino, Simplicio Rivera, and Erlindo Lumboy of the Western Police
District (WPD), Narcotics Division went to Zamora and Pandacan Streets, Manila to
confirm reports of drug pushing in the area. They saw petitioner selling something to
another person. After the alleged buyer left, they approached petitioner, identified
themselves as policemen, and frisked him. The search yielded two plastic cellophane
tea bags of marijuana.When asked if he had more marijuana, he replied that there was
more in his house. The policemen went to his residence where they found ten more
cellophane tea bags of marijuana. Petitioner was brought to the police headquarters
where he was charged with possession of prohibited drugs. On July 24, 1991, petitioner
posted bail and the trial court issued his order of release on July 29, 1991.
[3] [4]

Annabelle Alip, forensic chemist of the WPD Criminal Investigation Laboratory


Section, testified that the articles sent to her by Pat. Wilfredo Aquino regarding the
apprehension of a certain Rodolfo Espano for examination tested positive for marijuana,
with a total weight of 5.5 grams.
By way of defense, petitioner testified that on said evening, he was sleeping in his
house and was awakened only when the policemen handcuffed him. He alleged that the
policemen were looking for his brother-in-law Lauro, and when they could not find the
latter, he was instead brought to the police station for investigation and later indicted for
possession of prohibited drugs. His wife Myrna corroborated his story.
The trial court rejected petitioners defense as a mere afterthought and found the
version of the prosecution more credible and trustworthy.
Thus, on August 14, 1992, the trial court rendered a decision, convicting petitioner of
the crime charged, the dispositive portion of which reads:

WHEREFORE there being proof beyond reasonable doubt, the court finds the accused Rodolfo
Espano y Valeria guilty of the crime of violation of Section 8, Article II, in relation to Section 2
(e-L) (I) of Republic Act No. 6425 as amended by Batas Pambansa Blg. 179, and pursuant to
law hereby sentences him to suffer imprisonment of six (6) years and one (1) day to twelve (12)
years and to pay a fine of P6,000.00 with subsidiary imprisonment in case of default plus costs.

The marijuana is declared forfeited in favor of government and shall be turned over to the
Dangerous Drugs Board without delay.

SO ORDERED.[5]

Petitioner appealed the decision to the Court of Appeals. The appellate court,
however, affirmed the decision of the trial court in toto.
Hence, this petition.
Petitioner contends that the trial and appellate courts erred in convicting him on the
basis of the following: (a) the pieces of evidence seized were inadmissible; (b) the
superiority of his constitutional right to be presumed innocent over the doctrine of
presumption of regularity; (c) he was denied the constitutional right of confrontation and
to compulsory process; and (d) his conviction was based on evidence which was
irrelevant and not properly identified.
After a careful examination of the records of the case, this Court finds no compelling
reason sufficient to reverse the decisions of the trial and appellate courts.
First, it is a well settled doctrine that findings of trial courts on the credibility of
witnesses deserve a high degree of respect. Having observed the deportment of
witnesses during the trial, the trial judge is in a better position to determine the issue of
credibility and, thus, his findings will not be disturbed during appeal in the absence of
any clear showing that he had overlooked, misunderstood or misapplied some facts or
circumstances of weight and substance which could have altered the conviction of the
appellants. [6]

In this case, the findings of the trial court that the prosecution witnesses were more
credible than those of the defense must stand. Petitioner failed to show that Pat.
Pagilagan, in testifying against him, was motivated by reasons other than his duty to
curb drug abuse and had any intent to falsely impute to him such a serious crime as
possession of prohibited drugs. In the absence of such ill motive, the presumption of
regularity in the performance of his official duty must prevail.
In People v. Velasco, this Court reiterated the doctrine of presumption of regularity
[7]

in the performance of official duty which provides:


x x x. Appellant failed to establish that Pat. Godoy and the other members of the buy-bust team
are policemen engaged in mulcting or other unscrupulous activities who were motivated either
by the desire to extort money or exact personal vengeance, or by sheer whim and caprice, when
they entrapped her. And in the absence of proof of any intent on the part of the police authorities
to falsely impute such a serious crime against appellant, as in this case, the presumption of
regularity in the performance of official duty, . . ., must prevail over the self-serving and
uncorroborated claim of appellant that she had been framed. [8]

Furthermore, the defense set up by petitioner does not deserve any consideration.
He simply contended that he was in his house sleeping at the time of the incident. This
Court has consistently held that alibi is the weakest of all defenses; and for it to prosper,
the accused has the burden of proving that he was not at the scene of the crime at the
time of its commission and that it was physically impossible for him to be
there. Moreover, the claim of a frame-up, like alibi, is a defense that has been invariably
viewed by the Court with disfavor for it can just as easily be concocted but difficult to
prove, and is a common and standard line of defense in most prosecutions arising from
violations of the Dangerous Drugs Act. No clear and convincing evidence was
[9]

presented by petitioner to prove his defense of alibi.


Second, petitioner contends that the prosecutions failure to present the alleged
informant in court cast a reasonable doubt which warrants his acquittal. This is again
without merit, since failure of the prosecution to produce the informant in court is of no
moment especially when he is not even the best witness to establish the fact that a
buy-bust operation had indeed been conducted. In this case, Pat. Pagilagan, one of the
policemen who apprehended petitioner, testified on the actual incident of July 14, 1991,
and identified him as the one they caught in possession of prohibited drugs. Thus,

We find that the prosecution had satisfactorily proved its case against appellants. There is no compelling
reason for us to overturn the finding of the trial court that the testimony of Sgt. Gamboa, the lone witness for
the prosecution, was straightforward, spontaneous and convincing. The testimony of a sole witness, if
credible and positive and satisfies the court beyond reasonable doubt, is sufficient to convict.[10]

Thus on the basis of Pat. Pagilagans testimony, the prosecution was able to prove
that petitioner indeed committed the crime charged; consequently, the finding of
conviction was proper.
Lastly, the issue on the admissibility of the marijuana seized should likewise be ruled
upon. Rule 113 Section 5(a) of the Rules of Court provides:

A peace officer or a private person may, without a warrant, arrest a person:

a. when, in his presence, the person to be arrested has committed, is actually committing, or is
attempting to commit an offense;

x x x x x x x x x.

Petitioners arrest falls squarely under the aforecited rule. He was caught in
flagranti as a result of a buy-bust operation conducted by police officers on the basis of
information received regarding the illegal trade of drugs within the area of Zamora and
Pandacan Streets, Manila. The police officer saw petitioner handing over something to
an alleged buyer. After the buyer left, they searched him and discovered two
cellophanes of marijuana. His arrest was, therefore, lawful and the two cellophane bags
of marijuana seized were admissible in evidence, being the fruits of the crime.
As for the ten cellophane bags of marijuana found at petitioners residence, however,
the same are inadmissible in evidence.
The 1987 Constitution guarantees freedom against unreasonable searches and
seizures under Article III, Section 2 which provides:

The right of the people to be secure in their persons, houses, papers and effects against
unreasonable searches and seizures of whatever nature and for any purpose shall be inviolable,
and no search warrant or warrant of arrest shall issue except upon probable cause to be
determined personally by the judge after examination under oath or affirmation of the
complainant and the witnesses he may produce, and particularly describing the place to be
searched and the persons or things to be seized.

An exception to the said rule is a warrantless search incidental to a lawful arrest for
dangerous weapons or anything which may be used as proof of the commission of an
offense. It may extend beyond the person of the one arrested to include the premises
[11]

or surroundings under his immediate control. In this case, the ten cellophane bags of
marijuana seized at petitioners house after his arrest at Pandacan and Zamora Streets
do not fall under the said exceptions.
In the case of People v. Lua, this Court held:
[12]

As regards the brick of marijuana found inside the appellants house, the trial court correctly
ignored it apparently in view of its inadmissibility. While initially the arrest as well as the body
search was lawful, the warrantless search made inside the appellants house became unlawful
since the police operatives were not armed with a search warrant. Such search cannot fall under
search made incidental to a lawful arrest, the same being limited to body search and to that point
within reach or control of the person arrested, or that which may furnish him with the means of
committing violence or of escaping. In the case at bar, appellant was admittedly outside his
house when he was arrested.Hence, it can hardly be said that the inner portion of his house was
within his reach or control.

The articles seized from petitioner during his arrest were valid under the doctrine of
search made incidental to a lawful arrest. The warrantless search made in his house,
however, which yielded ten cellophane bags of marijuana became unlawful since the
police officers were not armed with a search warrant at the time. Moreover, it was
beyond the reach and control of petitioner.
In sum, this Court finds petitioner Rodolfo Espano guilty beyond reasonable doubt of
violating Article II, Section 8, in relation to Section 2 (e-L) (I) of Republic Act No. 6425,
as amended. Under the said provision, the penalty imposed is six years and one day to
twelve years and a fine ranging from six thousand to twelve thousand pesos. With the
passage of Republic Act No. 7659, which took effect on December 31, 1993, the
imposable penalty shall now depend on the quantity of drugs recovered. Under the
provisions of Republic Act No. 7629, Section 20, and as interpreted in People v.
Simon and People v. Lara, if the quantity of marijuana involved is less than 750
[13] [14]

grams, the imposable penalty ranges from prision correccional to reclusion


temporal. Taking into consideration that petitioner is not a habitual delinquent, the
amendatory provision is favorable to him and the quantity of marijuana involved is less
than 750 grams, the penalty imposed under Republic Act No. 7659 should be applied.
There being no mitigating nor aggravating circumstances, the imposable penalty shall
be prision correccional in its medium period. Applying the Indeterminate Sentence Law,
the maximum penalty shall be taken from the medium period of prision
correccional, which is two (2) years, four (4) months and one (1) day to four (4) years
and two (2) months, while the minimum shall be taken from the penalty next lower in
degree, which is one (1) month and one (1) day to six (6) months of arresto mayor.
WHEREFORE, the instant petition is hereby DENIED. The decision of the Court of
Appeals in C.A.-G.R. CR No. 13976 dated January 16, 1995 is AFFIRMED with the
MODIFICATION that petitioner Rodolfo Espano is sentenced to suffer an indeterminate
penalty of TWO (2) months and ONE (1) day of arresto mayor, as minimum to TWO (2)
years, FOUR (4) months and ONE (1) day of prision correccional, as maximum.
SO ORDERED.
[G.R. No. 163858. June 28, 2005]
UNITED LABORATORIES, INC., petitioner, vs. ERNESTO ISIP and/or
SHALIMAR PHILIPPINES and/or OCCUPANTS, Shalimar Building, No.
1571, Aragon Street, Sta. Cruz, Manila, respondents.

DECISION
CALLEJO, SR., J.:

Rolando H. Besarra, Special Investigator III of the National Bureau of Investigation


(NBI), filed an application, in the Regional Trial Court (RTC) of Manila, for the issuance
of a search warrant concerning the first and second floors of the Shalimar Building,
located at No. 1571, Aragon Street (formerly No. 1524, Lacson Avenue, Sta. Cruz,
Manila) occupied and/or used by Shalimar Philippines, owned/operated by Ernesto Isip;
and for the seizure of the following for violation of Section 4(a), in relation to Section 8,
of Republic Act (R.A.) No. 8203:
a. Finished or unfinished products of UNITED LABORATORIES (UNILAB), particularly REVICON
multivitamins;
b. Sundry items such as tags, labels, boxes, packages, wrappers, receptacles, advertisements and
other paraphernalia used in the offering for sale, sale and/or distribution of counterfeit REVICON
multivitamins;
c. Sales invoices, delivery receipts, official receipts, ledgers, journals, purchase orders and all other
books of accounts and documents used in recording the manufacture and/or importation,
distribution and/or sales of counterfeit REVICON multivitamins.[1]

The application was docketed as People v. Ernesto Isip, et al., Respondents, Search
Warrant Case No. 04-4916 and raffled to Branch 24 of the court. Appended thereto were
the following: (1) a sketch[2]showing the location of the building to be searched; (2) the
affidavit[3] of Charlie Rabe of the Armadillo Protection and Security Agency hired by
United Laboratories, Inc. (UNILAB), who allegedly saw the manufacture, production
and/or distribution of fake drug products such as Revicon by Shalimar Philippines; (3)
the letter-request of UNILAB, the duly licensed and exclusive manufacturer and/or
distributor of Revicon and Disudrin, for the monitoring of the unauthorized
production/manufacture of the said drugs and, if warranted, for their seizure; (4) the
letter-complaint[4] of UNILAB issued through its Director of the Security and Safety
Group; and (5) the joint affidavit[5] of NBI Agents Roberto Divinagracia and Rolando
Besarra containing the following allegations:
2. When learned that an Asset was already placed by ARMADILLO PROTECTIVE AND
SECURITY AGENCY named CHARLIE RABE, who was renting a room since November 2003,
at the said premises located at No. 1571 Aragon St., Sta. Cruz, Manila. MR. RABE averred that
the owner of the premises is a certain MR. ERNESTO ISIP and that the said premises which is
known as SHALIMAR PHILIPPINES, Shalimar Building, are being used to manufacture
counterfeit UNILAB products, particularly REVICON multivitamins, which was already patented
by UNILAB since 1985;
3. Upon verification of the report, we found out that the said premises is a six-story structure, with
an additional floor as a penthouse, and colored red-brown. It has a tight security arrangement
wherein non-residents are not allowed to enter or reconnoiter in the premises;
4. We also learned that its old address is No. 1524 Lacson Avenue, Sta. Cruz, Manila, and has a
new address as 1571 Aragon St., Sta. Cruz, Manila; and that the area of counterfeiting
operations are the first and second floors of Shalimar Building;
5. Since we cannot enter the premises, we instructed the Asset to take pictures of the area
especially the places wherein the clandestine manufacturing operations were being held. At a
peril to his well-being and security, the Asset was able to take photographs herein incorporated
into this Search Warrant Application.[6]

A representative from UNILAB, Michael Tome, testified during the hearing on the
application for the search warrant. After conducting the requisite searching questions,
the court granted the application and issued Search Warrant No. 04-4916 dated January
27, 2004, directing any police officer of the law to conduct a search of the first and
second floors of the Shalimar Building located at No. 1571, Aragon Street, Sta. Cruz,
Manila. The court also directed the police to seize the following items:
a. Finished or unfinished products of UNITED LABORATORIES (UNILAB), particularly REVICON
multivitamins;
b. Sundry items such as tags, labels, boxes, packages, wrappers, receptacles, advertisements and
other paraphernalia used in the offering for sale, sale and/or distribution of counterfeit REVICON
multivitamins;
c. Sales invoices, delivery receipts, official receipts, ledgers, journals, purchase orders and all other
books of accounts and documents used in recording the manufacture and/or importation,
distribution and/or sales of counterfeit REVICON multivitamins.[7]

The court also ordered the delivery of the seized items before it, together with a true
inventory thereof executed under oath.
The search warrant was implemented at 4:30 p.m. on January 27, 2004 by NBI
agents Besarra and Divinagracia, in coordination with UNILAB employees. No fake
Revicon multivitamins were found; instead, there were sealed boxes at the first and
second floors of the Shalimar Building which, when opened by the NBI agents in the
presence of respondent Isip, contained the following:
QUANTITY/UNIT DESCRIPTION

792 Bottles Disudrin 60 ml.


30 Boxes (100 pieces each) Inoflox 200 mg.[8]
NBI Special Investigator Divinagracia submitted an inventory of the things seized in
which he declared that the search of the first and second floors of the Shalimar Building
at No. 1571, Aragon Street, Sta. Cruz, Manila, the premises described in the warrant,
was done in an orderly and peaceful manner. He also filed a Return of Search
Warrant,[9] alleging that no other articles/items other than those mentioned in the warrant
and inventory sheet were seized. The agent prayed that of the items seized, ten boxes
of Disudrin 60 ml., and at least one box of Inoflox be turned over to the custody of the
Bureau of Food and Drugs (BFAD) for examination.[10] The court issued an order
granting the motion, on the condition that the turn over be made before the court, in the
presence of a representative from the respondents and the court.[11]
The respondents filed an Urgent Motion to Quash the Search Warrant or to
Suppress Evidence.[12] They contended that the implementing officers of the NBI
conducted their search at the first, second, third and fourth floors of the building at No.
1524-A, Lacson Avenue, Sta. Cruz, Manila, where items in open display were allegedly
found. They pointed out, however, that such premises was different from the address
described in the search warrant, the first and second floors of the Shalimar Building
located at No. 1571, Aragon Street, Sta. Cruz, Manila. The respondents, likewise,
asserted that the NBI officers seized Disudrin and Inoflox products which were not
included in the list of properties to be seized in the search warrant.
UNILAB, in collaboration with the NBI, opposed the motion, insisting that the search
was limited to the first and second floors of the Shalimar building located at the corner of
Aragon Street and Lacson Avenue, Sta. Cruz, Manila. They averred that, based on the
sketch appended to the search warrant application, Rabes affidavit, as well as the joint
affidavit of Besarra and Divinagracia, the building where the search was conducted was
located at No. 1571, Aragon Street corner Lacson Avenue, Sta. Cruz, Manila. They
pointed out that No. 1524 Lacson Avenue, Sta. Cruz, Manila was the old address, and
the new address was No. 1571, Aragon Street, Sta. Cruz, Manila. They maintained that
the warrant was not implemented in any other place.[13]
In reply, the respondents insisted that the items seized were different from those
listed in the search warrant. They also claimed that the seizure took place in the building
located at No. 1524-A which was not depicted in the sketch of the premises which the
applicant submitted to the trial court.[14] In accordance with the ruling of this Court
in People v. Court of Appeals,[15] the respondents served a copy of their pleading on
UNILAB.[16]
On March 11, 2004, the trial court issued an Order[17] granting the motion of the
respondents, on the ground that the things seized, namely, Disudrin and Inoflox, were
not those described in the search warrant. On March 16, 2004, the trial court issued an
advisory[18] that the seized articles could no longer be admitted in evidence against the
respondents in any proceedings, as the search warrant had already been quashed.
UNILAB, through the Ureta Law Office, filed a motion, in collaboration with the NBI
agents, for the reconsideration of the order, contending that the ground used by the
court in quashing the warrant was not that invoked by the respondents, and that the
seizure of the items was justified by the plain view doctrine. The respondents objected to
the appearance of the counsel of UNILAB, contending that the latter could not appear
for the People of the Philippines. The respondents moved that the motion for
reconsideration of UNILAB be stricken off the record. Disputing the claims of UNILAB,
they insisted that the items seized were contained in boxes at the time of the seizure at
No. 1524-A, Lacson Avenue corner Aragon Street, Sta. Cruz, Manila, and were not
apparently incriminating on plain view. Moreover, the seized items were not those
described and itemized in the search warrant application, as well as the warrant issued
by the court itself. The respondents emphasized that the Shalimar Laboratories is
authorized to manufacture galenical preparations of the following products:
Products:
- Povidone Iodine
- Chamomile Oil
- Salicylic Acid 10 g.
- Hydrogen Peroxide 3% Topical Solution
- Aceite de Alcamforado
- Aceite de Manzanilla[19]
In a manifestation and opposition, the respondents assailed the appearance of the
counsel of UNILAB, and insisted that it was not authorized to appear before the court
under the Rules of Court, and to file pleadings. They averred that the BFAD was the
authorized government agency to file an application for a search warrant.
In its counter-manifestation, UNILAB averred that it had the personality to file the
motion for reconsideration because it was the one which sought the filing of the
application for a search warrant; besides, it was not proscribed by Rule 126 of the
Revised Rules of Criminal Procedure from participating in the proceedings and filing
pleadings. The only parties to the case were the NBI and UNILAB and not the State or
public prosecutor. UNILAB also argued that the offended party, or the holder of a license
to operate, may intervene through counsel under Section 16 of Rule 110, in relation to
Section 7(e), of the Rules of Criminal Procedure.
UNILAB prayed that an ocular inspection be conducted of the place searched by the
NBI officers.[20] In their rejoinder, the respondents manifested that an ocular inspection
was the option to look forward to.[21] However, no such ocular inspection of the said
premises was conducted.
In the meantime, the BFAD submitted to the court the result of its examination of the
Disudrin and Inoflox samples which the NBI officers seized from the Shalimar Building.
On its examination of the actual component of Inoflox, the BFAD declared that the
substance failed the test.[22] The BFAD, likewise, declared that the examined Disudrin
syrup failed the test.[23] The BFAD had earlier issued the following report:
PRODUCT NAME Manufacturer L.N. E.D. FINDINGS
1.Phenylpropanolamine Unilab 21021552 3-06 -Registered, however,
(Disudrin) label/physical
12.5 mg./5mL Syrup appearance does not
conform with the
BFAD approved
label/ registered
specifications.
2.Ofloxacin (Inoflox) Unilab 99017407 3-05 -Registered, however,
200 mg. tablet. label/physical
appearance does not
conform with the
BFAD approved
label/ registered
specifications.[24]

On May 28, 2004, the trial court issued an Order[25] denying the motion for
reconsideration filed by UNILAB. The court declared that:

The Search Warrant is crystal clear: The seizing officers were only authorized to take possession
of finished or unfinished products of United Laboratories (UNILAB), particularly REVICON
Multivitamins, and documents evidencing the counterfeit nature of said products. The
Receipt/Inventory of Property Seized pursuant to the warrant does not, however, include
REVICON but other products. And whether or not these seized products are imitations of
UNILAB items is beside the point. No evidence was shown nor any was given during the
proceedings on the application for search warrant relative to the seized products.

On this score alone, the search suffered from a fatal infirmity and, hence, cannot be sustained. [26]

UNILAB, thus, filed the present petition for review on certiorari under Rule 45 of the
Rules of Court, where the following issues are raised:

Whether or not the seized 792 bottles of Disudrin 60 ml. and 30 boxes of Inoflox 200 mg. are
INADMISSIBLE as evidence against the respondents because they constitute the fruit of the
poisonous tree or, CONVERSELY, whether or not the seizure of the same counterfeit drugs is
justified and lawful under the plain view doctrine and, hence, the same are legally admissible as
evidence against the respondents in any and all actions?[27]

The petitioner avers that it was deprived of its right to a day in court when the trial
court quashed the search warrant for a ground which was not raised by the respondents
herein in their motion to quash the warrant. As such, it argues that the trial court ignored
the issue raised by the respondents. The petitioner insists that by so doing, the RTC
deprived it of its right to due process. The petitioner asserts that the description in the
search warrant of the products to be seized finished or unfinished products of UNILAB is
sufficient to include counterfeit drugs within the premises of the respondents not
covered by any license to operate from the BFAD, and/or not authorized or licensed to
manufacture, or repackage drugs produced or manufactured by UNILAB. Citing the
ruling of this Court in Padilla v. Court of Appeals,[28]the petitioner asserts that the
products seized were in plain view of the officers; hence, may be seized by them. The
petitioner posits that the respondents themselves admitted that the seized articles were
in open display; hence, the said articles were in plain view of the implementing officers.
In their comment on the petition, the respondents aver that the petition should have
been filed before the Court of Appeals (CA) because factual questions are raised. They
also assert that the petitioner has no locus standi to file the petition involving the validity
and the implementation of the search warrant. They argue that the petitioner merely
assisted the NBI, the BFAD and the Department of Justice; hence, it should have
impleaded the said government agencies as parties-petitioners. The petition should
have been filed by the Office of the Solicitor General (OSG) in behalf of the NBI and/or
the BFAD, because under the 1987 Revised Administrative Code, the OSG is mandated
to represent the government and its officers charged in their official capacity in cases
before the Supreme Court. The respondents further assert that the trial court may
consider issues not raised by the parties if such consideration would aid the court in the
just determination of the case.
The respondents, likewise, maintain that the raiding team slashed the sealed boxes
so fast even before respondent Isip could object. They argue that the seizure took place
at No. 1524-A, Lacson Avenue, Sta. Cruz, Manila covered by Transfer Certificate of Title
(TCT) No. 220778, and not at No. 1571, Aragon Street, Sta. Cruz, Manila covered by
TCT No. 174412 as stated in the search warrant. They assert that the ruling of the Court
in People v. Court of Appeals[29] is applicable in this case. They conclude that the
petitioner failed to prove the factual basis for the application of the plain view doctrine.[30]
In reply, the petitioner asserts that it has standing and is, in fact, the real
party-in-interest to defend the validity of the search warrant issued by the RTC; after all,
it was upon its instance that the application for a search warrant was filed by the NBI,
which the RTC granted. It asserts that it is not proscribed under R.A. No. 8203 from filing
a criminal complaint against the respondents and requesting the NBI to file an
application for a search warrant. The petitioner points out that the Rules of Criminal
Procedure does not specifically prohibit a private complainant from defending the
validity of a search warrant. Neither is the participation of a state prosecutor provided in
Rule 126 of the said Rules. After all, the petitioner insists, the proceedings for the
application and issuance of a search warrant is not a criminal action. The petitioner
asserts that the place sought to be searched was sufficiently described in the warrant for,
after all, there is only one building on the two parcels of land described in two titles
where Shalimar Philippines is located, the place searched by the NBI officers.[31] It also
asserts that the building is located at the corner of Aragon Street and Lacson Avenue,
Sta. Cruz, Manila.[32]
The petitioner avers that the plain view doctrine is applicable in this case because
the boxes were found outside the door of the respondents laboratory on the garage floor.
The boxes aroused the suspicion of the members of the raiding team precisely
because these were marked with the distinctive UNILAB logos. The boxes in which the
items were contained were themselves so designated to replicate true and original
UNILAB boxes for the same medicine. Thus, on the left hand corner of one side of some
of the boxes[33] the letters ABR under the words 60 ml, appeared to describe the
condition/quality of the bottles inside (as it is with genuine UNILAB box of the true
medicine of the same brand). The petitioner pointed out that ABR is the acronym for
amber bottle round describing the bottles in which the true and original Disudrin (for
children) is contained.
The petitioner points out that the same boxes also had their own license plates
which were instituted as among its internal control/countermeasures. The license plates
indicate that the items within are, supposedly, Disudrin. The NBI officers had reasonable
ground to believe that all the boxes have one and the same data appearing on their
supposedly distinctive license plates. The petitioner insists that although some of the
boxes marked with the distinctive UNILAB logo were, indeed, sealed, the tape or seal
was also a copy of the original because these, too, were marked with the distinctive
UNILAB logo. The petitioner appended to its pleading pictures of the Shalimar building
and the rooms searched showing respondent Isip;[34] the boxes seized by the police
officers containing Disudrin syrup;[35] and the boxes containing Inoflox and its
contents.[36]
The issues for resolution are the following: (1) whether the petitioner is the proper
party to file the petition at bench; (2) whether it was proper for the petitioner to file the
present petition in this Court under Rule 45 of the Rules of Court; and (3) whether the
search conducted by the NBI officers of the first and second floors of the Shalimar
building and the seizure of the sealed boxes which, when opened, contained Disudrin
syrup and Inoflox, were valid.
On the first issue, we agree with the petitioners contention that a search warrant
proceeding is, in no sense, a criminal action[37] or the commencement of a
prosecution.[38] The proceeding is not one against any person, but is solely for the
discovery and to get possession of personal property. It is a special and peculiar remedy,
drastic in nature, and made necessary because of public necessity. It resembles in
some respect with what is commonly known as John Doe proceedings.[39] While an
application for a search warrant is entitled like a criminal action, it does not make it such
an action.
A search warrant is a legal process which has been likened to a writ of discovery
employed by the State to procure relevant evidence of crime.[40] It is in the nature of a
criminal process, restricted to cases of public prosecutions.[41] A search warrant is a
police weapon, issued under the police power. A search warrant must issue in the name
of the State, namely, the People of the Philippines.[42]
A search warrant has no relation to a civil process. It is not a process for adjudicating
civil rights or maintaining mere private rights.[43] It concerns the public at large as
distinguished from the ordinary civil action involving the rights of private persons. [44] It
may only be applied for in the furtherance of public prosecution.[45]
However, a private individual or a private corporation complaining to the NBI or to a
government agency charged with the enforcement of special penal laws, such as the
BFAD, may appear, participate and file pleadings in the search warrant proceedings to
maintain, inter alia, the validity of the search warrant issued by the court and the
admissibility of the properties seized in anticipation of a criminal case to be filed; such
private party may do so in collaboration with the NBI or such government agency. The
party may file an opposition to a motion to quash the search warrant issued by the court,
or a motion for the reconsideration of the court order granting such motion to quash.[46]
In this case, UNILAB, in collaboration with the NBI, opposed the respondents motion
to quash the search warrant. The respondents served copies of their reply and
opposition/comment to UNILAB, through Modesto Alejandro, Jr.[47] The court a
quo allowed the appearance of UNILAB and accepted the pleadings filed by it and its
counsel.
The general rule is that the proper party to file a petition in the CA or Supreme Court
to assail any adverse order of the RTC in the search warrant proceedings is the People
of the Philippines, through the OSG. However, in Columbia Pictures Entertainment, Inc.
v. Court of Appeals,[48] the Court allowed a private corporation (the complainant in the
RTC) to file a petition for certiorari, and considered the petition as one filed by the OSG.
The Court in the said case even held that the petitioners therein could argue its case in
lieu of the OSG:

From the records, it is clear that, as complainants, petitioners were involved in the proceedings
which led to the issuance of Search Warrant No. 23. In People v. Nano, the Court declared that
while the general rule is that it is only the Solicitor General who is authorized to bring or defend
actions on behalf of the People or the Republic of the Philippines once the case is brought before
this Court or the Court of Appeals, if there appears to be grave error committed by the judge or a
lack of due process, the petition will be deemed filed by the private complainants therein as if it
were filed by the Solicitor General. In line with this ruling, the Court gives this petition due
course and will allow petitioners to argue their case against the questioned order in lieu of the
Solicitor General.[49]

The general rule is that a party is mandated to follow the hierarchy of courts.
However, in exceptional cases, the Court, for compelling reasons or if warranted by the
nature of the issues raised, may take cognizance of petitions filed directly before it. [50] In
this case, the Court has opted to take cognizance of the petition, considering the nature
of the issues raised by the parties.
The Court does not agree with the petitioners contention that the issue of whether
the Disudrin and Inoflox products were lawfully seized was never raised in the pleadings
of the respondents in the court a quo. Truly, the respondents failed to raise the issue in
their motion to quash the search warrant; in their reply, however, they averred that the
seized items were not included in the subject warrant and, therefore, were not lawfully
seized by the raiding team. They also averred that the said articles were not illegal per
se, like explosives and shabu, as to justify their seizure in the course of unlawful
search.[51] In their Opposition/Comment filed on March 15, 2004, the respondents even
alleged the following:

The jurisdiction of this Honorable Court is limited to the determination of whether there is a
legal basis to quash the search warrant and/or to suppress the seized articles in evidence. Since
the articles allegedly seized during the implementation of the search warrant Disudrin and
Inoflux products were not included in the search warrant, they were, therefore, not lawfully
seized by the raiding team; they are not illegal per se, as it were, like an arms cache, subversive
materials or shabu as to justify their seizure in the course of a lawful search, or being in plain
view or some such. No need whatever for some public assay.

The NBI manifestation is a glaring admission that it cannot tell without proper examination or
assay that the Disudrin and Inoflox samples allegedly seized from respondents place were
counterfeit. All the relevant presumptions are in favor of legality.[52]

The Court, therefore, finds no factual basis for the contention of the petitioner that
the respondents never raised in the court a quo the issue of whether the seizure of the
Disudrin and Inoflox products was valid.
In any event, the petitioner filed a motion for the reconsideration of the March 11,
2004 Order of the court a quo on the following claims:
2.01 The Honorable Court ERRED in ruling on a non-issue or the issue as to the alleged failure to
particularly describe in the search warrant the items to be seized but upon
which NO challenge was then existing and/or NO controversy is raised;
2.02 The Honorable Court ERRED in its ruling that finished or unfinished products of UNILAB
cannot stand the test of a particular description for which it then reasons that the search is,
supposedly unreasonable; and,
2.03 The Honorable Court ERRED in finding that the evidence seized is lawfully inadmissible
against respondents.[53]

The court a quo considered the motion of the petitioner and the issue raised by it
before finally resolving to deny the same. It cannot thus be gainsaid that the petitioner
was denied its right to due process.
On the validity of the seizure of the sealed boxes and its contents of Disudrin and
Inoflox, the Court, likewise, rejects the contention of the petitioner.
A search warrant, to be valid, must particularly describe the place to be searched
and the things to be seized. The officers of the law are to seize only those things
particularly described in the search warrant. A search warrant is not a sweeping
authority empowering a raiding party to undertake a fishing expedition to seize and
confiscate any and all kinds of evidence or articles relating to a crime. The search is
limited in scope so as not to be general or explanatory. Nothing is left to the discretion of
the officer executing the warrant.[54]
Objects, articles or papers not described in the warrant but on plain view of the
executing officer may be seized by him. However, the seizure by the officer of
objects/articles/papers not described in the warrant cannot be presumed as plain
view. The State must adduce evidence, testimonial or documentary, to prove the
confluence of the essential requirements for the doctrine to apply, namely: (a) the
executing law enforcement officer has a prior justification for an initial intrusion or
otherwise properly in a position from which he can view a particular order; (b) the officer
must discover incriminating evidence inadvertently; and (c) it must be immediately
apparent to the police that the items they observe may be evidence of a crime,
contraband, or otherwise subject to seizure.[55]
The doctrine is not an exception to the warrant. It merely serves to supplement the
prior justification whether it be a warrant for another object, hot pursuit, search as an
incident to a lawful arrest or some other legitimate reason for being present,
unconnected with a search directed against the accused. The doctrine may not be used
to extend a general exploratory search from one object to another until something
incriminating at last emerges. It is a recognition of the fact that when executing police
officers comes across immediately incriminating evidence not covered by the warrant,
they should not be required to close their eyes to it, regardless of whether it is evidence
of the crime they are investigating or evidence of some other crime. It would be
needless to require the police to obtain another warrant.[56]Under the doctrine, there is
no invasion of a legitimate expectation of privacy and there is no search within the
meaning of the Constitution.
The immediate requirement means that the executing officer can, at the time of
discovery of the object or the facts therein available to him, determine probable cause of
the objects incriminating evidence.[57] In other words, to be immediate, probable cause
must be the direct result of the officers instantaneous sensory perception of the
object.[58] The object is apparent if the executing officer had probable cause to connect
the object to criminal activity. The incriminating nature of the evidence becomes
apparent in the course of the search, without the benefit of any unlawful search or
seizure. It must be apparent at the moment of seizure.[59]
The requirement of inadvertence, on the other hand, means that the officer must not
have known in advance of the location of the evidence and intend to seize
it.[60] Discovery is not anticipated.[61]
The immediately apparent test does not require an unduly high degree of certainty
as to the incriminating character of evidence. It requires merely that the seizure be
presumptively reasonable assuming that there is probable cause to associate the
property with criminal activity; that a nexus exists between a viewed object and criminal
activity.[62]
Incriminating means the furnishing of evidence as proof of circumstances tending to
prove the guilt of a person.[63]
Indeed, probable cause is a flexible, common sense standard. It merely requires that
the facts available to the officer would warrant a man of reasonable caution and belief
that certain items may be contrabanded or stolen property or useful as evidence of a
crime. It does not require proof that such belief be correct or more likely than true. A
practical, non-traditional probability that incriminating evidence is involved is all that is
required. The evidence thus collected must be seen and verified as understood by those
experienced in the field of law enforcement.[64]
In this case, Disudrin and/or Inoflox were not listed in the search warrant issued by
the court a quo as among the properties to be seized by the NBI agents. The warrant
specifically authorized the officers only to seize counterfeit Revicon multivitamins,
finished or unfinished, and the documents used in recording, manufacture and/or
importation, distribution and/or sale, or the offering for sale, sale and/or distribution of
the said vitamins. The implementing officers failed to find any counterfeit Revicon
multivitamins, and instead seized sealed boxes which, when opened at the place where
they were found, turned out to contain Inoflox and Disudrin.
It was thus incumbent on the NBI agents and the petitioner to prove their claim that
the items were seized based on the plain view doctrine. It is not enough to prove that the
sealed boxes were in the plain view of the NBI agents; evidence should have been
adduced to prove the existence of all the essential requirements for the application of
the doctrine during the hearing of the respondents motion to quash, or at the very least,
during the hearing of the NBI and the petitioners motion for reconsideration on April 16,
2004. The immediately apparent aspect, after all, is central to the plain view exception
relied upon by the petitioner and the NBI. There is no showing that the NBI and the
petitioner even attempted to adduce such evidence. In fact, the petitioner and the NBI
failed to present any of the NBI agents who executed the warrant, or any of the
petitioners representative who was present at the time of the enforcement of the warrant
to prove that the enforcing officers discovered the sealed boxes inadvertently, and that
such boxes and their contents were incriminating and immediately apparent. It must be
stressed that only the NBI agent/agents who enforced the warrant had personal
knowledge whether the sealed boxes and their contents thereof were incriminating and
that they were immediately apparent.[65] There is even no showing that the NBI agents
knew the contents of the sealed boxes before they were opened.
In sum then, the Court finds and so hold that the petitioner and the NBI failed to
prove the essential requirements for the application of the plain view doctrine.
IN LIGHT OF ALL THE FOREGOING, the petition is DENIED for lack of merit. The
assailed orders of the Regional Trial Court are AFFIRMED.
SO ORDERED.

G.R. No. L-27360 February 28, 1968


HON. RICARDO G. PAPA, as Chief of Police of Manila; HON. JUAN PONCE ENRILE, as Commissioner of
Customs; PEDRO PACIS, as Collector of Customs of the Port of Manila; and MARTIN ALAGAO, as
Patrolman of the Manila Police Department, petitioners,
vs.
REMEDIOS MAGO and HILARION U. JARENCIO, as Presiding Judge of Branch 23, Court of First Instance
of Manila, respondents.

Office of the Solicitor General for petitioners.


Juan T. David for respondents.

ZALDIVAR, J.:

This is an original action for prohibition and certiorari, with preliminary injunction filed by Ricardo Papa,
Chief of Police of Manila; Juan once Enrile, Commissioner of Customs; Pedro Pacis, Collector of Customs of the
Port of Manila; and Martin Alagao, a patrolman of the Manila Police Department, against Remedios Mago and
Hon. Hilarion Jarencio, Presiding Judge of Branch 23 of the Court of First Instance of Manila, praying for the
annulment of the order issued by respondent Judge in Civil Case No. 67496 of the Court of First Instance of
Manila under date of March 7, 1967, which authorized the release under bond of certain goods which were seized
and held by petitioners in connection with the enforcement of the Tariff and Customs Code, but which were
claimed by respondent Remedios Mago, and to prohibit respondent Judge from further proceeding in any manner
whatsoever in said Civil Case No. 67496. Pending the determination of this case this Court issued a writ of
preliminary injunction restraining the respondent Judge from executing, enforcing and/or implementing the
questioned order in Civil Case No. 67496 and from proceeding with said case.

Petitioner Martin Alagao, head of the counter-intelligence unit of the Manila Police Department, acting
upon a reliable information received on November 3, 1966 to the effect that a certain shipment of personal effects,
allegedly misdeclared and undervalued, would be released the following day from the customs zone of the port of
Manila and loaded on two trucks, and upon orders of petitioner Ricardo Papa, Chief of Police of Manila and a duly
deputized agent of the Bureau of Customs, conducted surveillance at gate No. 1 of the customs zone. When the
trucks left gate No. 1 at about 4:30 in the afternoon of November 4, 1966, elements of the counter-intelligence unit
went after the trucks and intercepted them at the Agrifina Circle, Ermita, Manila. The load of the two trucks
consisting of nine bales of goods, and the two trucks, were seized on instructions of the Chief of Police. Upon
investigation, a person claimed ownership of the goods and showed to the policemen a "Statement and Receipts
of Duties Collected in Informal Entry No. 147-5501", issued by the Bureau of Customs in the name of a certain
Bienvenido Naguit.

Claiming to have been prejudiced by the seizure and detention of the two trucks and their cargo,
Remedios Mago and Valentin B. Lanopa filed with the Court of First Instance of Manila a petition
"for mandamus with restraining order or preliminary injunction, docketed as Civil Case No. 67496, alleging,
among others, that Remedios Mago was the owner of the goods seized, having purchased them from the Sta.
Monica Grocery in San Fernando, Pampanga; that she hired the trucks owned by Valentin Lanopa to transport,
the goods from said place to her residence at 1657 Laon Laan St., Sampaloc, Manila; that the goods were seized
by members of the Manila Police Department without search warrant issued by a competent court; that anila
Chief of Police Ricardo Papa denied the request of counsel for Remedios Mago that the bales be not opened and
the goods contained therein be not examined; that then Customs Commissioner Jacinto Gavino had illegally
assigned appraisers to examine the goods because the goods were no longer under the control and supervision
of the Commissioner of Customs; that the goods, even assuming them to have been misdeclared and,
undervalued, were not subject to seizure under Section 2531 of the Tariff and Customs Code because Remedios
Mago had bought them from another person without knowledge that they were imported illegally; that the bales
had not yet been opened, although Chief of Police Papa had arranged with the Commissioner of Customs
regarding the disposition of the goods, and that unless restrained their constitutional rights would be violated and
they would truly suffer irreparable injury. Hence, Remedios Mago and Valentin Lanopa prayed for the issuance of
a restraining order, ex parte, enjoining the above-named police and customs authorities, or their agents, from
opening the bales and examining the goods, and a writ of mandamus for the return of the goods and the trucks,
as well as a judgment for actual, moral and exemplary damages in their favor.

On November 10, 1966, respondent Judge Hilarion Jarencio issued an order ex parte restraining the
respondents in Civil Case No. 67496 — now petitioners in the instant case before this Court — from opening the
nine bales in question, and at the same time set the hearing of the petition for preliminary injunction on November
16, 1966. However, when the restraining order was received by herein petitioners, some bales had already been
opened by the examiners of the Bureau of Customs in the presence of officials of the Manila Police Department,
an assistant city fiscal and a representative of herein respondent Remedios Mago.

Under date of November 15, 1966, Remedios Mago filed an amended petition in Civil Case No. 67496,
including as party defendants Collector of Customs Pedro Pacis of the Port of Manila and Lt. Martin Alagao of the
Manila Police Department. Herein petitioners (defendants below) filed, on November 24, 1966, their "Answer with
Opposition to the Issuance of a Writ of Preliminary Injunction", denying the alleged illegality of the seizure and
detention of the goods and the trucks and of their other actuations, and alleging special and affirmative defenses,
to wit: that the Court of First Instance of Manila had no jurisdiction to try the case; that the case fell within the
exclusive jurisdiction of the Court of Tax Appeals; that, assuming that the court had jurisdiction over the case, the
petition stated no cause of action in view of the failure of Remedios Mago to exhaust the administrative remedies
provided for in the Tariff and Customs Code; that the Bureau of Customs had not lost jurisdiction over the goods
because the full duties and charges thereon had not been paid; that the members of the Manila Police
Department had the power to make the seizure; that the seizure was not unreasonable; and the persons
deputized under Section 2203 (c) of the Tariff and Customs Code could effect search, seizures and arrests in
inland places in connection with the enforcement of the said Code. In opposing the issuance of the writ of
preliminary injunction, herein petitioners averred in the court below that the writ could not be granted for the
reason that Remedios Mago was not entitled to the main reliefs she prayed for; that the release of the goods,
which were subject to seizure proceedings under the Tariff and Customs Code, would deprive the Bureau of
Customs of the authority to forfeit them; and that Remedios Mago and Valentin Lanopa would not suffer
irreparable injury. Herein petitioners prayed the court below for the lifting of the restraining order, for the denial of
the issuance of the writ of preliminary injunction, and for the dismissal of the case.

At the hearing on December 9, 1966, the lower Court, with the conformity of the parties, ordered that an
inventory of the goods be made by its clerk of court in the presence of the representatives of the claimant of the
goods, the Bureau of Customs, and the Anti-Smuggling Center of the Manila Police Department. On December
13, 1966, the above-named persons filed a "Compliance" itemizing the contents of the nine bales.

Herein respondent Remedios Mago, on December 23, 1966, filed an ex parte motion to release the
goods, alleging that since the inventory of the goods seized did not show any article of prohibited importation, the
same should be released as per agreement of the patties upon her posting of the appropriate bond that may be
determined by the court. Herein petitioners filed their opposition to the motion, alleging that the court had no
jurisdiction to order the release of the goods in view of the fact that the court had no jurisdiction over the case, and
that most of the goods, as shown in the inventory, were not declared and were, therefore, subject to forfeiture. A
supplemental opposition was filed by herein petitioners on January 19, 1967, alleging that on January 12, 1967
seizure proceedings against the goods had been instituted by the Collector of Customs of the Port of Manila, and
the determination of all questions affecting the disposal of property proceeded against in seizure and forfeiture
proceedings should thereby be left to the Collector of Customs. On January 30, 1967, herein petitioners filed a
manifestation that the estimated duties, taxes and other charges due on the goods amounted to P95,772.00. On
February 2, 1967, herein respondent Remedios Mago filed an urgent manifestation and reiteration of the motion
for the release under bond of the goods.

On March 7, 1967, the respondent Judge issued an order releasing the goods to herein respondent
Remedios Mago upon her filing of a bond in the amount of P40,000.00, and on March 13, 1967, said respondent
filed the corresponding bond.

On March 13, 1967, herein petitioner Ricardo Papa, on his own behalf, filed a motion for reconsideration
of the order of the court releasing the goods under bond, upon the ground that the Manila Police Department had
been directed by the Collector of Customs of the Port of Manila to hold the goods pending termination of the
seizure proceedings.

Without waiting for the court's action on the motion for reconsideration, and alleging that they had no
plain, speedy and adequate remedy in the ordinary course of law, herein petitioners filed the present action for
prohibition and certiorari with preliminary injunction before this Court. In their petition petitioners alleged, among
others, that the respondent Judge acted without jurisdiction in ordering the release to respondent Remedios
Mago of the disputed goods, for the following reasons: (1) the Court of First Instance of Manila, presided by
respondent Judge, had no jurisdiction over the case; (2) respondent Remedios Mago had no cause of action in
Civil Case No. 67496 of the Court of First Instance of Manila due to her failure to exhaust all administrative
remedies before invoking judicial intervention; (3) the Government was not estopped by the negligent and/or
illegal acts of its agent in not collecting the correct taxes; and (4) the bond fixed by respondent Judge for the
release of the goods was grossly insufficient.

In due time, the respondents filed their answer to the petition for prohibition and certiorari in this case. In
their answer, respondents alleged, among others: (1) that it was within the jurisdiction of the lower court presided
by respondent Judge to hear and decide Civil Case No. 67496 and to issue the questioned order of March 7,
1967, because said Civil Case No. 67496 was instituted long before seizure, and identification proceedings
against the nine bales of goods in question were instituted by the Collector of Customs; (2) that petitioners could
no longer go after the goods in question after the corresponding duties and taxes had been paid and said goods
had left the customs premises and were no longer within the control of the Bureau of Customs; (3) that
respondent Remedios Mago was purchaser in good faith of the goods in question so that those goods can not be
the subject of seizure and forfeiture proceedings; (4) that the seizure of the goods was affected by members of
the Manila Police Department at a place outside control of jurisdiction of the Bureau of Customs and affected
without any search warrant or a warrant of seizure and detention; (5) that the warrant of seizure and detention
subsequently issued by the Collector of Customs is illegal and unconstitutional, it not being issued by a judge; (6)
that the seizing officers have no authority to seize the goods in question because they are not articles of
prohibited importation; (7) that petitioners are estopped to institute the present action because they had agreed
before the respondent Judge that they would not interpose any objection to the release of the goods under bond
to answer for whatever duties and taxes the said goods may still be liable; and (8) that the bond for the release of
the goods was sufficient.

The principal issue in the instant case is whether or not, the respondent Judge had acted with jurisdiction
in issuing the order of March 7, 1967 releasing the goods in question.

The Bureau of Customs has the duties, powers and jurisdiction, among others, (1) to assess and collect
all lawful revenues from imported articles, and all other dues, fees, charges, fines and penalties, accruing under
the tariff and customs laws; (2) to prevent and suppress smuggling and other frauds upon the customs; and (3) to
enforce tariff and customs laws. 1 The goods in question were imported from Hongkong, as shown in the
"Statement and Receipts of Duties Collected on Informal Entry". 2 As long as the importation has not been
terminated the imported goods remain under the jurisdiction of the Bureau of customs. Importation is deemed
terminated only upon the payment of the duties, taxes and other charges upon the articles, or secured to be paid,
at the port of entry and the legal permit for withdrawal shall have been granted. 3 The payment of the duties, taxes,
fees and other charges must be in full. 4

The record shows, by comparing the articles and duties stated in the aforesaid "Statement and Receipts
of Duties Collected on Informal Entry" with the manifestation of the Office of the Solicitor General 5 wherein it is
stated that the estimated duties, taxes and other charges on the goods subject of this case amounted to
P95,772.00 as evidenced by the report of the appraiser of the Bureau of Customs, that the duties, taxes and other
charges had not been paid in full. Furthermore, a comparison of the goods on which duties had been assessed,
as shown in the "Statement and Receipts of Duties Collected on Informal Entry" and the "compliance" itemizing
the articles found in the bales upon examination and inventory, 6 shows that the quantity of the goods was
underdeclared, presumably to avoid the payment of duties thereon. For example, Annex B (the statement and
receipts of duties collected) states that there were 40 pieces of ladies' sweaters, whereas Annex H (the inventory
contained in the "compliance") states that in bale No. 1 alone there were 42 dozens and 1 piece of ladies'
sweaters of assorted colors; in Annex B, only 100 pieces of watch bands were assessed, but in Annex H, there
were in bale No. 2, 209 dozens and 5 pieces of men's metal watch bands (white) and 120 dozens of men's metal
watch band (gold color), and in bale No. 7, 320 dozens of men's metal watch bands (gold color); in Annex B, 20
dozens only of men's handkerchief were declared, but in Annex H it appears that there were 224 dozens of said
goods in bale No. 2, 120 dozens in bale No. 6, 380 dozens in bale No. 7, 220 dozens in bale No. 8, and another
200 dozens in bale No. 9. The articles contained in the nine bales in question, were, therefore, subject to
forfeiture under Section 2530, pars. e and m, (1), (3), (4), and (5) of the Tariff and Customs Code. And this Court
has held that merchandise, the importation of which is effected contrary to law, is subject to forfeiture, 7 and that
goods released contrary to law are subject to seizure and forfeiture. 8
Even if it be granted, arguendo, that after the goods in question had been brought out of the customs
area the Bureau of Customs had lost jurisdiction over the same, nevertheless, when said goods were intercepted
at the Agrifina Circle on November 4, 1966 by members of the Manila Police Department, acting under directions
and orders of their Chief, Ricardo C. Papa, who had been formally deputized by the Commissioner of
Customs, 9 the Bureau of Customs had regained jurisdiction and custody of the goods. Section 1206 of the Tariff
and Customs Code imposes upon the Collector of Customs the duty to hold possession of all imported articles
upon which duties, taxes, and other charges have not been paid or secured to be paid, and to dispose of the
same according to law. The goods in question, therefore, were under the custody and at the disposal of the
Bureau of Customs at the time the petition for mandamus, docketed as Civil Case No. 67496, was filed in the
Court of First Instance of Manila on November 9, 1966. The Court of First Instance of Manila, therefore, could not
exercise jurisdiction over said goods even if the warrant of seizure and detention of the goods for the purposes of
the seizure and forfeiture proceedings had not yet been issued by the Collector of Customs.

The ruling in the case of "Alberto de Joya, et al. v. Hon. Gregorio Lantin, et al.," G.R. No. L-24037,
decided by this Court on April 27, 1967, is squarely applicable to the instant case. In the De Joya case, it appears
that Francindy Commercial of Manila bought from Ernerose Commercial of Cebu City 90 bales of assorted
textiles and rags, valued at P117,731.00, which had been imported and entered thru the port of Cebu. Ernerose
Commercial shipped the goods to Manila on board an inter-island vessel. When the goods where about to leave
the customs premises in Manila, on October 6, 1964, the customs authorities held them for further verification,
and upon examination the goods were found to be different from the declaration in the cargo manifest of the
carrying vessel. Francindy Commercial subsequently demanded from the customs authorities the release of the
goods, asserting that it is a purchaser in good faith of those goods; that a local purchaser was involved so the
Bureau of Customs had no right to examine the goods; and that the goods came from a coastwise port. On
October 26, 1964, Francindy Commercial filed in the Court of First Instance of Manila a petition
for mandamus against the Commissioner of Customs and the Collector of Customs of the port of Manila to
compel said customs authorities to release the goods.

Francindy Commercial alleged in its petition for mandamus that the Bureau of Customs had no
jurisdiction over the goods because the same were not imported to the port of Manila; that it was not liable for
duties and taxes because the transaction was not an original importation; that the goods were not in the hands of
the importer nor subject to importer's control, nor were the goods imported contrary to law with its (Francindy
Commercial's) knowledge; and that the importation had been terminated. On November 12, 1964, the Collector of
Customs of Manila issued a warrant of seizure and identification against the goods. On December 3, 1964, the
Commissioner of Customs and the Collector of Customs, as respondents in the mandamus case, filed a motion to
dismiss the petition on the grounds of lack of jurisdiction, lack of cause of action, and in view of the pending
seizure and forfeiture proceedings. The Court of First Instance held resolution on the motion to dismiss in
abeyance pending decision on the merits. On December 14, 1964, the Court of First Instance of Manila issued a
preventive and mandatory injunction, on prayer by Francindy Commercial, upon a bond of P20,000.00. The
Commissioner of Customs and the Collector of Customs sought the lifting of the preliminary and mandatory
injunction, and the resolution of their motion to dismiss. The Court of First Instance of Manila, however, on
January 12, 1965, ordered them to comply with the preliminary and mandatory injunction, upon the filing by
Francindy Commercial of an additional bond of P50,000.00. Said customs authorities thereupon filed with this
Court, on January 14, 1965, a petition for certiorari and prohibition with preliminary injunction. In resolving the
question raised in that case, this Court held:

This petition raises two related issues: first, has the Customs bureau jurisdiction to seize the goods and
institute forfeiture proceedings against them? and (2) has the Court of First Instance jurisdiction to entertain the
petition for mandamus to compel the Customs authorities to release the goods?

Francindy Commercial contends that since the petition in the Court of first Instance was filed (on October
26, 1964) ahead of the issuance of the Customs warrant of seizure and forfeiture (on November 12, 1964),the
Customs bureau should yield the jurisdiction of the said court.

The record shows, however, that the goods in question were actually seized on October 6, 1964, i.e.,
before Francindy Commercial sued in court. The purpose of the seizure by the Customs bureau was to verify
whether or not Custom duties and taxes were paid for their importation. Hence, on December 23, 1964, Customs
released 22 bales thereof, for the same were found to have been released regularly from the Cebu Port (Petition
Annex "L"). As to goods imported illegally or released irregularly from Customs custody, these are subject to
seizure under Section 2530 m. of the Tariff and Customs Code (RA 1957).

The Bureau of Customs has jurisdiction and power, among others to collect revenues from imported
articles, fines and penalties and suppress smuggling and other frauds on customs; and to enforce tariff and
customs laws (Sec. 602, Republic Act 1957).

The goods in question are imported articles entered at the Port of Cebu. Should they be found to have
been released irregularly from Customs custody in Cebu City, they are subject to seizure and forfeiture, the
proceedings for which comes within the jurisdiction of the Bureau of Customs pursuant to Republic Act 1937.

Said proceeding should be followed; the owner of the goods may set up defenses therein (Pacis v.
Averia, L-22526, Nov. 20, 1966.) From the decision of the Commissioner of Customs appeal lies to the Court of
Tax Appeals, as provided in Sec. 2402 of Republic Act 1937 and Sec. 11 of Republic Act, 1125. To permit
recourse to the Court of First Instance in cases of seizure of imported goods would in effect render ineffective the
power of the Customs authorities under the Tariff and Customs Code and deprive the Court of Tax Appeals of
one of its exclusive appellate jurisdictions. As this Court has ruled in Pacis v. Averia, supra, Republic Acts 1937
and 1125 vest jurisdiction over seizure and forfeiture proceedings exclusively upon the Bureau of Customs and
the Court of Tax Appeals. Such law being special in nature, while the Judiciary Act defining the jurisdiction of
Courts of First Instance is a general legislation, not to mention that the former are later enactments, the Court of
First Instance should yield to the jurisdiction of the Customs authorities.

It is the settled rule, therefore, that the Bureau of Customs acquires exclusive jurisdiction over imported
goods, for the purposes of enforcement of the customs laws, from the moment the goods are actually in its
possession or control, even if no warrant of seizure or detention had previously been issued by the Collector of
Customs in connection with seizure and forfeiture proceedings. In the present case, the Bureau of Customs
actually seized the goods in question on November 4, 1966, and so from that date the Bureau of Customs
acquired jurisdiction over the goods for the purposes of the enforcement of the tariff and customs laws, to the
exclusion of the regular courts. Much less then would the Court of First Instance of Manila have jurisdiction over
the goods in question after the Collector of Customs had issued the warrant of seizure and detention on January
12, 1967. 10And so, it cannot be said, as respondents contend, that the issuance of said warrant was only an
attempt to divest the respondent Judge of jurisdiction over the subject matter of the case. The court presided by
respondent Judge did not acquire jurisdiction over the goods in question when the petition for mandamus was
filed before it, and so there was no need of divesting it of jurisdiction. Not having acquired jurisdiction over the
goods, it follows that the Court of First Instance of Manila had no jurisdiction to issue the questioned order of
March 7, 1967 releasing said goods.

Respondents also aver that petitioner Martin Alagao, an officer of the Manila Police Department, could
not seize the goods in question without a search warrant. This contention cannot be sustained. The Chief of the
Manila Police Department, Ricardo G. Papa, having been deputized in writing by the Commissioner of Customs,
could, for the purposes of the enforcement of the customs and tariff laws, effect searches, seizures, and
arrests, 11 and it was his duty to make seizure, among others, of any cargo, articles or other movable property
when the same may be subject to forfeiture or liable for any fine imposed under customs and tariff laws. 12 He
could lawfully open and examine any box, trunk, envelope or other container wherever found when he had
reasonable cause to suspect the presence therein of dutiable articles introduced into the Philippines contrary to
law; and likewise to stop, search and examine any vehicle, beast or person reasonably suspected of holding or
conveying such article as aforesaid. 13 It cannot be doubted, therefore, that petitioner Ricardo G. Papa, Chief of
Police of Manila, could lawfully effect the search and seizure of the goods in question. The Tariff and Customs
Code authorizes him to demand assistance of any police officer to effect said search and seizure, and the latter
has the legal duty to render said assistance. 14This was what happened precisely in the case of Lt. Martin Alagao
who, with his unit, made the search and seizure of the two trucks loaded with the nine bales of goods in question
at the Agrifina Circle. He was given authority by the Chief of Police to make the interception of the cargo. 15

Petitioner Martin Alagao and his companion policemen had authority to effect the seizure without any
search warrant issued by a competent court. The Tariff and Customs Code does not require said warrant in the
instant case. The Code authorizes persons having police authority under Section 2203 of the Tariff and Customs
Code to enter, pass through or search any land, inclosure, warehouse, store or building, not being a dwelling
house; and also to inspect, search and examine any vessel or aircraft and any trunk, package, or envelope or any
person on board, or to stop and search and examine any vehicle, beast or person suspected of holding or
conveying any dutiable or prohibited article introduced into the Philippines contrary to law, without mentioning the
need of a search warrant in said cases. 16 But in the search of a dwelling house, the Code provides that said
"dwelling house may be entered and searched only upon warrant issued by a judge or justice of the
peace. . . ." 17 It is our considered view, therefor, that except in the case of the search of a dwelling house, persons
exercising police authority under the customs law may effect search and seizure without a search warrant in the
enforcement of customs laws.

Our conclusion finds support in the case of Carroll v. United States, 39 A.L.R., 790, 799, wherein the
court, considering a legal provision similar to Section 2211 of the Philippine Tariff and Customs Code, said as
follows:

Thus contemporaneously with the adoption of the 4th Amendment, we find in the first Congress, and in
the following second and fourth Congresses, a difference made as to the necessity for a search warrant between
goods subject to forfeiture, when concealed in a dwelling house of similar place, and like goods in course of
transportation and concealed in a movable vessel, where readily they could be put out of reach of a search
warrant. . . .

Again, by the 2d section of the Act of March 3, 1815 (3 Stat. at L.231, 232, chap. 94), it was made lawful
for customs officers not only to board and search vessels within their own and adjoining districts, but also to stop,
search and examine any vehicle, beast or person on which or whom they should suspect there was merchandise
which was subject to duty, or had been introduced into the United States in any manner contrary to law, whether
by the person in charge of the vehicle or beast or otherwise, and if they should find any goods, wares, or
merchandise thereon, which they had probably cause to believe had been so unlawfully brought into the country,
to seize and secure the same, and the vehicle or beast as well, for trial and forfeiture. This Act was renewed April
27, 1816 (3 Sta. at L. 315, chap. 100), for a year and expired. The Act of February 28, 1865, revived § 2 of the Act
of 1815, above described, chap. 67, 13 Stat. at L. 441. The substance of this section was re-enacted in the 3d
section of the Act of July 18, 1866, chap. 201, 14 Stat. at L. 178, and was thereafter embodied in the Revised
Statutes as § 3061, Comp. Stat. § 5763, 2 Fed. Stat. Anno. 2d ed. p. 1161. Neither § 3061 nor any of its earlier
counterparts has ever been attacked as unconstitutional. Indeed, that section was referred to and treated as
operative by this court in Von Cotzhausen v. Nazro, 107 U.S. 215, 219, 27 L. ed. 540, 541, 2 Sup. Ct. Rep.
503. . . .

In the instant case, we note that petitioner Martin Alagao and his companion policemen did not have to
make any search before they seized the two trucks and their cargo. In their original petition, and amended petition,
in the court below Remedios Mago and Valentin Lanopa did not even allege that there was a search. 18 All that
they complained of was,

That while the trucks were on their way, they were intercepted without any search warrant near the
Agrifina Circle and taken to the Manila Police Department, where they were detained.

But even if there was a search, there is still authority to the effect that no search warrant would be
needed under the circumstances obtaining in the instant case. Thus, it has been held that:

The guaranty of freedom from unreasonable searches and seizures is construed as recognizing a
necessary difference between a search of a dwelling house or other structure in respect of which a search
warrant may readily be obtained and a search of a ship, motorboat, wagon, or automobile for contraband goods,
where it is not practicable to secure a warrant because the vehicle can be quickly moved out of the locality or
jurisdiction in which the warrant must be sought. (47 Am. Jur., pp. 513-514, citing Carroll v. United States, 267
U.S. 132, 69 L. ed., 543, 45 S. Ct., 280, 39 A.L.R., 790; People v. Case, 320 Mich., 379, 190 N.W., 389, 27 A.L.R.,
686.)

In the case of People v. Case (320 Mich., 379, 190 N.W., 389, 27 A.L.R., 686), the question raised by
defendant's counsel was whether an automobile truck or an automobile could be searched without search
warrant or other process and the goods therein seized used afterwards as evidence in a trial for violation of the
prohibition laws of the State. Same counsel contended the negative, urging the constitutional provision forbidding
unreasonable searches and seizures. The Court said:
. . . Neither our state nor the Federal Constitution directly prohibits search and seizure without a warrant,
as is sometimes asserted. Only "unreasonable" search and seizure is forbidden. . . .

. . . The question whether a seizure or a search is unreasonable in the language of the Constitution is a
judicial and not a legislative question; but in determining whether a seizure is or is not unreasonable, all of the
circumstances under which it is made must be looked to.

The automobile is a swift and powerful vehicle of recent development, which has multiplied by quantity
production and taken possession of our highways in battalions until the slower, animal-drawn vehicles, with their
easily noted individuality, are rare. Constructed as covered vehicles to standard form in immense quantities, and
with a capacity for speed rivaling express trains, they furnish for successful commission of crime a disguising
means of silent approach and swift escape unknown in the history of the world before their advent. The question
of their police control and reasonable search on highways or other public places is a serious question far deeper
and broader than their use in so-called "bootleging" or "rum running," which is itself is no small matter. While a
possession in the sense of private ownership, they are but a vehicle constructed for travel and transportation on
highways. Their active use is not in homes or on private premises, the privacy of which the law especially guards
from search and seizure without process. The baffling extent to which they are successfully utilized to facilitate
commission of crime of all degrees, from those against morality, chastity, and decency, to robbery, rape, burglary,
and murder, is a matter of common knowledge. Upon that problem a condition, and not a theory, confronts proper
administration of our criminal laws. Whether search of and seizure from an automobile upon a highway or other
public place without a search warrant is unreasonable is in its final analysis to be determined as a judicial
question in view of all the circumstances under which it is made.

Having declared that the seizure by the members of the Manila Police Department of the goods in
question was in accordance with law and by that seizure the Bureau of Customs had acquired jurisdiction over
the goods for the purpose of the enforcement of the customs and tariff laws, to the exclusion of the Court of First
Instance of Manila, We have thus resolved the principal and decisive issue in the present case. We do not
consider it necessary, for the purposes of this decision, to discuss the incidental issues raised by the parties in
their pleadings.

WHEREFORE, judgment is hereby rendered, as follows:

(a) Granting the writ of certiorari and prohibition prayed for by petitioners;

(b) Declaring null and void, for having been issued without jurisdiction, the order of respondent Judge
Hilarion U. Jarencio, dated March 7, 1967, in Civil Code No. 67496 of the Court of First Instance of Manila;

(c) Declaring permanent the preliminary injunction issued by this Court on March 31, 1967 restraining
respondent Judge from executing, enforcing and/or implementing his order of March 7, 1967 in Civil Case No.
67496 of the Court of First Instance of Manila, and from proceeding in any manner in said case;

(d) Ordering the dismissal of Civil Case No. 67496 of the Court of First Instance of Manila; and 1äwphï1.ñ ët

(e) Ordering the private respondent, Remedios Mago, to pay the costs.

It is so ordered.
G.R. No. 96177 January 27, 1993

PEOPLE OF THE PHILIPPINES, plaintiff-appellee,


vs.
MARI MUSA y HANTATALU, accused-appellant.

The Solicitor General for plaintiff-appellee.

Pablo L. Murillo for accused-appellant.

ROMERO, J.:

The appellant, Mari Musa, seeks, in this appeal, the reversal of the decision, dated August 31, 1990, of the
1

Regional Trial Court (RTC) of Zamboanga City, Branch XII, finding him guilty of selling marijuana in violation of
Article II, Section 4 of Republic Act No. 6425, as amended, otherwise known as the Dangerous Drugs Act of
1972.

The information filed on December 15, 1989 against the appellant reads:

That on or about December 14, 1989, in the City of Zamboanga, Philippines, and within the jurisdiction of this
Honorable Court, the
above-named accused, not being authorized by law, did then and there, wilfully, unlawfully and feloniously sell to
one SGT. AMADO ANI, two (2) wrappers containing dried marijuana leaves, knowing the same to be a prohibited
drug.

CONTRARY TO LAW. 2
Upon his arraignment on January 11, 1990, the appellant pleaded not guilty. 3

At the trial, the prosecution presented three (3) witnesses, namely: (1) Sgt. Amado Ani, Jr. of the 9th Narcotics
Command (NARCOM) of Zamboanga City, who acted as poseur-buyer in the buy-bust operation made against
the appellant; (2) T/Sgt. Jesus Belarga, also of the 9th Narcotics Command of Zamboanga City, who was the
NARCOM team leader of the buy-bust operation; and (3) Athena Elisa P. Anderson, the Document Examiner and
Forensic Chemist of PC-INP Crime Laboratory of Regional Command (RECOM) 9. The evidence of the
prosecution was summarized by the trial court as follows:

Prosecution evidence shows that in the morning of December 13, 1989, T/Sgt. Jesus Belarga, leader of a
NARCOTICS COMMAND (NARCOM) team based at Calarian, Zamboanga City, instructed Sgt. Amado Ani to
conduct surveillance and test buy on a certain Mari Musa of Suterville, Zamboanga City. Information received
from civilian informer was that this Mari Musa was engaged in selling marijuana in said place. So Sgt. Amado Ani,
another NARCOM agent, proceeded to Suterville, in company with a NARCOM civilian informer, to the house of
Mari Musa to which house the civilian informer had guided him. The same civilian informer had also described to
him the appearance of Mari Musa. Amado Ani was able to buy one newspaper-wrapped dried marijuana (Exh. "E")
for P10.00. Sgt. Ani returned to the NARCOM office and turned over the newspaper-wrapped marijuana to T/Sgt.
Jesus Belarga. Sgt. Belarga inspected the stuff turned over to him and found it to be marijuana.

The next day, December 14, 1989, about 1:30 P.M., a buy-bust was planned. Sgt. Amado Ani was assigned as
the poseur buyer for which purpose he was given P20.00 (with SN GA955883) by Belarga. The
buy-bust money had been taken by T/Sgt. Jesus Belarga from M/Sgt. Noh Sali Mihasun, Chief of Investigation
Section, and for which Belarga signed a receipt (Exh. "L" & "L-l" ) The team under Sgt. Foncargas was assigned
as back-up security. A pre-arranged signal was arranged consisting of Sgt. Ani's raising his right hand, after he
had succeeded to buy the marijuana. The two NARCOM teams proceeded to the target site in two civilian
vehicles. Belarga's team was composed of Sgt. Belarga, team leader, Sgt. Amado Ani, poseur buyer, Sgt. Lego
and Sgt. Biong.

Arriving at the target site, Sgt. Ani proceeded to the house of Mari Musa, while the rest of the NARCOM group
positioned themselves at strategic places about 90 to 100 meters from Mari Musa's house. T/Sgt. Belarga could
see what went on between Ani and suspect Mari Musa from where he was. Ani approached Mari Musa, who
came out of his house, and asked Ani what he wanted. Ani said he wanted some more stuff. Ani gave Mari Musa
the P20.00 marked money. After receiving the money, Mari Musa went back to his house and came back and
gave Amado Ani two newspaper wrappers containing dried marijuana. Ani opened the two wrappers and
inspected the contents. Convinced that the contents were marijuana, Ani walked back towards his companions
and raised his right hand. The two NARCOM teams, riding the two civilian vehicles, sped towards Sgt. Ani. Ani
joined Belarga's team and returned to the house.

At the time Sgt. Ani first approached Mari Musa, there were four persons inside his house: Mari Musa, another
boy, and two women, one of whom Ani and Belarga later came to know to be Mari Musa's wife. The second time,
Ani with the NARCOM team returned to Mari Musa's house, the woman, who was later known as Mari Musa's
wife, slipped away from the house. Sgt. Belarga frisked Mari Musa but could not find the P20.00 marked money
with him. Mari Musa was then asked where the P20.00 was and he told the NARCOM team he has given the
money to his wife (who had slipped away). Sgt. Belarga also found a plastic bag containing dried marijuana inside
it somewhere in the kitchen. Mari Musa was then placed under arrest and brought to the NARCOM office. At
Suterville, Sgt. Ani turned over to Sgt. Belarga the two newspaper-wrapped marijuana he had earlier bought from
Mari Musa (Exhs. "C" & "D").

In the NARCOM office, Mari Musa first gave his name as Hussin Musa. Later on, Mari Musa gave his true name
— Mari Musa. T/Sgt. Jesus Belarga turned over the two newspaper-wrapped marijuana (bought at the buy-bust),
the one newspaper-wrapped marijuana (bought at the test-buy) and the plastic bag containing more marijuana
(which had been taken by Sgt. Lego inside the kitchen of Mari Musa) to the PC Crime Laboratory, Zamboanga
City, for laboratory examination. The turnover of the marijuana specimen to the PC Crime Laboratory was by way
of a letter-request, dated December 14, 1989 (Exh. "B"), which was stamped "RECEIVED" by the PC Crime
Laboratory (Exh. "B-1") on the same day.

Mrs. Athena Elisa P. Anderson, the Forensic Chemist of the PC Crime Laboratory, examined the marijuana
specimens subjecting the same to her three tests. All submitted specimens she examined gave positive results
for the presence of marijuana. Mrs. Anderson reported the results of her examination in her Chemistry Report
D-100-89, dated December 14, 1989, (Exh. "J", "J-1", "J-2", "J-3", "J-4" and "J-5"). Mrs. Anderson identified in
court the two newspaper wrapped marijuana bought at the
buy-bust on December 14, 1989, through her initial and the weight of each specimen written with red ink on each
wrapper (Exhs. "C-1" and "D-1"). She also identified the one newspaper-wrapped marijuana bought at the
test-buy on December 13, 1989, through her markings (Exh. "E-1"). Mrs. Anderson also identified her Chemistry
Report (Exh. "J" & sub-markings.)

T. Sgt. Belarga identified the two buy-bust newspaper wrapped marijuana through his initial, the words "buy-bust"
and the words "December 14, 1989, 2:45 P.M." (written on Exhs. "C" and "D"). Belarga also identified the receipt
of the P20 marked money (with SN GA955883) (Exh. "L"), dated December 14, 1989, and his signature thereon
(Exh.
"L-1"). He also identified the letter-request, dated December 14, 1989, addressed to the PC Crime Laboratory
(Exh. "B") and his signature thereon (Exh. "B-2") and the stamp of the PC Crime Laboratory marked "RECEIVED"
(Exh. "B-1").4

For the defense, the following testified as witnesses: (1) the accused-appellant Mari H. Musa; and (2) Ahara R.
Musa, his wife. The trial court summarized the version of the defense, thus:

[O]n December 14, 1989, at about 1:30 in the afternoon, Mari Musa was in his house at Suterville, Zamboanga
City. With him were his wife, Ahara Musa, known as Ara, his one-year old child, a woman manicurist, and a male
cousin named Abdul Musa. About 1:30 that afternoon, while he was being manicured at one hand, his wife was
inside the one room of their house, putting their child to sleep. Three NARCOM agents, who introduced
themselves as NARCOM agents, dressed in civilian clothes, got inside Mari Musa's house whose door was open.
The NARCOM agents did not ask permission to enter the house but simply announced that they were NARCOM
agents. The NARCOM agents searched Mari Musa's house and Mari Musa asked them if they had a search
warrant. The NARCOM agents were just silent. The NARCOM agents found a red plastic bag whose contents,
Mari Musa said, he did not know. He also did not know if the plastic bag belonged to his brother, Faisal, who was
living with him, or his father, who was living in another house about ten arms-length away. Mari Musa, then, was
handcuffed and when Mari Musa asked why, the NARCOM agents told him for clarification.

Mari Musa was brought in a pick-up, his wife joining him to the NARCOM Office at Calarian, Zamboanga City.
Inside the NARCOM Office, Mari Musa was investigated by one NARCOM agent which investigation was reduced
into writing. The writing or document was interpreted to Mari Musa in Tagalog. The document stated that the
marijuana belonged to Mari Musa and Mari Musa was asked to sign it. But Mari Musa refused to sign because the
marijuana did not belong to him. Mari Musa said he was not told that he was entitled to the assistance of counsel,
although he himself told the NARCOM agents he wanted to be assisted by counsel.

Mari Musa said four bullets were then placed between the fingers of his right hand and his fingers were pressed
which felt very painful. The NARCOM agents boxed him and Mari Musa lost consciousness. While Mari Musa
was maltreated, he said his wife was outside the NARCOM building. The very day he was arrested (on
cross-examination Mari Musa said it was on the next day), Mari Musa was brought to the Fiscal's Office by three
NARCOM agents. The fiscal asked him if the marijuana was owned by him and he said "not." After that single
question, Mari Musa was brought to the City Jail. Mari Musa said he did not tell the fiscal that he had been
maltreated by the NARCOM agents because he was afraid he might be maltreated in the fiscal's office.

Mari Musa denied the NARCOM agents' charge that he had sold two wrappers of marijuana to them; that he had
received from them a P20.00 bill which he had given to his wife. He did not sell marijuana because he was afraid
that was against the law and that the person selling marijuana was caught by the authorities; and he had a wife
and a very small child to support. Mari Musa said he had not been arrested for selling marijuana before.5

After trial, the trial court rendered the assailed decision with the following disposition:

WHEREFORE, finding accused Mari Musa y Hantatalu guilty beyond reasonable doubt of selling marijuana and
pursuant to Sec. 4, Art II of Rep. Act No. 6425, he is sentenced to life imprisonment and to pay the fine of
P20,000.00, the latter imposed without subsidiary imprisonment. 6
In this appeal, the appellant contends that his guilt was not proved beyond reasonable doubt and impugns the
credibility of the prosecution witnesses.

The appellant claims that the testimony of Sgt. Ani, the poseur-buyer, is not credible because: (1) prior to the
buy-bust operation, neither Sgt. Ani nor the other NARCOM agents were personally known by the appellant or
vice-versa; and (2) there was no witness to the alleged giving of the two wrappers of marijuana by the appellant to
Sgt. Ani.

Sgt. Ani testified that on December 13, 1989, upon instruction by T/Sgt. Jesus Belarga, he conducted a test-buy
operation on the appellant whereby he bought one wrapper of marijuana for P15.00 from the latter. He reported
7

the successful operation to T/Sgt. Belarga on the same day. Whereupon, T/Sgt. Belarga conducted a conference
8

to organize a buy-bust operation for the following day. 9

On December 14, 1989, at 1:30 p.m., two NARCOM teams in separate vehicles headed by T/Sgt. Belarga and a
certain Sgt. Foncardas went to the place of operation, which was the appellant's house located in Laquian
Compound, Suterville, Zamboanga City. Sgt. Ani was with the team of T/Sgt. Belarga, whose other members
were Sgts. Lego and Biong. Sgt. Ani was given a marked P20.00 bill by T/Sgt. Belarga, which was to be used in
10

the operation.

Upon reaching the place, the NARCOM agents positioned themselves at strategic places. Sgt. Ani approached
11

the house. Outside the house, the appellant asked Sgt. Ani what he wanted. Sgt. Ani asked him for some more
marijuana. Sgt. Ani gave him the marked P20.00 bill and the appellant went inside the house and brought back
12

two paper wrappers containing marijuana which he handed to Sgt. Ani. From his position, Sgt. Ani could see that
13

there were other people in the house. 14

After the exchange, Sgt. Ani approached the other NARCOM agents and made the pre-arranged signal of raising
his right hand. The NARCOM agents, accompanied by Sgt. Ani, went inside the house and made the arrest. The
15

agents searched the appellant and unable to find the marked money, they asked him where it was. The appellant
said that he gave it to his wife. 16

The Court, after a careful reading of the record, finds the testimony of Sgt. Ani regarding the buy-bust operation,
which resulted in the apprehension, prosecution and subsequent conviction of the appellant, to be direct, lucid
and forthright. Being totally untainted by contradictions in any of the material points, it deserves credence.

The contention that the appellant could not have transacted with Sgt. Ani because they do not know each other is
without merit. The day before the
buy-bust operation, Sgt. Ani conducted a test-buy and he successfully bought a wrapper of marijuana from the
appellant. Through this previous transaction, Sgt. Ani was able to gain the appellant's confidence for the latter to
sell more marijuana to Sgt. Ani the following day, during the buy-bust operation. Moreover, the Court has held
that what matters is not an existing familiarity between the buyer and the seller, for quite often, the parties to the
transaction may be strangers, but their agreement and the acts constituting the sale and delivery of the
marijuana. 17

The appellant, again to cast doubt on the credibility of Sgt. Ani, argues that it was impossible for the appellant to
sell marijuana while his wife, cousin and manicurist were present. But the place of the commission of the crime of
selling prohibited drugs has been held to be not crucial and the presence of other people apart from the buyer
18

and seller will not necessarily prevent the consummation of the illegal sale. As the Court observed in People v.
Paco, these factors may sometimes camouflage the commission of the crime. In the instant case, the fact that
19

the other people inside the appellant's house are known to the appellant may have given him some assurance
that these people will not report him to the authorities.

The appellant, besides assailing Sgt. Ani's credibility, also questions the credibility of T/Sgt. Belarga. The
appellant submits that since T/Sgt. Belarga admitted that he was about 90 meters away from Sgt. Ani and the
appellant, he could not have possibly witnessed the sale. The appellant invokes People v.
Ale where the Court observed that from a distance of 10-15 meters, a policeman cannot distinguish between
20

marijuana cigarette from ordinary ones by the type of rolling done on the cigarette sticks. And since T/Sgt.
Belarga allegedly did not see the sale, the appellant contends that the uncorroborated testimony of Sgt. Ani can
not stand as basis for his conviction.
People v. Ale does not apply here because the policeman in that case testified that he and his companion were
certain that the appellant therein handed marijuana cigarettes to the poseur-buyer based on the appearance of
the cigarette sticks. The Court rejected this claim, stating that:

This Court cannot give full credit to the testimonies of the prosecution witnesses marked as they are with
contradictions and tainted with inaccuracies.

Biñan testified that they were able to tell that the four cigarettes were marijuana cigarettes because according to
him, the rolling of ordinary cigarettes are different from those of marijuana cigarettes. (tsn, November 13, 1984, p.
10).

It is however, incredible to believe that they could discern the type of rolling done on those cigarettes from the
distance where they were observing the alleged sale of more or less 10 to 15 meters. 21

In the case at bar, however, T/Sgt. Belarga did not positively claim that he saw the appellant hand over marijuana
to Sgt. Ani. What he said was that there was an exchange of certain articles between the two. The relevant
portion of T/Sgt. Belarga's testimony reads: 22

Q Now, do you remember whether Sgt. Ani was able to reach the house of Mari Musa?

A Yes, ma'am.

Q After reaching Mari Musa, did you see what happened (sic)?

A Yes, ma'am.

Q Could you please tell us?

A From our vehicle the stainless owner type jeep where Sgt. Lego, Sgt. Biong were boarded, I saw that Sgt. Ani
proceeded to the house near the road and he was met by one person and later known as Mari Musa who was at
the time wearing short pants and later on I saw that Sgt. Ani handed something to him, thereafter received by
Mari Musa and went inside the house and came back later and handed something to Sgt. Ani.

Contrary to the contention of the appellant, it was not impossible for T/Sgt. Belarga to have seen, from a distance
of 90-100 meters, Sgt. Ani hand to the appellant "something" and for the latter to give to the former "something."

Notwithstanding the fact that T/Sgt. Belarga could not have been certain that what Sgt. Ani received from the
appellant was marijuana because of the distance, his testimony, nevertheless, corroborated the direct evidence,
which the Court earlier ruled to be convincing, presented by Sgt. Ani on the following material points: (1) T/Sgt.
Belarga instructed Sgt. Ani to conduct a surveillance and test-buy operation on the appellant at Suterville,
Zamboanga City on December 13, 1989; (2) later that same day, Sgt. Ani went back to their office and reported
23

a successful operation and turned over to T/Sgt. Belarga one wrapper of marijuana; (3) T/Sgt. Belarga then
24

organized a team to conduct a buy-bust operation the following day; (4) on December 14, 1989, T/Sgt. Belarga
25

led a team of NARCOM agents who went to Suterville, Zamboanga City; (5) T/Sgt. Belarga gave a P20.00
26

marked bill to Sgt. Ani which was to be used in the buy-bust operation; (6) upon the arrival of the NARCOM
27

agents in Suterville, Zamboanga City, Sgt. Ani proceeded to the house of the appellant while some agents stayed
in the vehicles and others positioned themselves in strategic places; the appellant met Sgt. Ani and an exchange
28

of articles took place. 29

The corroborative testimony of T/Sgt. Belarga strengthens the direct evidence given by Sgt. Ani. Additionally, the
Court has ruled that the fact that the police officers who accompanied the poseur-buyer were unable to see
exactly what the appellant gave the poseur-buyer because of their distance or position will not be fatal to the
prosecution's case provided there exists other evidence, direct or circumstantial, e.g., the testimony of the
30

poseur-buyer, which is sufficient to prove the consummation of the sale of the prohibited drug

The appellant next assails the seizure and admission as evidence of a plastic bag containing marijuana which the
NARCOM agents found in the appellant's kitchen. It appears that after Sgt. Ani gave the pre-arranged signal to
the other NARCOM agents, the latter moved in and arrested the appellant inside the house. They searched him
to retrieve the marked money but didn't find it. Upon being questioned, the appellant said that he gave the marked
money to his wife. Thereafter, T/Sgt. Belarga and Sgt. Lego went to the kitchen and noticed what T/Sgt. Belarga
31

described as a "cellophane colored white and stripe hanging at the corner of the kitchen." They asked the
32

appellant about its contents but failing to get a response, they opened it and found dried marijuana leaves. At the
trial, the appellant questioned the admissibility of the plastic bag and the marijuana it contains but the trial court
issued an Order ruling that these are admissible in evidence. 33

Built into the Constitution are guarantees on the freedom of every individual against unreasonable searches and
seizures by providing in Article III, Section 2, the following:

The right of the people to be secure in their persons, houses, papers, and effects against unreasonable searches
and seizures of whatever nature and for any purpose shall be inviolable, and no search warrant or warrant of
arrest shall issue except upon probable cause to be determined personally by the judge after examination under
oath or affirmation of the complainant and the witness he may produce, and particularly describing the place to be
searched and the persons or things to be seized.

Furthermore, the Constitution, in conformity with the doctrine laid down in Stonehill v. Diokno, declares 34

inadmissible, any evidence obtained in violation of the freedom from unreasonable searches and seizures. 35

While a valid search warrant is generally necessary before a search and seizure may be effected, exceptions to
this rule are recognized. Thus, in Alvero v. Dizon, the Court stated that. "[t]he most important exception to the
36

necessity for a search warrant is the right of search and seizure as an incident to a lawful arrest." 37

Rule 126, Section 12 of the Rules of Court expressly authorizes a warrantless search and seizure incident to a
lawful arrest, thus:

Sec. 12. Search incident to lawful arrest. — A person lawfully arrested may be searched for dangerous weapons
or anything which may be used as proof of the commission of an offense, without a search warrant.

There is no doubt that the warrantless search incidental to a lawful arrest authorizes the arresting officer to make
a search upon the person of the person arrested. As early as 1909, the Court has ruled that "[a]n officer making
an arrest may take from the person arrested any money or property found upon his person which was used in the
commission of the crime or was the fruit of the crime or which might furnish the prisoner with the means of
committing
violence or of escaping, or which may be used as evidence in the trial of the cause . . . " Hence, in a buy-bust
38

operation conducted to entrap a drug-pusher, the law enforcement agents may seize the marked money found on
the person
of the pusher immediately after the arrest even without arrest and search warrants. 39

In the case at bar, the NARCOM agents searched the person of the appellant after arresting him in his house but
found nothing. They then searched the entire house and, in the kitchen, found and seized a plastic bag hanging in
a corner.

The warrantless search and seizure, as an incident to a suspect's lawful arrest, may extend beyond the person of
the one arrested to include the premises or surroundings under his immediate control. Objects in the "plain view"
40

of an officer who has the right to be in the position to have that view are subject to seizure and may be presented
as evidence. 41

In Ker v. California police officers, without securing a search warrant but having information that the defendant
42

husband was selling marijuana from his apartment, obtained from the building manager a passkey to defendants'
apartment, and entered it. There they found the defendant husband in the living room. The defendant wife
emerged from the kitchen, and one of the officers, after identifying himself, observed through the open doorway of
the kitchen, a small scale atop the kitchen sink, upon which lay a brick-shaped package containing green leafy
substance which he recognized as marijuana. The package of marijuana was used as evidence in prosecuting
defendants for violation of the Narcotic Law. The admissibility of the package was challenged before the U.S.
Supreme Court, which held, after observing that it was not unreasonable for the officer to walk to the doorway of
the adjacent kitchen on seeing the defendant wife emerge therefrom, that "the discovery of the brick of marijuana
did not constitute a search, since the officer merely saw what was placed before him in full view. The U.S. 43
Supreme Court ruled that the warrantless seizure of the marijuana was legal on the basis of the "plain view"
doctrine and upheld the admissibility of the seized drugs as part of the prosecution's evidence. 44

The "plain view" doctrine may not, however, be used to launch unbridled searches and indiscriminate seizures
nor to extend a general exploratory search made solely to find evidence of defendant's guilt. The "plain view"
doctrine is usually applied where a police officer is not searching for evidence against the accused, but
nonetheless inadvertently comes across an incriminating object. Furthermore, the U.S. Supreme Court stated
45

the following limitations on the application of the doctrine:

What the "plain view" cases have in common is that the police officer in each of them had a prior justification for
an intrusion in the course of which he came inadvertently across a piece of evidence incriminating the accused.
The doctrine serves to supplement the prior justification — whether it be a warrant for another object, hot pursuit,
search incident to lawful arrest, or some other legitimate reason for being present unconnected with a search
directed against the accused — and permits the warrantless seizure. Of course, the extension of the original
justification is legitimate only where it is immediately apparent to the police that they have evidence before them;
the "plain view" doctrine may not be used to extend a general exploratory search from one object to another until
something incriminating at last emerges. 46

It has also been suggested that even if an object is observed in "plain view," the "plain view" doctrine will not
justify the seizure of the object where the incriminating nature of the object is not apparent from the "plain view" of
the object. Stated differently, it must be immediately apparent to the police that the items that they observe may
47

be evidence of a crime, contraband, or otherwise subject to seizure.

In the instant case, the appellant was arrested and his person searched in the living room. Failing to retrieve the
marked money which they hoped to find, the NARCOM agents searched the whole house and found the plastic
bag in the kitchen. The plastic bag was, therefore, not within their "plain view" when they arrested the appellant as
to justify its seizure. The NARCOM agents had to move from one portion of the house to another before they
sighted the plastic bag. Unlike Ker vs. California, where the police officer had reason to walk to the doorway of the
adjacent kitchen and from which position he saw the marijuana, the NARCOM agents in this case went from room
to room with the obvious intention of fishing for more evidence.

Moreover, when the NARCOM agents saw the plastic bag hanging in one corner of the kitchen, they had no clue
as to its contents. They had to ask the appellant what the bag contained. When the appellant refused to respond,
they opened it and found the marijuana. Unlike Ker v. California, where the marijuana was visible to the police
officer's eyes, the NARCOM agents in this case could not have discovered the inculpatory nature of the contents
of the bag had they not forcibly opened it. Even assuming then, that the NARCOM agents inadvertently came
across the plastic bag because it was within their "plain view," what may be said to be the object in their "plain
view" was just the plastic bag and not the marijuana. The incriminating nature of the contents of the plastic bag
was not immediately apparent from the "plain view" of said object. It cannot be claimed that the plastic bag clearly
betrayed its contents, whether by its distinctive configuration, its transprarency, or otherwise, that its contents are
obvious to an observer. 48

We, therefore, hold that under the circumstances of the case, the "plain view" doctrine does not apply and the
marijuana contained in the plastic bag was seized illegally and cannot be presented in evidence pursuant to
Article III, Section 3(2) of the Constitution.

The exclusion of this particular evidence does not, however, diminish, in any way, the damaging effect of the
other pieces of evidence presented by the prosecution to prove that the appellant sold marijuana, in violation of
Article II, Section 4 of the Dangerous Drugs Act of 1972. We hold that by virtue of the testimonies of Sgt. Ani and
T/Sgt. Belarga and the two wrappings of marijuana sold by the appellant to Sgt. Ani, among other pieces of
evidence, the guilt of the appellant of the crime charged has been proved beyond reasonable doubt.

WHEREFORE, the appeal is DISMISSED and the judgment of the Regional Trial Court AFFIRMED.

SO ORDERED.
[G.R. No. 145176. March 30, 2004]
PEOPLE OF THE PHILIPPINES, appellee, vs. SANTIAGO PERALTA y POLIDARIO (at large), ARMANDO DATUIN
JR. y GRANADOS (at large), ULYSSES GARCIA yTUPAS, MIGUELITO DE LEON y LUCIANO,
LIBRANDO FLORES y CRUZ AND ANTONIO LOYOLA y SALISI, accused, ULYSSES GARCIA Y TUPAS,
MIGUELITO DE LEON y LUCIANO, LIBRANDO FLORES y CRUZ AND ANTONIO
LOYOLA y SALISI, appellants.

DECISION
PANGANIBAN, J.:

The right of the accused to counsel demands effective, vigilant and independent representation. The lawyers role
cannot be reduced to being that of a mere witness to the signing of an extra-judicial confession.

The Case

Before the Court is an appeal from the August 21, 2000 Decision [1] of the Regional Trial Court (RTC) of Manila
(Branch 18) in Criminal Case No. 92-112322. Appellants Ulysses Garcia y Tupas, Miguelito de Leon y Luciano,
Librando Flores y Cruz and Antonio Loyola y Salisi, as well as their co-accused -- Santiago Peralta y Polidario and
Armando Datuin Jr. y Granados -- were convicted therein of qualified theft.The dispositive portion of the Decision
reads:

WHEREFORE, the accused, Santiago Peralta y Polidario, Armando Datuin, Jr. y Granados, Ulysses Garcia y Tupas, Miguelito De
Leon y Luciano, Librando Flores y Cruz and Antonio Loyola y Salisi, are hereby convicted of the crime of qualified theft
of P194,190.00 and sentenced to suffer the penalty of reclusion perpetua with all the accessory penalties provided by law, and to
pay the costs. Moreover, all the accused are ordered to pay the Central Bank of the Philippines, now Bangko Sentral ng Pilipinas,
actual damages in the sum of P194,190.00 with interest thereon at the legal rate from the date of the filing of this action,
November 9, 1992, until fully paid.[2]

In an Information dated November 9, 1992,[3] appellants and their co-accused were charged as follows:

That sometime in the year 1990 and including November 4, 1992, in the City of Manila, Philippines, the said accused, conspiring
and confederating with others whose true names, identities and present whereabouts are still unknown and helping one another,
did then and there wilfully, unlawfully and feloniously, with intent to gain and without the knowledge and consent of the owner
thereof, take, steal and carry away punctured currency notes due for shredding in the total amount of P194,190.00, belonging to
the Central Bank of the Philippines as represented by Pedro Labita y Cabriga, to the damage and prejudice of the latter in the
aforesaid sum of P194,190.00 Philippine currency;

That said accused Santiago Peralta y Polidario, Armando Datuin, Jr. y Granados, Ulysses Garcia y Tupas, Miguelito de Leon y
Luciano and Antonio Loyola y Salisi committed said offense with grave abuse of confidence they being at the time employed as
Currency Reviewers, Driver, Currency Assistant I and Money Counter of the offended party and as such they had free access to
the property stolen.[4]

Garcia was arrested on November 4, 1992; and his co-accused, on November 9, 1992. Appellants, however,
obtained two Release Orders from RTC Vice Executive Judge Corona Ibay-Somera on November 9 and 10, 1992,
upon their filing of a cash bond to secure their appearance whenever required by the trial court. [5]
During their arraignment on May 4, 1993, appellants, assisted by their respective counsels, pleaded not
guilty.[6] On September 30, 1998, the trial court declared that Datuin Jr. and Peralta were at large, because they had
failed to appear in court despite notice.[7]
After trial in due course, they were all found guilty and convicted of qualified theft in the appealed Decision.

The Facts

Version of the Prosecution


The Office of the Solicitor General (OSG) presents the prosecutions version of the facts as follows:

About 10:00 oclock in the morning of November 4, 1992, Pedro Labita of Central Bank of the Philippines (CBP) [now Bangko
Sentral ng Pilipinas (BSP)] went to the Theft and Robbery Section of Western Police District Command (WPDC), and filed a
complaint for Qualified Theft against Santiago Peralta, Armando Datuin, Jr., Ulysses Garcia, Miguelito de Leon, Librando Flores
and Antonio S. Loyola.

Pedro Labita submitted to SPO4 Cielito Coronel, the investigating officer at WPDC, punctured currency notes in P100.00
and P500.00 bills with a face value of Php194,190.00. Said notes were allegedly recovered by the BSP Cash Department during
its cash counting of punctured currency bills submitted by different banks to the latter. The punctured bills were rejected by the
BSP money counter machine and were later submitted to the investigation staff of the BSP Cash Department. As a result of the
investigation, it was determined that said rejected currency bills were actually punctured notes already due for shredding. These
currency bills were punctured because they were no longer intended for circulation. Before these notes could be shredded, they
were stolen from the BSP by the above-named accused.

On the basis of the complaint filed by Pedro Labita, Ulysses Garcia was apprehended in front of Golden Gate Subdivision, Las
Pias City, while he was waiting for a passenger bus on his way to the BSP. Garcia was brought to the police station for
investigation.

On November 4, 5 and 6, 1992, while in the custody of the police officers, Garcia gave three separate statements admitting his
guilt and participation in the crime charged. He also identified the other named accused as his cohorts and accomplices and
narrated the participation of each and everyone of them.

On the basis of Garcias sworn statements, the other named accused were invited for questioning at the police station and were
subsequently charged with qualified theft together with Garcia. [8] (Citations omitted)

Version of the Defense

The defense states its version of the facts in the following manner:

Accused-appellant Garcia served as a driver of the armored car of the Central Bank from 1978 to 1994.

On November 4, 1992, between 7:00 a.m. and 8:00 a.m., a man who had identified himself as a police officer arrested
accused-appellant Garcia while waiting for a passenger bus in front of the Golden Gate Subdivision, Las Pias City. He was
arrested without any warrant for his arrest. The police officer who had arrested accused-appellant Garcia dragged the latter across
the street and forced him to ride x x x a car.

While inside the car, he was blindfolded, his hands were handcuffed behind his back, and he was made to bend with his chest
touching his knees. Somebody from behind hit him and he heard some of the occupants of the car say that he would be salvaged if
he would not tell the truth. When the occupants of the car mentioned perforated notes, he told them that he does not know
anything about those notes.

After the car had stopped, he was dragged out of the car and x x x up and down x x x the stairs. While being dragged out of the car,
he felt somebody frisk his pocket.

At a safe house, somebody mentioned to him the names of his co-accused and he told them that he does not know his co-accused
x x x. Whenever he would deny knowing his co-accused, somebody would box him on his chest.Somebody poured water on
accused-appellant Garcias nose while lying on the bench. He was able to spit out the water that had been poured on his nose [at
first], but somebody covered his mouth. As a result, he could not breath[e].

When accused-appellant Garcia realized that he could not bear the torture anymore, he decided to cooperate with the police, and
they stopped the water pouring and allowed him to sit down.

Accused-appellant Garcia heard people talking and he heard somebody utter, may nakikinig. Suddenly his two ears were hit with
open palm[s] x x x. As he was being brought down, he felt somebody return his personal belongings to his
pocket. Accused-appellant Garcias personal belongings consisted of [his] drivers license, important papers and coin purse.
He was forced to ride x x x the car still with blindfold. His blindfold and handcuffs were removed when he was at the office of
police officer Dante Dimagmaliw at the Western Police District, U.N. Avenue, Manila.

SPO4 Cielito Coronel asked accused-appellant Garcia about the latters name, age and address. The arrival of Mr. Pedro Labita of
the Cash Department, Central Bank of the Philippines, interrupted the interview, and Mr. Labita instructed SPO4 Coronel to get
accused-appellant Garcias wallet and examine the contents thereof. SPO4 Coronel supposedly found three pieces of P100
perforated bill in accused-appellant Garcias wallet and the former insisted that they recovered the said perforated notes from
accused-appellants wallet. SPO4 Coronel took down the statement of Mr. Labita.

It was actually Mr. Labita, and not accused-appellant Garcia, who gave the answers appearing in accused-appellant Garcias
alleged three sworn statements dated November 4, 1992, November 5, 1992 and x x x November 6, 1992.

At or about 6:00 p.m. on November 5, 1992, accused-appellant Garcia was brought to the cell of the Theft and Robbery Section of
the WPD. At or about 8:00 p.m., he was brought to the office of Col. Alladin Dimagmaliw where his co-accused were also
inside. He did not identify his co-accused, but he merely placed his hands on the shoulders of each of his co-accused, upon being
requested, and Mr. Labita took x x x pictures while he was doing the said act.

Accused-appellant Garcia came to know Atty. Francisco Sanchez of the Public Attorneys Office on November 4, 1992, at the
office of police officer Dante Dimagmaliw, when SPO4 Coronel introduced Atty. Sanchez to accused-appellant Garcia and told
him that Atty. Sanchez would be his lawyer. However, accused-appellant Garcia did not agree to have Atty. Sanchez to be his
lawyer. Atty. Sanchez left after talking to SPO4 Coronel, and accused-appellant Garcia had not met Atty. Sanchez anymore since
then. He was not present when Atty. Sanchez allegedly signed x x x the alleged three (3) sworn statements.

During the hearing of the case on April 6, 2000, Atty. Sanchez manifested in open court that he did not assist accused-appellant
Garcia when the police investigated accused-appellant Garcia, and that he signed x x x the three (3) sworn statements only as a
witness thereto.

Accused-appellant Garcia signed the alleged three sworn statements due to SPO4 Coronels warning that if he would not do so, he
would again be tortured by water cure.

SPO[4] Coronel caused the arrest without any warrant of accused appellants De Leon, Loyola, [Flores] on the basis of the
complaint of Mr. Pedro Labita, and which arrest was effected on November 5, 1992, by SPO1 Alfredo Silva and SPO1 Redelico.

SPO4 Coronel, in his letter dated November 6, 1992, forwarded the case to the Duty Inquest Prosecutor assigned at the WPDC
Headquarters.[9] (Citations omitted)

Ruling of the Trial Court

The trial court found that all the accused used to work for the BSP. Garcia was a driver assigned to the Security
and Transport Department; while Peralta, Datuin Jr., De Leon, Flores and Loyola were laborers assigned to the
Currency Retirement Division. Their main task was to haul perforated currency notes from the currency retirement vault
to the basement of the BSP building for shredding.
On several occasions, during the period 1990-1992, they handed to Garcia perforated currency notes placed in a
coin sack that he, in turn, loaded in an armored escort van and delivered to someone waiting outside the premises of
the building. The trial court held that the coordinated acts of all the accused unerringly led to the conclusion that they
had conspired to pilfer the perforated currency notes belonging to the BSP.
The RTC rejected the disclaimer by Garcia of his own confessions, as such disclaimer was an eleventh hour
concoction to exculpate himself and his co-accused. The trial court found his allegations of torture and coerced
confessions unsupported by evidence. Moreover, it held that the recovery of three pieces of perforated P100 bills from
Garcias wallet and the flight of Peralta and Datuin Jr. were indicative of the guilt of the accused.
Hence, this appeal.[10]

Issues

In his Brief, Garcia raises the following issues:


1
The trial court erred in admitting in evidence the alleged three Sworn Statements of Accused-appellant
Garcia and the alleged three pieces of P100 perforated notes
2
The trial court erred in finding the accused-appellant guilty of qualified theft.[11]
In their joint Brief, De Leon, Loyola and Flores interpose this additional assignment of errors:
1
The trial court erred in admitting in evidence the alleged three sworn statements of Accused Ulysses
Garcia (Exhibits I, J and K) and the alleged three pieces of P100 perforated notes (Exhibits N to N-2) over
the objections of the accused-appellants.
2
The trial court erred in denying the demurrer to evidence of Accused-appellants De Leon, Loyola and
Flores;
3
The trial court erred in denying the Motion for Reconsideration of the Order denying the demurrer to
evidence;
4
The trial court erred when it failed to consider the evidence adduced by the accused-appellants, consisting of
exhibits 1, 2 to 2-B, 3 and 4 and the testimony of their witness, State Auditor Esmeralda Elli;
5
The trial court erred in finding the accused-appellants guilty of qualified theft.[12]
Simplified, the issues are as follows: (1) the sufficiency of the evidence against appellants, including the
admissibility of Garcias confessions and of the three perforated P100 currency notes; and (2) the propriety of the denial
of their demurrer to evidence.

The Courts Ruling

The appeal has merit.

First Issue:
Sufficiency of Evidence

The trial court convicted appellants mainly on the strength of the three confessions given by Garcia and the three
perforated P100 currency notes confiscated from him upon his arrest. Appellants, however, contend that these pieces
of evidence are inadmissible.

Extrajudicial Confessions

Appellants aver that the alleged three Sworn Statements of Garcia were obtained without the assistance of
counsel in violation of his rights under Article III, Section 12 (1) and (2) of the 1987 Constitution, which provides thus:

Sec. 12. (1) Any person under investigation for the commission of an offense shall have the right to be informed of his right to
remain silent and to have competent and independent counsel, preferably of his own choice. If the person cannot afford the
services of counsel, he must be provided with one. These rights cannot be waived except in writing and in the presence of counsel.
(2) No torture, force, violence, threat, intimidation, or any other means which vitiate the free will shall be used against him. Secret
detention places, solitary, incomunicado, or other similar forms of detention are prohibited.

On the other hand, the OSG contends that counsel, Atty. Francisco Sanchez III of the Public Attorneys Office, duly
assisted Garcia during the custodial investigation.
It is clear from a plain reading of the three extrajudicial confessions [13] that Garcia was not assisted by Atty.
Sanchez. The signature of the latter on those documents was affixed after the word SAKSI. Moreover, he appeared in
court and categorically testified that he had not assisted Garcia when the latter was investigated by the police, and that
the former had signed the Sworn Statement only as a witness.[14]
The written confessions, however, were still admitted in evidence by the RTC on the ground that Garcia had
expressed in writing his willingness and readiness to give the Sworn Statements without the assistance of counsel. The
lower courts action is manifest error.
The right to counsel has been written into our Constitution in order to prevent the use of duress and other undue
influence in extracting confessions from a suspect in a crime. The basic law specifically requires that any waiver of this
right must be made in writing and executed in the presence of a counsel. In such case, counsel must not only ascertain
that the confession is voluntarily made and that the accused understands its nature and consequences, but also advise
and assist the accused continuously from the time the first question is asked by the investigating officer until the signing
of the confession.
Hence, the lawyers role cannot be reduced to being that of a mere witness to the signing of a pre-prepared
confession, even if it indicated compliance with the constitutional rights of the accused. [15] The accused is entitled to
effective, vigilant and independent counsel.[16]
A waiver in writing, like that which the trial court relied upon in the present case, is not enough. Without the
assistance of a counsel, the waiver has no evidentiary relevance.[17] The Constitution states that [a]ny confession or
admission obtained in violation of [the aforecited Section 12] shall be inadmissible in evidence x x x. Hence, the trial
court was in error when it admitted in evidence the uncounseled confessions of Garcia and convicted appellants on the
basis thereof. The question of whether he was tortured becomes moot.

Perforated Currency Notes

Appellants contend that the three P100 perforated currency notes (Exhibits N to N-2) allegedly confiscated from
Garcia after his arrest were fruits of the poisonous tree and, hence, inadmissible in evidence.
The solicitor general evades the issue and argues, instead, that appellants waived the illegality of their arrest when
they entered a plea. He further contends that the exclusion from the evidence of the three punctured currency bills
would not alter the findings of the trial court.
The police arrested Garcia without a warrant, while he had merely been waiting for a passenger bus after being
pointed out by the Cash Department personnel of the BSP. At the time of his arrest, he had not committed, was not
committing, and was not about to commit any crime. Neither was he acting in a manner that would engender a
reasonable ground to suspect that he was committing a crime. None of the circumstances justifying an arrest without a
warrant under Section 5 of Rule 113 of the Rules of Court was present.
Hence, Garcia was not lawfully arrested. Nonetheless, not having raised the matter before entering his plea, he is
deemed to have waived the illegality of his arrest. Note, however, that this waiver is limited to the arrest. It does not
extend to the search made as an incident thereto or to the subsequent seizure of evidence allegedly found during the
search.
The Constitution proscribes unreasonable searches and seizures [18] of whatever nature. Without a judicial warrant,
these are allowed only under the following exceptional circumstances: (1) a search incident to a lawful arrest, (2)
seizure of evidence in plain view, (3) search of a moving motor vehicle, (4) customs search, (5) stop and frisk situations,
and (6) consented search.[19]
Where the arrest was incipiently illegal, it follows that the subsequent search was similarly illegal.[20] Any evidence
obtained in violation of the constitutional provision is legally inadmissible in evidence under the exclusionary rule. [21] In
the present case, the perforated P100 currency notes were obtained as a result of a search made without a warrant
subsequent to an unlawful arrest; hence, they are inadmissible in evidence.
Moreover, untenable is the solicitor generals argument that Appellants De Leon, Flores and Loyola waived the
illegality of the arrest and seizure when, without raising objections thereto, they entered a plea of guilty. It was Garcia
who was unlawfully arrested and searched, not the aforementioned three appellants. The legality of an arrest can be
contested only by the party whose rights have been impaired thereby.Objection to an unlawful search and seizure is
purely personal, and third parties cannot avail themselves of it.[22]
Indeed, the prosecution sufficiently proved the theft of the perforated currency notes for retirement. It failed,
however, to present sufficient admissible evidence pointing to appellants as the authors of the crime.
The evidence presented by the prosecution shows that there were other people who had similar access to the
shredding machine area and the currency retirement vault.[23] Appellants were pinpointed by Labita because of an
anonymous phone call informing his superior of the people allegedly behind the theft; and of the unexplained increase
in their spending, which was incompatible with their income. Labita, however, did not submit sufficient evidence to
support his allegation.
Without the extrajudicial confession and the perforated currency notes, the remaining evidence would be utterly
inadequate to overturn the constitutional presumption of innocence.

Second Issue:
Demurrer to Evidence

Appellants contend that the trial court seriously erred when it denied the demurrer to evidence filed by Appellants
Loyola, De Leon and Flores. Not one of the documents offered by the prosecution and admitted in evidence by the
RTC established the alleged qualified theft of perforated notes, and not one of the pieces of evidence showed
appellants participation in the commission of the crime.
On the exercise of sound judicial discretion rests the trial judges determination of the sufficiency or the
insufficiency of the evidence presented by the prosecution to establish a prima facie case against the accused. Unless
there is a grave abuse of discretion amounting to lack of jurisdiction, the trial courts denial of a motion to dismiss may
not be disturbed.[24]
As discussed earlier, the inadmissibility of the confessions of Garcia did not become apparent until after Atty.
Francisco had testified in court. Even if the confiscated perforated notes from the person of the former were held to be
inadmissible, the confessions would still have constituted prima facie evidence of the guilt of appellants. On that basis,
the trial court did not abuse its discretion in denying their demurrer to evidence.
WHEREFORE, the assailed Decision is REVERSED and SET ASIDE. Appellants are hereby ACQUITTED and
ordered immediately RELEASED, unless they are being detained for any other lawful cause.The director of the Bureau
of Corrections is hereby directed to submit his report on the release of the appellant or the reason for his continued
detention within five (5) days from notice of this Decision. No costs.
SO ORDERED.
G.R. No. 83988 September 29, 1989

RICARDO C. VALMONTE AND UNION OF LAWYERS AND ADVOCATES FOR PEOPLE'S RIGHTS
(ULAP), petitioners,
vs.
GEN. RENATO DE VILLA AND NATIONAL CAPITAL REGION DISTRICT COMMAND, respondents.

Ricardo C. Valmonte for himself and his co-petitioners.

PADILLA, J.:

This is a petition for prohibition with preliminary injunction and/or temporary restraining order, seeking the
declaration of checkpoints in Valenzuela, Metro Manila or elsewhere, as unconstitutional and the dismantling and
banning of the same or, in the alternative, to direct the respondents to formulate guidelines in the implementation
of checkpoints, for the protection of the people.
Petitioner Ricardo C. Valmonte sues in his capacity as citizen of the Republic, taxpayer, member of the Integrated
Bar of the Philippines (IBP), and resident of Valenzuela, Metro Manila; while petitioner Union of Lawyers and
Advocates for People's Rights (ULAP) sues in its capacity as an association whose members are all members of
the IBP.

The factual background of the case is as follows:

On 20 January 1987, the National Capital Region District Command (NCRDC) was activated pursuant to Letter of
Instruction 02/87 of the Philippine General Headquarters, AFP, with the mission of conducting security operations
within its area of responsibility and peripheral areas, for the purpose of establishing an effective territorial defense,
maintaining peace and order, and providing an atmosphere conducive to the social, economic and political
development of the National Capital Region. As part of its duty to maintain peace and order, the NCRDC installed
1

checkpoints in various parts of Valenzuela, Metro Manila.

Petitioners aver that, because of the installation of said checkpoints, the residents of Valenzuela are worried of
being harassed and of their safety being placed at the arbitrary, capricious and whimsical disposition of the
military manning the checkpoints, considering that their cars and vehicles are being subjected to regular searches
and check-ups, especially at night or at dawn, without the benefit of a search warrant and/or court order. Their
alleged fear for their safety increased when, at dawn of 9 July 1988, Benjamin Parpon, a supply officer of the
Municipality of Valenzuela, Bulacan, was gunned down allegedly in cold blood by the members of the NCRDC
manning the checkpoint along McArthur Highway at Malinta, Valenzuela, for ignoring and/or refusing to submit
himself to the checkpoint and for continuing to speed off inspire of warning shots fired in the air. Petitioner
Valmonte also claims that, on several occasions, he had gone thru these checkpoints where he was stopped and
his car subjected to search/check-up without a court order or search warrant.

Petitioners further contend that the said checkpoints give the respondents a blanket authority to make searches
and/or seizures without search warrant or court order in violation of the Constitution; and, instances have
2

occurred where a citizen, while not killed, had been harassed.

Petitioners' concern for their safety and apprehension at being harassed by the military manning the checkpoints
are not sufficient grounds to declare the checkpoints as per se illegal. No proof has been presented before the
Court to show that, in the course of their routine checks, the military indeed committed specific violations of
petitioners' right against unlawful search and seizure or other rights.

In a case filed by the same petitioner organization, Union of Lawyers and Advocates for People's Right (ULAP) vs.
Integrated National Police, it was held that individual petitioners who do not allege that any of their rights were
3

violated are not qualified to bring the action, as real parties in interest.

The constitutional right against unreasonable searches and seizures is a personal right invocable only by those
whose rights have been infringed, or threatened to be infringed. What constitutes a reasonable or unreasonable
4

search and seizure in any particular case is purely a judicial question, determinable from a consideration of the
circumstances involved. 5

Petitioner Valmonte's general allegation to the effect that he had been stopped and searched without a search
warrant by the military manning the checkpoints, without more, i.e., without stating the details of the incidents
which amount to a violation of his right against unlawful search and seizure, is not sufficient to enable the Court to
determine whether there was a violation of Valmonte's right against unlawful search and seizure. Not all searches
and seizures are prohibited. Those which are reasonable are not forbidden. A reasonable search is not to be
determined by any fixed formula but is to be resolved according to the facts of each case. 6

Where, for example, the officer merely draws aside the curtain of a vacant vehicle which is parked on the public
fair grounds, or simply looks into a vehicle, or flashes a light therein, these do not constitute unreasonable
7 8 9

search.

The setting up of the questioned checkpoints in Valenzuela (and probably in other areas) may be considered as a
security measure to enable the NCRDC to pursue its mission of establishing effective territorial defense and
maintaining peace and order for the benefit of the public. Checkpoints may also be regarded as measures to
thwart plots to destabilize the government, in the interest of public security. In this connection, the Court may take
judicial notice of the shift to urban centers and their suburbs of the insurgency movement, so clearly reflected in
the increased killings in cities of police and military men by NPA "sparrow units," not to mention the abundance of
unlicensed firearms and the alarming rise in lawlessness and violence in such urban centers, not all of which are
reported in media, most likely brought about by deteriorating economic conditions — which all sum up to what
one can rightly consider, at the very least, as abnormal times. Between the inherent right of the state to protect its
existence and promote public welfare and an individual's right against a warrantless search which is
however reasonably conducted, the former should prevail.

True, the manning of checkpoints by the military is susceptible of abuse by the men in uniform, in the same
manner that all governmental power is susceptible of abuse. But, at the cost of occasional inconvenience,
discomfort and even irritation to the citizen, the checkpoints during these abnormal times, when conducted within
reasonable limits, are part of the price we pay for an orderly society and a peaceful community.

Finally, on 17 July 1988, military and police checkpoints in Metro Manila were temporarily lifted and a review and
refinement of the rules in the conduct of the police and military manning the checkpoints was ordered by the
National Capital Regional Command Chief and the Metropolitan Police Director. 10

WHEREFORE, the petition is DISMISSED.

SO ORDERED.

Fernan, C.J., Narvasa, Melencio-Herrera, Gutierrez, Jr., Paras, Feliciano, Gancayco, Bidin, Cortes, Griño-Aquino,
Medialdea and Regalado, JJ., concur.

Separate Opinions

CRUZ, J., dissenting:

I dissent. The sweeping statements in the majority opinion are as dangerous as the checkpoints it would sustain
and fraught with serious threats to individual liberty. The bland declaration that individual rights must yield to the
demands of national security ignores the fact that the Bill of Rights was intended precisely to limit the authority of
the State even if asserted on the ground of national security. What is worse is that the searches and seizures are
peremptorily pronounced to be reasonable even without proof of probable cause and much less the required
warrant. The improbable excuse is that they are aimed at 'establishing an effective territorial defense, maintaining
peace and order, and providing an atmosphere conducive to the social, economic and political development of
the National Capital Region." For these purposes, every individual may be stopped and searched at random and
at any time simply because he excites the suspicion, caprice, hostility or malice of the officers manning the
checkpoints, on pain of arrest or worse, even being shot to death, if he resists.

I have no quarrel with a policeman flashing a light inside a parked vehicle on a dark street as a routine measure of
security and curiosity. But the case at bar is different. Military officers are systematically stationed at strategic
checkpoint to actively ferret out suspected criminals by detaining and searching any individual who in their
opinion might impair "the social, economic and political development of the National Capital Region." It is
incredible that we can sustain such a measure. And we are not even under martial law.

Unless we are vigilant of our rights, we may find ourselves back to the dark era of the truncheon and the barbed
wire, with the Court itself a captive of its own complaisance and sitting at the death-bed of liberty.

SARMIENTO, J., dissenting:


I join Justice Isagani Cruz in his dissent, delivered so staightforwardly and eloquently. I am agreed that the
existence alone of checkpoints makes search done therein, unreasonable and hence, repugnant to the
Constitution.

The Charter says that the people enjoy the right of security of person, home, and effects. (CONST., art. III, sec. 2.)
It is also the bedrock — the right of the people to be left alone — on which the regime of law and constitutionalism
rest. It is not, as the majority would put it, a matter of "occasional inconveniences, discomfort and even irritation."
(Resolution, 4.) To say that it is, is — so I submit — to trivialize the plain command of the Constitution.

Checkpoints, I further submit, are things of martial rule, and things of the past. They first saw the light of day by
virtue of General Order No. 66 (AUTHORIZING THE CHIEF OF CONSTABULARY TO ESTABLISH
CHECKPOINTS, UPDATE LISTS OF WANTED PERSONS AND CONDUCT DRAGNET OPERATIONS AND
FOR OTHER PURPOSES), a martial law issuance, as amended by General Order No. 67 (AMENDING AND
AMPLIFYING PARAGRAPH 7 OF GENERAL ORDER NO. 66 DATED SEPTEMBER 12, 1980), yet another
martial law issuance. (See O.G. 4224-4226; 4226-4227 [Aug., 1983].) They are, so I strongly submit, repressive
measures, the same measures against which we had fought so painstakingly in our quest for liberty, a quest that
ended at EDSA and a quest that terminated a dictatorship. How soon we forget.

While the right against unreasonable searches and seizures, as my brethren advance, is a right personal to the
aggrieved party, the petitioners, precisely, have come to Court because they had been, or had felt, aggrieved. I
submit that in that event, the burden is the State's, to demonstrate the reasonableness of the search. The
petitioners, Ricardo Valmonte in particular, need not, therefore, have illustrated the "details of the incident"
(Resolution, supra, 4) in all their gore and gruesomeness.

In any event, the absence alone of a search warrant, as I have averred, makes checkpoint searches
unreasonable, and by itself, subject to constitutional challenges. (Supra.) As it is, "checkpoints", have become
"search warrants" unto themselves a roving one at that.

That "[n]ot all searches and seizures are prohibited," the majority points out, is fine. And so is "a reasonable
search is not to be determined by any fixed formula but is to be resolved according to the facts of each case."
(Supra) But the question, exactly, is: Is (are) the search(es) in this case reasonable? I submit that it (they) is (are)
not, for one simple reason: No search warrant has been issued by a judge.

I likewise do not find this case to be a simple matter of an "officer merely draw(ing) aside the curtain of a vacant
vehicle ... or simply look(ing) (supra) there, "or flash(ing) a light therein." (Supra) What we have here is Orwell's
Big Brother watching every step we take and every move we make.

As it also is, "checkpoints" are apparently, State policy. The American cases the majority refers to involve routine
checks compelled by "probable cause". What we have here, however, is not simply a policeman on the beat but
armed men, CAFGU or Alsa Masa, who hold the power of life or death over the citizenry, who fire with no
provocation and without batting an eyelash. They likewise shoot you simply because they do not like your face. I
have witnessed actual incidents.

Washington said that militia can not be made to dictate the terms for the nation. He can not be anymore correct
here.

"Between the inherent right of the state to protect its existence ... and on individual's right against a warrantless
search, which is reasonably conducted, "so my brethren go on, the former shall prevail. (Supra) First, this is the
same lie that the hated despot foisted on the Filipino people. It is a serious mistake to fall for it a second time
around. Second, the checkpoint searches herein are unreasonable: There was no warrant.

A final word. After twenty years of tyranny, the dawn is upon us. The country is once again the "showcase of
democracy" in Asia. But if in many cases, it has been "paper democracy", let this Court anyway bring to pass its
stand, and make liberty in the land, a living reality.

I vote then, to grant the petition.


Separate Opinions

CRUZ, J., dissenting:

I dissent. The sweeping statements in the majority opinion are as dangerous as the checkpoints it would sustain
and fraught with serious threats to individual liberty. The bland declaration that individual rights must yield to the
demands of national security ignores the fact that the Bill of Rights was intended precisely to limit the authority of
the State even if asserted on the ground of national security. What is worse is that the searches and seizures are
peremptorily pronounced to be reasonable even without proof of probable cause and much less the required
warrant. The improbable excuse is that they are aimed at 'establishing an effective territorial defense, maintaining
peace and order, and providing an atmosphere conducive to the social, economic and political development of
the National Capital Region." For these purposes, every individual may be stopped and searched at random and
at any time simply because he excites the suspicion, caprice, hostility or malice of the officers manning the
checkpoints, on pain of arrest or worse, even being shot to death, if he resists.

I have no quarrel with a policeman flashing a light inside a parked vehicle on a dark street as a routine measure of
security and curiosity. But the case at bar is different. Military officers are systematically stationed at strategic
checkpoint to actively ferret out suspected criminals by detaining and searching any individual who in their
opinion might impair "the social, economic and political development of the National Capital Region." It is
incredible that we can sustain such a measure. And we are not even under martial law.

Unless we are vigilant of our rights, we may find ourselves back to the dark era of the truncheon and the barbed
wire, with the Court itself a captive of its own complaisance and sitting at the death-bed of liberty.

SARMIENTO, J., dissenting:

I join Justice Isagani Cruz in his dissent, delivered so staightforwardly and eloquently. I am agreed that the
existence alone of checkpoints makes search done therein, unreasonable and hence, repugnant to the
Constitution.

The Charter says that the people enjoy the right of security of person, home, and effects. (CONST., art. III, sec. 2.)
It is also the bedrock — the right of the people to be left alone — on which the regime of law and constitutionalism
rest. It is not, as the majority would put it, a matter of "occasional inconveniences, discomfort and even irritation."
(Resolution, 4.) To say that it is, is — so I submit — to trivialize the plain command of the Constitution.

Checkpoints, I further submit, are things of martial rule, and things of the past. They first saw the light of day by
virtue of General Order No. 66 (AUTHORIZING THE CHIEF OF CONSTABULARY TO ESTABLISH
CHECKPOINTS, UPDATE LISTS OF WANTED PERSONS AND CONDUCT DRAGNET OPERATIONS AND
FOR OTHER PURPOSES), a martial law issuance, as amended by General Order No. 67 (AMENDING AND
AMPLIFYING PARAGRAPH 7 OF GENERAL ORDER NO. 66 DATED SEPTEMBER 12, 1980), yet another
martial law issuance. (See O.G. 4224-4226; 4226-4227 [Aug., 1983].) They are, so I strongly submit, repressive
measures, the same measures against which we had fought so painstakingly in our quest for liberty, a quest that
ended at EDSA and a quest that terminated a dictatorship. How soon we forget.

While the right against unreasonable searches and seizures, as my brethren advance, is a right personal to the
aggrieved party, the petitioners, precisely, have come to Court because they had been, or had felt, aggrieved. I
submit that in that event, the burden is the State's, to demonstrate the reasonableness of the search. The
petitioners, Ricardo Valmonte in particular, need not, therefore, have illustrated the "details of the incident"
(Resolution, supra, 4) in all their gore and gruesomeness.

In any event, the absence alone of a search warrant, as I have averred, makes checkpoint searches
unreasonable, and by itself, subject to constitutional challenges. (Supra.) As it is, "checkpoints", have become
"search warrants" unto themselves a roving one at that.

That "[n]ot all searches and seizures are prohibited," the majority points out, is fine. And so is "a reasonable
search is not to be determined by any fixed formula but is to be resolved according to the facts of each case."
(Supra) But the question, exactly, is: Is (are) the search(es) in this case reasonable? I submit that it (they) is (are)
not, for one simple reason: No search warrant has been issued by a judge.

I likewise do not find this case to be a simple matter of an "officer merely draw(ing) aside the curtain of a vacant
vehicle ... or simply look(ing) (supra) there, "or flash(ing) a light therein." (Supra) What we have here is Orwell's
Big Brother watching every step we take and every move we make.

As it also is, "checkpoints" are apparently, State policy. The American cases the majority refers to involve routine
checks compelled by "probable cause". What we have here, however, is not simply a policeman on the beat but
armed men, CAFGU or Alsa Masa, who hold the power of life or death over the citizenry, who fire with no
provocation and without batting an eyelash. They likewise shoot you simply because they do not like your face. I
have witnessed actual incidents.

Washington said that militia can not be made to dictate the terms for the nation. He can not be anymore correct
here.

"Between the inherent right of the state to protect its existence ... and on individual's right against a warrantless
search, which is reasonably conducted, "so my brethren go on, the former shall prevail. (Supra) First, this is the
same lie that the hated despot foisted on the Filipino people. It is a serious mistake to fall for it a second time
around. Second, the checkpoint searches herein are unreasonable: There was no warrant.

A final word. After twenty years of tyranny, the dawn is upon us. The country is once again the "showcase of
democracy" in Asia. But if in many cases, it has been "paper democracy", let this Court anyway bring to pass its
stand, and make liberty in the land, a living reality.

I vote then, to grant the petition.

S-ar putea să vă placă și